Loading documents preview...
Intellectual Property Law Prof. Susan D. Villanueva 2 n d Semester, AY 2011-2012 Katrina Michelle Mancao Sp eci al t han k s to K a rich i Sa n tos for hel pi n g m e c omp let e m y no tes , a nd to M a rie E strel la a n d Di Bo n ill a fo r sh ari ng thei r correc t a n s w ers to o ur hw J
Contents
I.
INTRODUCTION
1
II.
CONSTITUTION
2
III.
INTERNATIONAL CONVENTIONS
3
IV.
CIVIL CODE
5
V.
COPYRIGHT AND RELATED RIGHTS
6
VI.
LAW ON TRADEMARKS, TRADENAMES AND SERVICE MARKS
VII.
LAW ON PATENTS
I.
63 126
-‐
Introduction
Justin Hughes, The Philosophy of Intellectual Property,
-‐
77 Geo. L.J. 287 (1988)
-‐
Marshall A. Leaffer, Protecting United States Intellectual Property Abroad: Toward a New Multilateralism, 76 Iowa L. Rev. 273 (1991) A. Samuel Oddi, The International Patent System and Third World Development, 1987 Duke L. J. 831 (1987) Michael A. Heller and Rebecca Eisenberg, Can Patents Deter Innovation? The Anti-‐commons in Biomedical Research, 280 Science 698 (1998)
-‐
24 November 2011 Intellectual Property: -‐ It is a form of property. (Note that under the Civil Code, intellectual creation is one of the modes of acquiring ownership.) -‐ It is an intangible property right that belongs to the creator.
It is separate and distinct from the thing created. o Thus, it can be subject to a separate contract. Rules for IP are different from rules governing traditional property. What’s the point of granting IP? o IP is an incentive system designed to encourage individuals to create, to invent, to innovate. o BUT the higher purpose of IP is to improve the lives of the public. It operates on the assumption that everyone would benefit from the creation of more works. Thus, IP is granted under the “beneficial to the people” clause of the Constitution. IP, however, can be in conflict with public access. It grants a form of monopoly that restricts people’s access to something. Thus, IP Law must be balanced with a strong competition law.
TRIPS – it is mandatory to those who would like to enter or join the WTO to agree to this Agreement.
Katrina Michelle Mancao their intellectual property and creations, particularly when beneficial to the people, for such period as may be provided by law. ARTS AND CULTURE
II. Constitution ARTICLE XII, SECTION 6. The use of property bears a social function, and all economic agents shall contribute to the common good. Individuals and private groups, including corporations, cooperatives, and similar collective organizations, shall have the right to own, establish, and operate economic enterprises, subject to the duty of the State to promote distributive justice and to intervene when the common good so demands.
SECTION 14. The State shall foster the preservation, enrichment, and dynamic evolution of a Filipino national culture based on the principle of unity in diversity in a climate of free artistic and intellectual expression.
ARTICLE XII, SECTION 14.
SECTION 15.
The sustained development of a reservoir of national talents consisting of Filipino scientists, entrepreneurs, professionals, managers, high-‐level technical manpower and skilled workers and craftsmen in all fields shall be promoted by the State. The State shall encourage appropriate technology and regulate its transfer for the national benefit. The practice of all professions in the Philippines shall be limited to Filipino citizens, save in cases prescribed by law.
Arts and letters shall enjoy the patronage of the State. The State shall conserve, promote, and popularize the nation's historical and cultural heritage and resources, as well as artistic creations.
SECTION 16. All the country's artistic and historic wealth constitutes the cultural treasure of the nation and shall be under the protection of the State which may regulate its disposition.
ARTICLE XIV, SECTIONS 10 TO 18.
SECTION 17.
SCIENCE AND TECHNOLOGY
The State shall recognize, respect, and protect the rights of indigenous cultural communities to preserve and develop their cultures, traditions, and institutions. It shall consider these rights in the formulation of national plans and policies.
SECTION 10. Science and technology are essential for national development and progress. The State shall give priority to research and development, invention, innovation, and their utilization; and to science and technology education, training, and services. It shall support indigenous, appropriate, and self-‐reliant scientific and technological capabilities, and their application to the country's productive systems and national life.
SECTION 18. The State shall ensure equal access to cultural opportunities through the educational system, public or private cultural entities, scholarships, grants and other incentives, and community cultural centers, and other public venues. The State shall encourage and support researches and studies on the arts and culture. 08 December 2011 Note that under the Constitution (Section 11, Article XIV), there are “other forms of incentives” granted to encourage innovation and creation. Why is it still necessary to have IP laws despite the presence of this clause in the Constitution? -‐ We are obligated by the TRIPS, as a signatory to it, to pass laws that will protect IP. What makes the IP system more superior (than the mere granting of awards)? What makes it more sustainable? -‐ Term and duration of the benefit. They may be enjoyed for a longer period of time. -‐ IP System is more sustainable because the rules
SECTION 11. The Congress may provide for incentives, including tax deductions, to encourage private participation in programs of basic and applied scientific research. Scholarships, grants-‐in-‐aid, or other forms of incentives shall be provided to deserving science students, researchers, scientists, inventors, technologists, and specially gifted citizens.
SECTION 12. The State shall regulate the transfer and promote the adaptation of technology from all sources for the national benefit. It shall encourage the widest participation of private groups, local governments, and community-‐based organizations in the generation and utilization of science and technology.
SECTION 13. The State shall protect and secure the exclusive rights of scientists, inventors, artists, and other gifted citizens to
2z
Intellectual Property Law in placed and the standards of protection are objective and open. The rights granted under the IP system attached even when the work produced is not good in the subjective sense at the time it was granted (note that standards change).
III.
International Conventions
It is important to know about the conventions because the States are mandated to amend their laws to comply with it.
It is possible for 2 forms of IP to protect a single work. -‐ e.g. Winnie the Pooh and Friends – protected by trademark and copyright.
A. Berne Convention for the Protection of Literary and Artistic Works as revised in Brussels (Brussels Act) It was concluded in 1886.
B. Paris Convention for the Protection of Industrial Property Rights (Paris Convention) The Paris Convention was concluded in 1883. It covers industrial property in its broadest sense including patents, trademarks, industrial design, utility models (sometimes called “little patents”), tradenames, geographical indications, and methods of unfair competition. (Catindig)
C.
International Convention for the Protection of Performers, Producers of Phonograms and Broadcasting Organizations (Rome Convention)
It was concluded in 1961.
D. Convention Establishing the World Intellectual Property Organization
E.
Budapest Treaty on the International Recognition of the Deposit of Microorganisms for the Purposes of Patent Procedure
Concluded in 1977, this Treaty requires a Contracting State, which allows or requires the deposit of microorganisms for purposes of patent procedure, to recognize, for such purposes, the deposit of a microorganism with any “international depositary authority.”
F.
Agreement on Trade Related Aspects of Intellectual Property, Including Trade in Counterfeit Goods of the General Agreement in Tariffs and Trade
TANADA V. ANGARA (1997) Relevant issue: 4th issue Petitioners aver that paragraph 1, Article 34 of the General Provisions and Basic Principles of the Agreement
o3
Katrina Michelle Mancao on Trade-‐Related Aspects of Intellectual Property Rights (TRIPS) intrudes on the power of the Supreme Court to promulgate rules concerning pleading, practice and procedures. To understand the scope and meaning of Article 34, TRIPS, it will be fruitful to restate its full text as follows: “Article 34. Process Patents: Burden of Proof 1. For the purposes of civil proceedings in respect of the infringement of the rights of the owner referred to in paragraph 1(b) of Article 28, if the subject matter of a patent is a process for obtaining a product, the judicial authorities shall have the authority to order the defendant to prove that the process to obtain an identical product is different from the patented process. Therefore, Members shall provide, in at least one of the following circumstances, that any identical product when produced without the consent of the patent owner shall, in the absence of proof to the contrary, be deemed to have been obtained by the patented process: (a) if the product obtained by the patented process is new; (b) if there is a substantial likelihood that the identical product was made by the process and the owner of the patent has been unable through reasonable efforts to determine the process actually used. 2. Any Member shall be free to provide that the burden of proof indicated in paragraph 1 shall be on the alleged infringer only if the condition referred to in subparagraph (a) is fulfilled or only if the condition referred to in subparagraph (b) is fulfilled. 3. In the adduction of proof to the contrary, the legitimate interests of defendants in protecting their manufacturing and business secrets shall be taken into account.” From the above, a WTO Member is required to provide a rule of disputable (note the words “in the absence of proof to the contrary”) presumption that a product shown to be identical to one produced with the use of a patented process shall be deemed to have been obtained by the (illegal) use of the said patented process, (1) where such product obtained by the patented product is new, or (2) where there is “substantial likelihood” that the identical product was made with the use of the said patented process but the owner of the
patent could not determine the exact process used in obtaining such identical product. Hence, the “burden of proof” contemplated by Article 34 should actually be understood as the duty of the alleged patent infringer to overthrow such presumption. Such burden, properly understood, actually refers to the “burden of evidence” (burden of going forward) placed on the producer of the identical (or fake) product to show that his product was produced without the use of the patented process. The foregoing notwithstanding, the patent owner still has the “burden of proof” since, regardless of the presumption provided under paragraph 1 of Article 34, such owner still has to introduce evidence of the existence of the alleged identical product, the fact that it is “identical” to the genuine one produced by the patented process and the fact of “newness” of the genuine product or the fact of “substantial likelihood” that the identical product was made by the patented process. The foregoing should really present no problem in changing the rules of evidence as the present law on the subject, Republic Act No. 165, as amended, otherwise known as the Patent Law, provides a similar presumption in cases of infringement of patented design or utility model, thus: “SEC. 60. Infringement. -‐ Infringement of a design patent or of a patent for utility model shall consist in unauthorized copying of the patented design or utility model for the purpose of trade or industry in the article or product and in the making, using or selling of the article or product copying the patented design or utility model. Identity or substantial identity with the patented design or utility model shall constitute evidence of copying.” (underscoring supplied) Moreover, it should be noted that the requirement of Article 34 to provide a disputable presumption applies only if (1) the product obtained by the patented process is NEW or (2) there is a substantial likelihood that the identical product was made by the process and the process owner has not been able through reasonable effort to determine the process used. Where either of these two provisos does not obtain, members shall be free to determine the appropriate method of implementing the provisions of TRIPS within their own internal systems and processes. By and large, the arguments adduced in connection with our disposition of the third issue -‐-‐ derogation of legislative power -‐ will apply to this fourth issue also. Suffice it to say that the reciprocity clause more than justifies such intrusion, if any actually exists. Besides, Article 34 does not contain an unreasonable burden,
4z
Intellectual Property Law consistent as it is with due process and the concept of adversarial dispute settlement inherent in our judicial system. So too, since the Philippine is a signatory to most international conventions on patents, trademarks and copyrights, the adjustment in legislation and rules of procedure will not be substantial.
ARTICLE 723. Letters and other private communications in writing are owned by the person to whom they are addressed and delivered, but they cannot be published or disseminated without the consent of the writer or his heirs. However, the court may authorize their publication or dissemination if the public good or the interest of justice so requires.
G. Patent Cooperation Treaty
ARTICLE 724.
Question: Was it the right time to enter it in 2001?
Special laws govern copyright and patent.
A trade-‐mark or trade-‐name duly registered in the proper government bureau or office is owned by and pertains to the person, corporation, or firm registering the same, subject to the provisions of special laws.
ARTICLE 520.
IV. Civil Code ARTICLE 712. ★ Ownership is acquired by occupation and by intellectual creation. Ownership and other real rights over property are acquired and transmitted by law, by donation, by testate and intestate succession, and in consequence of certain contracts, by tradition. They may also be acquired by means of prescription.
ARTICLE 521. The goodwill of a business is property, and may be transferred together with the right to use the name under which the business is conducted.
ARTICLE 522. Trade-‐marks and trade-‐names are governed by special laws.
ARTICLE 721. By intellectual creation, the following persons acquire ownership: (1) The author with regard to his literary, dramatic, historical, legal, philosophical, scientific or other work; (2) The composer; as to his musical composition; (3) The painter, sculptor, or other artist, with respect to the product of his art; (4) The scientist or technologist or any other person with regard to his discovery or invention.
Differences between trademarks and patent KHO V. COURT OF APPEALS (2002) •
copyright,
Refresher: Elidad C. Kho, doing business under the name of KEC Cosmetics Laboratory, alleged that she is the registered owner of the copyrights Chin Chun Su and Oval Facial Cream Container/Case, and that she has the patent rights on Chin Chun Su & Device and Chin Chun Su for medicated cream. Petitioner likewise alleged that respondent Summerville General Merchandising and Co. advertised and sold petitioner's cream and products under the brand name Chin Chun Su in similar containers that petitioner uses, thereby misleading the public and resulting in the decline in the petitioner's sales and income. The respondents, on the other hand, alleged as their defense that Summerville is the exclusive and authorized importer, re-‐packer and distributor of Chin Chun Su products manufactured by Shun Yi Factory of Taiwan; that the said Taiwanese manufacturing company authorized Summerville to register its trade name Chin Chun Su Medicated Cream with the Philippine Patent Office and other appropriate governmental agencies; that KEC Cosmetics Laboratory of the petitioner obtained the copyrights through misrepresentation and falsification; and, that the authority of Quintin Cheng,
ARTICLE 722. The author and the composer, mentioned in Nos. 1 and 2 of the preceding article, shall have the ownership of their creations even before the publication of the same. Once their works are published, their rights are governed by the Copyright laws. The painter, sculptor or other artist shall have dominion over the product of his art even before it is copyrighted. The scientist or technologist has the ownership of his discovery or invention even before it is patented.
o5
Katrina Michelle Mancao assignee of the patent registration certificate, to distribute and market Chin Chun Su products in the Philippines had already been terminated by the said Taiwanese Manufacturing Company. Issue: Whether the copyright and patent over the name and container of a beauty cream product would entitle the registrant to the use and ownership over the same to the exclusion of others. Doctrine: Trademark, copyright and patents are different intellectual property rights that cannot be interchanged with one another. Ø A trademark is any visible sign capable of distinguishing the goods (trademark) or services (service mark) of an enterprise and shall include a stamped or marked container of goods. Ø In relation thereto, a trade name means the name or designation identifying or distinguishing an enterprise. Ø Meanwhile, the scope of a copyright is confined to literary and artistic works which are original intellectual creations in the literary and artistic domain protected from the moment of their creation. Ø Patentable inventions, on the other hand, refer to any technical solution of a problem in any field of human activity which is new, involves an inventive step and is industrially applicable. Petitioner has no right to support her claim for the exclusive use of the subject trade name and its container. The name and container of a beauty cream product are proper subjects of a trademark inasmuch as the same falls squarely within its definition. In order to be entitled to exclusively use the same in the sale of the beauty cream product, the user must sufficiently prove that she registered or used it before anybody else did. The petitioner’s copyright and patent registration of the name and container would not guarantee her the right to the exclusive use of the same for the reason that they are not appropriate subjects of the said intellectual rights. Consequently, a preliminary injunction order cannot be issued for the reason that the petitioner has not proven that she has a clear right over the said name and container to the exclusion of others, not having proven that she has registered a trademark thereto or used the same before anyone did.
V.
Copyright and Related Rights
A. Legislative history of the law on Copyright and Related rights 1. Spanish Law on Intellectual Property 2. US Copyright Law 3. Act 3134 (Copyright Law) 4. Presidential Decree No. 49 (took effect on 27 December 1972) 5. Presidential Decree No. 1988 (amended by PD 49 b y inserting Section 56) 6. RA 8293 – Intellectual Property Code (took effect on 01 January 1998) The Philippines its one of the first developing countries to amend its laws.
B. Law on Copyright SECTION 241. EFFECTIVITY. – This Act shall take effect on 1 January 1998.
SECTION 239. REPEALS. 239.3. The provisions of this Act shall apply to works in which copyright protection obtained prior to the effectivity of this Act is subsisting: Provided, That the application of this Act shall not result in the diminution of such protection.
SECTION 236. PRESERVATION OF EXISTING RIGHTS. – Nothing herein shall adversely affect the rights on the enforcement of rights in patents, utility models, industrial designs, marks and works, acquired in good faith prior to the effective date of this Act.
1.
Definition of Copyright SECTION 177. COPYRIGHT OR ECONOMIC RIGHTS. – Subject to the provisions of Chapter VIII, copyright or economic rights shall consist of the exclusive right to carry out, authorize or prevent the following acts: 177.1. Reproduction of the work or substantial portion of the work; 177.2. Dramatization, translation, adaptation, abridgment, arrangement or other transformation of the work;
6z
Intellectual Property Law 177.3. The first public distribution of the original and each copy of the work by sale or other forms of transfer of ownership; 177.4. Rental of the original or a copy of an audiovisual or cinematographic work, a work embodied in a sound recording, a computer program, a compilation of data and other materials or a musical work in graphic form, irrespective of the ownership of the original or the copy which is the subject of the rental; 177.5. Public display of the original or a copy of the work; 177.6. Public performance of the work; and 177.7. Other communication to the public of the work.
RULE 12, COPYRIGHT SAFEGUARDS AND REGULATIONS DEFINITION OF TERMS SECTION 1. FIRST PUBLIC DISTRIBUTION OF WORK. An exclusive right of first distribution of work includes all acts involving distribution, specifically including the first importation of an original and each copy of the work into the jurisdiction of the Republic of the Philippines.
2.
Standard for Copyright Protection SECTION 172. LITERARY AND ARTISTIC WORKS. – 172.1. Literary and artistic works, hereinafter referred to as "works", are original intellectual creations in the literary and artistic domain protected from the moment of their creation and shall include in particular: (a) Books, pamphlets, articles and other writings; (b) Periodicals and newspapers; (c) Lectures, sermons, addresses, dissertations prepared for oral delivery, whether or not reduced in writing or other material form; (d) Letters; (e) Dramatic or dramatico-‐musical compositions; choreographic works or entertainment in dumb shows; (f) Musical compositions, with or without words; (g) Works of drawing, painting, architecture, sculpture, engraving, lithography or other works of art; models or designs for works of art1; (h) Original ornamental designs or models for articles of manufacture, whether or not registrable as an industrial design, and other works of applied art; (i) Illustrations, maps, plans, sketches, charts and three-‐dimensional works relative to geography, topography, architecture or science; (j) Drawings or plastic works of a scientific or technical character;
RULE 2, COPYRIGHT SAFEGUARDS AND REGULATIONS DEFINITION OF TERMS For the purpose of these Copyright Safeguards and Regulations, the following terms are herein defined: x x x Copyright is a right granted by statute to the author or originator of literary, scholarly, scientific, or artistic productions, including computer programs. A copyright gives him the legal right to determine how the work is used and to obtain economic benefits from the work. For example, the owner of a copyright for a book or a piece of software has the exclusive rights to use, copy, distribute, and sell copies of the work, including later editions or versions of the work. If another person improperly uses material covered by a copyright, the copyright owner can obtain legal relief x x x
RULE 11, COPYRIGHT SAFEGUARDS AND REGULATIONS DEFINITION OF TERMS SECTION 1. COMMUNICATION TO THE PUBLIC OF COPYRIGHTED WORKS. “Communication to the public” or “communicate to the public,” also includes point-‐to-‐point transmission of a work, including video on demand, and providing access to an electronic retrieval system, such as computer databases, servers, or similar electronic storage devices. Broadcasting, rebroadcasting, retransmission by cable, and broadcast and retransmission by satellite are all acts of “communication to the public” within the meaning of the IPC.
1
o7
“Works of applied art” à artistic creation with utilitarian value
Katrina Michelle Mancao (k) Photographic works including works produced by a process analogous to photography; lantern slides; (l) Audiovisual works and cinematographic works and works produced by a process analogous to cinematography or any process for making audio-‐visual recordings; (m) Pictorial illustrations and advertisements; (n) Computer programs2; and (o) Other literary, scholarly, scientific and artistic works.
Doctrine: A person to be entitled to a copyright must be the original creator of the work. He must have created it by his own skill, labor and judgment without directly copying or evasively imitating the work of another. The grant of preliminary injunction in a case rests on the sound discretion of the court with the caveat that it should be made with extreme caution. Its grant depends chiefly on the extent of doubt on the validity of the copyright, existence of infringement, and the damages sustained by such infringement. In our view, the copies of the certificates of copyright registered in the name of Ceroilfood Shandong sufficiently raise reasonable doubt. With such a doubt preliminary injunction is unavailing.
172.2. Works are protected by the sole fact of their creation, irrespective of their mode or form of expression, as well as of their content, quality and purpose.
SAMBAR V. LEVI STRAUSS (2002) Refresher: On 28 September 1987, Levi Strauss & Co. and Levi Strauss (Phil.), Inc. demanded that CVS Garment Enterprises (CVSGE) desist from using their stitched arcuate design on the Europress jeans which CVSGE advertised in Manila Bulletin. Atty. Benjamin Gruba, counsel of CVSGE, replied that the arcuate design on the back pockets of Europress jeans was different from the design on the back pockets of Levi’s jeans. He further asserted that his client had a copyright on the design it was using. Thereafter, private respondents filed a complaint against Sambar, doing business under the name and style of CVSGE. Private respondents also impleaded the Director of the National Library. Levi Strauss and Co. (LS&Co.), an internationally known clothing manufacturer, owns the arcuate design trademark which was registered under U.S. Trademark Registration No. 404, 248 on November 16, 1943, and in the Principal Register of trademarks with the Philippine Patent Office under Certificate of Registration No. 20240 issued on October 8, 1973; that through a Trademark Technical Data and Technical Assistance Agreement with Levi Strauss (Phil.) Inc. (LSPI) in 1972, LS&Co. granted LSPI a non-‐exclusive license to use the arcuate trademark in its manufacture and sale of Levi’s pants, jackets and shirts in the Philippines; that in 1983, LS&Co. also appointed LSPI as its agent and attorney-‐in-‐fact to protect its trademark in the Philippines; and that sometime in 1987, CVSGIC and Venancio Sambar, without the consent and authority of private respondents and in infringement and unfair competition, sold and advertised, and despite demands to cease and desist, continued to manufacture, sell and advertise denim pants under the brand name “Europress” with back pockets bearing a design similar to the arcuate trademark of private respondents, thereby causing confusion on the buying public, prejudicial to private respondents’ goodwill and property right.
CHING KIAN CHUAN V. CA (2001) Refresher: Petitioner Wilson Ong Ching Kian Chuan (“Ong”) imports vermicelli from China National Cereals Oils and Foodstuffs Import and Export Corporation, based in Beijing, China, under the firm name C.K.C. Trading. He repacks it in cellophane wrappers with a design of two-‐ dragons and the TOWER trademark on the uppermost portion. Ong acquired a Certificate of Copyright Registration from the National Library on June 9, 1993 on the said design. Ong discovered that private respondent Lorenzo Tan repacked his vermicelli he imports from the same company but based in Qingdao, China in a "nearly" identical wrapper. On September 16, 1993, Ong filed against Tan a verified complaint for infringement of copyright with damages and prayer for temporary restraining order or writ of preliminary injunction with the Regional Trial Court in Quezon City. Ong alleged that he was the holder of a Certificate of Copyright Registration over the cellophane wrapper with the two-‐ dragon design, and that Tan used an identical wrapper in his business. In his prayer for a preliminary injunction in addition to damages, he asked that Tan be restrained from using the wrapper. He said he would post a bond to guarantee the payment of damages resulting from the issuance of the writ of preliminary injunction. 2
Class notes: This is the outcome of the lobbying in the US. At that time, technology was not as advanced. They were concerned more on DURATION (copyright protection is for 50 years!). They found out later that duration is not important in computer programs because they are immediately replaced or upgraded.
8z
Intellectual Property Law Doctrine: To be entitled to copyright, the thing being copyrighted must be original, created by the author through his own skill, labor and judgment, without directly copying or evasively imitating the work of another.
3.
He has a right to determine whether it shall be published at all, and if published, when, where, by whom, and in what form. This exclusive right is confined to the first publication. When once published, it is dedicated to the public, and the author loses the exclusive right to control subsequent publication by others, unless the work is placed under the protection of the copyright law. Remarks: Decided under the old law.
When does Copyright vest?
SANTOS V. MCCULLOUGH PRINTING CO. (1964)
FILIPINO SOCIETY OF COMPOSERS V. BENJAMIN TAN (1987)
Refresher: The complaint alleges that plaintiff Mauro Malang Santos designed for former Ambassador Felino Neri, for his personal Christmas Card greetings for the year 1959 (Rural Christmas time scene), the artistic motif in question. The following year the defendant McCullough Printing Company, without the knowledge and authority of plaintiff, displayed the very design in its album of Christmas cards and offered it for sale, for a price. Doctrine: Paragraph 33 of Patent Office Administrative Order No. 3 (as amended dated September 18, 1947) entitled "Rules of Practice in the Philippines Patent Office relating to the Registration of Copyright Claims" promulgated pursuant to Republic Act 165, provides, among others, that an intellectual creation should be copyrighted 30 days after its publication, if made in Manila, or within 60 day's if made elsewhere, failure of which renders such creation public property. In the case at bar, even as of this moment, there is no copyright for the design in question. We are not also prepared to accept the contention of appellant that the publication of the design was a limited one, or that there was an understanding that only Ambassador Neri should, have absolute right to use the same. In the first place, if such were the condition then Ambassador Neri would be the aggrieved party, and not the appellant. In the second place, if there was such a limited publication or prohibition, the same was not shown on the face of the design. When the purpose is a limited publication, but the effect is general publication, irrevocable rights thereupon become vested in the public, in consequence of which enforcement of the restriction becomes impossible. It has been held that the effect of offering for sale a dress, for example manufactured in accordance with an original design which is not protected by either a copyright or a patent, is to divest the owner of his common law rights therein by virtue of the publication of a 'copy' and thereafter anyone is free to copy the design or the dress. When Ambassador Neri distributed 800 copies of the design in controversy, the plaintiff lost control of his design and the necessary implication was that there had been a general publication, there having been no showing of a clear indication that a limited publication was intended. The author of a literary composition has a light to the first publication thereof.
Refresher: Plaintiff-‐appellant is a non-‐profit association of authors, composers and publishers duly organized under the Corporation Law of the Philippines and registered with the Securities and Exchange Commission. Said association is the owner of certain musical compositions among which are the songs entitled: "Dahil Sa Iyo", "Sapagkat Ikaw Ay Akin," "Sapagkat Kami Ay Tao Lamang" and "The Nearness Of You." On the other hand, defendant-‐appellee is the operator of a restaurant known as "Alex Soda Foundation and Restaurant" where a combo with professional singers, hired to play and sing musical compositions to entertain and amuse customers therein, were playing and singing the above-‐mentioned compositions without any license or permission from the appellant to play or sing the same. Accordingly, appellant demanded from the appellee payment of the necessary license fee for the playing and singing of aforesaid compositions but the demand was ignored. Hence, on November 7, 1967, appellant filed a complaint with the lower court for infringement of copyright against defendant-‐appellee for allowing the playing in defendant-‐appellee's restaurant of said songs copyrighted in the name of the former. Issue: Whether or not the playing and signing of musical compositions which have been copyrighted under the provisions of the Copyright Law (Act 3134) inside the establishment of the defendant-‐appellee constitute a public performance for profit within the meaning and contemplation of the Copyright Law of the Philippines; and assuming that there were indeed public performances for profit, whether or not appellee can be held liable therefor. Doctrine: There were "public performances for profit. " The word "perform" as used in the Act has been applied to "one who plays a musical composition on a piano, thereby producing in the air sound waves which are heard as music ... and if the instrument he plays on is a piano plus a broadcasting apparatus, so that waves are
o9
Katrina Michelle Mancao thrown out, not only upon the air, but upon the other, then also he is performing the musical composition." In relation thereto, it has been held that "The playing of music in dine and dance establishment which was paid for by the public in purchases of food and drink constituted "performance for profit" within a Copyright Law." Thus, it has been explained that while it is possible in such establishments for the patrons to purchase their food and drinks and at the same time dance to the music of the orchestra, the music is furnished and used by the orchestra for the purpose of inducing the public to patronize the establishment and pay for the entertainment in the purchase of food and drinks. The defendant conducts his place of business for profit, and it is public; and the music is performed for profit. In a similar case, the Court ruled that "The Performance in a restaurant or hotel dining room, by persons employed by the proprietor, of a copyrighted musical composition, for the entertainment of patrons, without charge for admission to hear it, infringes the exclusive right of the owner of the copyright." In delivering the opinion of the Court in said two cases, Justice Holmes elaborated thus: If the rights under the copyright are infringed only by a performance where money is taken at the door, they are very imperfectly protected. Performances not different in kind from those of the defendants could be given that might compete with and even destroy the success of the monopoly that the law intends the plaintiffs to have. It is enough to say that there is no need to construe the statute so narrowly. The defendants' performances are not eleemosynary. They are part of a total for which the public pays, and the fact that the price of the whole is attributed to a particular item which those present are expected to order is not important. It is true that the music is not the sole object, but neither is the food, which probably could be got cheaper elsewhere. The object is a repast in surroundings that to people having limited power of conversation or disliking the rival noise, give a luxurious pleasure not to be had from eating a silent meal. If music did not pay, it would be given up. If it pays, it pays out of the public's pocket. Whether it pays or not, the purpose of employing it is profit, and that is enough. In the case at bar, it is admitted that the patrons of the restaurant in question pay only for the food and drinks and apparently not for listening to the music. As found by the trial court, the music provided is for the purpose of entertaining and amusing the customers in order to make the establishment more attractive and desirable. It will
be noted that for the playing and singing the musical compositions involved, the combo was paid as independent contractors by the appellant. It is therefore obvious that the expenses entailed thereby are added to the overhead of the restaurant which are either eventually charged in the price of the food and drinks or to the overall total of additional income produced by the bigger volume of business which the entertainment was programmed to attract. Consequently, it is beyond question that the playing and singing of the combo in defendant-‐appellee's restaurant constituted performance for profit contemplated by the Copyright Law. Nevertheless, appellee cannot be said to have infringed upon the Copyright Law. Appellee's allegation that the composers of the contested musical compositions waived their right in favor of the general public when they allowed their intellectual creations to become property of the public domain before applying for the corresponding copyrights for the same is correct. The Supreme Court has ruled that "Paragraph 33 of Patent Office Administrative Order No. 3 entitled 'Rules of Practice in the Philippines Patent Office relating to the Registration of Copyright Claims' promulgated pursuant to Republic Act 165, provides among other things that an intellectual creation should be copyrighted 30 days after its publication, if made in Manila, or within the 60 days if made elsewhere, failure of which renders such creation public property." Indeed, if the general public has made use of the object sought to be copyrighted for 30 days prior to the copyright application the law deems the object to have been donated to the public domain and the same can no longer be copyrighted.
SECTION 2, PD 49 (DECREE ON THE PROTECTION OF INTELLECTUAL PROPERTY) The Rights granted by this Decree shall, from the moment of creation, subsist with respect to any of the following classes of works: (A) Books, including composite and encyclopedic works, manuscripts, directories, and gazetteers; (B) Periodicals, including pamphlets and newspapers; (C) Lectures, sermons, addresses, dissertations prepared for oral delivery; (D) Letters; (E) Dramatic or dramatico-‐musical compositions; choreographic works and entertainments in dumb shows, the acting form of which is fixed in writing or otherwise;
10 z
Intellectual Property Law (F) Musical compositions, with or without words; (G) Works of drawing, painting, architecture, sculpture, engraving, lithography, and other works of art; models or designs for works of art; (H) Reproductions of a work of art; (I) Original ornamental designs or models for articles of manufacture, whether or not patentable, and other works of applied art; (J) Maps, plans, sketches, and charts; (K) Drawings, or plastic works of a scientific or technical character; (L) Photographic works and works produced by a process analogous to photography; lantern slides; (M) Cinematographic works and works produced by a process analogous to cinematography or any process for making audio-‐visual recordings; (N) Computer programs; (O) Prints, pictorial, illustration, advertising copies, labels, tags, and box wraps; (P) Dramatization, translations, adaptations, abridgements, arrangements and other alterations of literary, musical or artistic works or of works of the Philippine Government as herein defined, which shall be protected as provided in Section 8 of this Decree. (Q) Collection of literary, scholarly, or artistic works or of works referred to in Section 9 of this Decree which by reason of the selection and arrangement of their contents constitute intellectual creations, the same to be protected as such in accordance with Section 8 of this Decree. (R) Other literary, scholarly, scientific and artistic works.
protection in the country of origin of the work. Consequently, apart from the provisions of this Convention, the extent of protection, as well as the means of redress afforded to the author to protect his rights, shall be governed exclusively by the laws of the country where protection is claimed.
SECTION 172 AND 172.2, SUPRA
RULE 7, SECTIONS 2-‐4, COPYRIGHT SAFEGUARDS AND REGULATIONS SECTION 2. EFFECTS OF REGISTRATION AND DEPOSIT OF WORK. — The registration and deposit of the work is purely for recording the date of registration and deposit of the work and shall not be conclusive as to copyright ownership or the term of copyrights or the rights of the copyright owner, including neighboring rights. SECTION 3. EFFECT OF NON-‐REGISTRATION AND D EPOSIT. — If, within three (3) weeks after receipt by the copyright owner of a written demand from TNL and/or SCL3 for the deposit of a work listed in Rule 5 Sec. 4 of this SAR, the required copies are not delivered and the fee for registration and deposit is not paid, the copyright owner, his assignee, or his agent shall be liable to pay a fine equivalent to the required fee per month of delay and to pay to TNL and SCL the amount of the retail price of the best edition of the work. SECTION 4. OTHER LAWS. — Upon issuance of a certificate of deposit, the copyright owner shall be exempt from making additional deposits of the work with TNL or the SCL under other laws.
4.
Scope of Copyright
ARTICLE 2, BERNE CONVENTION FOR THE PROTECTION OF LITERARY AND ARTISTIC WORKS Protected Works: 1. “Literary and artistic works”; 2. Possible requirement of fixation; 3. Derivative works; 4. Official texts; 5. Collections; 6. Obligation to protect; beneficiaries of protection; 7. Works of applied art and industrial designs; 8. News (1) The expression “literary and artistic works” shall include every production in the literary, scientific and artistic domain, whatever may be the mode or form of
ARTICLE 5(2), BERNE CONVENTION FOR THE PROTECTION OF LITERARY AND ARTISTIC W ORKS RIGHTS GUARANTEED (2) The enjoyment and the exercise of these rights shall not be subject to any formality; such enjoyment and such exercise shall be independent of the existence of
3
o 11
TNL – The National Library; SCL – Library of the Supreme Court
Katrina Michelle Mancao its expression, such as books, pamphlets and other writings; lectures, addresses, sermons and other works of the same nature; dramatic or dramatico-‐musical works; choreographic works and entertainments in dumb show; musical compositions with or without words; cinematographic works to which are assimilated works expressed by a process analogous to cinematography; works of drawing, painting, architecture, sculpture, engraving and lithography; photographic works to which are assimilated works expressed by a process analogous to photography; works of applied art; illustrations, maps, plans, sketches and three-‐dimensional works relative to geography, topography, architecture or science. (2) It shall, however, be a matter for legislation in the countries of the Union to prescribe that works in general or any specified categories of works shall not be protected unless they have been fixed in some material form. (3) Translations, adaptations, arrangements of music and other alterations of a literary or artistic work shall be protected as original works without prejudice to the copyright in the original work. (4) It shall be a matter for legislation in the countries of the Union to determine the protection to be granted to official texts of a legislative, administrative and legal nature, and to official translations of such texts. (5) Collections of literary or artistic works such as encyclopaedias and anthologies which, by reason of the selection and arrangement of their contents, constitute intellectual creations shall be protected as such, without prejudice to the copyright in each of the works forming part of such collections. (6) The works mentioned in this Article shall enjoy protection in all countries of the Union. This protection shall operate for the benefit of the author and his successors in title. (7) Subject to the provisions of Article 7(4) of this Convention, it shall be a matter for legislation in the countries of the Union to determine the extent of the application of their laws to works of applied art and industrial designs and models, as well as the conditions under which such works, designs and models shall be protected. Works protected in the country of origin solely as designs and models shall be entitled in another country of the Union only to such special protection as is granted in that country to designs and models; however, if no such special protection is granted in that country, such works shall be protected as artistic works.
(8) The protection of this Convention shall not apply to news of the day or to miscellaneous facts having the character of mere items of press information.
SECTION 172 AND 172.2, SUPRA (ON ORIGINAL WORKS)
SECTION 173. DERIVATIVE W ORKS. – 173.1. The following derivative works shall also be protected by copyright: (a) Dramatizations, translations, adaptations, abridgments, arrangements, and other alterations of literary or artistic works; and (b) Collections of literary, scholarly or artistic works, and compilations of data and other materials which are original by reason of the selection or coordination or arrangement of their contents. 173.2. The works referred to in paragraphs (a) and (b) of Subsection 173.1 shall be protected as new works: Provided however, That such new work shall not affect the force of any subsisting copyright upon the original works employed or any part thereof, or be construed to imply any right to such use of the original works, or to secure or extend copyright in such original works.
SECTION 174. PUBLISHED EDITION OF WORK. – In addition to the right to publish granted by the author, his heirs, or assigns, the publisher shall have a copyright consisting merely of the right of reproduction of the typographical arrangement of the published edition of the work.
SECTION 175. UNPROTECTED SUBJECT MATTER. – Notwithstanding the provisions of Sections 172 and 173, no protection shall extend, under this law, to any idea, procedure, system, method or operation, concept, principle, discovery or mere data as such, even if they are expressed, explained, illustrated or embodied in a work; news of the day and other miscellaneous facts having the character of mere items of press information; or any official text of a legislative, administrative or legal nature, as well as any official translation thereof.
SECTION 176. WORKS OF THE GOVERNMENT. – 176.1. No copyright shall subsist in any work of the Government of the Philippines. However, prior approval of the government agency or office wherein the work is created shall be necessary for exploitation of such work for profit. Such agency or office may, among other things, impose as a condition the payment of royalties. No prior approval or conditions shall be required for the use of any purpose of statutes, rules and regulations, and speeches, lectures, sermons, addresses, and
12 z
Intellectual Property Law dissertations, pronounced, read or rendered in courts of justice, before administrative agencies, in deliberative assemblies and in meetings of public character. 176.2. The author of speeches, lectures, sermons, addresses, and dissertations mentioned in the preceding paragraphs shall have the exclusive right of making a collection of his works. 176.3. Notwithstanding the foregoing provisions, the Government is not precluded from receiving and holding copyrights transferred to it by assignment, bequest or otherwise; nor shall publication or republication by the Government in a public document of any work in which copyright is subsisting be taken to cause any abridgment or annulment of the copyright or to authorize any use or appropriation of such work without the consent of the copyright owner.
Copyright, in the strict sense of the term, is purely a statutory right. It is a new or independent right granted by the statute, and not simply a pre-‐existing right regulated by the statute. Being a statutory grant, the rights are only such as the statute confers, and may be obtained and enjoyed only with respect to the subjects and by the persons, and on terms and conditions specified in the statute. Since copyright in published works is purely a statutory creation, a copyright may be obtained only for a work falling within the statutory enumeration or description. Regardless of the historical viewpoint, it is authoritatively settled in the United States that there is no copyright except that which is both created and secured by act of Congress. P.D. No. 49, §2, in enumerating what are subject to copyright, refers to finished works and not to concepts. The copyright does not extend to an idea, procedure, process, system, method of operation, concept, principle, or discovery, regardless of the form in which it is described, explained, illustrated, or embodied in such work. In fact, Section 175 of the Intellectual Property Code expressly excluded these as protected subject matter. What then is the subject matter of petitioners’ copyright? This Court is of the opinion that petitioner BJPI’s copyright covers audio-‐visual recordings of each episode of Rhoda and Me, as falling within the class of works mentioned in P.D. 49, §2(M), to wit: Cinematographic works and works produced by a process analogous to cinematography or any process for making audio-‐visual recordings; The copyright does not extend to the general concept or format of its dating game show. Accordingly, by the very nature of the subject of petitioner BJPI’s copyright, the investigating prosecutor should have the opportunity to compare the videotapes of the two shows. Mere description by words of the general format of the two dating game shows is insufficient; the presentation of the master videotape in evidence was indispensable to the determination of the existence of probable cause. As aptly observed by respondent Secretary of Justice: A television show includes more than mere words can describe because it involves a whole spectrum of visuals and effects, video and audio, such that no similarity or dissimilarity may be found by merely describing the general copyright/format of both dating game shows.
JOAQUIN, JR. V. DRILON (1999) Refresher: Petitioner BJ Productions, Inc. (BJPI) is the holder/grantee of Certificate of Copyright No. M922, dated January 28, 1971, of Rhoda and Me, a dating game show aired from 1970 to 1977. On June 28, 1973, petitioner BJPI submitted to the National Library an addendum to its certificate of copyright specifying the show’s format and style of presentation. On July 14, 1991, while watching television, petitioner Francisco Joaquin, Jr., president of BJPI, saw on RPN Channel 9 an episode of It’s a Date, which was produced by IXL Productions, Inc. (IXL). On July 18, 1991, he wrote a letter to private respondent Gabriel M. Zosa, president and general manager of IXL, informing Zosa that BJPI had a copyright to Rhoda and Me and demanding that IXL discontinue airing It’s a Date, because the latter’s game format was substantially the same as the former. The substance of the television productions of the complainant’s “RHODA AND ME” and Zosa’s “IT’S A DATE” is that two matches are made between a male and a female, both single, and the two couples are treated to a night or two of dining and/or dancing at the expense of the show. The major concepts of both shows is the same. Any difference appear mere variations of the major concepts. Doctrine: To begin with, the format of a show is not copyrightable. The format or mechanics of a television show is not included in the list of protected works in §2 of P.D. No. 49, nor in §172 of the Intellectual Property Code. For this reason, the protection afforded by the law cannot be extended to cover them.
o 13
Katrina Michelle Mancao license its white pages listings to Feist for a directory covering 11 different telephone service areas, Feist extracted the listings it needed from Rural's directory without Rural's consent. Although Feist altered many of Rural's listings, several were identical to listings in Rural's white pages. The District Court granted summary judgment to Rural in its copyright infringement suit, holding that telephone directories are copyrightable. The Court of Appeals affirmed. Doctrine: Rural's white pages are not entitled to copyright, and therefore Feist's use of them does not constitute infringement. Article I, § 8, cl. 8, of the Constitution mandates originality as a prerequisite for copyright protection. The constitutional requirement necessitates independent creation plus a modicum of creativity. Since facts do not owe their origin to an act of authorship, they are not original, and thus are not copyrightable. Although a compilation of facts may possess the requisite originality because the author typically chooses which facts to include, in what order to place them, and how to arrange the data so that readers may use them effectively, copyright protection extends only to those components of the work that are original to the author, not to the facts themselves. This fact/expression dichotomy severely limits the scope of protection in fact-‐based works. The Copyright Act of 1976 and its predecessor, the Copyright Act of 1909, leave no doubt that originality is the touchstone of copyright protection in directories and other fact-‐based works. The 1976 Act explains that copyright extends to "original works of authorship," 17 U.S.C. § 102(a), and that there can be no copyright in facts, § 102(b). A compilation is not copyrightable per se, but is copyrightable only if its facts have been "selected, coordinated, or arranged in such a way that the resulting work as a whole constitutes an original work of authorship. Thus, the statute envisions that some ways of selecting, coordinating, and arranging data are not sufficiently original to trigger copyright protection. Even a compilation that is copyrightable receives only limited protection, for the copyright does not extend to facts contained in the compilation. Lower courts that adopted a "sweat of the brow" or "industrious collection" test -‐-‐ which extended a compilation's copyright protection beyond selection and arrangement to the facts themselves – misconstrued the 1909 Act and eschewed the fundamental axiom of copyright law that no one may copyright facts or ideas. Rural's white pages do not meet the constitutional or statutory requirements for copyright protection. While
UNITED FEATURE SYNDICATE, INC. V. MUNSINGWEAR CREATION MANUFACTURING COMPANY (1989) Refresher: This case arose from petition filed by petitioner for the cancellation of the registration of trademark CHARLIE BROWN (Registration No. SR. 4224) in the name of respondent MUNSINGWEAR in Inter Partes Case No. 1350 entitled "United Feature Syndicate, Inc. v. Munsingwear Creation Mfg. Co.", with the Philippine Patent Office alleging that petitioner is damaged by the registration of the trademark CHARLIE BROWN of T-‐ Shirts under Class 25 with the Registration No. SR-‐4224 dated September 12, 1979 in the name of Munsingwear Creation Manufacturing Co., Inc., on the following grounds: (1) that respondent was not entitled to the registration of the mark CHARLIE BROWN, & DEVICE at the time of application for registration; (2) that CHARLIE BROWN is a character creation or a pictorial illustration, the copyright to which is exclusively owned worldwide by the petitioner; (3) that as the owner of the pictorial illustration CHARLIE BROWN, petitioner has since 1950 and continuously up to the present, used and reproduced the same to the exclusion of others; (4) that the respondent-‐registrant has no bona fide use of the trademark in commerce in the Philippines prior to its application for registration. Doctrine: Since the name "CHARLIE BROWN" and its pictorial representation were covered by a copyright registration way back in 1950, the same are entitled to protection under PD No. 49. Aside from its copyright registration, petitioner is also the owner of several trademark registrations and application for the name and likeness of "CHARLIE BROWN" which is the duly registered trademark and copyright of petitioner United Feature Syndicate Inc. as early as 1957 and additionally also as TV SPECIALS featuring the “PEANUTS” characters “CHARLIE BROWN.”
FEIST PUBLICATIONS, INC. V. RURAL TELEPHONE SERVICE CO. (1991) Refresher: Respondent Rural Telephone Service Company is a certified public utility providing telephone service to several communities in Kansas. Pursuant to state regulation, Rural publishes a typical telephone directory, consisting of white pages and yellow pages. It obtains data for the directory from subscribers, who must provide their names and addresses to obtain telephone service. Petitioner Feist Publications, Inc., is a publishing company that specializes in area-‐wide telephone directories covering a much larger geographic range than directories such as Rural's. When Rural refused to
14 z
Intellectual Property Law Rural has a valid copyright in the directory as a whole because it contains some forward text and some original material in the yellow pages, there is nothing original in Rural's white pages. The raw data are uncopyrightable facts, and the way in which Rural selected, coordinated, and arranged those facts is not original in any way. Rural's selection of listings -‐-‐ subscribers' names, towns, and telephone numbers -‐-‐ could not be more obvious, and lacks the modicum of creativity necessary to transform mere selection into copyrightable expression. In fact, it is plausible to conclude that Rural did not truly "select" to publish its subscribers' names and telephone numbers, since it was required to do so by state law. Moreover, there is nothing remotely creative about arranging names alphabetically in a white pages directory. It is an age-‐old practice, firmly rooted in tradition and so commonplace that it has come to be expected as a matter of course. Remarks: US case.
5.
the person who so commissioned the work shall have ownership of the work, but the copyright thereto shall remain with the creator, unless there is a written stipulation to the contrary; 178.5. In the case of audiovisual work, the copyright shall belong to the producer, the author of the scenario, the composer of the music, the film director, and the author of the work so adapted. However, subject to contrary or other stipulations among the creators, the producer shall exercise the copyright to an extent required for the exhibition of the work in any manner, except for the right to collect performing license fees for the performance of musical compositions, with or without words, which are incorporated into the work; and 178.6. In respect of letters, the copyright shall belong to the writer subject to the provisions of Article 723 of the Civil Code.
SECTION 179. ANONYMOUS AND PSEUDONYMOUS WORKS. –
Who owns Copyright?
For purposes of this Act, the publishers shall be deemed to represent the authors of articles and other writings published without the names of the authors or under pseudonyms, unless the contrary appears, or the pseudonyms or adopted name leaves no doubt as to the author's identity, or if the author of the anonymous works discloses his identity.
SECTION 178. RULES ON COPYRIGHT OWNERSHIP. – Copyright ownership shall be governed by the following rules: 178.1. Subject to the provisions of this section, in the case of original literary and artistic works, copyright shall belong to the author of the work; 178.2. In the case of works of joint authorship, the co-‐ authors shall be the original owners of the copyright and in the absence of agreement, their rights shall be governed by the rules on co-‐ownership. If, however, a work of joint authorship consists of parts that can be used separately and the author of each part can be identified, the author of each part shall be the original owner of the copyright in the part that he has created; 178.3. In the case of work created by an author during and in the course of his employment, the copyright shall belong to: (a) The employee, if the creation of the object of copyright is not a part of his regular duties even if the employee uses the time, facilities and materials of the employer. (b) The employer, if the work is the result of the performance of his regularly-‐assigned duties, unless there is an agreement, express or implied, to the contrary. 178.4. In the case of a work commissioned by a person other than an employer of the author and who pays for it and the work is made in pursuance of the commission,
SECTION 174. PUBLISHED EDITION OF WORK. – In addition to the right to publish granted by the author, his heirs, or assigns, the publisher shall have a copyright consisting merely of the right of reproduction of the typographical arrangement of the published edition of the work.
ARTICLE 722, CIVIL C ODE. The author and the composer, mentioned in Nos. 1 and 2 of the preceding article, shall have the ownership of their creations even before the publication of the same. Once their works are published, their rights are governed by the Copyright laws. The painter, sculptor or other artist shall have dominion over the product of his art even before it is copyrighted. The scientist or technologist has the ownership of his discovery or invention even before it is patented.
ARTICLE 723, CIVIL CODE. Letters and other private communications in writing are owned by the person to whom they are addressed and delivered, but they cannot be published or disseminated without the consent of the writer or his heirs. However, the court may authorize their publication or
o 15
Katrina Michelle Mancao dissemination if the public good or the interest of justice so requires.
(5) The term of protection subsequent to the death of the author and the terms provided by paragraphs (2), (3) and (4) shall run from the date of death or of the event referred to in those paragraphs, but such terms shall always be deemed to begin on the first of January of the year following the death or such event. (6) The countries of the Union may grant a term of protection in excess of those provided by the preceding paragraphs. (7) Those countries of the Union bound by the Rome Act of this Convention which grant, in their national legislation in force at the time of signature of the present Act, shorter terms of protection than those provided for in the preceding paragraphs shall have the right to maintain such terms when ratifying or acceding to the present Act. (8) In any case, the term shall be governed by the legislation of the country where protection is claimed; however, unless the legislation of that country otherwise provides, the term shall not exceed the term fixed in the country of origin of the work.
ARTICLE 520, CIVIL CODE. A trade-‐mark or trade-‐name duly registered in the proper government bureau or office is owned by and pertains to the person, corporation, or firm registering the same, subject to the provisions of special laws.
6.
Duration of Copyright
ARTICLE 7, BERNE CONVENTION FOR THE PROTECTION OF LITERARY AND ARTISTIC WORKS TERM OF PROTECTION: 1. Generally; 2. For cinematographic works; 3. For anonymous and pseudonymous works; 4. For photographic works and works of applied art; 5. Starting date of computation; 6. Longer terms; 7. Shorter terms; 8. Applicable law; “comparison” of terms (1) The term of protection granted by this Convention shall be the life of the author and fifty years after his death. (2) However, in the case of cinematographic works, the countries of the Union may provide that the term of protection shall expire fifty years after the work has been made available to the public with the consent of the author, or, failing such an event within fifty years from the making of such a work, fifty years after the making. (3) In the case of anonymous or pseudonymous works, the term of protection granted by this Convention shall expire fifty years after the work has been lawfully made available to the public. However, when the pseudonym adopted by the author leaves no doubt as to his identity, the term of protection shall be that provided in paragraph (1). If the author of an anonymous or pseudonymous work discloses his identity during the above-‐mentioned period, the term of protection applicable shall be that provided in paragraph (1). The countries of the Union shall not be required to protect anonymous or pseudonymous works in respect of which it is reasonable to presume that their author has been dead for fifty years. (4) It shall be a matter for legislation in the countries of the Union to determine the term of protection of photographic works and that of works of applied art in so far as they are protected as artistic works; however, this term shall last at least until the end of a period of twenty-‐five years from the making of such a work.
ARTICLE 7BIS, BERNE CONVENTION FOR THE PROTECTION OF LITERARY AND ARTISTIC W ORKS TERM OF PROTECTION FOR WORKS OF JOINT AUTHORSHIP The provisions of the preceding Article shall also apply in the case of a work of joint authorship, provided that the terms measured from the death of the author shall be calculated from the death of the last surviving author.
SECTION 213. TERM OF PROTECTION. – 213.1. Subject to the provisions of Subsections 213.2 to 213.5, the copyright in works under Sections 172 and 173 shall be protected during the life of the author and for fifty (50) years after his death. This rule also applies to posthumous works. 213.2. In case of works of joint authorship, the economic rights shall be protected during the life of the last surviving author and for fifty (50) years after his death. 213.3. In case of anonymous or pseudonymous works, the copyright shall be protected for fifty (50) years from the date on which the work was first lawfully published: Provided, That where, before the expiration of the said period, the author's identity is revealed or is no longer in doubt, the provisions of Subsections 213.1. and 213.2 shall apply, as the case may be: Provided, further, That such works if not published before shall be protected for fifty (50) years counted from the making of the work.
16 z
Intellectual Property Law Government or any of its subdivisions and instrumentalities, including government-‐owned or controlled corporations as a part of his regularly prescribed official duties.
213.4. In case of works of applied art the protection shall be for a period of twenty-‐five (25) years from the date of making. 213.5. In case of photographic works, the protection shall be for fifty (50) years from publication of the work and, if unpublished, fifty (50) years from the making. 213.6. In case of audio-‐visual works including those produced by process analogous to photography or any process for making audio-‐visual recordings, the term shall be fifty (50) years from date of publication and, if unpublished, from the date of making.
SECTION 184. LIMITATIONS ON COPYRIGHT. – 184.1. Notwithstanding the provisions of Chapter V, the following acts shall not constitute infringement of copyright: (a) The recitation or performance of a work, once it has been lawfully made accessible to the public, if done privately and free of charge or if made strictly for a charitable or religious institution or society; (b) The making of quotations from a published work if they are compatible with fair use and only to the extent justified for the purpose, including quotations from newspaper articles and periodicals in the form of press summaries: Provided, That the source and the name of the author, if appearing on the work, are mentioned; (c) The reproduction or communication to the public by mass media of articles on current political, social, economic, scientific or religious topic, lectures, addresses and other works of the same nature, which are delivered in public if such use is for information purposes and has not been expressly reserved: Provided, That the source is clearly indicated; (d) The reproduction and communication to the public of literary, scientific or artistic works as part of reports of current events by means of photography, cinematography or broadcasting to the extent necessary for the purpose; (e) The inclusion of a work in a publication, broadcast, or other communication to the public, sound recording or film, if such inclusion is made by way of illustration for teaching purposes and is compatible with fair use: Provided, That the source and of the name of the author, if appearing in the work, are mentioned; (f) The recording made in schools, universities, or educational institutions of a work included in a broadcast for the use of such schools, universities or educational institutions: Provided, That such recording must be deleted within a reasonable period after they were first broadcast: Provided, further, That such recording may not be made from audiovisual
SECTION 214. CALCULATION OF TERM. – The term of protection subsequent to the death of the author provided in the preceding Section shall run from the date of his death or of publication, but such terms shall always be deemed to begin on the first day of January of the year following the event which gave rise to them.
7.
Limitations on Copyright
SECTION 176. WORKS OF THE GOVERNMENT. – 176.1. No copyright shall subsist in any work of the Government of the Philippines. However, prior approval of the government agency or office wherein the work is created shall be necessary for exploitation of such work for profit. Such agency or office may, among other things, impose as a condition the payment of royalties. No prior approval or conditions shall be required for the use of any purpose of statutes, rules and regulations, and speeches, lectures, sermons, addresses, and dissertations, pronounced, read or rendered in courts of justice, before administrative agencies, in deliberative assemblies and in meetings of public character. 176.2. The author of speeches, lectures, sermons, addresses, and dissertations mentioned in the preceding paragraphs shall have the exclusive right of making a collection of his works. 176.3. Notwithstanding the foregoing provisions, the Government is not precluded from receiving and holding copyrights transferred to it by assignment, bequest or otherwise; nor shall publication or republication by the Government in a public document of any work in which copyright is subsisting be taken to cause any abridgment or annulment of the copyright or to authorize any use or appropriation of such work without the consent of the copyright owner. SECTION 171.11. A "WORK OF THE GOVERNMENT OF THE PHILIPPINES" is a work created by an officer or employee of the Philippine
o 17
Katrina Michelle Mancao works which are part of the general cinema repertoire of feature films except for brief excerpts of the work; (g) The making of ephemeral recordings by a broadcasting organization by means of its own facilities and for use in its own broadcast; (h) The use made of a work by or under the direction or control of the Government, by the National Library or by educational, scientific or professional institutions where such use is in the public interest and is compatible with fair use; (i) The public performance or the communication to the public of a work, in a place where no admission fee is charged in respect of such public performance or communication, by a club or institution for charitable or educational purpose only, whose aim is not profit making, subject to such other limitations as may be provided in the Regulations; (j) Public display of the original or a copy of the work not made by means of a film, slide, television image or otherwise on screen or by means of any other device or process: Provided, That either the work has been published, or, that the original or the copy displayed has been sold, given away or otherwise transferred to another person by the author or his successor in title; and (k) Any use made of a work for the purpose of any judicial proceedings or for the giving of professional advice by a legal practitioner.
whether the use made of a work in any particular case is fair use, the factors to be considered shall include: (a) The purpose and character of the use, including whether such use is of a commercial nature or is for non-‐profit educational purposes; (b) The nature of the copyrighted work; (c) The amount and substantiality of the portion used in relation to the copyrighted work as a whole; and (d) The effect of the use upon the potential market for or value of the copyrighted work. 185.2. The fact that a work is unpublished shall not by itself bar a finding of fair use if such finding is made upon consideration of all the above factors.
SECTION 186. WORK OF ARCHITECTURE. – Copyright in a work of architecture shall include the right to control the erection of any building which reproduces the whole or a substantial part of the work either in its original form or in any form recognizably derived from the original: Provided, That the copyright in any such work shall not include the right to control the reconstruction or rehabilitation in the same style as the original of a building to which that copyright relates.
SECTION 187. REPRODUCTION OF PUBLISHED WORK. – 187.1. Notwithstanding the provision of Section 177, and subject to the provisions of Subsection 187.2, the private reproduction of a published work in a single copy, where the reproduction is made by a natural person exclusively for research and private study, shall be permitted, without the authorization of the owner of copyright in the work. 187.2. The permission granted under Subsection 187.1 shall not extend to the reproduction of: (a) A work of architecture in the form of building or other construction; (b) An entire book, or a substantial part thereof, or of a musical work in graphic form by reprographic means; (c) A compilation of data and other materials; (d) A computer program except as provided in Section 189; and (e) Any work in cases where reproduction would unreasonably conflict with a normal
184.2. The provisions of this section shall be interpreted in such a way as to allow the work to be used in a manner which does not conflict with the normal exploitation of the work and does not unreasonably prejudice the right holder's legitimate interests.4
SECTION 185. FAIR USE OF A COPYRIGHTED WORK. – 185.1. The fair use of a copyrighted work for criticism, comment, news reporting, teaching including multiple copies for classroom use, scholarship, research, and similar purposes is not an infringement of copyright. Decompilation, which is understood here to be the reproduction of the code and translation of the forms of the computer program to achieve the inter-‐operability of an independently created computer program with other programs may also constitute fair use. In determining 4
IMPORTANT NOTE: 184.1 must be read with 184.2.
18 z
Intellectual Property Law
exploitation of the work or would otherwise unreasonably prejudice the legitimate interests of the author.
and to the extent, for which the computer program has been obtained; and (b) Archival purposes, and, for the replacement of the lawfully owned copy of the computer program in the event that the lawfully obtained copy of the computer program is lost, destroyed or rendered unusable.
SECTION 188. REPROGRAPHIC REPRODUCTION BY LIBRARIES. – 188.1. Notwithstanding the provisions of Subsection 177.6, any library or archive whose activities are not for profit may, without the authorization of the author of copyright owner, make a single copy of the work by reprographic reproduction: (a) Where the work by reason of its fragile character or rarity cannot be lent to user in its original form; (b) Where the works are isolated articles contained in composite works or brief portions of other published works and the reproduction is necessary to supply them, when this is considered expedient, to persons requesting their loan for purposes of research or study instead of lending the volumes or booklets which contain them; and (c) Where the making of such a copy is in order to preserve and, if necessary in the event that it is lost, destroyed or rendered unusable, replace a copy, or to replace, in the permanent collection of another similar library or archive, a copy which has been lost, destroyed or rendered unusable and copies are not available with the publisher. 188.2. Notwithstanding the above provisions, it shall not be permissible to produce a volume of a work published in several volumes or to produce missing tomes or pages of magazines or similar works, unless the volume, tome or part is out of stock: Provided, That every library which, by law, is entitled to receive copies of a printed work, shall be entitled, when special reasons so require, to reproduce a copy of a published work which is considered necessary for the collection of the library but which is out of stock.
189.2. No copy or adaptation mentioned in this Section shall be used for any purpose other than the ones determined in this Section, and any such copy or adaptation shall be destroyed in the event that continued possession of the copy of the computer program ceases to be lawful. 189.3. This provision shall be without prejudice to the application of Section 185 whenever appropriate.
SECTION 190. IMPORTATION FOR PERSONAL PURPOSES. – 190.1. Notwithstanding the provision of Subsection 177.6, but subject to the limitation under the Subsection 185.2, the importation of a copy of a work by an individual for his personal purposes shall be permitted without the authorization of the author of, or other owner of copyright in, the work under the following circumstances: (a) When copies of the work are not available in the Philippines and: (i) Not more than one (1) copy at one time is imported for strictly individual use only; or (ii) The importation is by authority of and for the use of the Philippine Government; or (iii) The importation, consisting of not more than three (3) such copies or likenesses in any one invoice, is not for sale but for the use only of any religious, charitable, or educational society or institution duly incorporated or registered, or is for the encouragement of the fine arts, or for any state school, college, university, or free public library in the Philippines. (b) When such copies form parts of libraries and personal baggage belonging to persons or families arriving from foreign countries and are not intended for sale: Provided, That such copies do not exceed three (3).
SECTION 189. REPRODUCTION OF COMPUTER PROGRAM. – 189.1. Notwithstanding the provisions of Section 177, the reproduction in one (1) back-‐up copy or adaptation of a computer program shall be permitted, without the authorization of the author of, or other owner of copyright in, a computer program, by the lawful owner of that computer program: Provided, That the copy or adaptation is necessary for: (a) The use of the computer program in conjunction with a computer for the purpose,
o 19
Katrina Michelle Mancao 190.2. Copies imported as allowed by this Section may not lawfully be used in any way to violate the rights of owner the copyright or annul or limit the protection secured by this Act, and such unlawful use shall be deemed an infringement and shall be punishable as such without prejudice to the proprietor's right of action. 190.3. Subject to the approval of the Secretary of Finance, the Commissioner of Customs is hereby empowered to make rules and regulations for preventing the importation of articles the importation of which is prohibited under this Section and under treaties and conventions to which the Philippines may be a party and for seizing and condemning and disposing of the same in case they are discovered after they have been imported.
SECTION 2. USE IN JUDICIAL PROCEEDINGS OR BY LEGAL PRACTITIONERS. — Without prejudice to other exceptions to protection, the use of copyrighted works as provided in the IPC “for the giving of professional advise by a legal practitioner” is limited to those uses directly connected to the rendering of legal advice with respect to such works, or to individuals, institutions, events or circumstances to which the creation of exploitation of such works are directly related. This exception does not apply to the exercise of exclusive rights of copyright owners in legal research materials, legal-‐related computer software, legal-‐related online material, or the other works utilized in the practice of law on a regular basis. SECTION 3. USES COMPATIBLE WITH FAIR USE. — To determine whether use of a copyrighted work is “compatible with fair use,” the criteria set forth in Sec. 185 of the IPC shall be applied. SECTION 4. FAIR USE AND DECOMPILATION OF COMPUTER SOFTWARE. — An act of decompilation of software may qualify for analysis under the fair use provisions if it meets all of the criteria set out in the IPC, viz: (a) It consist only of the reproduction of code and translation of the forms of a computer program; (b) The reproduction and translation are indispensable to obtain information, such that information can be obtained in no other way than through decompilation; (c) The information is necessary in order to achieve interoperability (i.e., interoperability cannot be achieved without obtaining the information and without using decompilation to obtain it) ; and (d) The interoperability that is sought is between the decompiled program and an independently created computer program (i.e., a computer program that was created prior to and without reference to the decompilation). If the above criteria are satisfied, then the fair use factors set forth in the IPC can be applied in order to decide whether or not the decompilation, in the particular circumstances involved, constitutes a fair use. The fact that a use is decompilation (as defined by the statutory criteria) creates no presumption that the use is fair.
RULE 14 – LIMITATIONS ON COPYRIGHT, COPYRIGHT SAFEGUARDS AND REGULATIONS SECTION 1. PUBLIC PERFORMANCE OR COMMUNICATION TO THE PUBLIC OF WORK BY NON-‐PROFIT INSTITUTION. — The public performance or the communication to the public of a work, in a place where no admission fee is charged in respect of such public performance or communication, by a club or institution for charitable or educational purposes only, whose aim is not profit-‐ making, shall not constitute infringement of copyright, if: (a) The works are limited to non-‐dramatic literary works and non-‐copyrighted musical compositions, and do not include audiovisual works or computer programs; (b) The clubs or institutions to which this subsection applies are limited to those organized exclusively for charitable or educational purposes; and if the following conditions are satisfied: (i) No fee or compensation is paid to any performers, promoters or organizers of the public performance; (ii) There is no direct or indirect admission charge to the place where the performance occurs; and (iii) Admission to the performance is restricted to persons who are, and for at least the preceding thirty days have been, members in good standing of the club or the institution for whose charitable or education purposes the performance is being carried out.
20 z
Intellectual Property Law publication of the article "we reserved all rights," which was legally equivalent to a notice "that their publication is reserved." To give that paragraph any other construction would eliminate, take from it, and wipe out, the words "that their publication is reserved," and this court has no legal right to do that. It was contended that this construction would nullify the use and value of the whole Copyright Law, but it will be noted that this exception is specifically confined and limited to "news items editorial paragraphs, and articles in periodicals," and hence could not be made to apply to any other provisions of the Copyright Law. It will also be noted that in the instant case, the defendant had the legal right to publish the article in question by giving "the source of the reproduction." The plaintiff bought and paid for the article and published it with the notice that "we reserve all rights," and the defendant published the article in question without citing "the source of the reproduction," and for aught that appeared in his paper, the article was purchase and paid for by the defendant. We are clearly of the opinion that the language in question in the Copyright Law of the Philippine Islands, which is not found in the Copyright Law of the United States, was inserted for a specific purpose, and it was intended to prohibit the doing of the very thing which the defendant did in this case; otherwise, the use of all of those words is a nullity. This construction does not least impair the Copyright Law, except as to "news items, editorial paragraphs, and articles in periodicals," and it protects an enterprising newspaper or magazine that invests its money and pays for the right to publish an original article, and that was the reason why the Legislature saw fit to use the language in question. Ma’am comment on the case: This is erroneous!!!
SECTION 5. EFFECT ON VALUE OF THE WORK. — The effect of the use upon the value of the copyrighted work shall also constitute a limitation on fair use of works, particularly whenever their reproduction will affect their usefulness, reliability, and validity such as psychological tests and others of similar nature. Reproduction of these works shall, therefore, need prior authority of the copyright owner. SECTION 6. REPRODUCTION OR COMMUNICATION TO THE PUBLIC BY MASS M EDIA OF ARTICLES, LECTURE, ETC. — The reproduction or communication to the public by mass media of articles, such as those publish in newspapers or periodicals on current political, social, economic, scientific, or religious topics, as well as lectures, addresses, and other works of the same nature, which are delivered in public, shall not constitute infringement of copyright if such use is for information purposes and has not been expressly reserved: Provided, that the source is clearly indicated and that the use of the work should not unreasonably prejudice the right holder’s legitimate interests.
FILIPINO SOCIETY OF COMPOSERS V. BENJAMIN TAN (1987), SUPRA
PHILIPPINE EDUCATION CO. V. SOTTO AND ALINDADA (1929) Refresher: The True Story of Mrs. Rizal, written by Prof. Craig, published by Petitioner in Philippine Education Magazine. It was alleged that Respondent reproduced the same in The Independent, without the Petitioner’s permission. Applicable law is Act No. 3134. Doctrine: Analyzing the language used, it says, first, that such news items, editorial paragraphs, and articles in periodicals may be reproduced, unless they contain a notice that their publication is reserved, or, second, that may also be reproduced, unless they contain a notice of copyright. But in either event, the law specifically provides that "the source of the reproduction or original reproduced shall be cited," and is not confined or limited to case in which there is "a notice of copyright," and specifically says that in either event "the source of the reproduction or original reproduced shall be cited." To give this section any other construction would be to nullify, eliminate and take from the paragraph the words "they contain a notice that their publication is reserved," and to say that the Legislature never intended to say what it did say. This court must construe the language found in the act. In the instant case, the plaintiff did not give notice of its copyright, for the simple reason that it did not have a copyright, but it did notify the defendant that in the
SONY CORPORATION OF AMERICA V. UNIVERSAL STUDIOS, INC. (1983) Refresher: Petitioner Sony Corp. manufactures home video tape recorders (VTR's), and markets them through retail establishments, some of which are also petitioners. Respondents own the copyrights on some of the television programs that are broadcast on the public airwaves. Respondents brought an action against petitioners in Federal District Court, alleging that VTR consumers had been recording some of respondents' copyrighted works that had been exhibited on commercially sponsored television and thereby infringed respondents' copyrights, and further that petitioners were liable for such copyright infringement because of their marketing of the VTR's. Respondents sought money damages, an equitable accounting of profits, and an injunction against the manufacture and marketing of the VTR's.
o 21
Katrina Michelle Mancao The District Court denied respondents all relief, holding that noncommercial home use recording of material broadcast over the public airwaves was a fair use of copyrighted works and did not constitute copyright infringement, and that petitioners could not be held liable as contributory infringers even if the home use of a VTR was considered an infringing use. The Court of Appeals reversed, holding petitioners liable for contributory infringement and ordering the District Court to fashion appropriate relief. Doctrine: The sale of the VTR's to the general public does not constitute contributory infringement of respondents' copyrights. The protection given to copyrights is wholly statutory, and, in a case like this, in which Congress has not plainly marked the course to be followed by the judiciary, this Court must be circumspect in construing the scope of rights created by a statute that never contemplated such a calculus of interests. Any individual may reproduce a copyrighted work for a "fair use"; the copyright owner does not possess the exclusive right to such a use. Petitioner is not a “contributory infringer.” Kalem Co. v. Harper Brothers does not support respondents' novel theory that supplying the "means" to accomplish an infringing activity and encouraging that activity through advertisement are sufficient to establish liability for copyright infringement. This case does not fall in the category of those in which it is manifestly just to impose vicarious liability because the "contributory" infringer was in a position to control the use of copyrighted works by others and had authorized the use without permission from the copyright owner. Here, the only contact between petitioners and the users of the VTR's occurred at the moment of sale. And there is no precedent for imposing vicarious liability on the theory that petitioners sold the VTR's with constructive knowledge that their customers might use the equipment to make unauthorized copies of copyrighted material. The sale of copying equipment, like the sale of other articles of commerce, does not constitute contributory infringement if the product is widely used for legitimate, unobjectionable purposes, or, indeed, is merely capable of substantial noninfringing uses. The record and the District Court's findings show: (1) that there is a significant likelihood that substantial numbers of copyright holders who license their works for broadcast on free television would not object to having their broadcast time-‐shifted by private viewers (i. e., recorded at a time when the VTR owner cannot
view the broadcast so that it can be watched at a later time); and (2) that there is no likelihood that time-‐shifting would cause nonminimal harm to the potential market for, or the value of, respondents' copyrighted works. The VTR's are therefore capable of substantial noninfringing uses. Private, noncommercial time-‐shifting in the home satisfies this standard of noninfringing uses both because respondents have no right to prevent other copyright holders from authorizing such time-‐shifting for their programs, and because the District Court's findings reveal that even the unauthorized home time-‐shifting of respondents' programs is legitimate fair use. Remarks: US Case.
HARPER & ROW, PUBLISHERS, INC. V. NATION ENTERPRISES (1985) Refresher: In 1977, former President Ford contracted with petitioners to publish his as yet unwritten memoirs. The agreement gave petitioners the exclusive first serial right to license prepublication excerpts. Two years later, as the memoirs were nearing completion, petitioners, as the copyright holders, negotiated a prepublication licensing agreement with Time Magazine under which Time agreed to pay $25,000 ($12,500 in advance and the balance at publication) in exchange for the right to excerpt 7,500 words from Mr. Ford's account of his pardon of former President Nixon. Shortly before the Time article's scheduled release, an unauthorized source provided The Nation Magazine with the unpublished Ford manuscript. Working directly from this manuscript, an editor of The Nation produced a 2,250-‐word article, at least 300 to 400 words of which consisted of verbatim quotes of copyrighted expression taken from the manuscript. It was timed to "scoop" the Time article. As a result of the publication of The Nation's article, Time canceled its article and refused to pay the remaining $12,500 to petitioners. Petitioners then brought suit in Federal District Court against respondent publishers of The Nation, alleging, inter alia, violations of the Copyright Act (Act). The District Court held that the Ford memoirs were protected by copyright at the time of The Nation publication and that respondents' use of the copyrighted material constituted an infringement under the Act, and the court awarded actual damages of $12,500. The Court of Appeals reversed, holding that The Nation's publication of the 300 to 400 words it identified as copyrightable expression was sanctioned as a "fair use" of the copyrighted material under 107 of the Act. Section 107 provides that notwithstanding the provisions of 106 giving a copyright owner the exclusive right to reproduce the copyrighted work and to prepare
22 z
Intellectual Property Law derivative works based on the copyrighted work, the fair use of a copyrighted work for purposes such as comment and news reporting is not an infringement of copyright. Section 107 further provides that in determining whether the use was fair the factors to be considered shall include: (1) the purpose and character of the use; (2) the nature of the copyrighted work; (3) the substantiality of the portion used in relation to the copyrighted work as a whole; and (4) the effect on the potential market for or value of the copyrighted work. Doctrine: The Nation's article was not a "fair use" sanctioned by 107. In using generous verbatim excerpts of Mr. Ford's unpublished expression to lend authenticity to its account of the forthcoming memoirs, The Nation effectively arrogated to itself the right of first publication, an important marketable subsidiary right. Though the right of first publication, like other rights enumerated in 106, is expressly made subject to the fair use provisions of 107, fair use analysis must always be tailored to the individual case. The nature of the interest at stake is highly relevant to whether a given use is fair. The unpublished nature of a work is a key, though not necessarily determinative, factor tending to negate a defense of fair use. And under ordinary circumstances, the author's right to control the first public appearance of his undisseminated expression will outweigh a claim of fair use. In view of the First Amendment's protections embodied in the Act's distinction between copyrightable expression and uncopyrightable facts and ideas, and the latitude for scholarship and comment traditionally afforded by fair use, there is no warrant for expanding, as respondents contend should be done, the fair use doctrine to what amounts to a public figure exception to copyright. Whether verbatim copying from a public figure's manuscript in a given case is or is not fair must be judged according to the traditional equities of fair use. Taking into account the FOUR FACTORS enumerated in 107 as especially relevant in determining fair use, leads to the conclusion that the use in question here was not fair. (i) The fact that news reporting was the general purpose of The Nation's use is simply one factor. While The Nation had every right to be the first to publish the information, it went beyond simply reporting uncopyrightable information and actively sought to exploit the headline value of its infringement, making a "news event" out of its unauthorized first publication. The fact that the publication was commercial as opposed to nonprofit is a
separate factor tending to weigh against a finding of fair use. Fair use presupposes good faith. The Nation's unauthorized use of the undisseminated manuscript had not merely the incidental effect but the intended purpose of supplanting the copyright holders' commercially valuable right of first publication. (ii) While there may be a greater need to disseminate works of fact than works of fiction, The Nation's taking of copyrighted expression exceeded that necessary to disseminate the facts and infringed the copyright holders' interests in confidentiality and creative control over the first public appearance of the work. (iii) Although the verbatim quotes in question were an insubstantial portion of the Ford manuscript, they qualitatively embodied Mr. Ford's distinctive expression and played a key role in the infringing article. (iv) As to the effect of The Nation's article on the market for the copyrighted work, Time's cancellation of its projected article and its refusal to pay $12,500 were the direct effect of the infringing publication. Once a copyright holder establishes a causal connection between the infringement and loss of revenue, the burden shifts to the infringer to show that the damage would have occurred had there been no taking of copyrighted expression. Petitioners established a prima facie case of actual damage that respondents failed to rebut. More important, to negate a claim of fair use it need only be shown that if the challenged use should become widespread, it would adversely affect the potential market for the copyrighted work. Here, The Nation's liberal use of verbatim excerpts posed substantial potential for damage to the marketability of first serialization rights in the copyrighted work. Remarks: US Case
ANN BARTOW, EDUCATIONAL FAIR USE IN COPYRIGHT: RECLAIMING THE RIGHT TO PHOTOCOPY FREELY (1998) The doctrine of fair use was codified in § 107 of the Copyright Act of 1976, which states in pertinent part: The fair use of a copyrighted work, including such use by reproduction in copies or phonorecords or by any other means specified [in §§ 106 and 106A], for purposes such as criticism, comment, news reporting, teaching (including multiple copies for classroom use), scholarship, or research, is not an infringement of copyright. In determining whether the use made of a
o 23
Katrina Michelle Mancao work in any particular case is a fair use the factors to be considered shall include-‐ (1) the purpose and character of the use, including whether such use is of a commercial nature or is for nonprofit educational purposes; (2) the nature of the copyrighted work; (3) the amount and substantiality of the portion used in relation to the copyrighted work as a whole; and (4) the effect of the use upon the potential market for or value of the copyrighted work. The omissions from § 107 are immediately apparent. Fair use is not defined, and no guidance is offered with respect to an ordering of priorities in the application of the four articulated factors to be considered. Fair use has been called an equitable rule of reason with no real definition. Fair use is, however, widely recognized as a doctrine under which a copyrighted work can be used without permission from, or even against the express wishes of the copyright owner, in certain circumstances. Fair use doctrine embodies a policy against freighting ideas and information with proprietary rights. Although one of the bedrock principles of copyright law is that only the expression of ideas is copyrightable, and not the ideas that are expressed, as a practical matter it is often very difficult to separate the two. If, for example, a unique idea is expressed in a highly original, copyrighted book, unless one is prepared to "re-‐express" the idea, thereby rewriting the tome in one's own words (and preferably in a manner that does not induce the book's author to sue for copyright infringement, while still retaining the "essence" of whatever makes the work unique and valuable), monopolistic control over the idea contained in the work is essentially ceded to the work's copyright owner. Though ideas are part of the public domain, one requires words ("expression") to convey them. When an idea is complex, it will be onerous, if not impossible, for another person to communicate using other words without slanting or shading underlying concepts. Preservation and accurate transmission of an elaborate original idea compel use of intact, original expression. The Copyright Act does not bestow absolute exclusive rights upon a copyright owner, but rather confers a limited monopoly with respect to use of the copyrighted work. A copyright is intangible (or "intellectual") property that is vested with a public interest, intended to achieve an "important public purpose." There is a societal bargain implicit in the copyright law. Copyright owners are given tools in the form of exclusive rights with which to exploit creative endeavors financially, but this gift is conditioned upon an understanding that the ultimate goal of copyrights is to maximize the number of creative works available to the public, and not to benefit
individual copyright owners. Alternatively expressed, "copyright is a bargain between the public and publishers, in which the public consents to restrict its rights as a kind of bribe to publishers." To effectuate this bargain, copyrights should be no more restrictive than is necessary to create incentives for the promotion of knowledge and learning. Where public interests conflict with those of copyright owners, the public interests should prevail. Fair use therefore represents an amplitude of control that is withheld from a copyright owner when a grant of copyright is made, rather than a right or privilege copyright owners earn or acquire that is subsequently "taken." Fair use is the "only limitation on copyright that applies to all rights of the copyright owner and to all copyrighted works." The public's statutorily circumscribed right to the fair use of copyrighted works can be justified in several ways. Fair use access to copyrighted works may be grounded in recognition that no newly created, copyrighted work can be truly original. All authors are consciously or unconsciously, directly or indirectly exposed to, informed and inspired by the earlier works and thoughts of others, and are therefore obligated to contribute "free samples" from their own works to the marketplace of ideas. In this construction, fair use functions almost as a tax or fee that is levied upon individual copyright works for the common welfare, and justified by the copyright owner's consumption of common or public knowledge. Fair use may also be perceived as a tax perhaps paid as partial reparation for the costs to society of the "copyright infrastructure" comprised of the Copyright Office, the federal court system (which has subject matter jurisdiction over copyrights), law enforcement agencies, and other governmental entities dedicated to delineating, allocating, and protecting copyrights. Fair use thus functions as a burden on intellectual property that reciprocally accompanies the benefits provided by the copyright infrastructure. Fair use can also be viewed as somewhat similar to an easement. The recent publisher-‐propelled rush to expand the scope of copyright protections has been aptly analogized to "a land grab the likes of which we have not seen since the Government opened up the Northwest Territory for settlement." Fair use may be seen as society's easement across "servient" creative works. Under this construction authors create and publish new works subject to the burden of fair use, and should consider the possible impact of the doctrine just as a landowner would factor in the ramifications of an easement when deciding whether and how to develop land. Unlike the typical effect of an easement on land, however, an easement doesn't necessarily burden or devalue intellectual property. A book can be equally
24 z
Intellectual Property Law valuable to each in a series of new readers no matter how many times it is read, with no diminution in the quality of the work. As Thomas Jefferson stated: "He who receives an idea from me, receives instructions himself without lessening mine; as he who lights his taper at mine, receives light without darkening me." Fair users don't erode the desirability of a work, and may actually augment royalty generating demand for it by increasing its visibility or elevating its prominence. Fair use has also been justified as a response to market failure, based on the premise that the public ought to have free access to information and ideas manifested in copyrighted works which could not be purchased with reasonable ease or efficiency. Under this notion of fair use, the availability of a work or portion thereof has an inverse relationship with the prospective scope of its fair usability. The scope of fair use would therefore be broader with respect to a book that was out of print, and narrower for a book that was inexpensive and widely distributed. Where there was no market failure, there could be no fair use. The idea that there was a fair use right to use a copyrighted work without the copyright owner's permission was first raised in 1841 in Folsom v. Marsh. The plaintiffs in this case owned a copyright in a multivolume autobiography of George Washington, which defendants were accused of infringing when they copied 353 pages of the treatise. Though this sounds like an appallingly extensive appropriation, it actually amounted to less than 6% of the original lengthy work. Defendant's fair use claim failed because the court concluded that the most important parts of the work had been copied in an attempt to supplant uses (and presumably purchases) of the original work. In Folsom, Justice Story articulated the underpinnings of the modern test of fair use, subsequently incorporated into § 107 of the Copyright Act, which comprised a review of "the nature and objects of the selections made, the quantity and value of the materials used, and the degree in which the use may prejudice the sale, or diminish the profits, or supersede the objects, of the original work." Justice Story was most concerned about the market effect of the accused infringing usage, presaging the extensive (though misdirected) focus by modern courts on the profitability implications of nonpermissive uses asserted to be fair. It has been argued that rather than being "a typical statutory provision representing Congress's creation of new law and establishment of rules for the courts to apply [§ 107] is instead a direction to the courts to continue to develop the common law." Under this view, § 107 is not a codification of the doctrine of fair use, but rather "statutory recognition of a judge-‐made rule of reason." One commentator has referred to fair use as a "paradigmatic "catch-‐all'," and asserted that "the
doctrine has no crisp outlines, no precise standards, and no obvious center or core. Its flexibility allows judges to adjust the contours of copyright protection to fit individual circumstances as they arise, in the true common law tradition." In any event, though § 107 has not been legislatively altered with respect to the fair use of published works since it went into effect in 1978, the scope of fair use that it delineates has been steadily shrinking. SECTION 107'S FOUR-‐PART TEST Section 107 sets out a non-‐exhaustive list of factors a court must consider when deciding whether non-‐ permissive utilization of a copy-‐righted work is "fair use." In the context of educational photocopying, § 107's four-‐ part test was intended to be the arbiter of fair use for photocopying. 1. "The purpose and character of the use including whether such use is of a commercial nature or is for nonprofit educational purposes:" If the reproduction is made for profit, with commercial gain as its primary goal, it is unlikely to fall within the fair use doctrine. The phrase "including whether such use is of a commercial nature or is for nonprofit educational purposes" was added to this first prong of § 107's four-‐ part fair use test by the House Judiciary Committee shortly before passage of the Copyright Act of 1976 "in order to mollify educators who had lobbied unsuccessfully for an across-‐the-‐board exemption for nonprofit educational uses." While it would seem obvious that the purpose and character of photocopying multiple copies for classroom use is educational and nonprofit in nature, courts have held that educational copying by a commercial photocopy business was a commercial undertaking because the actual photocopying of work selected by educators was performed by for-‐profit vendors. 2. "The nature of the copyrighted work:" In applying this prong of the fair use test, courts have considered the originality, informative nature, intended use, and availability of the work. In an educational context, courts are less likely to find fair use where text books and other materials prepared for educational markets are copied than when works created for general public distribution are reproduced for classroom use. Additionally, in part because facts cannot by copyrighted, use of informational works is more likely to be considered fair than use of creative works. Courts sometimes have difficulty classifying a copyrighted work as distinctly informational or distinctly creative. For example, on one occasion the Ninth Circuit decided that it could not categorize a cake decorating booklet which had both informational and creative aspects. As one observer uncharitably put it, "If cake decorating
o 25
Katrina Michelle Mancao booklets can stump the Ninth Circuit, then the ability of courts to classify more complicated materials photocopied by educators and scholars at universities is doubtful." 3. "The amount and substantiality of the portion used in relation to the copyrighted work as a whole:" Photocopying small excerpts of a copyrighted work for educational purposes is more likely to appear fair than photocopying large portions of an article or book. Once the quantities set out in the Guidelines are exceeded, there is no bright line number of pages or percentage of a work one can confidently assume will be deemed fair or unfair. If the amount of copying is substantial enough, some courts will not find fair use regardless of the outcome of the other test factors. In addition to doing a quantitative assessment, courts will also do a qualitative assessment of the portion of the work that was reproduced. Photocopying the "heart" or critical parts of a work may be deemed unfair even if the number of pages or actual percentage of the work copied is insubstantial. When a work is excerpted for educational usage, the very fact that certain portions of a work are chosen by a faculty member will probably be sufficient to implicate them as "critical parts," which will weigh against a finding of fair use. 4. "The effect of the use upon the potential market for or value of the copyrighted work:" This factor echoes aspects of the second and third factors, as both the nature of the work and the amount and substantiality copied will be partially determinative of whether a copyright owner has been economically injured. Market effect has been called "undoubtedly the single most important element of fair use" by the Supreme Court in a non-‐educational context. However, the exact extent of its preeminence among § 107's four factors has not been delineated.
eventually became the working draft of the decision. She said that, for most parts, she did her research electronically. For international materials, she sourced these mainly from Westlaw, an online research service for legal and law-‐related materials to which the Court subscribes. In the old days, the common practice was that after a Justice would have assigned a case for study and report, the researcher would source his materials mostly from available law books and published articles on print. When he found a relevant item in a book, whether for one side of the issue or for the other, he would place a strip of paper marker on the appropriate page, pencil mark the item, and place the book on his desk where other relevant books would have piled up. He would later paraphrase or copy the marked out passages from some of these books as he typed his manuscript on a manual typewriter. This occasion would give him a clear opportunity to attribute the materials used to their authors or sources. With the advent of computers, however, as Justice Del Castillo’s researcher also explained, most legal references, including the collection of decisions of the Court, are found in electronic diskettes or in internet websites that offer virtual libraries of books and articles. Here, as the researcher found items that were relevant to her assignment, she downloaded or copied them into her “main manuscript,” a smorgasbord plate of materials that she thought she might need. The researcher’s technique in this case is not too far different from that employed by a carpenter. The carpenter first gets the pieces of lumber he would need, choosing the kinds and sizes suitable to the object he has in mind, say a table. When ready, he would measure out the portions he needs, cut them out of the pieces of lumber he had collected, and construct his table. He would get rid of the scraps. Here, Justice Del Castillo’s researcher did just that. She electronically “cut” relevant materials from books and journals in the Westlaw website and “pasted” these to a “main manuscript” in her computer that contained the issues for discussion in her proposed report to the Justice. She used the Microsoft Word program. Later, after she decided on the general shape that her report would take, she began pruning from that manuscript those materials that did not fit, changing the positions in the general scheme of those that remained, and adding and deleting paragraphs, sentences, and words as her continuing discussions with Justice Del Castillo, her chief editor, demanded. Parenthetically, this is the standard scheme that computer-‐literate court researchers use everyday in their work. Justice Del Castillo’s researcher showed the Committee the early drafts of her report in the Vinuya case and
IN THE MATTER OF THE CHARGES OF PLAGIARISM, ETC. AGAINST ASSOCIATE JUSTICE MARIANO C. DEL CASTILLO (2010) Court decision: Petitioners point out that the Vinuya decision lifted passages from Tams’ book, Enforcing Erga Omnes Obligations in International Law (2006) and used them in Footnote 69 with what the author thought was a mere generic reference. But, although Tams himself may have believed that the footnoting in this case was not “an appropriate form of referencing,” Unless amply explained, the above lifting from the works of Ellis and Criddle-‐Descent could be construed as plagiarism. But one of Justice Del Castillo’s researchers, a court-‐ employed attorney, explained how she accidentally deleted the attributions, originally planted in the beginning drafts of her report to him, which report
26 z
Intellectual Property Law these included the passages lifted from the separate articles of Criddle-‐Descent and of Ellis with proper attributions to these authors. But, as it happened, in the course of editing and cleaning up her draft, the researcher accidentally deleted the attributions. She and petitioners cannot deny that the decision did attribute the source or sources of such passages. Justice Del Castillo did not pass off Tams’ work as his own. The Justice primarily attributed the ideas embodied in the passages to Bruno Simma, whom Tams himself credited for them. Still, Footnote 69 mentioned, apart from Simma, Tams’ article as another source of those ideas. The Court believes that whether or not the footnote is sufficiently detailed, so as to satisfy the footnoting standards of counsel for petitioners is not an ethical matter but one concerning clarity of writing. The statement “See Tams, Enforcing Obligations Erga Omnes in International Law (2005)” in the Vinuya decision is an attribution no matter if Tams thought that it gave him somewhat less credit than he deserved. Such attribution altogether negates the idea that Justice Del Castillo passed off the challenged passages as his own. That it would have been better had Justice Del Castillo used the introductory phrase “cited in” rather than the phrase “See” would make a case of mere inadvertent slip in attribution rather than a case of “manifest intellectual theft and outright plagiarism.” If the Justice’s citations were imprecise, it would just be a case of bad footnoting rather than one of theft or deceit. If it were otherwise, many would be target of abuse for every editorial error, for every mistake in citing pagination, and for every technical detail of form. Notably, neither Justice Del Castillo nor his researcher had a motive or reason for omitting attribution for the lifted passages to Criddle-‐Descent or to Ellis. The latter authors are highly respected professors of international law. The law journals that published their works have exceptional reputations. It did not make sense to intentionally omit attribution to these authors when the decision cites an abundance of other sources. Citing these authors as the sources of the lifted passages would enhance rather than diminish their informative value. Both Justice Del Castillo and his researcher gain nothing from the omission. Thus, the failure to mention the works of Criddle-‐Decent and Ellis was unquestionably due to inadvertence or pure oversight. Petitioners of course insist that intent is not material in committing plagiarism since all that a writer has to do, to avoid the charge, is to enclose lifted portions with quotation marks and acknowledge the sources from which these were taken. Petitioners point out that the Court should apply to this case the ruling in University of the Philippines Board of Regents v. Court of Appeals and Arokiaswamy William Margaret Celine. They argue that
standards on plagiarism in the academe should apply with more force to the judiciary. But petitioners’ theory ignores the fact that plagiarism is essentially a form of fraud where intent to deceive is inherent. Their theory provides no room for errors in research, an unrealistic position considering that there is hardly any substantial written work in any field of discipline that is free of any mistake. The theory places an automatic universal curse even on errors that, as in this case, have reasonable and logical explanations. Indeed, the 8th edition of Black’s Law Dictionary defines plagiarism as the “deliberate and knowing presentation of another person's original ideas or creative expressions as one's own.” Thus, plagiarism presupposes intent and a deliberate, conscious effort to steal another’s work and pass it off as one’s own. Besides, the Court said nothing in U.P. Board of Regents that would indicate that an intent to pass off another’s work as one’s own is not required in plagiarism. The Court merely affirmed the academic freedom of a university to withdraw a master’s degree that a student obtained based on evidence that she misappropriated the work of others, passing them off as her own. This is not the case here since, as already stated, Justice Del Castillo actually imputed the borrowed passages to others.
IN THE MATTER OF THE CHARGES OF PLAGIARISM, ETC. AGAINST ASSOCIATE JUSTICE MARIANO C. DEL CASTILLO (2011) Court decision: Mainly, petitioners claim that the Court has by its decision legalized or approved of the commission of plagiarism in the Philippines. This claim is absurd. The Court, like everyone else, condemns plagiarism as the world in general understands and uses the term. Plagiarism, a term not defined by statute, has a popular or common definition. To plagiarize, says Webster, is “to steal and pass off as one’s own” the ideas or words of another. Stealing implies malicious taking. Black’s Law Dictionary, the world’s leading English law dictionary quoted by the Court in its decision, defines plagiarism as the “deliberate and knowing presentation of another person's original ideas or creative expressions as one’s own.” The presentation of another person’s ideas as one’s own must be deliberate or premeditated—a taking with ill intent. There is no commonly-‐used dictionary in the world that embraces in the meaning of plagiarism errors in attribution by mere accident or in good faith. Certain educational institutions of course assume different norms in its application. For instance, the
o 27
Katrina Michelle Mancao Loyola Schools Code of Academic Integrity ordains that “plagiarism is identified not through intent but through the act itself. The objective act of falsely attributing to one’s self what is not one’s work, whether intentional or out of neglect, is sufficient to conclude that plagiarism has occurred. Students who plead ignorance or appeal to lack of malice are not excused.” But the Court’s decision in the present case does not set aside such norm. The decision makes this clear, thus: To paraphrase Bast and Samuels, while the academic publishing model is based on the originality of the writer’s thesis, the judicial system is based on the doctrine of stare decisis, which encourages courts to cite historical legal data, precedents, and related studies in their decisions. The judge is not expected to produce original scholarship in every respect. The strength of a decision lies in the soundness and general acceptance of the precedents and long held legal opinions it draws from. Original scholarship is highly valued in the academe and rightly so. A college thesis, for instance, should contain dissertations embodying results of original research, substantiating a specific view. This must be so since the writing is intended to earn for the student an academic degree, honor, or distinction. He earns no credit nor deserves it who takes the research of others, copies their dissertations, and proclaims these as his own. There should be no question that a cheat deserves neither reward nor sympathy. But the policy adopted by schools of disregarding the element of malicious intent found in dictionaries is evidently more in the nature of establishing what evidence is sufficient to prove the commission of such dishonest conduct than in rewriting the meaning of plagiarism. Since it would be easy enough for a student to plead ignorance or lack of malice even as he has copied the work of others, certain schools have adopted the policy of treating the mere presence of such copied work in his paper sufficient objective evidence of plagiarism. Surely, however, if on its face the student’s work shows as a whole that he has but committed an obvious mistake or a clerical error in one of hundreds of citations in his thesis, the school will not be so unreasonable as to cancel his diploma. In contrast, decisions of courts are not written to earn merit, accolade, or prize as an original piece of work or art. Deciding disputes is a service rendered by the government for the public good. Judges issue decisions to resolve everyday conflicts involving people of flesh and blood who ache for speedy justice or juridical beings which have rights and obligations in law that need to be protected. The interest of society in written decisions is
not that they are originally crafted but that they are fair and correct in the context of the particular disputes involved. Justice, not originality, form, and style, is the object of every decision of a court of law. There is a basic reason for individual judges of whatever level of courts, including the Supreme Court, not to use original or unique language when reinstating the laws involved in the cases they decide. Their duty is to apply the laws as these are written. But laws include, under the doctrine of stare decisis, judicial interpretations of such laws as are applied to specific situations. Under this doctrine, Courts are “to stand by precedent and not to disturb settled point.” Once the Court has “laid down a principle of law as applicable to a certain state of facts, it will adhere to that principle, and apply it to all future cases, where facts are substantially the same; regardless of whether the parties or property are the same.” And because judicial precedents are not always clearly delineated, they are quite often entangled in apparent inconsistencies or even in contradictions, prompting experts in the law to build up regarding such matters a large body of commentaries or annotations that, in themselves, often become part of legal writings upon which lawyers and judges draw materials for their theories or solutions in particular cases. And, because of the need to be precise and correct, judges and practitioners alike, by practice and tradition, usually lift passages from such precedents and writings, at times omitting, without malicious intent, attributions to the originators. Is this dishonest? No. Duncan Webb, writing for the International Bar Association puts it succinctly. When practicing lawyers (which include judges) write about the law, they effectively place their ideas, their language, and their work in the public domain, to be affirmed, adopted, criticized, or rejected. Being in the public domain, other lawyers can thus freely use these without fear of committing some wrong or incurring some liability. Thus: The tendency to copy in law is readily explicable. In law accuracy of words is everything. Legal disputes often centre round the way in which obligations have been expressed in legal documents and how the facts of the real world fit the meaning of the words in which the obligation is contained. This, in conjunction with the risk-‐aversion of lawyers means that refuge will often be sought in articulations that have been tried and tested. In a sense therefore the community of lawyers have together contributed to this body of knowledge, language, and expression which is common property and may be utilized, developed and bettered by anyone.
28 z
Intellectual Property Law The implicit right of judges to use legal materials regarded as belonging to the public domain is not unique to the Philippines. As Joyce C. George, whom Justice Maria Lourdes Sereno cites in her dissenting opinion, observed in her Judicial Opinion Writing Handbook: A judge writing to resolve a dispute, whether trial or appellate, is exempted from a charge of plagiarism even if ideas, words or phrases from a law review article, novel thoughts published in a legal periodical or language from a party’s brief are used without giving attribution. Thus judges are free to use whatever sources they deem appropriate to resolve the matter before them, without fear of reprisal. This exemption applies to judicial writings intended to decide cases for two reasons: the judge is not writing a literary work and, more importantly, the purpose of the writing is to resolve a dispute. As a result, judges adjudicating cases are not subject to a claim of legal plagiarism. If the Court were to inquire into the issue of plagiarism respecting its past decisions from the time of Chief Justice Cayetano S. Arellano to the present, it is likely to discover that it has not on occasion acknowledged the originators of passages and views found in its decisions. These omissions are true for many of the decisions that have been penned and are being penned daily by magistrates from the Court of Appeals, the Sandiganbayan, the Court of Tax Appeals, the Regional Trial Courts nationwide and with them, the municipal trial courts and other first level courts. Never in the judiciary’s more than 100 years of history has the lack of attribution been regarded and demeaned as plagiarism. This is not to say that the magistrates of our courts are mere copycats. They are not. Their decisions analyze the often conflicting facts of each case and sort out the relevant from the irrelevant. They identify and formulate the issue or issues that need to be resolved and evaluate each of the laws, rulings, principles, or authorities that the parties to the case invoke. The decisions then draw their apt conclusions regarding whether or not such laws, rulings, principles, or authorities apply to the particular cases before the Court. These efforts, reduced in writing, are the product of the judges’ creativity. It is here— actually the substance of their decisions—that their genius, originality, and honest labor can be found, of which they should be proud. In Vinuya, Justice Del Castillo examined and summarized the facts as seen by the opposing sides in a way that no one has ever done. He identified and formulated the core of the issues that the parties raised. And when he had done this, he discussed the state of the law relevant to their resolution. It was here that he drew materials from various sources, including the three foreign authors
cited in the charges against him. He compared the divergent views these present as they developed in history. He then explained why the Court must reject some views in light of the peculiar facts of the case and applied those that suit such facts. Finally, he drew from his discussions of the facts and the law the right solution to the dispute in the case. On the whole, his work was original. He had but done an honest work. The Court will not, therefore, consistent with established practice in the Philippines and elsewhere, dare permit the filing of actions to annul the decisions promulgated by its judges or expose them to charges of plagiarism for honest work done. This rule should apply to practicing lawyers as well. Counsels for the petitioners, like all lawyers handling cases before courts and administrative tribunals, cannot object to this. Although as a rule they receive compensation for every pleading or paper they file in court or for every opinion they render to clients, lawyers also need to strive for technical accuracy in their writings. They should not be exposed to charges of plagiarism in what they write so long as they do not depart, as officers of the court, from the objective of assisting the Court in the administration of justice. As Duncan Webb said: In presenting legal argument most lawyers will have recourse to either previous decisions of the courts, frequently lifting whole sections of a judge’s words to lend weight to a particular point either with or without attribution. The words of scholars are also sometimes given weight, depending on reputation. Some encyclopaedic works are given particular authority. In England this place is given to Halsbury’s Laws of England which is widely considered authoritative. A lawyer can do little better than to frame an argument or claim to fit with the articulation of the law in Halsbury’s. While in many cases the very purpose of the citation is to claim the authority of the author, this is not always the case. Frequently commentary or dicta of lesser standing will be adopted by legal authors, largely without attribution. x x x x The converse point is that originality in the law is viewed with skepticism. It is only the arrogant fool or the truly gifted who will depart entirely from the established template and reformulate an existing idea in the belief that in doing so they will improve it. While over time incremental changes occur, the wholesale
o 29
Katrina Michelle Mancao abandonment of established expression is generally considered foolhardy.
8.
Transfer of Copyright
The Court probably should not have entertained at all the charges of plagiarism against Justice Del Castillo, coming from the losing party. But it is a case of first impression and petitioners, joined by some faculty members of the University of the Philippines school of law, have unfairly maligned him with the charges of plagiarism, twisting of cited materials, and gross neglect for failing to attribute lifted passages from three foreign authors. These charges as already stated are false, applying the meaning of plagiarism as the world in general knows it. True, Justice Del Castillo failed to attribute to the foreign authors materials that he lifted from their works and used in writing the decision for the Court in the Vinuya case. But, as the Court said, the evidence as found by its Ethics Committee shows that the attribution to these authors appeared in the beginning drafts of the decision. Unfortunately, as testified to by a highly qualified and experienced court-‐employed researcher, she accidentally deleted the same at the time she was cleaning up the final draft. The Court believed her since, among other reasons, she had no motive for omitting the attribution. The foreign authors concerned, like the dozens of other sources she cited in her research, had high reputations in international law. Notably, those foreign authors expressly attributed the controversial passages found in their works to earlier writings by others. The authors concerned were not themselves the originators. As it happened, although the ponencia of Justice Del Castillo accidentally deleted the attribution to them, there remained in the final draft of the decision attributions of the same passages to the earlier writings from which those authors borrowed their ideas in the first place. In short, with the remaining attributions after the erroneous clean-‐up, the passages as it finally appeared in the Vinuya decision still showed on their face that the lifted ideas did not belong to Justice Del Castillo but to others. He did not pass them off as his own. With our ruling, the Court need not dwell long on petitioners’ allegations that Justice Del Castillo had also committed plagiarism in writing for the Court his decision in another case, Ang Ladlad v. Commission on Elections. Petitioners are nit-‐picking. Upon close examination and as Justice Del Castillo amply demonstrated in his comment to the motion for reconsideration, he in fact made attributions to passages in such decision that he borrowed from his sources although they at times suffered in formatting lapses.
SECTION 180. RIGHTS OF ASSIGNEE. – 180.1. The copyright may be assigned in whole or in part. Within the scope of the assignment, the assignee is entitled to all the rights and remedies which the assignor had with respect to the copyright. 180.2. The copyright is not deemed assigned inter vivos in whole or in part unless there is a written indication of such intention. 180.3. The submission of a literary, photographic or artistic work to a newspaper, magazine or periodical for publication shall constitute only a license to make a single publication unless a greater right is expressly granted. If two (2) or more persons jointly own a copyright or any part thereof, neither of the owners shall be entitled to grant licenses without the prior written consent of the other owner or owners.
SECTION 181. COPYRIGHT AND MATERIAL OBJECT. – The copyright is distinct from the property in the material object subject to it. Consequently, the transfer or assignment of the copyright shall not itself constitute a transfer of the material object. Nor shall a transfer or assignment of the sole copy or of one or several copies of the work imply transfer or assignment of the copyright.
SECTION 182. FILING OF ASSIGNMENT OR LICENSE. – An assignment or exclusive license may be filed in duplicate with the National Library upon payment of the prescribed fee for registration in books and records kept for the purpose. Upon recording, a copy of the instrument shall be returned to the sender with a notation of the fact of record. Notice of the record shall be published in the IPO Gazette.
SECTION 183. DESIGNATION OF SOCIETY. – The copyright owners or their heirs may designate a society of artists, writers or composers to enforce their economic rights and moral rights on their behalf.
SECTION 4.2. The term "TECHNOLOGY TRANSFER ARRANGEMENTS" refers to contracts or agreements involving the transfer of systematic knowledge for the manufacture of a product, the application of a process, or rendering of a service including management contracts; and the transfer, assignment or licensing of all forms of intellectual property rights, including licensing of computer software except computer software developed for mass market.
30 z
Intellectual Property Law conditions or to initiate research and development programs in connection with new products, processes or equipment; 87.13. Those which prevent the licensee from adapting the imported technology to local conditions, or introducing innovation to it, as long as it does not impair the quality standards prescribed by the licensor; 87.14. Those which exempt the licensor for liability for non-‐fulfilment of his responsibilities under the technology transfer arrangement and/or liability arising from third party suits brought about by the use of the licensed product or the licensed technology; and 87.15. Other clauses with equivalent effects.
SECTION 87. PROHIBITED CLAUSES. – Except in cases under Section 91, the following provisions shall be deemed prima facie to have an adverse effect on competition and trade: 87.1. Those which impose upon the licensee the obligation to acquire from a specific source capital goods, intermediate products, raw materials, and other technologies, or of permanently employing personnel indicated by the licensor; 87.2. Those pursuant to which the licensor reserves the right to fix the sale or resale prices of the products manufactured on the basis of the license; 87.3. Those that contain restrictions regarding the volume and structure of production; 87.4. Those that prohibit the use of competitive technologies in a non-‐exclusive technology transfer agreement; 87.5. Those that establish a full or partial purchase option in favor of the licensor; 87.6. Those that obligate the licensee to transfer for free to the licensor the inventions or improvements that may be obtained through the use of the licensed technology; 87.7. Those that require payment of royalties to the owners of patents for patents which are not used; 87.8. Those that prohibit the licensee to export the licensed product unless justified for the protection of the legitimate interest of the licensor such as exports to countries where exclusive licenses to manufacture and/or distribute the licensed product(s) have already been granted; 87.9. Those which restrict the use of the technology supplied after the expiration of the technology transfer arrangement, except in cases of early termination of the technology transfer arrangement due to reason(s) attributable to the licensee; 87.10. Those which require payments for patents and other industrial property rights after their expiration, termination arrangement; 87.11. Those which require that the technology recipient shall not contest the validity of any of the patents of the technology supplier; 87.12. Those which restrict the research and development activities of the licensee designed to absorb and adapt the transferred technology to local
SECTION 88. MANDATORY PROVISIONS. – The following provisions shall be included in voluntary license contracts: 88.1. That the laws of the Philippines shall govern the interpretation of the same and in the event of litigation, the venue shall be the proper court in the place where the licensee has its principal office; 88.2. Continued access to improvements in techniques and processes related to the technology shall be made available during the period of the technology transfer arrangement; 88.3. In the event the technology transfer arrangement shall provide for arbitration, the Procedure of Arbitration of the Arbitration Law of the Philippines or the Arbitration Rules of the United Nations Commission on International Trade Law (UNCITRAL) or the Rules of Conciliation and Arbitration of the International Chamber of Commerce (ICC) shall apply and the venue of arbitration shall be the Philippines or any neutral country; and 88.4. The Philippine taxes on all payments relating to the technology transfer arrangement shall be borne by the licensor.
SECTION 92. NON-‐REGISTRATION WITH THE DOCUMENTATION, INFORMATION AND TECHNOLOGY TRANSFER BUREAU . – Technology transfer arrangements that conform with the provisions of Sections 86 and 87 need not be registered with the Documentation, Information and Technology Transfer Bureau. Non-‐conformance with any of the provisions of Sections 87 and 88, however, shall automatically render the technology transfer arrangement unenforceable, unless said technology transfer arrangement is approved and registered with
o 31
Katrina Michelle Mancao the Documentation, Information and Technology Transfer Bureau under the provisions of Section 91 on exceptional cases.
SECTION 228. PUBLIC RECORDS. – The section or division of the National Library and the Supreme Court Library charged with receiving copies and instruments deposited and with keeping records required under this Act and everything in it shall be opened to public inspection. The Director of the National Library is empowered to issue such safeguards and regulations as may be necessary to implement this Section and other provisions of this Act.
SECTION 237. NOTIFICATION ON BERNE APPENDIX. – The Philippines shall by proper compliance with the requirements set forth under the Appendix of the Berne Convention (Paris Act, 1971) avail itself of the special provisions regarding developing countries, including provisions for licenses grantable by competent authority under the Appendix.
9.
SECTION 229. COPYRIGHT DIVISION; FEES. – The Copyright Section of the National Library shall be classified as a Division upon the effectivity of this Act. The National Library shall have the power to collect, for the discharge of its services under this Act, such fees as may be promulgated by it from time to time subject to the approval of the Department Head.
Deposit of Copyrightable Materials
SECTION 191. REGISTRATION AND DEPOSIT WITH NATIONAL LIBRARY AND THE SUPREME COURT LIBRARY. – After the first public dissemination of performance by authority of the copyright owner of a work falling under Subsections 172.1, 172.2 and 172.3 of this Act, there shall, for the purpose of completing the records of the National Library and the Supreme Court Library, within three (3) weeks, be registered and deposited with it, by personal delivery or by registered mail two (2) complete copies or reproductions of the work in such form as the directors of said libraries may prescribe. A certificate of deposit shall be issued for which the prescribed fee shall be collected and the copyright owner shall be exempt from making additional deposit of the works with the National Library and the Supreme Court Library under other laws. If, within three (3) weeks after receipt by the copyright owner of a written demand from the directors for such deposit, the required copies or reproductions are not delivered and the fee is not paid, the copyright owner shall be liable to pay a fine equivalent to the required fee per month of delay and to pay to the National Library and the Supreme Court Library the amount of the retail price of the best edition of the work. Only the above mentioned classes of work shall be accepted for deposit by the National Library and the Supreme Court Library.
RULE 5 -‐ REGISTRATION AND DEPOSIT OF WORK, COPYRIGHT SAFEGUARDS AND REGULATIONS SECTION 1. WHO MAY APPLY. — The owner or assignee of the copyright or his duly authorized agent or representative, may apply for a certificate of registration and deposit of the work: Provided, That if an author could not claim the benefit of copyright protection, his assignee or agent cannot claim it. If the applicant is not the owner or author or assignee of the work, he shall be required to submit his authority to apply. An assignee is a person to whom an author may assign copyright in whole or in part. The assignee is entitled to all the rights and remedies which the assignor has with respect to the copyright. Although no copyright should subsist in any work of the government, any employee may claim it by submitting for registration any work that has been created during the time of his regularly prescribed official duties. SECTION 2. IDENTIFICATION OF AUTHOR OR AUTHORS. — An application for copyright certificate shall identify the author or authors, as far as practicable, without prejudice to the provisions of Sections 171.2 and 179 of the IPC. SECTION 3. NON-‐RESIDENT APPLICANT. — A non-‐resident applicant shall appoint a resident agent, by special power of attorney (SPA), who shall be authorized to pursue the copyright application for his/her/its behalf with TNL and/or the SCL and to receive service of notice or other legal process relating to the application and the copyright. In the event of death, absence or incapacity of the resident agent, the applicant shall appoint a new resident agent, by SPA
SECTION 192. NOTICE OF COPYRIGHT. – Each copy of a work published or offered for sale may contain a notice bearing the name of the copyright owner, and the year of its first publication, and, in copies produced after the creator's death, the year of such death.
SECTION 227. OWNERSHIP OF DEPOSIT AND INSTRUMENTS. – All copies deposited and instruments in writing filed with the National Library and the Supreme Court Library in accordance with the provisions of this Act shall become the property of the Government.
32 z
Intellectual Property Law with revocation of the prior SPA, and file notice and a copy thereof with TNL and/or the SCL. SECTION 4. WORKS THAT SHALL BE REGISTERED AND DEPOSITED. — Two (2) copies or reproductions of the following classes of works, and transfers and assignments related thereto, shall be registered and deposited with TNL Copyright Division and another two (2) copies with the SCL: • Books, pamphlets, articles and other writings; • Periodicals and newspapers; • Lectures, sermons, addresses, dissertations prepared for oral delivery whether or not reduced in writing or other material form; • Letters; • Musical compositions with or without words. SECTION 5. REPLICAS AND PICTURES. — For practical purposes, only replicas and pictures of the following classes of works, shall be registered and deposited with TNL Copyright Division: • Works of drawing, painting, architecture, sculpture, engraving, lithography or other works of art, models or designs for works of art; • Original ornamental designs or models for articles of manufacture, whether or not registerable as an industrial design, and other works of applied art; • Illustrations, maps, plans, sketches, charts and three-‐dimensional works relative to geography, topography, architecture or science; • Drawings or plastic works of a scientific or technical character. SECTION 6. WORKS THAT MAY BE REGISTERED AND DEPOSITED. — The following works may be registered and deposited: • Dramatic or dramatic-‐musical compositions, choreographic works or entertainment in shows; • Photographic works including works produced by a process analogous to photography, lantern slides; • Audiovisual works and cinematographic works and works produced by a process analogous to cinematography or any process for making audio-‐visual recordings; • Pictorial illustrations and advertisements; • Computer programs; • Other literary, scholarly, scientific and artistic works; • Sound recordings; • Broadcast recordings.
SECTION 7. When to Register and Deposit. — The registration and deposit of copies or reproductions of the work or works, using the prescribed form, shall be made personally or by registered mail within three (3) weeks after the first public dissemination or publication as authorized by the author. 08 and 15 December 2011 COPYRIGHT What is copyright? -‐ It is a bundle of rights enumerated in Section 177. Why is copyright called an “economic right”? -‐ Because the purpose of the establishment of the right is to allow the author or originator of the work to maximize the value that can be derived from the work. -‐ How does it allow the copyright owner to maximize the value of the work? o By allowing him to assign his rights, grant licenses for the use of the work, etc., thus allowing him to get royalties from the work. Beauty of copyright: Duration. It protects the owner during his lifetime + 50 years after death. Standard of copyright protection: Originality. -‐ “Original” means it is the product of one’s skill, labor/employment -‐ Work does not need to be groundbreaking or revolutionary. -‐ What it merely requires is that the work is your own. This is an incredibly low standard! Why is the standard so low? -‐ To remain very objective. Having subjective standards will erode the IP system = if every new work that challenges the status quo will not be protected, no one might challenge the status quo. -‐ Thus, IP should not look into the substance of the work. (See Section 172.2) -‐ Consequence of the low standard: It is hard to prove infringement. To prove infringement, you have to prove ACCESS and COPYING. 3 things covered by IP: 1. Traditional works – literary and artistic works 2. Derivative works – translation 3. Published works Issue: Can a work that was transformed without the consent of the original creator be considered as original and be protected? -‐ It can be understood that the derivative work should be AUTHORIZED. (See 173.2)
o 33
Katrina Michelle Mancao -‐
BUT: Ideas are too important to be kept in the private domain. If done, you are increasing the cost for the creation of the work. Thus, the la protects NOT the underlying idea by the EXPRESSION of the idea. Ideas are basic building blocks. If you allow the monopoly of idea through copyright, you would increase the cost for further work and this may lead to stagnation. Remember, the primary purpose of IP is really the advancement of art and literature, and this won’t be achieved if you monopolized the fundamental building blocks of these, i.e. the ideas. o Reconcile this with Section 173.1(b) à data is NOT protected, but the COMPILATIONS of data are protected? o Note that in Feist (which is a US case, thus merely persuasive), the standard of protection for compilation à modicum of creativity. It required the compilation to be original by reason of their selection or arrangement. There must be something unusual or exceptional with the collection of data.
-‐
b.
If not registered, buy only the thing. Post-‐PD49 à Were all the rights assigned to one person?
Ownership of copyright by the government: -‐ GR: Government cannot acquire copyright. -‐ However, if somebody would exploit the work for profit, there must be prior approval from the government agency or office where the work is created. -‐ Exception to the prior approval requirement exception: 176.3 Ma’am is a government employee. Are her oral lectures protected by copyright? -‐ Yes. Copyright protects the work regardless of its form. -‐ What right does she have? o 176.2. She has the exclusive right to reproduces, etc. Ownership of copyright: -‐ GR: Author of the work -‐ If more than 1 author: o Apply their contract if there is any. o If no contract: apply the rules on co-‐ ownership If the work is made by an employee: -‐ Test: Whether the work is part of the employee’s regularly-‐assigned duties: o If Yes: the work is owned by the employer. o If No: the work is owned by the employee. -‐ Note: See 178.3 o 178.3(b) expressly allows the employer and the employee to have an agreement to the contrary o 178.3(a) does not have a similar proviso o the difference in treatment implies that an agreement to make the employer own the work in instances where the employee created something and such is not a part of his regularly assigned duties is PROHIBITED. § But Ma’am also said that you can still argue the other way. If the work is created by commission: -‐ Work: owned by the person who commissioned it -‐ Copyright: owned by the creator o UNLESS there is a contract to the
When does copyright vests? -‐ Under the IP Code: from the moment of creation (we follow the principle of automatic protection under the Berne Convention) -‐ The principle of automatic protection was followed since 1972 under PD49. o Under PD 49, the copyright owner can still sue the infringer even if the former failed to deposit the material. The only effect of failure to register is that the copyright owner won’t be allowed to obtain damages from the defendant. -‐ Prior to that: Under Act 3134, registration and deposit were needed to vest copyright. o Reglamentary period for registration: 30 days from creation. o Effect of failure to register within that period: it may be the basis for questioning the copyright o Santos v. McCullough Printing: failure to deposit the copyrighted material made the material part of the public domain. -‐ PD 49 was actually a revolutionary piece of legislation! So, for due diligence investigation: 1. Ask when the work was created. 2. If: a. Pre-‐PD49 à Was it registered or not?
34 z
Intellectual Property Law contrary If the work is a letter: -‐ Letter: owned by the recipient -‐ Copyright: owned by the writer Duration of copyright: lifetime + 50 years after death -‐ Copyright is best for works with long shelf life. -‐ Note calculation for the period after death: begins on January 1 of the year following the death o Why? For easier computation -‐ After the expiration of the period, the work becomes part of the public domain à why classic works are significantly cheaper! 05 January 2012 Limitations on copyright (exception to the exclusive right given to holder): 1. Section 184 (limitations of copyright), 176 (works of the government) 2. Exceptions for certain works: library 3. Fair use Q: Showing of Harry Potter, done by the Religious Society of Mary, for P50. Is the Religious Society of Mary liable? -‐ No! 184.1(a) embodies 2 exceptions: 1. Privately and free of charge; OR 2. Charitable or religious institution/society -‐ BUT the showing must be done AFTER the first publication of the work. -‐ Follow-‐up question: What if they show the movie all over the Philippines, are they now liable? o Yes! See 184.2. The act must not unreasonably prejudice the right holder’s legitimate interests. o ALWAYS READ 184.1 IN CONJUNCTION WITH 184.2 184.1 (b) – drawing from work of others -‐ Conditions: o Work has been published o Compatible with fair use o Only to the extent justified for the purpose o Attribution! 184.1(c) – exception for mass media -‐ Conditions: o Delivered in public o For information purposes o No “all rights reserved” in the article o Attribution
184.1(i) v. 184.1(a) 184.1(i) “The public performance or the communication to the public of a work, in a place where no admission fee is charged in respect of such public performance or communication, by a club or institution for charitable or educational purpose only, whose aim is not profit making, subject to such other limitations as may be provided in the Regulations” BOTH must be present: 1. Educational/ charitable purpose 2. Free of charge
184.1(a) “The recitation or performance of a work, once it has been lawfully made accessible to the public, if done privately and free of charge or if made strictly for a charitable or religious institution or society”
Only EITHER of the 2 is required: 1. Religious/ charitable institution 2. Free of charge and private
Filipino Society of Composers v. Tan: Standard used by the court to determine whether the performance was done free or charge or not à whether or not it is designed to induce more business. Plus costs already included in other costs. Philippine Educ. Co. v. Sotto: Erroneous case! There was no copyright on the material but with an “all rights reserved” clause. -‐ Ma’am: If no copyright, what rights were reserved? “All rights reserved” applicable only for copyrighted works. Del Castillo cases: -‐ 1st Court: No intent on the part of Del Castillo to plagiarize. o Ma’am: Intent shouldn't be required in special laws! (malum prohibitum) -‐ 2nd Court: Court made a distinction between the standards of an academic institution and the standards for members of the bar. o Acad: Higher standards o Bar: Lower standards – stare decisis o Ma’am: 184.1(k) was extended! They rewrote the IP Coe! Court, in effect, said lawyers and justices were exempted from the application of the IP Code. They did not even bother with 184.2! Other exceptions: 1. Work of architecture 2. Reproduction of published work
o 35
Katrina Michelle Mancao Old law (PD49) = complete exception granted to natural person o Now = subject to certain conditions. o IP Code abolished PD 285 For libraries Importation -‐ No express prohibition of parallel importation
rights are assigned.
o
3. 4.
12 January 2012 Deposit -‐ Not a requirement for the vesting of copyright. -‐ Where made: TNL and SCL o Note that the IPO and the SCL now has a Memorandum of Agreement. -‐ Office is exercising a purely ministerial function. -‐ Certificate of deposit o Legally does not mean anything. o It is merely a proof of the fact of deposit o Can establish that as of the date of deposit, work was already created. Thus, may help establish that the depositor is not an appropriator but the creator of the work. o With respect to third person who may want to deal with the depositor: may impress upon the third person that the depositor is the right person to deal with. § Philippines is a formalistic society. § This may reduce transaction costs. § Removes lingering doubt about copyright. -‐ Fine for failure to deposit – kicks in only AFTER a demand -‐ Why require a deposit? o Public interest = promote availability of information to the public by making it available in the National Library (so that there is a public repository of these works of art)
FAIR USE: -‐ Drawn from US jurisprudence -‐ Harper & Row case: o Factors of fair use: § Purpose of use – for commercial purpose à presumption that use is not fair § Nature of work – unpublished à unfair use § Amount and substantiality – measured by quality, not quantity (heart of work) § Effect on the market – MOST IMPORTANT factor -‐ Q: Did we completely adopt the US concept of fair use? o Not really. § 185.2 – fact that work is unpublished… § 185.1 – did not establish a hierarchy among the 4 factors § commercial… -‐ Sony Corp. v. Universal Studios: very important case o The ruling in this case is still standard. § Standard: capable of single, non-‐infringing use = no infringement. § In this case, time shifting is a substantial, non-‐infringing use. o No contributory infringement. Transfer of copyright: -‐ Must be written. Oral assignment is invalid. -‐ Purpose of the requirement = protect rights of the creator. To ensure that the copyright owner knows exactly what he/she is giving away. Assignment v. License Assignment License Transfer of ownership of Limited. Only permission right = ceding right to use. Right remains with the copyright owner who may declare conditions with which
10. Infringement
a.
Definition
Ballantine’s Law Dictionary: It is a violation of a right; an encroachment.
b.
Remedies
SECTION 221. POINTS OF ATTACHMENT FOR WORKS UNDER SECTIONS 172 AND 173. – 221.1. The protection afforded by this Act to copyrightable works under Sections 172 and 173 shall apply to:
36 z
Intellectual Property Law (a) Works of authors who are nationals of, or have their habitual residence in, the Philippines; (b) Audio-‐visual works the producer of which has his headquarters or habitual residence in the Philippines; (c) Works of architecture erected in the Philippines or other artistic works incorporated in a building or other structure located in the Philippines; (d) Works first published in the Philippines; and (e) Works first published in another country but also published in the Philippines within thirty days, irrespective of the nationality or residence of the authors.
223.2. Sound recordings that were first published in the Philippines.
SECTION 224. POINTS OF ATTACHMENT FOR BROADCASTS. – 224.1. The provisions of this Act on the protection of broadcasts shall apply to: (a) Broadcasts of broadcasting organizations the headquarters of which are situated in the Philippines; and (b) Broadcasts transmitted from transmitters situated in the Philippines. 224.2. The provisions of this Act shall also apply to performers who, and to producers of sound recordings and broadcasting organizations which, are to be protected by virtue of and in accordance with any international convention or other international agreement to which the Philippines is a party.
221.2. The provisions of this Act shall also apply to works that are to be protected by virtue of and in accordance with any international convention or other international agreement to which the Philippines is a party.
SECTION 3. INTERNATIONAL RECIPROCITY. –
SECTION 222. POINTS OF ATTACHMENT FOR PERFORMERS. –
CONVENTIONS
AND
Any person who is a national or who is domiciled or has a real and effective industrial establishment in a country which is a party to any convention, treaty or agreement relating to intellectual property rights or the repression of unfair competition, to which the Philippines is also a party, or extends reciprocal rights to nationals of the Philippines by law, shall be entitled to benefits to the extent necessary to give effect to any provision of such convention, treaty or reciprocal law, in addition to the rights to which any owner of an intellectual property right is otherwise entitled by this Act.
The provisions of this Act on the protection of performers shall apply to: 222.1. Performers who are nationals of the Philippines; 222.2. Performers who are not nationals of the Philippines but whose performances: (a) Take place in the Philippines; or (b) Are incorporated in sound recordings that are protected under this Act; or (c) Which has not been fixed in sound recording but are carried by broadcast qualifying for protection under this Act.
SECTION 10.2. (a) Exercise original jurisdiction in administrative complaints for violations of laws involving intellectual property rights: Provided, That its jurisdiction is limited to complaints where the total damages claimed are not less than Two hundred thousand pesos (P200,000): Provided further, That availment of the provisional remedies may be granted in accordance with the Rules of Court. The Director of Legal Affairs shall have the power to hold and punish for contempt all those who disregard orders or writs issued in the course of the proceedings. (b) After formal investigation, the Director for Legal Affairs may impose one (1) or more of the following administrative penalties: (i) The issuance of a cease and desist order which shall specify the acts that the respondent shall cease and desist from and shall require him to submit a compliance report
SECTION 223. POINTS OF ATTACHMENT FOR SOUND RECORDINGS. – The provisions of this Act on the protection of sound recordings shall apply to: 223.1. Sound recordings the producers of which are nationals of the Philippines; and
o 37
Katrina Michelle Mancao within a reasonable time which shall be fixed in the order; (ii) The acceptance of a voluntary assurance of compliance or discontinuance as may be imposed. Such voluntary assurance may include one or more of the following:
(vi) The cancellation of any permit, license, authority, or registration which may have been granted by the Office, or the suspension of the validity thereof for such period of time as the Director of Legal Affairs may deem reasonable which shall not exceed one (1) year; (vii) The withholding of any permit, license, authority, or registration which is being secured by the respondent from the Office; (viii) The assessment of damages; (ix) Censure; and (x) Other analogous penalties or sanctions.
(1) An assurance to comply with the provisions of the intellectual property law violated; (2) An assurance to refrain from engaging in unlawful and unfair acts and practices subject of the formal investigation; (3) An assurance to recall, replace, repair, or refund the money value of defective goods distributed in commerce; and (4) An assurance to reimburse the complainant the expenses and costs incurred in prosecuting the case in the Bureau of Legal Affairs. The Director of Legal Affairs may also require the respondent to submit periodic compliance reports and file a bond to guarantee compliance of his undertaking;
SECTION 216. REMEDIES FOR INFRINGEMENT. – 216.1. Any person infringing a right protected under this law shall be liable: (a) To an injunction restraining such infringement. The court may also order the defendant to desist from an infringement, among others, to prevent the entry into the channels of commerce of imported goods that involve an infringement, immediately after customs clearance of such goods. (b) Pay to the copyright proprietor or his assigns or heirs such actual damages, including legal costs and other expenses, as he may have incurred due to the infringement as well as the profits the infringer may have made due to such infringement, and in proving profits the plaintiff shall be required to prove sales only and the defendant shall be required to prove every element of cost which he claims, or, in lieu of actual damages and profits, such damages which to the court shall appear to be just and shall not be regarded as penalty. (c) Deliver under oath, for impounding during the pendency of the action, upon such terms and conditions as the court may prescribe, sales invoices and other documents evidencing sales, all articles and their packaging alleged to infringe a copyright and implements for making them. (d) Deliver under oath for destruction without any compensation all infringing copies or devices, as well as all plates, molds, or other means for making such infringing copies as the court may order. (e) Such other terms and conditions, including the payment of moral and exemplary damages,
(iii) The condemnation or seizure of products which are subject of the offense. The goods seized hereunder shall be disposed of in such manner as may be deemed appropriate by the Director of Legal Affairs, such as by sale, donation to distressed local governments or to charitable or relief institutions, exportation, recycling into other goods, or any combination thereof, under such guidelines as he may provide; (iv) The forfeiture of paraphernalia and all real and personal properties which have been used in the commission of the offense; (v) The imposition of administrative fines in such amount as deemed reasonable by the Director of Legal Affairs, which shall in no case be less than Five thousand pesos (P5,000) nor more than One hundred fifty thousand pesos (P150,000). In addition, an additional fine of not more than One thousand pesos (P1,000) shall be imposed for each day of continuing violation;
38 z
Intellectual Property Law which the court may deem proper, wise and equitable and the destruction of infringing copies of the work even in the event of acquittal in a criminal case.
(c) Trade exhibit of the article in public, shall be guilty of an offense and shall be liable on conviction to imprisonment and fine as above mentioned.
216.2. In an infringement action, the court shall also have the power to order the seizure and impounding of any article which may serve as evidence in the court proceedings.
SECTION 218. AFFIDAVIT EVIDENCE. – 218.1. In an action under this Chapter, an affidavit made before a notary public by or on behalf of the owner of the copyright in any work or other subject matter and stating that: (a) At the time specified therein, copyright subsisted in the work or other subject matter; (b) He or the person named therein is the owner of the copyright; and (c) The copy of the work or other subject matter annexed thereto is a true copy thereof, shall be admitted in evidence in any proceedings for an offense under this Chapter and shall be prima facie proof of the matters therein stated until the contrary is proved, and the court before which such affidavit is produced shall assume that the affidavit was made by or on behalf of the owner of the copyright. 218.2. In an action under this Chapter: (a) Copyright shall be presumed to subsist in the work or other subject matter to which the action relates if the defendant does not put in issue the question whether copyright subsists in the work or other subject matter; and (b) Where the subsistence of the copyright is established, the plaintiff shall be presumed to be the owner of the copyright if he claims to be the owner of the copyright and the defendant does not put in issue the question of his ownership. (c) Where the defendant, without good faith, puts in issue the questions of whether copyright subsists in a work or other subject matter to which the action relates, or the ownership of copyright in such work or subject matter, thereby occasioning unnecessary costs or delay in the proceedings, the court may direct that any costs to the defendant in respect of the action shall not be allowed by him and that any costs occasioned by the defendant to other parties shall be paid by him to such other parties.
SECTION 217. CRIMINAL PENALTIES. – 217.1. Any person infringing any right secured by provisions of Part IV of this Act or aiding or abetting such infringement shall be guilty of a crime punishable by: (a) Imprisonment of one (1) year to three (3) years plus a fine ranging from Fifty thousand pesos (P50,000) to One hundred fifty thousand pesos (P150,000) for the first offense. (b) Imprisonment of three (3) years and one (1) day to six (6) years plus a fine ranging from One hundred fifty thousand pesos (P150,000) to Five hundred thousand pesos (P500,000) for the second offense. (c) Imprisonment of six (6) years and one (1) day to nine (9) years plus a fine ranging from five hundred thousand pesos (P500,000) to One million five hundred thousand pesos (P1,500,000) for the third and subsequent offenses. (d) In all cases, subsidiary imprisonment in cases of insolvency. 217.2. In determining the number of years of imprisonment and the amount of fine, the court shall consider the value of the infringing materials that the defendant has produced or manufactured and the damage that the copyright owner has suffered by reason of the infringement. 217.3. Any person who at the time when copyright subsists in a work has in his possession an article which he knows, or ought to know, to be an infringing copy of the work for the purpose of: (a) Selling, letting for hire, or by way of trade offering or exposing for sale, or hire, the article; (b) Distributing the article for purpose of trade, or for any other purpose to an extent that will prejudice the rights of the copyright owner in the work; or
o 39
Katrina Michelle Mancao the Philippines, Article 50 of the Agreement on Trade Related Aspects of Intellectual Property Rights, otherwise known as TRIPS and other related laws and international conventions. SECTION 2. THE WRIT OF SEARCH AND SEIZURE. – Where any delay is likely to cause irreparable harm to the intellectual property right holder or where there is demonstrable risk of evidence being destroyed, the intellectual property right holder or his duly authorized representative in a pending civil action for infringement or who intends to commence such an action may apply ex parte for the issuance of a writ of search and seizure directing the alleged infringing defendant or expected adverse party to admit into his premises the persons named in the order and to allow the search, inspection, copying, photographing, audio and audiovisual recording or seizure of any document and article specified in the order. SECTION 3. WHERE APPLICATION FILED. – The application shall be filed with any of the Regional Trial Courts of the judicial region designated to try violations of intellectual property rights stationed at the place where the alleged violation occurred or is to occur, or the place to be searched, at the election of the applicant: Provided, however, that where the complaint for infringement has already been filed, the application shall be made in the court where the case is pending. SECTION 4. VERIFIED APPLICATION AND AFFIDAVITS. – The applicant shall file a verified application alleging the ground upon which it is based and the specific description and location of the documents and articles to be searched, inspected, copied or seized and their value. It shall also state the name applicant, his representative, witnesses and counsel who will attend the search in the event that the application is granted. The application shall be supported by affidavits of witnesses who personally know the facts and by the authenticated or certified documents. The application shall contain a certification against forum shopping as prescribed by Section 5, Rule 7 of the 1997 Rules of Civil Procedure. The applicant shall undertake in his application that he will not use any of the documents, articles or information obtained by reason of the search and seizure for any purpose other than in the action in which the writ is issued. SECTION 5. EXAMINATION OF APPLICANT; RECORD; CONFIDENTIALITY OF PROCEEDINGS. – The application shall be acted upon within twenty-‐four (24) hours from its filing. The judge must, before issuing the writ, examine in the form of searching questions and
SECTION 219. PRESUMPTION OF AUTHORSHIP. – 219.1. The natural person whose name is indicated on a work in the usual manner as the author shall, in the absence of proof to the contrary, be presumed to be the author of the work. This provision shall be applicable even if the name is a pseudonym, where the pseudonym leaves no doubt as to the identity of the author. 219.2. The person or body corporate whose name appears on a audio-‐visual work in the usual manner shall, in the absence of proof to the contrary, be presumed to be the maker of said work.
SECTION 220. INTERNATIONAL REGISTRATION OF WORKS. – A statement concerning a work, recorded in an international register in accordance with an international treaty to which the Philippines is or may become a party, shall be construed as true until the contrary is proved except: 220.1. Where the statement cannot be valid under this Act or any other law concerning intellectual property. 220.2. Where the statement is contradicted by another statement recorded in the international register.
SECTION 225. JURISDICTION. – Without prejudice to the provisions of Subsection 7.1(c), actions under this Act shall be cognizable by the courts with appropriate jurisdiction under existing law.
SECTION 226. DAMAGES. – No damages may be recovered under this Act after four (4) years from the time the cause of action arose.
SECTION 231. REVERSE RECIPROCITY OF FOREIGN LAWS. – Any condition, restriction, limitation, diminution, requirement, penalty or any similar burden imposed by the law of a foreign country on a Philippine national seeking protection of intellectual property rights in that country, shall reciprocally be enforceable upon nationals of said country, within Philippine jurisdiction.
RULE ON SEARCH AND SEIZURE IN CIVIL ACTIONS FOR INFRINGEMENT OF INTELLECTUAL PROPERTY RIGHTS SECTION 1. COVERAGE. – This Rule shall govern the provisional seizure and impounding of documents and articles in pending and intended civil actions for the purpose of preventing infringement and preserving relevant evidence in regard to alleged infringement under Republic Act No. 8293, otherwise known as the Intellectual Property Code of
40 z
Intellectual Property Law answers, in writing an under oath or affirmation, the applicant and the witnesses he may produce on facts personally known to them. The examination of the applicant and his witnesses shall be recorded. Their sworn statements and their affidavits shall form part of the record of the case. The hearing on the application for the writ shall be held in the chambers of the judge. Court personnel shall maintain the confidentiality of the application proceeding. The court may require the applicant to give other information necessary for the identification of the articles and documents to be searched, inspected, copied or seized and the premises to be searched. Where feasible, it may direct the applicant to submit copies and photographs of the documents or articles to be seized and impounded. SECTION 6. GROUNDS FOR THE ISSUANCE OF THE ORDER. – Before the Order can be issued, the evidence proffered by the applicant and personally evaluated by the judge must show that: (a) the applicant is the right holder or his duly authorized representative; (b) there is probable cause to believe that the applicant’s right is being infringed or that such infringement is imminent and there is a prima facie case for final relief against the alleged infringing defendant or expected adverse party; (c) damage, potential or actual, likely to be caused to the applicant is irreparable; (d) there is demonstrable risk of evidence that the alleged infringing defendant or expected adverse party may destroy, hide or remove the documents or articles before any application inter partes can be made; and (e) the documents and articles to be seized constitute evidence of the alleged infringing defendant’s or expected adverse party’s infringing activity or that they infringe upon the intellectual property right of the applicant or that they are used or intended to be used as means of infringing the applicant’s intellectual property right. SECTION 7. WHEN WRIT MAY ISSUE. – If the judge is satisfied with the proof of facts upon which the application is based, he shall issue the writ requiring the search, inspection or copying of the subject documents or articles or commanding the sheriff to take
them into his custody subject to the control of the court. The enforcement of the writ shall be supervised by an independent Commissioner to be appointed by the court. SECTION 8. CONTENTS OF THE WRIT. – The writ shall contain the following: (a) an order to the alleged infringing defendant, expected adverse party or to the person who appears to be in charge or in control of the premises or residing or working therein to permit the persons named in the writ to enter into the premises for the purpose of searching, inspecting, copying, or removing from the premises and transferring to the custody of the sheriff and subject to the control of the court the subject documents and articles; (b) an order to the alleged infringing defendant, expected adverse party or to the person in charge or in control of the premises to disclose to the sheriff serving the writ the location of the documents and articles subject of the writ. (c) the period when the writ shall be enforced which in no case shall be more than ten (10) days from the date of issuance by the court. (d) the names of the applicant or his agent or representative and the Commissioner who shall supervise the enforcement of the writ; and (e) other terms and conditions that will insure the proper execution of the writ with due regard to the rights of the alleged infringing defendant or expected adverse party. It shall also contain a warning that violation of any of the terms and conditions of the writ shall constitute contempt of court. SECTION 9. BOND AND ITS CONDITIONS. – The applicant shall be required to post a cash bond, surety bond or other equivalent security executed in favor of the defendant or expected adverse party in a reasonable amount to be fixed by the court in its order granting the issuance of a writ of search and seizure. The bond shall be conditioned on the undertaking of the applicant that he will pay all the costs which may be adjudged to defendant or expected adverse party and all damages which the latter may sustain by reason of the issuance of the writ. SECTION 10. WHEN WRIT SHALL BE SERVED . – The writ shall be served only on weekdays and from 8 o'clock in the morning to 5 o'clock in the afternoon. However, the court may direct that the writ be served on
o 41
Katrina Michelle Mancao any day and any time for compelling reasons stated in the application and duly proved. SECTION 11. TO WHOM WRIT SHALL BE SERVED. – The writ shall be served on the alleged infringing defendant or expected adverse party in the place to be searched. If the alleged infringing defendant or expected adverse party cannot be found in the premises, the writ shall be served on his agent or representative. In the absence of an agent or representative, it shall be served on the person in charge or in control of the premises, or residing or working therein who is of sufficient age and discretion. If such person is absent, the sheriff or proper officer shall post the papers on the premises and proceed with the enforcement of the writ. SECTION 12. COMMISSIONER, DUTIES, QUALIFICATIONS AND FEES. – The enforcement of the writ shall be supervised by the independent Commissioner appointed by the court in the performance of his duty, the Commissioner shall: (a) give impartial advice to the alleged infringing defendant, expected adverse party or to the person in charge of the premises to be searched as to the meaning and coverage of the writ. (b) attempt to achieve agreement on a suitable search procedure (c) assess what documents or articles come within the terms of the writ (d) ensure the accuracy of the list of documents and articles searched, inspected, copied or seized by the sheriff; (e) prepare his own report on the search and seizure and verify and sign the return prepared by the sheriff, and (f) generally, assist in the proper execution of the writ. The Commissioner shall be a member of the Philippine Bar and of proven competence, integrity and probity. He shall receive such reasonable compensation as may be determined by the court which can be charged as cost of suit. SECTION 13. SEARCH TO BE CONDUCTED IN THE PRESENCE OF DEFENDANT, HIS REPRESENTATIVE, PERSON IN CHARGE OF THE PREMISES, OR WITNESSES. – The premises may not be searched except in the presence of the alleged infringing defendant, expected adverse party or his representative or the person in
charge or in control of the premises or residing or working therein who shall be given the opportunity to read the writ before its enforcement and seek its interpretation from the Commissioner. In the absence of the latter, two persons of sufficient age and discretion residing in the nearest locality. SECTION 14. MANNER OF SEARCH AND SEIZURE; DUTIES OF THE SHERIFF. – Upon service of the writ in accordance with section 11 hereof, the sheriff, under the supervision of the Commissioner, shall search for the documents and articles specified in the writ, and take them in his custody subject to the control of the court. If the subject articles are not capable of manual delivery, the sheriff shall attach to them a tag or label stating the fact of seizure and warning all persons from tampering with them. The sheriff shall, in the presence of the applicant or his representative, and under the supervision of the Commissioner, prepare a detailed list of the seized documents and articles. He shall give an accurate copy of the same to the alleged infringing defendant, expected adverse party, his agent or representative, to the person in charge or in control of the premises or residing or working therein in whose presence the search and seizure were made. In the absence of the person in charge or in control of the premises or residing or working therein, the sheriff must, in the presence of at least two witnesses of sufficient age and discretion residing in the same locality, leave a copy of the receipt in the place in which he found the seized property. Where no witnesses are available in the same locality, the copy of the receipt shall be left by the sheriff in the presence of two witnesses residing in the nearest locality The applicant or his representative and the Commissioner shall also be given a copy of the receipt. After the sheriff has taken possession of the documents and articles, he shall deliver them to a bonded warehouse or government warehouse for safekeeping. The applicant or his representative shall be allowed access to said materials for the purpose of examining them. The applicant shall be responsible for the necessary expenses incurred in the seizure and safekeeping of the documents and articles in a bonded warehouse or government warehouse. SECTION 15. USE OF REASONABLE FORCE TO EFFECT WRIT. – The sheriff, if refused admittance to the premises after giving notice of his purpose and authority or in the absence of the alleged infringing defendant or expected adverse party, his agent or representative, or person in charge or in control of the premises or residing or
42 z
Intellectual Property Law working therein who is of sufficient age and discretion, may use reasonable force to gain entry to the premises or any part of the building or anything therein, to enforce the writ or to liberate himself or any person lawfully aiding him when unlawfully detained therein. SECTION 16. SEIZURE OF COMPUTER DISKS OR OTHER STORAGE DEVICES. – The seizure of a computer disk or any storage device may be executed in any of the following manner: (a) by the physical taking thereof, (b) by copying its contents in suitable device or disk provided by the applicant; or (c) by printing out the contents of the disk or device with the use of a printer. When the computer disks or storage device cannot be readily removed from the computer to which they are fitted, the sheriff may take the subject computer from the custody of the alleged infringing defendant, expected adverse party or person in charge or in control of the premises or residing or working therein. SECTION 17. SHERIFF’S RETURN. – The sheriff who executed the writ shall, within three (3) days from its enforcement, make a verified return to the court which issued the writ. The return shall contain a full statement of the proceedings under the writ and a complete inventory of the documents and articles searched, inspected or copied or seized and impounded, with copies served on the applicant, the defendant or expected adverse party and the Commissioner. If not all of the documents and articles enumerated in the order and writ were seized, the sheriff shall so report to the court and state the reasons therefor. All objections of the defendant expected adverse party or person in charge of the premises on the manner and regularity of the service of the writ shall be included by the sheriff in his return. SECTION 18. DISCHARGE OF WRIT BY THE DEFENDANT OR EXPECTED ADVERSE PARTY. – Without waiting for the return to be filed by the sheriff, the defendant, expected adverse party or the party whose property has been searched, inspected, copied or seized may file a motion with the court which issued the writ for its discharge with prayer for the return of the documents and articles seized. The writ may be discharged on any of the following grounds: (a) that the writ was improperly or irregularly issued, or excessively enforced;
(b) that the bond is insufficient; (c) that the safeguards provided in the writ have been violated by the applicant or the sheriff; or (d) that the documents and articles seized are not infringing copies or means for making the materials alleged to infringe the intellectual property right of the applicant. The writ may be discharged in a summary hearing by the court after notice to the applicant, the sheriff and the Commissioner. If the court finds that the bond is insufficient, it shall order a new bond to be filed by the applicant within a reasonable time. The discharge of the writ based on the insufficiency of the bond may only be made if the applicant fails to post the new bond within the period fixed by the court. SECTION 19. PROCEEDINGS ON RETURN. – Five (5) days after issuance of the writ, the issuing judge shall ascertain if the writ has not been served or the return has been made by the sheriff. If the writ was not served or no return was made, it shall summon the sheriff and the applicant to whom the writ was issued and require them to explain why the writ was not served or why no return has been filed as the case may be. If the return has been made, the judge shall, after notice to the applicant, the alleged infringing defendant or expected adverse party, the sheriff and the Commissioner, ascertain whether the provisions of this Rule and applicable laws have been complied with. SECTION 20. FAILURE TO FILE COMPLAINT. – The writ shall also, upon motion of the expected adverse party, be set aside and the seized documents and articles returned to the expected adverse party if no case is filed with the appropriate court or authority within thirty-‐one (31) calendar days from the date of issuance of the writ. SECTION 21. CLAIM FOR DAMAGES. – When the writ is discharged on any of the grounds provided in this Rule, or where it is found after trial that there has been no infringement or threat of infringement of an intellectual property right, the court upon motion of the alleged infringing defendant or expected adverse and after due hearing, shall order the applicant to compensate the defendant or expected adverse party upon the cash bond, surety bond or other equivalent security for any injury or damage the latter suffered by the issuance and enforcement of the writ. Should the damages exceed the amount of the bond, the applicant shall be liable for the payment of the excess.
o 43
Katrina Michelle Mancao When a complaint is already filed in court, the motion shall be filed with the same court during the trial or before appeal is perfected or before judgment becomes executory, with due notice to the applicant setting forth the facts showing the defendant’s right to damages and amount thereof. The award of damages shall be included in the judgment in the main case. Where no complaint is filed against the expected adverse party, the motion shall be filed with the court which issued the writ. In such a case, the court shall set the motion for summary hearing and immediately determine the expected adverse party’s right to damages. A judgment in favor of the applicant in its principal claim should not necessarily bar the alleged infringing defendant from recovering damages where he suffered losses by reasons of the wrongful issuance or enforcement of the writ. The damages provided f or in this section shall be independent from the damages claimed by the defendant in his counterclaim SECTION 22. JUDGMENT. – If it appears after trial that the seized documents and articles are bound to infringe the intellectual property right of the applicant or that they constitute the means the means for the production of infringing goods, the court shall order their destruction or donation to charitable, educational or religious institutions with the prohibition against bringing the same in to the channels of commerce. In the latter case, infringing trademarks or trade names found on labels, tags and other portions of the infringing materials shall be removed or defaced before the donation. In no case shall the infringing materials be returned to the defendant. If the court finds no infringement, the seized material shall be immediately returned to the defendant. SECTION 23. DIRECT FILING, PROVISIONAL DOCKETING AND DEPOSIT OF PRESCRIBED FILING FEE. – The Regional Trial Courts specially designated to try violations of intellectual property rights shall keep a distinct and separate logbook for writs of search and seizure. The application for a writ of search and seizure filed directly with the said courts shall be given a provisional docket number. The prescribed filing fee shall be deposited with the branch clerk of court and properly receipted for and transmitted to the Clerk of Court within twenty-‐four (24) hours from issuance of the order granting or denying the application for said writ. If a formal complaint is filed thereafter, the Clerk of Court may make a reassessment of the filing fee.
SECTION 24. SEPARATE LOGBOOK. – In every court, there shall be a logbook under the custody of the Clerk of Court wherein shall be docketed and entered within twenty-‐four (24) hours after the issuance or denial of the writ search and seizure, the filing of such application and other particulars thereof. All the subsequent proceedings concerning the writ of search and seizure shall be faithfully recorded in the separate logbook. SECTION 25. EFFECT OF VIOLATION. – A violation of any of the terms and conditions of the order and the writ of search and seizure or any provision of the Rule shall constitute contempt of court. SECTION 26. WRIT NOT A BAR TO OTHER MEASURES. – The availment of the writ of search and seizure under this Rule shall not prevent the applicant from resorting to other provisional measures or remedies provided in existing laws and procedural rules. SECTION 27. EFFECTIVITY. – This Rule shall take effect on February 15, 2002 after its publication in two (2) newspapers of general circulation not later than January 30, 2002.
RE: CONSOLIDATION OF INTELLECTUAL PROPERTY COURTS WITH COMMERCIAL COURTS [A.M. NO . 03-‐03-‐03-‐SC 2003-‐06-‐17] RESOLUTION WHEREAS, to implement the provisions of Section 5.2 of Republic Act No. 8799 (The Securities Regulation Code), and in the interest of a speedy and efficient administration of justice, the Supreme Court en banc, in the (a) Resolution dated 21 November 2000 (Annex 1), 4 July 2001 (Annex 1-‐a), 12 November 2002 (Annex 1-‐b), and 9 July 2002 (Annex 1-‐c), all issued in A.M. No. 00-‐11-‐03-‐SC; (b) Resolution dated 27 August 2001 in A.M. No. 01-‐5-‐298 RTC (Annex 2); and (c) Resolution dated 8 July 2002 in A.M. No. 01-‐12-‐656-‐RTC (Annex 3), resolved to designate certain branches of the Regional Trial Courts to try and decide cases formerly cognizable by the Securities and Exchange Commission; WHEREAS, pursuant to the same Resolution, sixty-‐five (65) Regional Trial Courts, distributed in all regions (NCJR and Regions 1-‐12), were designated as SEC courts ("SEC Courts"), which courts have presently a total of 812 pending SEC cases (see Annex 6, Table); WHEREAS, in A.O No. 113-‐95, dated 2 October 1995, as amended by A.O. No. 104-‐96, dated 21 October 1996, the Regional Trial Courts in the National Capital Region and Regions 3, 4, 6, 7, 9, 10 and 11, with-‐twenty seven (27)
44 z
Intellectual Property Law judges, were specially designated to try and decide cases for violations of Intellectual Property Rights (Annex 4), and to ensure the speedy disposition of cases involving violations of intellectual property rights under the Intellectual Property Code (Rep. Act No. 8293), the Supreme Court en banc, in A.M. No. 02-‐1-‐11-‐ SC, dated February 19, 2002, designated the Regional Trial Courts in Regions 1, 2, 5, 8 and 12, with a total of seven (7) judges, and Branch 24 of the Regional Trial Court of Manila with one (1) judge, as Special Intellectual Property Courts ("Special IP Courts") (Annex 5) WHEREAS, pursuant to A.M. No. 02-‐1-‐11 SC and A.O. No. 113-‐95, these Special IP Courts have a total caseload of 503 cases. Of this number 434 IP cases are pending in the NCJR (Annex 6, Table); WHEREAS, since the establishment of Special IP Courts (except for the Special IP Courts in Manila), 15 designated courts, in Regions 1, 2, 3, 4, 5, 6, 7, 8, 9 and 12 have zero (0) IP cases, and do not warrant their continued designations as Intellectual Property Courts (Annex 7, Table); WHEREAS, intellectual property cases are commercial in nature; WHEREAS, to streamline the court structure and to promote expediency and efficiency in handling such special cases, the jurisdiction to hear and decide IPC and SEC cases are best consolidated in one court; NOW, THEREFORE, the Court Resolves: 1. The Regional Courts previously designated as SEC Courts through the: (a) Resolutions of this Court dated 21 November 2000, 4 July 2001, 12 November 2002, and 9 July 2002, all issued in A.M. No. 00-‐11-‐03-‐SC, (b) Resolution dated 27 August 2001 in A.M. No. 01-‐5-‐298-‐ RTC; and (c) Resolution dated 8 July 2002 in A.M. No. 01-‐ 12-‐656-‐RTC are hereby DESIGNATED and shall be CALLED as Special Commercial Courts to try and decide cases involving violations of Intellectual Property Rights which fall within their jurisdiction and those cases formerly cognizable by the Securities and Exchange Commission; 2. The designation of Intellectual Property Courts under Administrative Order No. 113-‐95 dated 2 October 1995, as amended by Administrative Order No. 104-‐96 dated 21 October 1996 and Resolution dated 19 February 2002 in A.M. No. 02-‐1-‐11-‐SC, is hereby revoked. However, the Regional Trial Court, Branch 24, Manila is hereby designated as an additional Special Commercial Court in the City of Manila; 3. Upon the effectivity of this Resolution, all IP cases shall be transferred to the designated Special Commercial Courts except those which have undergone
the pretrial stage in civil cases or those where any of the accused has been arraigned in criminal cases which shall be retained by the court previously assigned to try them; 4. The Special Commercial Courts shall have jurisdiction over cases arising within their respective territorial jurisdiction with respect to the National Capital Judicial Region and within the respective provinces with respect to the First to Twelfth Judicial Regions. Thus, cases shall be filed in the Office of the Clerk of Court in the official station of the designated Special Commercial Court; 5. In the event of inhibition of the judge of a designated Special Commercial Court, the following guidelines shall be observed: (a) where there is only one (1) Special Commercial Court, the case shall be raffled among the other judges in the station; (b) where there are two (2) Special Commercial Courts in the station, the Executive Judge shall immediately assign the case to the other Special Commercial Court; and (c) in case of inhibition of both judges of the Special Commercial Courts, the Executive Judge shall raffle the case among the judges in the station; and 6. In order to ensure a just and equitable distribution of cases, the designated Special Commercial Courts shall continue to participate in the raffles of other cases. Provided, however, that the Executive Judge concerned shall adopt a procedure whereby every IP and SEC case assigned to a Special Commercial Court should be considered a case raffled to it and duly credited to such court. This Resolution shall take effect on 1 July 2003 and shall be published in two (2) newspapers of general circulation. 17 June 2003
20TH CENTURY FOX FILM CORP. V. CA, BARRETO, SAGULLO, LEDESMA (1988) Refresher: In a letter-‐complaint dated August 26, 1985, petitioner 20th Century Fox Film Corporation through counsel sought the NBI’s assistance in the conduct of searches and seizures in connection with the latter's anti-‐film piracy campaign. Specifically, the letter-‐complaint alleged that certain videotape outlets all over Metro Manila are engaged in the unauthorized sale and renting out of copyrighted films in videotape form which constitute a flagrant violation of PD 49. Acting on the letter-‐complaint, the NBI conducted surveillance and investigation of the outlets pinpointed by the petitioner and subsequently filed 3 applications for search warrants against the video outlets owned by the private respondents.
o 45
Katrina Michelle Mancao On September 4, 1985, the lower court issued the desired search warrants. Armed with the search warrants, the NBI accompanied by the petitioner's agents, raided the video outlets and seized the items described therein. An inventory of the items seized was made and left with the private respondents. However, the lower court subsequently lifted the search warrants issued. Issue: whether or not the judge properly lifted the search warrants he issued earlier upon the application of the National Bureau of Investigation on the basis of the complaint filed by the petitioner. Held: The presentation of the master tapes of the copyrighted films from which the pirated films were allegedly copied, was necessary for the validity of search warrants against those who have in their possession the pirated films. The petitioner's argument to the effect that the presentation of the master tapes at the time of application may not be necessary as these would be merely evidentiary in nature and not determinative of whether or not a probable cause exists to justify the issuance of the search warrants is not meritorious. The court cannot presume that duplicate or copied tapes were necessarily reproduced from master tapes that it owns. The application for search warrants was directed against video tape outlets which allegedly were engaged in the unauthorized sale and renting out of copyrighted films belonging to the petitioner pursuant to P.D. 49. The essence of a copyright infringement is the similarity or at least substantial similarity of the purported pirated works to the copyrighted work. Hence, the applicant must present to the court the copyrighted films to compare them with the purchased evidence of the video tapes allegedly pirated to determine whether the latter is an unauthorized reproduction of the former. This linkage of the copyrighted films to the pirated films must be established to satisfy the requirements of probable cause. Mere allegations as to the existence of the copyrighted films cannot serve as basis for the issuance of a search warrant.
Acting on the applications, then RTC Judge Maria Alicia M. Austria conducted a joint hearing during which she made a personal examination of the applicant and his witnesses. Finding just and probable cause for granting the application at the time, Judge Austria issued the corresponding Search Warrants numbered 95, 96, and 97. Doctrine: It is not correct to say that "the basic fact" to be proven to establish probable cause in the instant cases is not the "unauthorized transfer" of a motion picture that has been recorded but the "sale, lease, or distribution of pirated video tapes of copyrighted films." In applying for the search warrants the NBI charged violation of the entire provisions of Section 56 of P.D. No. 49 as amended by P.D. No. 1988. This included not only the sale, lease or distribution of pirated tapes but also the transfer or causing to be transferred of any sound recording or motion picture or other audio visual work. But even assuming, as appellants argue, that only the sale, lease, or distribution of pirated video tapes is involved, the fact remains that there is need to establish probable cause that the tapes being sold, leased or distributed are pirated tapes, hence the issue reverts back to the question of whether there was unauthorized transfer, directly or indirectly, of a sound recording or motion picture or other audio visual work that has been recorded.
COLUMBIA PICTURES, INC., ET AL. V. CA (1996) Refresher: Complainants thru counsel lodged a formal complaint with the NBI for violation of PD 49, as amended, and sought its assistance in their anti-‐film piracy drive. Agents of the NBI and private researchers made discreet surveillance on various video establishments in Metro Manila including Sunshine Home Video Inc. (Sunshine for brevity), owned and operated by Danilo A. Pelindario. NBI Senior Agent Lauro C. Reyes applied for a search warrant with the court a quo against Sunshine seeking the seizure, among others, of pirated video tapes of copyrighted films all of which were enumerated in a list attached to the application; and, television sets, video cassettes and/or laser disc recordings equipment and other machines and paraphernalia used or intended to be used in the unlawful exhibition, showing, reproduction, sale, lease or disposition of videograms tapes in the premises above described. Main issue: Retroactive application to the present controversy of the ruling in 20th Century Fox Film Corporation vs. Court of Appeals, et al., promulgated on August 19, 1988, that for the determination of probable
COLUMBIA PICTURES, INC., ET AL. V. CA (1994) Refresher: On 07 April 1988, the NBI, through its Agent Lauro C. Reyes, filed with the Pasig RTC 3 applications for search warrant against private respondents Tube Video Enterprises and Edward C. Cham, the Blooming Rose Tape Center and Ma. Jajorie T. Uy, and the Video Channel and Lydia Nabong, charging said respondents with violation of Section 56 of PD 49, as amended by P.D. No. 1988.
46 z
Intellectual Property Law cause to support the issuance of a search warrant in copyright infringement cases involving videograms, the production of the master tape for comparison with the allegedly pirated copies is necessary. Doctrine: Mindful as we are of the ramifications of the doctrine of stare decisis and the rudiments of fair play, it is our considered view that the 20th Century Fox ruling cannot be retroactively applied to the instant case to justify the quashal of Search Warrant No. 87-‐053. Herein petitioners’ consistent position that the order of the lower court of September 5, 1988 denying therein defendants’ motion to lift the order of search warrant was properly issued, there having been satisfactory compliance with the then prevailing standards under the law for determination of probable cause, is indeed well taken. The lower court could not possibly have expected more evidence from petitioners in their application for a search warrant other than what the law and jurisprudence, then existing and judicially accepted, required with respect to the finding of probable cause.
done after the ten-‐year period has lapsed — since 1960 and 1964 were both leap years and the case was thus filed two (2) days too late. Since this case was filed on September 3, 1965, it was filed one day too late; considering that the 730th day fell on September 2, 1965 — the year 1964 being a leap year.
LAKTAW V. PAGLINAWAN (1918) Refresher: In the complaint presented in the Court of First Instance of the City of Manila on February 20, 1915, it was alleged: (1) That the plaintiff was, according to the laws regulating literary properties, the registered owner and author of a literary work entitled Diccionario Hispano-‐ Tagalog (Spanish-‐Tagalog Dictionary) published in the City of Manila in 1889 by the printing establishment La Opinion, and a copy of which was attached to the complaint, as Exhibit A; (2) that the defendant, without the consent of the plaintiff, reproduced said literary work, improperly copied the greater part thereof in the work published by him and entitled Diccionariong Kastila-‐Tagalog (Spanish-‐Tagalog Dictionary), a copy of which was also attached to the complaint as Exhibit B; (3) that said act of the defendant, which is a violation of article 7 of the Law of January 10, 1879, on Intellectual Property, caused irreparable injuries to the plaintiff, who was surprised when, on publishing his new work entitled Diccionario Tagalog-‐Hispano (Tagalog-‐Spanish Dictionary) he learned of the fact, and (4) that the damages occasioned to the plaintiff by the publication of defendant's work amounted to $10,000. The plaintiff therefore prayed the court to order the defendant to withdraw from sale all stock of the work herein identified as Exhibit B and to pay the plaintiff the sum of $10,000, with costs. Doctrine: In the judgment appealed from, the court gives one to understand that the reproduction of another's dictionary without the owner's consent does not constitute a violation of the Law of Intellectual Property for the court's idea of a dictionary is stated in the decision itself, as follows: Dictionaries have to be made with the aid of others, and they are improved by the increase of words. What may be said of a pasture ground may be said also of a dictionary, i. e., that it should be common property for all who may desire to write a new dictionary, and the defendant has come to this pasture ground and taken whatever he needed from it in the exercise of a perfect right. Such idea is very erroneous, especially in relation to the Law of Intellectual Property. Danvilla y Collado the author of the Law of January 10, 1879, on Intellectual Property, which was discussed and approved in the
PEOPLE V. RAMOS (1978) Refresher: Soccorro Ramos, as the proprietor aid general manager of the National Book Store, as enterprise engaged in the business of publishing, selling and distributing books, was charged with copyright infringement for willfully and illegally selling and distributing spurious and pirated copies of the high school textbook, entitled General Science Today for Philippine School, First Year, by Gilam, Van Houten and Cornista, which was duly copyrighted by the Phoenix Publishing House, Inc., and was being distributed exclusively by its sister corporation, Alemar's or Sibal and Sons, Inc. The alleged act happened on July – September 1963. The criminal cases were filed on September 1965. Issue: Whether or not the extra day in the leap year, 1964 should be taken into consideration in the computation of the two-‐year period of prescription provided in Section 24 of the copyright law. Held: Resolution of this issue hinges, in turn, on whether February 28, and 29 of a leap year, 1964, should be counted as one day, as proposed by the prosecution; or as separate days, as alleged by the defense. This issue which was in 1965 still undetermined is now a settled matter. It was held in 1969 in Namarco vs. Tuazon that February 28 and 29 of a leap year should be counted as separate days in computing periods of prescription. Thus, this Court, speaking thru former Chief Justice Roberto Concepcion, held that where the prescriptive period was supposed to commence on December 21, 1955, the filing of the action on December 21, 1965, was
o 47
Katrina Michelle Mancao Spanish Cortes, in his work entitled La Propiedad Intelectual (page 362, 1st ed.) states with respect to dictionaries and in relation to article 7 of said law: The protection of the law cannot be denied to the author of a dictionary, for although words are not the property of anybody, their definitions, the example that explain their sense, and the manner of expressing their different meanings, may constitute a special work. On this point, the correctional court of the Seine held, on August 16, 1864, that a dictionary constitutes property, although some of the words therein are explained by mere definitions expressed in a few lines and sanctioned by usage, provided that the greater part of the other words contain new meanings; new meanings which evidently may only belonged to the first person who published them. In addition to what has been said, according to article 428 of the Civil Code, the author of a literary, scientific, or artistic work, has the right to exploit it and dispose thereof at will. In relation to this right, there exists the exclusive right of the author, who is the absolute owner of his own work, to produce it, according to article 2 of the Law of January 10, 1879, and consequently, nobody may reproduce it, without his permission, not even to annotate or add something to it, or to improve any edition thereof, according to article 7 of said law. Manresa, in his commentaries on article 429 of the Civil Code (vol. 3, p. 633, 3d ed.) says that the concrete statement of the right to literary properties is found in the legal doctrine according to which nobody may reproduce another person's work, without the consent of his owner, or even to annotate or add something to it or to improve any edition thereof. And on page 616 of said volume, Manresa says the following: He who writes a book, or carves a statue, or makes an invention, has the absolute right to reproduce or sell it, just as the owner of land has the absolute right to sell it or its fruits. But while the owner of land, by selling it and its fruits, perhaps fully realizes all its economic value, by receiving its benefits and utilities, which are presented, for example, by the price, on the other hand the author of a book, statue or invention, does not reap all the benefits and advantages of his own property by disposing of it, for the most important form of realizing the economic advantages of a book, statue or invention, consists in the right to reproduce it in similar or like copies, everyone of which serves to give to the person reproducing them all the conditions which the original requires in order to give the author the full enjoyment
thereof. If the author of a book, after its publication, cannot prevent its reproduction by any person who may want to reproduce it, then the property right granted him is reduced to a very insignificant thing and the effort made in the production of the book is no way rewarded. Indeed the property right recognized and protected by the Law of January 10, 1879, on Intellectual Property, would be illusory if, by reason of the fact that said law is no longer in force as a consequence of the change of sovereignty in these Islands, the author of a work, who has the exclusive right to reproduce it, could not prevent another person from so doing without his consent, and could not enforce this right through the courts of justice in order to prosecute the violator of this legal provision and the defrauder or usurper of his right, for he could not obtain the full enjoyment of the book or other work, and his property right thereto, which is recognized by law, would be reduced, as Manresa says, to an insignificant thing, if he should have no more right than that of selling his work.
HABANA, CINCO AND FERNANDO V. ROBLES AND GOODWILL TRADING CO., INC. (1999) Refresher: Petitioners are authors and copyright owners of duly issued certificates of copyright registration covering their published works, produced through their combined resources and efforts, entitled COLLEGE ENGLISH FOR TODAY (CET for brevity), Books 1 and 2, and WORKBOOK FOR COLLEGE FRESHMAN ENGLISH, Series 1. Respondent Felicidad Robles and Goodwill Trading Co., Inc. are the author/publisher and distributor/seller of another published work entitled “DEVELOPING ENGLISH PROFICIENCY” (DEP for brevity), Books 1 and 2 (1985 edition) which book was covered by copyrights issued to them. In the course of revising their published works, petitioners scouted and looked around various bookstores to check on other textbooks dealing with the same subject matter. By chance they came upon the book of respondent Robles and upon perusal of said book they were surprised to see that the book was strikingly similar to the contents, scheme of presentation, illustrations and illustrative examples in their own book, CET. After an itemized examination and comparison of the two books (CET and DEP), petitioners found that several pages of the respondent’s book are similar, if not all together a copy of petitioners’ book, which is a case of plagiarism and copyright infringement.
48 z
Intellectual Property Law Respondent Robles denied the allegations of plagiarism and copying that petitioners claimed. Respondent stressed that (1) the book DEP is the product of her independent researches, studies and experiences, and was not a copy of any existing valid copyrighted book; (2) DEP followed the scope and sequence or syllabus which are common to all English grammar writers as recommended by the Association of Philippine Colleges of Arts and Sciences (APCAS), so any similarity between the respondents book and that of the petitioners was due to the orientation of the authors to both works and standards and syllabus; and (3) the similarities may be due to the authors’ exercise of the “right to fair use of copyrighted materials, as guides.” Doctrine: We believe that respondent Robles’ act of lifting from the book of petitioners substantial portions of discussions and examples, and her failure to acknowledge the same in her book is an infringement of petitioners’ copyrights. When is there a substantial reproduction of a book? It does not necessarily require that the entire copyrighted work, or even a large portion of it, be copied. If so much is taken that the value of the original work is substantially diminished, there is an infringement of copyright and to an injurious extent, the work is appropriated. In determining the question of infringement, the amount of matter copied from the copyrighted work is an important consideration. To constitute infringement, it is not necessary that the whole or even a large portion of the work shall have been copied. If so much is taken that the value of the original is sensibly diminished, or the labors of the original author are substantially and to an injurious extent appropriated by another, that is sufficient in point of law to constitute piracy. The essence of intellectual piracy should be essayed in conceptual terms in order to underscore its gravity by an appropriate understanding thereof. Infringement of a copyright is a trespass on a private domain owned and occupied by the owner of the copyright, and, therefore, protected by law, and infringement of copyright, or piracy, which is a synonymous term in this connection, consists in the doing by any person, without the consent of the owner of the copyright, of anything the sole right to do which is conferred by statute on the owner of the copyright. The respondents’ claim that the copied portions of the book CET are also found in foreign books and other grammar books, and that the similarity between her style and that of petitioners can not be avoided since they come from the same background and orientation
may be true. However, in this jurisdiction under Sec 184 of Republic Act 8293 it is provided that: Limitations on Copyright. Notwithstanding the provisions of Chapter V, the following shall not constitute infringement of copyright: x x x x (c) The making of quotations from a published work if they are compatible with fair use and only to the extent justified for the purpose, including quotations from newspaper articles and periodicals in the form of press summaries: Provided, That the source and the name of the author, if appearing on the work, are mentioned. A copy of a piracy is an infringement of the original, and it is no defense that the pirate, in such cases, did not know whether or not he was infringing any copyright; he at least knew that what he was copying was not his, and he copied at his peril. The next question to resolve is to what extent can copying be injurious to the author of the book being copied. Is it enough that there are similarities in some sections of the books or large segments of the books are the same? In cases of infringement, copying alone is not what is prohibited. The copying must produce an “injurious effect”. Here, the injury consists in that respondent Robles lifted from petitioners’ book materials that were the result of the latter’s research work and compilation and misrepresented them as her own. She circulated the book DEP for commercial use and did not acknowledge petitioners as her source. Hence, there is a clear case of appropriation of copyrighted work for her benefit that respondent Robles committed. Petitioners’ work as authors is the product of their long and assiduous research and for another to represent it as her own is injury enough. In copyrighting books the purpose is to give protection to the intellectual product of an author. This is precisely what the law on copyright protected, under Section 184.1 (b). Quotations from a published work if they are compatible with fair use and only to the extent justified by the purpose, including quotations from newspaper articles and periodicals in the form of press summaries are allowed provided that the source and the name of the author, if appearing on the work, are mentioned. In the case at bar, the least that respondent Robles could have done was to acknowledge petitioners Habana et. al. as the source of the portions of DEP. The final product of an author’s toil is her book. To allow
o 49
Katrina Michelle Mancao another to copy the book without appropriate acknowledgment is injury enough.
the terms thereof. In his reply dated February 17, 1986, Vergara protested the unilateral action of SMI, saying it was without basis. In the same letter, he pushed for the signing of the contract for SM Cubao. Two years later, Metro Industrial Services, the company formerly contracted by Pearl and Dean to fabricate its display units, offered to construct light boxes for Shoemart’s chain of stores. SMI approved the proposal and ten (10) light boxes were subsequently fabricated by Metro Industrial for SMI. After its contract with Metro Industrial was terminated, SMI engaged the services of EYD Rainbow Advertising Corporation to make the light boxes. Some 300 units were fabricated in 1991. These were delivered on a staggered basis and installed at SM Megamall and SM City. Sometime in 1989, Pearl and Dean, received reports that exact copies of its light boxes were installed at SM City and in the fastfood section of SM Cubao. Upon investigation, Pearl and Dean found out that aside from the two (2) reported SM branches, light boxes similar to those it manufactures were also installed in two (2) other SM stores. It further discovered that defendant-‐ appellant North Edsa Marketing Inc. (NEMI), through its marketing arm, Prime Spots Marketing Services, was set up primarily to sell advertising space in lighted display units located in SMI’s different branches. Pearl and Dean noted that NEMI is a sister company of SMI. Doctrine: On copyright infringement: Petitioner’s application for a copyright certificate clearly stated that it was for a class “O” work under Section 2(O) of PD 49 (The Intellectual Property Decree) which was the statute then prevailing. Said Section 2 expressly enumerated the works subject to copyright: (O) Prints, pictorial illustrations, advertising copies, labels, tags, and box wraps; Although petitioner’s copyright certificate was entitled “Advertising Display Units” (which depicted the box-‐ type electrical devices), its claim of copyright infringement cannot be sustained. Copyright, in the strict sense of the term, is purely a statutory right. Being a mere statutory grant, the rights are limited to what the statute confers. It may be obtained and enjoyed only with respect to the subjects and by the persons, and on terms and conditions specified in the statute. Accordingly, it can cover only the works falling within the statutory enumeration or description. P & D secured its copyright under the classification class “O” work. This being so, petitioner’s copyright protection extended only to the technical drawings and
JOAQUIN AND BJ PRODUCTIONS, INC. V. DRILON, ET AL. (1999), SUPRA Refresher: Rhoda and Me and It’s a Date dating game show. Doctrine: P.D. No. 49, 2, in enumerating what are subject to copyright, refers to finished works and not to concepts. The copyright does not extend to an idea, procedure, process, system, method of operation, concept, principle, or discovery, regardless of the form in which it is described, explained, illustrated, or embodied in such work.
PEARL & DEAN (PHIL), INC. V. SHOEMART, INC. AND NORTH EDSA MARKETING, INC. (2003) Refresher: Plaintiff-‐appellant Pearl and Dean (Phil.), Inc. is a corporation engaged in the manufacture of advertising display units simply referred to as light boxes. These units utilize specially printed posters sandwiched between plastic sheets and illuminated with back lights. Pearl and Dean was able to secure a Certificate of Copyright Registration dated January 20, 1981 over these illuminated display units. The advertising light boxes were marketed under the trademark “Poster Ads”. The application for registration of the trademark was filed with the Bureau of Patents, Trademarks and Technology Transfer on June 20, 1983, but was approved only on September 12, 1988, per Registration No. 41165. From 1981 to about 1988, Pearl and Dean employed the services of Metro Industrial Services to manufacture its advertising displays. Sometime in 1985, Pearl and Dean negotiated with defendant-‐appellant Shoemart, Inc. (SMI) for the lease and installation of the light boxes in SM City North Edsa. Since SM City North Edsa was under construction at that time, SMI offered as an alternative, SM Makati and SM Cubao, to which Pearl and Dean agreed. On September 11, 1985, Pearl and Dean’s General Manager, Rodolfo Vergara, submitted for signature the contracts covering SM Cubao and SM Makati to SMI’s Advertising Promotions and Publicity Division Manager, Ramonlito Abano. Only the contract for SM Makati, however, was returned signed. On October 4, 1985, Vergara wrote Abano inquiring about the other contract and reminding him that their agreement for installation of light boxes was not only for its SM Makati branch, but also for SM Cubao. SMI did not bother to reply. Instead, in a letter dated January 14, 1986, SMI’s house counsel informed Pearl and Dean that it was rescinding the contract for SM Makati due to non-‐performance of
50 z
Intellectual Property Law not to the light box itself because the latter was not at all in the category of “prints, pictorial illustrations, advertising copies, labels, tags and box wraps.” Stated otherwise, even as we find that P & D indeed owned a valid copyright, the same could have referred only to the technical drawings within the category of “pictorial illustrations.” It could not have possibly stretched out to include the underlying light box. The strict application of the law’s enumeration in Section 2 prevents us from giving petitioner even a little leeway, that is, even if its copyright certificate was entitled “Advertising Display Units.” What the law does not include, it excludes, and for the good reason: the light box was not a literary or artistic piece which could be copyrighted under the copyright law. And no less clearly, neither could the lack of statutory authority to make the light box copyrightable be remedied by the simplistic act of entitling the copyright certificate issued by the National Library as “Advertising Display Units.” On unfair competition: By the nature of things, there can be no unfair competition under the law on copyrights although it is applicable to disputes over the use of trademarks. Even a name or phrase incapable of appropriation as a trademark or tradename may, by long and exclusive use by a business (such that the name or phrase becomes associated with the business or product in the mind of the purchasing public), be entitled to protection against unfair competition. In this case, there was no evidence that P & D’s use of “Poster Ads” was distinctive or well-‐ known. As noted by the Court of Appeals, petitioner’s expert witnesses himself had testified that “ ‘Poster Ads’ was too generic a name. So it was difficult to identify it with any company, honestly speaking.” This crucial admission by its own expert witness that “Poster Ads” could not be associated with P & D showed that, in the mind of the public, the goods and services carrying the trademark “Poster Ads” could not be distinguished from the goods and services of other entities. 12 January 2012 Infringement -‐ Violation of the rights exclusively granted by law to the copyright owner. -‐ Exception: o Fair use o Limitations to copyright Remedies: 1. Civil 2. Criminal 3. Administrative NOTE: SC issued a circular à IP case shall be filed in courts designated as commercial courts
Issue: Validity of the rule? -‐ Q: If you were a commercial court judge hearing a criminal case (1st offense) and a motion to quash was filed by, how would you uphold your jurisdiction? o Note: BP 129, jurisdiction for the 1st offense is lodged in the MTC HW Answer (Marie Estrella) Jurisdiction over the subject matter of a case is conferred by law and is determined by the allegations of the complaint, irrespective of whether the plaintiff is entitled to all or some of the claims asserted therein. (Intestate Estate of Alexander Ty v. Court of Appeals, 356 SCRA 661) What determines the jurisdiction of the court in criminal cases is the extent of the penalty which the law imposes for the misdemeanor, crime or violation charged in the complaint. (People vs. Fajardo, 49 Phil., 206, 210) With the penal provisions of the Intellectual Property Code, we are baffled with issue, “ Which court has jurisdiction.” The provision of the Code on jurisdiction is not any much help either. It simply provides that without prejudice to the provisions of Subsection 7.1(c), actions under this Act shall be cognizable by the courts with appropriate jurisdiction under existing law. The existing law conferring jurisdiction to our courts is Batas Pambansa Blg. 129 (BP 129), or the Judiciary Reorganization Act of 1980. Under Section 32 of BP 129, Municipal Trial Courts (MTC) shall have exclusive original jurisdiction over all offenses punishable with imprisonment not exceeding six (6) years irrespective of the amount of fine, and regardless of all other impossible of accessory penalties. Taking only this into consideration, we are led to conclude that with the first and second offenses, the MTC has jurisdiction. However, we also know that all IP cases are heard by the Regional Trial Court (RTC) acting as Special Commercial Court. (A.M. N0. 03-‐03-‐ 03-‐SC, JULY 01, 2003) To reconcile the seeming conflict, I think it is also useful to refer to Section 23 of BP 129 which enables the Supreme Court to designate certain branches of the Regional Trial Courts to handle exclusively criminal cases, juvenile and domestic cases, agrarian cases, urban land reform cases which do not fall under the jurisdiction of quasi-‐judicial bodies and agencies and/or such other special cases as the Supreme Court may determine in the interest of a speedy and efficient administration of
o 51
Katrina Michelle Mancao justice. With this provision, the Supreme Court, to streamline and promote expediency and efficiency, designated the Regional Trial Courts, as Special Commercial Courts to exclusively hear IP cases. I think, in the construction of laws such as the one before us, we are enjoined to read and construe them together and try to reconcile them. In this case, I think we can reconcile the IPC and BP 129 by construing that IPC is a special law and, as such, is an exception to the general rule that jurisdiction is determined by the extent of the penalty the law imposes. We can also use the interpretation that the general law yields to the special law. Lastly, taking into consideration the purpose of BP 129, which I think is to ultimately promote justice, expediency and efficiency, the designation of the RTC as Special Commercial Courts works to towards this end.
nationwide Criminal penalties -‐ No accessories/ accomplices o It is a special law -‐ Infringer and those who aids and abets = same penalty -‐ Note: the law does not have a definition of “aiding or abetting” = quite broad Issue: Can a consumer be liable for aiding or abetting? HW Answer (Katrina Mancao): The buyer of products that infringe on the copyright of another may not be held liable for aiding and abetting. Section 217.1 of the Intellectual Property Code makes any person aiding or abetting an infringement criminally liable. The law, however, does not define what actions may be considered as aiding or abetting. Unlike the other acts of infringement, this provision is vague, and thus calls for an application of the rule on statutory construction. On this issue, I propose that the provision be construed liberally in favor of the buyers. The act of aiding and abetting an infringement imposes a criminal liability upon the person charged with the offense. The first offense alone merits an imprisonment of 1 year to 3 years, plus a fine ranging from P50,000.00 to P150,000.00. I believe that the gravity the law placed on the offense, and the excessiveness of the penalty and fines imposed on it calls for the application of the legal maxim nullum crimen, nulla poena sine lege. There is no crime when there is no law punishing it. To be punishable, the act must be clearly defined by the legislature as a criminal act. The Intellectual Property Code does not explicitly define aiding or abetting. Neither does it punish mere buying or possession of an infringing material. The absence of any intention by the legislature to punish the buyers of an infringing material makes it not punishable under our jurisdiction. Moreover, punishing the act violates Article III, Section 14, paragraph 1 of the 1987 Constitution which states, “No person shall be held to answer for a criminal offense without due process of law.” The constitutional right to due process requires that the law accord people fair notice of which conduct to avoid. Otherwise, the law may be said to undermine due process in two respects: (1) in failing to accord persons fair notice of the conduct to
Civil Remedies: 1. Injunction – this is a critical provisional remedy so that the infringing activity stops 2. Damages – plaintiff is only required to prove gross sales o If gross sales was not proved – court is given leeway 3. Impounding 4. Destruction 5. Other damages Criminal remedy: -‐ Why is this more effective than civil and administrative remedies? o Because of the notoriety of criminal cases à defendant has reputation concerns; moreover, people value liberty How do you best initiate case against infringers? -‐ By obtaining a search warrant. o Timeliness o Ex parte (no notice) o No premature disclosure of the fact -‐ Search and seizure in civil actions o Ma’am: Dangerous to constitutional rights. It is very much similar to a criminal search warrant. § Differences: • Bond requirement • Independent commissioner o Where do you apply? § Metro Manila § Writs are imposable
52 z
Intellectual Property Law avoid, and (2) in giving law enforcers unbridled discretion in implementing the provisions of the law. The provision on aiding and abetting does just that. In failing to define or describe acts that may be considered as falling within the provision, it does not reasonably notify the public that mere buying of infringing material is criminally punishable. It does not, therefore, afford them with the opportunity to stop doing the act to avoid criminal liability. HW Answer (Marie Estrella): It is an elementary rule of statutory construction that penal laws are strictly construed against the State and liberally in favor of the accused. (People vs Subido, 66 SCRA 545) Penal statutes cannot be enlarged or extended by intendment, implication, or any equitable consideration. (People vs Garcia, 85 Phil. 651) Section 217 of the Intellectual property Code (IPC) of the Philippines provides: Art. 217-‐217.1. Any person infringing any right secured by provisions of Part IV of this Act or aiding or abetting such infringement shall be guilty of a crime punishable by: (a) Imprisonment of one (1) year to three (3) years plus a fine ranging from fifty thousand pesos (P50,000) to one hundred and fifty thousand pesos (P150,000) for the first offense. (b) Imprisonment of three (3) years and one (1) day to six (6) years plus a fine ranging from one hundred and fifty thousand pesos (P150,000) to five hundred thousand pesos (P500,000) for the second offense. (c) Imprisonment of six (6) years and one (1) day to nine (9) years plus a fine ranging from five hundred thousand pesos (P500,000) to one million five hundred thousand pesos (P1,500,000) for the third and subsequent offenses. (d) In all cases, subsidiary
imprisonment in cases of insolvency. 217.2. In determining the number of years of imprisonment and the amount of fine, the court shall consider the value of the infringing materials that the defendant has produced or manufactured and the damage that the copyright owner has suffered by reason of the infringement. 217.3. Any person who at the time when copyright subsists in a work has in his possession an article which he knows, or ought to know, to be an infringing copy of the work for the purpose of: (a) Selling, letting for hire, or by way of trade, offering or exposing for sale, or hire, the article; (b) Distributing the article for purpose of trade or for any other purpose to an extent that will prejudice the rights of the copyright owner in the work; or (c) Trade exhibit of the article in public, shall be guilty of an offense and shall be liable on conviction to imprisonment and fine as above mentioned. Certainly, Section 217 punishes acts that infringe or aids or abets the infringement of copyright as provided in the chapter on copyright. There is no dispute as to the definition of infringement, that is, to directly violate the rights of the copyright owner as provided in the law. Anyone violating any of the rights provided in Section 177, and other sections, of the IPC is liable for infringement. There is no question as to the culpability of direct infringers. However, there is a quandary as to what constitutes “ aiding and abetting” . There seems no established legal definition of aiding and abetting. The phrase, commonly used in criminal prosecution, connotes the commission of a crime without direct involvement. The phrase was first used during
o 53
Katrina Michelle Mancao the Nuremberg trial in pinning liability on the officers and directors of a company that manufactured the gas used by the Nazis in the
cannot interpret the law to include acts which it does not penalize. Copyright ALLOWS parallel creation. Standard only is originality. To prove infringement, you must prove ACCESS + COPYING.
5
annihilation of the Jews . In aiding and abetting, a person does not actually and directly performs the acts constitutive of the crime but however is instrumental in its commission because without him, the crime would not have been possible. In the field of Intellectual Property, we learn of the concept “ contributory infringement” . In the case of Sony Corp. vs Universal City Studios, contributory infringement was used in referring to the act of selling a machine which for the absolute purpose of committing infringing acts. In this sense, contributory infringement reflects the same premise in aiding and abetting. The person who sells the machine, although he does not do the actual reproduction of copyrighted materials, is guilty of contributory infringement. Aiding and abetting presupposes that the person charged has actual or constructive knowledge that his or her actions are likely to facilitate infringement by another, and that he
C.
Related Rights
1.
Moral Rights
ARTICLE 6BIS. Moral Rights: 1. To claim authorship; to object to certain modifications and other derogatory actions; 2. After the author's death; 3. Means of redress (1) Independently of the author's economic rights, and even after the transfer of the said rights, the author shall have the right to claim authorship of the work and to object to any distortion, mutilation or other modification of, or other derogatory action in relation to, the said work, which would be prejudicial to his honor or reputation. (2) The rights granted to the author in accordance with the preceding paragraph shall, after his death, be maintained, at least until the expiry of the economic rights, and shall be exercisable by the persons or institutions authorized by the legislation of the country where protection is claimed. However, those countries whose legislation, at the moment of their ratification of or accession to this Act, does not provide for the protection after the death of the author of all the rights set out in the preceding paragraph may provide that some of these rights may, after his death, cease to be maintained. (3) The means of redress for safeguarding the rights granted by this Article shall be governed by the legislation of the country where protection is claimed.
6
materially contributes to the infringement . In all the above definitions of aiding and abetting, and contributory negligence, there is a positive act, although not directly infringing, that contributes to the ultimate act of infringement. This positive act makes the infringement possible. Conversely, without the positive act of the contributory infringer, the infringement would not have been committed. This is not the case with the purchasers of pirated DVDs. The act of the purchasers in buying the DVDs is not necessary. The infringement would be committed even without them and their patronage. Having said this, the purchasers of pirated DVDs or other copyrighted materials cannot be held criminally liable. The IPC provisions are clear. It criminalizes infringement and the acts of aiding and abetting its commission. It does not penalize the purchase of infringing materials. We
SECTION 193. SCOPE OF MORAL RIGHTS. – The author of a work shall, independently of the economic rights in Section 177 or the grant of an assignment or license with respect to such right, have the right: 193.1. To require that the authorship of the works be attributed to him, in particular, the right that his name, as far as practicable, be indicated in a prominent way on the copies, and in connection with the public use of his work;
5
Prof. Harry Roque. “Ampatuan Victims vs Arroyo,” blog entry from http://harryroque.com/tag/aiding-‐and-‐abetting/, last accessed January 20, 2012. 6
Mark Bartholomew. “Contributory Infringers and Good Samaritans,” electronic copy from http://ssrn.com/abstract=1276286, last accessed January 20, 2012.
54 z
Intellectual Property Law 193.2. To make any alterations of his work prior to, or to withhold it from publication; 193.3. To object to any distortion, mutilation or other modification of, or other derogatory action in relation to, his work which would be prejudicial to his honor or reputation; and 193.4. To restrain the use of his name with respect to any work not of his own creation or in a distorted version of his work.
SECTION 198. TERM OF MORAL RIGHTS. – 198.1. The rights of an author under this chapter shall last during the lifetime of the author and for fifty (50) years after his death and shall not be assignable or subject to license. The person or persons to be charged with the posthumous enforcement of these rights shall be named in writing to be filed with the National Library. In default of such person or persons, such enforcement shall devolve upon either the author's heirs, and in default of the heirs, the Director of the National Library. 198.2. For purposes of this Section, "Person" shall mean any individual, partnership, corporation, association, or society. The Director of the National Library may prescribe reasonable fees to be charged for his services in the application of provisions of this Section.
SECTION 194. BREACH OF CONTRACT. – An author cannot be compelled to perform his contract to create a work or for the publication of his work already in existence. However, he may be held liable for damages for breach of such contract.
SECTION 199. ENFORCEMENT REMEDIES. –
SECTION 195. WAIVER OF MORAL RIGHTS. –
Violation of any of the rights conferred by this Chapter shall entitle those charged with their enforcement to the same rights and remedies available to a copyright owner. In addition, damages which may be availed of under the Civil Code may also be recovered. Any damage recovered after the creator's death shall be held in trust for and remitted to his heirs, and in default of the heirs, shall belong to the government.
An author may waive his rights mentioned in Section 193 by a written instrument, but no such waiver shall be valid where its effects is to permit another: 195.1. To use the name of the author, or the title of his work, or otherwise to make use of his reputation with respect to any version or adaptation of his work which, because of alterations therein, would substantially tend to injure the literary or artistic reputation of another author; or 195.2. To use the name of the author with respect to a work he did not create.
2.
Right to Proceeds in Subsequent Transfers (Droit De Suite or Follow Up Rights)
SECTION 200. SALE OR LEASE OF W ORK. – In every sale or lease of an original work of painting or sculpture or of the original manuscript of a writer or composer, subsequent to the first disposition thereof by the author, the author or his heirs shall have an inalienable right to participate in the gross proceeds of the sale or lease to the extent of five percent (5%). This right shall exist during the lifetime of the author and for fifty (50) years after his death.
SECTION 196. CONTRIBUTION TO COLLECTIVE WORK. – When an author contributes to a collective work, his right to have his contribution attributed to him is deemed waived unless he expressly reserves it.
SECTION 197. EDITING, ARRANGING AND ADAPTATION OF WORK. – In the absence of a contrary stipulation at the time an author licenses or permits another to use his work, the necessary editing, arranging or adaptation of such work, for publication, broadcast, use in a motion picture, dramatization, or mechanical or electrical reproduction in accordance with the reasonable and customary standards or requirements of the medium in which the work is to be used, shall not be deemed to contravene the author's rights secured by this chapter. Nor shall complete destruction of a work unconditionally transferred by the author be deemed to violate such rights.
SECTION 201. WORKS NOT COVERED. – The provisions of this Chapter shall not apply to prints, etchings, engravings, works of applied art, or works of similar kind wherein the author primarily derives gain from the proceeds of reproductions.
3.
Neighboring Rights
SECTION 202. DEFINITIONS. – For the purpose of this Act, the following terms shall have the following meanings:
o 55
Katrina Michelle Mancao Sections 203, 208 and 209 shall not apply where the acts referred to in those Sections are related to: 212.1. The use by a natural person exclusively for his own personal purposes; 212.2. Using short excerpts for reporting current events; 212.3. Use solely for the purpose of teaching or for scientific research; and 212.4. Fair use of the broadcast subject to the conditions under Section 185.
202.1. "PERFORMERS" are actors, singers, musicians, dancers, and other persons who act, sing, declaim, play in, interpret, or otherwise perform literary and artistic work; 202.2. "SOUND RECORDING" means the fixation of the sounds of a performance or of other sounds, or representation of sound, other than in the form of a fixation incorporated in a cinematographic or other audiovisual work; 202.3. An "AUDIOVISUAL WORK OR FIXATION" is a work that consists of a series of related images which impart the impression of motion, with or without accompanying sounds, susceptible of being made visible and, where accompanied by sounds, susceptible of being made audible; 202.4. "FIXATION" means the embodiment of sounds, or of the representations thereof, from which they can be perceived, reproduced or communicated through a device; 202.5. "PRODUCER OF A SOUND RECORDING" means the person, or the legal entity, who or which takes the initiative and has the responsibility for the first fixation of the sounds of a performance or other sounds, or the representation of sounds; 202.6. "PUBLICATION OF A FIXED PERFORMANCE OR A SOUND RECORDING" means the offering of copies of the fixed performance or the sound recording to the public, with the consent of the right holder: Provided, That copies are offered to the public in reasonable quality; 202.7. "BROADCASTING" means the transmission by wireless means for the public reception of sounds or of images or of representations thereof; such transmission by satellite is also "broadcasting" where the means for decrypting are provided to the public by the broadcasting organization or with its consent; 202.8. "BROADCASTING ORGANIZATION" shall include a natural person or a juridical entity duly authorized to engage in broadcasting; and 202.9 "COMMUNICATION TO THE PUBLIC OF A PERFORMANCE OR A SOUND RECORDING" means the transmission to the public, by any medium, otherwise than by broadcasting, of sounds of a performance or the representations of sounds fixed in a sound recording. For purposes of Section 209, "communication to the public" includes making the sounds or representations of sounds fixed in a sound recording audible to the public.
a.
Rights of Performers
SECTION 203. SCOPE OF PERFORMERS' RIGHTS. – Subject to the provisions of Section 212, performers shall enjoy the following exclusive rights: 203.1. As regards their performances, the right of authorizing: (a) The broadcasting and other communication to the public of their performance; and (b) The fixation of their unfixed performance. 203.2. The right of authorizing the direct or indirect reproduction of their performances fixed in sound recordings, in any manner or form; 203.3. Subject to the provisions of Section 206, the right of authorizing the first public distribution of the original and copies of their performance fixed in the sound recording through sale or rental or other forms of transfer of ownership; 203.4. The right of authorizing the commercial rental to the public of the original and copies of their performances fixed in sound recordings, even after distribution of them by, or pursuant to the authorization by the performer; and 203.5. The right of authorizing the making available to the public of their performances fixed in sound recordings, by wire or wireless means, in such a way that members of the public may access them from a place and time individually chosen by them.
SECTION 204. MORAL RIGHTS OF PERFORMERS. –
SECTION 212. LIMITATIONS ON RIGHTS. –
56 z
Intellectual Property Law 204.1. Independently of a performer's economic rights, the performer, shall, as regards his live aural performances or performances fixed in sound recordings, have the right to claim to be identified as the performer of his performances, except where the omission is dictated by the manner of the use of the performance, and to object to any distortion, mutilation or other modification of his performances that would be prejudicial to his reputation. 204.2. The rights granted to a performer in accordance with Subsection 203.1 shall be maintained and exercised fifty (50) years after his death, by his heirs, and in default of heirs, the government, where protection is claimed.
215.2. In case of broadcasts, the term shall be twenty (20) years from the date the broadcast took place. The extended term shall be applied only to old works with subsisting protection under the prior law.
b.
Rights of Producers of Sound Recording
SECTION 208. SCOPE OF RIGHT. – Subject to the provisions of Section 212, producers of sound recordings shall enjoy the following exclusive rights: 208.1. The right to authorize the direct or indirect reproduction of their sound recordings, in any manner or form; the placing of these reproductions in the market and the right of rental or lending; 208.2. The right to authorize the first public distribution of the original and copies of their sound recordings through sale or rental or other forms of transferring ownership; and 208.3. The right to authorize the commercial rental to the public of the original and copies of their sound recordings, even after distribution by them by or pursuant to authorization by the producer.
SECTION 205. LIMITATION ON RIGHT. – 205.1. Subject to the provisions of Section 206, once the performer has authorized the broadcasting or fixation of his performance, the provisions of Sections 203 shall have no further application. 205.2. The provisions of Section 184 and Section 185 shall apply mutatis mutandis to performers.
SECTION 206. ADDITIONAL REMUNERATION FOR SUBSEQUENT COMMUNICATIONS OR BROADCASTS. – Unless otherwise provided in the contract, in every communication to the public or broadcast of a performance subsequent to the first communication or broadcast thereof by the broadcasting organization, the performer shall be entitled to an additional remuneration equivalent to at least five percent (5%) of the original compensation he or she received for the first communication or broadcast.
SECTION 209. COMMUNICATION TO THE PUBLIC. – If a sound recording published for commercial purposes, or a reproduction of such sound recording, is used directly for broadcasting or for other communication to the public, or is publicly performed with the intention of making and enhancing profit, a single equitable remuneration for the performer or performers, and the producer of the sound recording shall be paid by the user to both the performers and the producer, who, in the absence of any agreement shall share equally.
SECTION 207. CONTRACT TERMS. – Nothing in this Chapter shall be construed to deprive performers of the right to agree by contracts on terms and conditions more favorable for them in respect of any use of their performance.
SECTION 215. TERM OF PROTECTION FOR PERFORMERS, PRODUCERS AND BROADCASTING ORGANIZATIONS. –
SECTION 210. LIMITATION OF RIGHT. – Sections 184 and 185 shall apply mutatis mutandis to the producer of sound recordings.
215.1. The rights granted to performers and producers of sound recordings under this law shall expire: (a) For performances not incorporated in recordings, fifty (50) years from the end of the year in which the performance took place; and (b) For sound or image and sound recordings and for performances incorporated therein, fifty (50) years from the end of the year in which the recording took place.
SECTION 215. TERM OF PROTECTION FOR PERFORMERS, PRODUCERS AND BROADCASTING ORGANIZATIONS. – 215.1. The rights granted to performers and producers of sound recordings under this law shall expire: (a) For performances not incorporated in recordings, fifty (50) years from the end of the year in which the performance took place; and (b) For sound or image and sound recordings and for performances incorporated
o 57
Katrina Michelle Mancao therein, fifty (50) years from the end of the year in which the recording took place.
digital direct-‐to-‐home (DTH) television via satellite to its subscribers all over the Philippines. Herein individual respondents are members of PMSI’s Board of Directors. PMSI was granted a legislative franchise under RA 86305 on May 7, 1998 and was given a Provisional Authority by the National Telecommunications Commission (NTC) on February 1, 2000 to install, operate and maintain a nationwide DTH satellite service. When it commenced operations, it offered as part of its program line-‐up ABS-‐ CBN Channels 2 and 23, NBN, Channel 4, ABC Channel 5, GMA Channel 7, RPN Channel 9, and IBC Channel 13, together with other paid premium program channels. However, on April 25, 2001, ABS-‐CBN demanded for PMSI to cease and desist from rebroadcasting Channels 2 and 23. On April 27, 2001, PMSI replied that the rebroadcasting was in accordance with the authority granted it by NTC and its obligation under NTC Memorandum Circular No. 4-‐08-‐88, Section 6.2 of which requires all cable television system operators operating in a community within Grade “A” or “B” contours to carry the television signals of the authorized television broadcast stations. ABS-‐CBN contends that PMSI’s unauthorized rebroadcasting of Channels 2 and 23 is an infringement of its broadcasting rights and copyright under the Intellectual Property Code (IP Code); that Memorandum Circular No. 04-‐08-‐88 excludes DTH satellite television operators; that the Court of Appeals’ interpretation of the must-‐carry rule violates Section 9 of Article III of the Constitution because it allows the taking of property for public use without payment of just compensation. Respondents, on the other hand, argue that PMSI’s rebroadcasting of Channels 2 and 23 is sanctioned by Memorandum Circular No. 04-‐08-‐88; that the must-‐carry rule under the Memorandum Circular is a valid exercise of police power; and that the Court of Appeals correctly dismissed CA-‐G.R. SP No. 90762 since it found no need to exercise its power of contempt. Doctrine: Section 202.7 of the IP Code defines broadcasting as “the transmission by wireless means for the public reception of sounds or of images or of representations thereof; such transmission by satellite is also ‘broadcasting’ where the means for decrypting are provided to the public by the broadcasting organization or with its consent.” On the other hand, rebroadcasting as defined in Article 3(g) of the International Convention for the Protection of Performers, Producers of Phonograms and Broadcasting Organizations, otherwise known as the 1961 Rome Convention, of which the Republic of the Philippines is a signatory, is “the simultaneous
215.2. In case of broadcasts, the term shall be twenty (20) years from the date the broadcast took place. The extended term shall be applied only to old works with subsisting protection under the prior law.
c.
Rights Broadcasting Organizations
of
SECTION 211. SCOPE OF RIGHT. – Subject to the provisions of Section 212, broadcasting organizations shall enjoy the exclusive right to carry out, authorize or prevent any of the following acts: 211.1. The rebroadcasting of their broadcasts; 211.2. The recording in any manner, including the making of films or the use of video tape, of their broadcasts for the purpose of communication to the public of television broadcasts of the same; and 211.3. The use of such records for fresh transmissions or for fresh recording.
SUBSECTION 215.2. In case of broadcasts, the term shall be twenty (20) years from the date the broadcast took place. The extended term shall be applied only to old works with subsisting protection under the prior law.
ABS-‐CBN BROADCASTING COMMUNICATION V. PHILIPPINE MULTIMEDIA SYSTEM, ET AL. (2009) Refresher: ABS-‐CBN is licensed to engage in television and radio broadcasting. It broadcasts television programs by wireless means to Metro Manila and nearby provinces, and by satellite to provincial stations through Channel 2 on Very High Frequency (VHF) and Channel 23 on Ultra High Frequency (UHF). The programs aired over Channels 2 and 23 are either produced by ABS-‐CBN or purchased from or licensed by other producers. ABS-‐CBN also owns regional television stations which pattern their programming in accordance with perceived demands of the region. Thus, television programs shown in Metro Manila and nearby provinces are not necessarily shown in other provinces. Respondent Philippine Multi-‐Media System, Inc. (PMSI) is the operator of Dream Broadcasting System. It delivers
58 z
Intellectual Property Law broadcasting by one broadcasting organization of the broadcast of another broadcasting organization.” Under the Rome Convention, rebroadcasting is “the simultaneous broadcasting by one broadcasting organization of the broadcast of another broadcasting organization.” The Working Paper prepared by the Secretariat of the Standing Committee on Copyright and Related Rights defines broadcasting organizations as “entities that take the financial and editorial responsibility for the selection and arrangement of, and investment in, the transmitted content.” Evidently, PMSI would not qualify as a broadcasting organization because it does not have the aforementioned responsibilities imposed upon broadcasting organizations, such as ABS-‐ CBN. ABS-‐CBN creates and transmits its own signals; PMSI merely carries such signals which the viewers receive in its unaltered form. PMSI does not produce, select, or determine the programs to be shown in Channels 2 and 23. Likewise, it does not pass itself off as the origin or author of such programs. Insofar as Channels 2 and 23 are concerned, PMSI merely retransmits the same in accordance with Memorandum Circular 04-‐08-‐88. With regard to its premium channels, it buys the channels from content providers and transmits on an as-‐is basis to its viewers. Clearly, PMSI does not perform the functions of a broadcasting organization; thus, it cannot be said that it is engaged in rebroadcasting Channels 2 and 23. The Director-‐General of the IPO and the Court of Appeals also correctly found that PMSI’s services are similar to a cable television system because the services it renders fall under cable “retransmission,” as described in the Working Paper, to wit: (G) Cable Retransmission 47. When a radio or television program is being broadcast, it can be retransmitted to new audiences by means of cable or wire. In the early days of cable television, it was mainly used to improve signal reception, particularly in so-‐called “shadow zones,” or to distribute the signals in large buildings or building complexes. With improvements in technology, cable operators now often receive signals from satellites before retransmitting them in an unaltered form to their subscribers through cable. 48. In principle, cable retransmission can be either simultaneous with the broadcast over-‐ the-‐air or delayed (deferred transmission) on the basis of a fixation or a reproduction of a fixation. Furthermore, they might be unaltered or altered, for example through replacement of
commercials, etc. In general, however, the term “retransmission” seems to be reserved for such transmissions which are both simultaneous and unaltered. 49. The Rome Convention does not grant rights against unauthorized cable retransmission. Without such a right, cable operators can retransmit both domestic and foreign over the air broadcasts simultaneously to their subscribers without permission from the broadcasting organizations or other rightholders and without obligation to pay remuneration. Thus, while the Rome Convention gives broadcasting organizations the right to authorize or prohibit the rebroadcasting of its broadcast, however, this protection does not extend to cable retransmission. The retransmission of ABS-‐CBN’s signals by PMSI – which functions essentially as a cable television – does not therefore constitute rebroadcasting in violation of the former’s intellectual property rights under the IP Code. It must be emphasized that the law on copyright is not absolute. The IP Code provides that: Sec. 184. Limitations on Copyright. -‐ 184.1. Notwithstanding the provisions of Chapter V, the following acts shall not constitute infringement of copyright: x x x x (h) The use made of a work by or under the direction or control of the Government, by the National Library or by educational, scientific or professional institutions where such use is in the public interest and is compatible with fair use; The carriage of ABS-‐CBN’s signals by virtue of the must-‐ carry rule in Memorandum Circular No. 04-‐08-‐88 is under the direction and control of the government though the NTC which is vested with exclusive jurisdiction to supervise, regulate and control telecommunications and broadcast services/facilities in the Philippines. The imposition of the must-‐carry rule is within the NTC’s power to promulgate rules and regulations, as public safety and interest may require, to encourage a larger and more effective use of communications, radio and television broadcasting facilities, and to maintain effective competition among private entities in these activities whenever the Commission finds it reasonably feasible. PD 285. Authorizing the Compulsory Licensing or Reprinting of Educational, Scientific or Cultural Books and Materials as a Temporary or Emergency Measure
o 59
Katrina Michelle Mancao Whenever the Prices thereof become so Exorbitant as to be Detrimental to the National Interest WHEREAS, the spiralling of prices of educational, scientific or cultural books and materials has reached very serious proportion; WHEREAS, it is in the national interest that such books and materials be made available to everybody at the least cost; and WHEREAS, this can be achieved by the compulsory licensing and reprinting of both domestic and foreign educational, scientific or cultural books and materials, as a temporary or emergency measure, whenever their prices become exorbitant; NOW, THEREFORE, I, FERDINAND E. MARCOS, President of the Philippines, by virtue of the powers vested in me by the Constitution as Commander-‐in-‐Chief of all the Armed Forces of the Philippines, and pursuant to Proclamation No. 1081 dated September 21, 1972, and General Order No. 1 dated September 22, 1972, do hereby order and decree as follows: SECTION 1. Whenever the price of any educational, scientific or cultural book, pamphlet and other written materials, whether of domestic or foreign origin, has become so exorbitant as to be detrimental to the national interest, as determined and declared by a committee composed of the Secretary of Education and Culture, such book, pamphlet or written material may be reprinted by the Government or by any private printer or printers for a limited period and only for the purpose of making the same available to the people at reasonable cost. SECTION 2. The reprinted books, pamphlets and materials shall bear proper acknowledgment of the source, authorship, copyright proprietors and past printers, if known, as well as the names and addresses of the reprinter or reprinters. If abbreviated or edited in any manner, such fact shall be stated be stated. The following inscription shall also appear on the covers thereof: "Export of this Book or material from the Philippines is punishable by law." The export of such reprints under any circumstance is hereby prohibited. SECTION 3. The reprinting of the above books, pamphlets and materials shall be subject to the condition that the reprinter shall pay, in local currency, a royalty of three per centum (3%) of the gross selling price, if so demanded by the authors, publishers or copyright proprietors concerned, whoever is legally entitled thereto: Provided, That in the case of non-‐resident authors, publishers or copyright proprietors, the payment of the royalties shall be made only to them
personally in the Philippines or their respective representative or branch offices in the Philippines. SECTION 4. Any person, natural or judicial, who shall violate the provisions of this Decree shall, upon conviction thereof, be punished by imprisonment of not less than Five Thousand Pesos nor more than two Ten Thousand Pesos. If the violation is committed by a firm, company or corporation, the manager or person in charge of the management of the business thereof shall be responsible thereof. Books and materials printed or published or exported in violation of this Decree shall be immediately confiscated and the establishment that printed or published or exported them shall forthwith be closed and its operation discontinued. SECTION 5. The Committee herein named shall promulgate rules and regulations for the implementation of this Degree. SECTION 6. This Decree shall take effect immediately, the provisions of any law, decree, executive order, treaty or executive agreement to the contrary notwithstanding. Done in the City of Manila, this 3rd day of September, in the year of Our Lord, nineteen hundred and seventy-‐ three. Remarks: Repealed by the IP Code. SECTION 239.1. All Acts and parts of Acts inconsistent herewith, more particularly Republic Act No. 165, as amended; Republic Act No. 166, as amended; and Articles 188 and 189 of the Revised Penal Code; Presidential Decree No. 49, including Presidential Decree No. 285, as amended, are hereby repealed. 26 January 2012 RELATED RIGHTS -‐ Separate and independent from copyright MORAL RIGHTS -‐ Consists of (193.1, 193.2, 193.3, 193.4) o Right to attribution o Right to integrity of work -‐ Separate from economic rights -‐ Personal to the author -‐ Retained by the author despite assignment of the copyright. -‐ Not assignable/ subject to license (198.1) o BUT the same may be waived! § Subject to exceptions -‐ Illustration: Producer obtained the copyright of the work. Such does not include the assignment of moral rights.
60 z
Intellectual Property Law What is the point of giving moral right? -‐ To protect the author à preserve how the author is represented to the public. -‐ The work is the representation of the author – how the author is represented to the world. Can you compel the author to write? -‐ No. See Section 194. Your remedy is to ask for damages. Is destruction of the work allowed? -‐ Yes, only if the destruction is COMPLETE. -‐ Incomplete destruction is not allowed. -‐ Why? In complete destruction, there would be no misrepresentation of the author to the public. The purpose of the law is to prevent misrepresentation of the author to the public. If purpose of the law ceases to exist, no need for the application of the law. Term of moral right: Lifetime of the author + 50 years after death (same as copyright) -‐ Note the persons charged with posthumous enforcement of the right is enumerated in Section 198.1 o Those named by the author in writing, o In default of the former, author’s heirs o In default of the heirs, Director of TNL FOLLOW-‐UP RIGHTS (SECTION 200) -‐ Of French origin, adopted because of the Convention -‐ Applicable to specific class only: o Painting o Sculpture o Original manuscript (of a writer or composer) -‐ Ratio: These works appreciate in value. Thus, the purpose of the law is to allow the creator to participate in the appreciation in value of their works. -‐ Basis: Gross proceeds. -‐ Problem with these rights: Enforcement (how do you enforce these?) o We have no registry system for these works; thus, no way to monitor it. NEIGHBORING RIGHTS: -‐ Granted to: o Performers o Producers of sound recording o Broadcasting corporations -‐ Performers: o Defined in Section 202.1 o Some performers are better off than others. How?
Right of all performers in general – Section 203.1 ONLY § Right of those whose performances are fixed in sound recordings – Section 203.1 + 203.2, 203.3, 203.4, 203.5 o Why not include other works? § Ma’am: It is not financially feasible to apportion all rights to all performers in audiovisual works because these are collaborative works. o But can other performers demand the rights in 203.2-‐203.5? § YES. Section 207 allows the parties to agree for better terms. The law does not provide for ceilings. Producers of sound recording and Broadcasting organization o Can they also claim copyright? Yes. o When can they claim copyright? § The work is original, they produced content itself, and such is created by their employees in the exercise of their functions. OR § Commissioned work AND the copyright is vested in them by the contract with the author of work. o Independent of copyright, they also have other rights. o Why give them copyright and other rights? § Because it is possible that they do not own the copyright of the work that they produced. Thus, if they do not own the copyrights, they can at least invoke other rights. §
-‐
Issue: Why would copyright infringement be the better right to be invoked by a broadcasting organization against a cable operator based in Lanao rather than the rights of a broadcasting organization under Chapter XIV of the IP Code? What is the LEGAL risk if the broadcasting organization applies for a search warrant on the basis of the rights of a broadcasting organization under chapter XIV? HW Answer (Di Bonilla): The search warrant is susceptible to being quashed for failure to establish probable cause
o 61
Katrina Michelle Mancao for violation of the rights of a broadcasting organization. First, as emphasized in the case of ABS-‐CBN vs. PMMI, the Rome Convention defines rebroadcasting as the “the simultaneous broadcasting by one broadcasting organization of the broadcast of another broadcasting organization.” For there to be rebroadcasting, the alleged rebroadcaster must be a broadcasting organization. However, the Lanao cable operator does not meet the definition of broadcasting organizations, which according to the foregoing case, are “entities that take the financial and editorial responsibility for the selection and arrangement of, and investment in, the transmitted content.” The Lanao cable operator does not select, arrange or invest in the content it transmits. It only transmits but does not take financial and editorial responsibility for the content. It is HBO that creates, selects, arranges and transmits its own channel to its distributors in whole -‐ including movies it has been licensed to show and its own originally created miniseries. It is part of the package it offers and paid for by its distributors. On the other hand, the Lanao cable operator only carries the signals of HBO in its unchanged form. it does not perform the functions of a broadcasting organization so that it cannot be said to be engaged in rebroadcasting of HBO's broadcasts. Second, the Lanao cable operator is not engaged in broadcasting. As a cable television, it is engaged in mere cable retransmission. The right of broadcasting organizations against rebroadcasting without their consent does not cover cable retransmission which, according to the same ABSCBN case, is a mere simultaneous and unaltered transmission by a cable television through wire or cable. Sec. 202.7 of the IP Code defines broadcasting as "the transmission by wireless means for the public reception of sounds or of images or of representations thereof; such transmission by satellite is also "broadcasting" where the means for decrypting are provided to the public by the broadcasting organization or with its consent." Thus, the Lanao cable television operator which transmits through wire or cables is not broadcasting because the latter is limited to transmission by wireless and by satellite. Also, Lanao's transmission is not for the public reception in general but only for its subscribers. Finally, since any violation of the rights of a broadcasting
organization entitles the latter to the same criminal remedies, among others, in case of infringement of copyright, the same rights must be construed strictly against the State. Thus, these rights and the corresponding criminal violations thereof must not be deemed to include acts which do not clearly and explicitly fall under them. Criminal acts must not be deemed to exist by implication.
62 z
Intellectual Property Law VI.
industrial designs, marks and works, acquired in good faith prior to the effective date of this Act.
Law on Trademarks, Tradenames and Service Marks
SECTION 241. EFFECTIVITY. –
A. Legislative History
This Act shall take effect on 1 January 1998.
REPUBLIC ACT 166
B. Definition of Trademarks
An Act to Provide for the Registration and Protection of Trade-‐marks, Trade-‐names and Service-‐marks, defining Unfair Competition and False Marking and Providing Remedies against the Same, and for Other Purposes -‐ Took effect 20 June 1947
SECTION 121.1. "MARK" means any visible sign capable of distinguishing the goods (trademark) or services (service mark) of an enterprise and shall include a stamped or marked container of goods DISTILLERIA WASHINGTON, INC. V. CA (1996)
SECTION 239. REPEALS. –
Refresher: La Tondeña Distillers, Inc. (LTDI) filed a case for manual delivery with dmages against Distilleria Washington. LTDI, under a claim of ownership, sought to seize from Distilleria Washington 18,157 empty "350 c.c. white flint bottles" bearing the blown-‐in marks of "La Tondeña Inc." and "Ginebra San Miguel." LTDI asserted that, being the owner and registrant of the bottles, it was entitled to the protection so extended by Republic Act ("R.A.") No. 623, as amended, notwithstanding its sale of the Ginebra San Miguel gin product contained in said bottles. Washington countered that R.A. No. 623, invoked by LTDI, should not apply to gin, an alcoholic beverage which is unlike that of "soda water, mineral or aerated water, ciders, milks, cream, or other lawful beverages" mentioned in the law, and that, in any case, ownership of the bottles should, considering the attendant facts and circumstances, be held lawfully transferred to the buyers upon the sale of the gin and containers at a single price. Doctrine: The case before us goes beyond just seeking to have such use stopped but it so takes on even the ownership issue as well. Parenthetically, petitioner is not here being charged with a violation of Section 2 of R.A. No. 623 or of the Trademark Law. The instant suit is one for replevin (manual delivery) where the claimant must be able to show convincingly that he is either the owner or clearly entitled to the possession of the object sought to be recovered. Replevin is a possessory action the gist of which focuses on the right of possession that, in turn, is dependent on a legal basis that, not infrequently, looks to the ownership of the object sought to be replevied. It is to be pointed out that a trademark refers to a word, name, symbol, emblem, sign or device or any combination thereof adopted and used by a merchant to identify, and distinguish from others, his goods of commerce. It is basically an intellectual creation that is susceptible to ownership and, consistently therewith,
239.1. All Acts and parts of Acts inconsistent herewith, more particularly Republic Act No. 165, as amended; Republic Act No. 166, as amended; and Articles 188 and 189 of the Revised Penal Code; Presidential Decree No. 49, including Presidential Decree No. 285, as amended, are hereby repealed. 239.2. Marks registered under Republic Act No. 166 shall remain in force but shall be deemed to have been granted under this Act and shall be due for renewal within the period provided for under this Act and, upon renewal shall be reclassified in accordance with the International Classification. Trade names and marks registered in the Supplemental Register under Republic Act No. 166 shall remain in force but shall no longer be subject to renewal. 239.3. The provisions of this Act shall apply to works in which copyright protection obtained prior to the effectivity of this Act is subsisting: Provided, That the application of this Act shall not result in the diminution of such protection.
SECTION 235.2. All applications for registration of marks or trade names pending in the Bureau of Patents, Trademarks and Technology Transfer at the effective date of this Act may be amended, if practicable to bring them under the provisions of this Act. The prosecution of such applications so amended and the grant of registrations thereon shall be proceeded with in accordance with the provisions of this Act. If such amendments are not made, the prosecution of said applications shall be proceeded with and registrations thereon granted in accordance with the Acts under which said applications were filed, and said Acts are hereby continued in force to this extent for this purpose only, notwithstanding the foregoing general repeal thereof.
SECTION 236. PRESERVATION OF EXISTING RIGHTS. – Nothing herein shall adversely affect the rights on the enforcement of rights in patents, utility models,
o 63
Katrina Michelle Mancao gives rise to its own elements of jus posidendi, jus utendi, jus fruendi, jus disponendi, and jus abutendi, along with the applicable jus lex, comprising that ownership. The incorporeal right, however, is distinct from the property in the material object subject to it. Ownership in one does not necessarily vest ownership in the other. Thus, the transfer or assignment of the intellectual property will not necessarily constitute a conveyance of the thing it covers, nor would a conveyance of the latter imply the transfer or assignment of the intellectual right. R.A. No. 623 evidently does not disallow the sale or transfer of ownership of the marked bottles or containers. In fact, the contrary is implicit in the law; thus — Sec. 5. No action shall be brought under this Act against any person to whom the registered manufacturer, bottler or seller, has transferred by way of sale, any of the containers herein referred to, but the sale of the beverage contained in the said containers shall not include the sale of the containers unless specifically so provided. Sec. 6. The provisions of this Act shall not be interpreted as prohibiting the use of bottles as containers for "sisi," "bagoong," "patis," and similar native products. Scarcely disputed are certain and specific industry practices in the sale of gin: The manufacturer sells the product in marked containers, through dealers, to the public in supermarkets, grocery shops, retail stores and other sales outlets. The buyer takes the item; he is neither required to return the bottle nor required to make a deposit to assure its return to the seller. He could return the bottle and get a refund. A number of bottles at times find their way to commercial users. It cannot be gainsaid that ownership of the containers does pass on to the consumer albeit subject to the statutory limitation on the use of the registered containers and to the trademark right of the registrant. The statement in Section 5 of R.A. 623 to the effect that the "sale of beverage contained in the said containers shall not include the sale of the containers unless specifically so provided" is not a rule of proscription. It is a rule of construction that, in keeping with the spirit and intent of the law, establishes at best a presumption (of non-‐ conveyance of the container) and which by no means can be taken to be either interdictive or conclusive in character. Upon the other hand, LTDI's sales invoice, stipulating that the "sale does not include the bottles with the blown-‐in marks of ownership of La Tondeña Distillers," cannot affect those who are not privies thereto.
C.
Functions of Trademarks
ANG V. TEODORO (1942) Refresher: Trademark involved: “Ang Tibay” Toribo Teodoro – used it for slippers, shoes, and indoor baseballs since 1910. Registered it as trademark in 1915, as trade-‐name in 1933. Ana L. Ang – registered the trademark for pants and shirts in 1932. Doctrine: We find it necessary to go into the etymology and meaning of the Tagalog words "Ang Tibay" to determine whether they are a descriptive term, i.e., whether they relate to the quality or description of the merchandise to which respondent has applied them as a trademark. The word "ang" is a definite article meaning "the" in English. It is also used as an adverb, a contraction of the word "anong" (what or how). For instance, instead of saying, "Anong ganda!" ("How beautiful!"), we ordinarily say, "Ang ganda!" Tibay is a root word from which are derived the verb magpatibay (to strenghten; the nouns pagkamatibay (strength, durability), katibayan (proof, support, strength), katibay-‐tibayan (superior strength); and the adjectives matibay (strong, durable, lasting), napakatibay (very strong), kasintibay or magkasintibay (as strong as, or of equal strength). The phrase "Ang Tibay" is an exclamation denoting administration of strength or durability. For instance, one who tries hard but fails to break an object exclaims, "Ang tibay!" (How strong!") It may also be used in a sentence thus, "Ang tibay ng sapatos mo!" (How durable your shoes are!") The phrase "ang tibay" is never used adjectively to define or describe an object. One does not say, "ang tibay sapatos" or "sapatos ang tibay" is never used adjectively to define or describe an object. One does not say, "ang tibay sapatos" or "sapatos ang tibay" to mean "durable shoes," but "matibay na sapatos" or "sapatos na matibay." From all of this we deduce that "Ang Tibay" is not a descriptive term within the meaning of the Trade-‐Mark Law but rather a fanciful or coined phrase which may properly and legally be appropriated as a trademark or trade-‐name. In this connection we do not fail to note that when the petitioner herself took the trouble and expense of securing the registration of these same words as a trademark of her products she or her attorney as well as the Director of Commerce was undoubtedly convinced that said words (Ang Tibay) were not a descriptive term and hence could be legally used and validly registered as a trade-‐mark. It seems stultifying and puerile for her now to contend otherwise, suggestive of the story of sour grapes. Counsel for the petitioner says that the function of a trademark is to
64 z
Intellectual Property Law point distinctively, either by its own meaning or by association, to the origin or ownership of the wares to which it is applied. That is correct, and we find that "Ang Tibay," as used by the respondent to designate his wares, had exactly performed that function for twenty-‐two years before the petitioner adopted it as a trade-‐mark in her own business. Ang Tibay shoes and slippers are, by association, known throughout the Philippines as products of the Ang Tibay factory owned and operated by the respondent Toribio Teodoro.
Barbizon Corporation – NY corp. = opposed the application. Escobar later assigned all her rights and interest over the trademark to petitioner Pribhdas J. Mirpuri who, under his firm name then, the "Bonito Enterprises," was the sole and exclusive distributor of Escobar's "Barbizon" products. Doctrine: In Philippine jurisprudence, the function of a trademark is to point out distinctly the origin or ownership of the goods to which it is affixed; to secure to him, who has been instrumental in bringing into the market a superior article of merchandise, the fruit of his industry and skill; to assure the public that they are procuring the genuine article; to prevent fraud and imposition; and to protect the manufacturer against substitution and sale of an inferior and different article as his product. Modern authorities on trademark law view trademarks as performing three distinct functions: (1) they indicate origin or ownership of the articles to which they are attached; (2) they guarantee that those articles come up to a certain standard of quality; and (3) they advertise the articles they symbolize. Symbols have been used to identify the ownership or origin of articles for several centuries. As early as 5,000 B.C., markings on pottery have been found by archaeologists. Cave drawings in southwestern Europe show bison with symbols on their flanks. Archaeological discoveries of ancient Greek and Roman inscriptions on sculptural works, paintings, vases, precious stones, glassworks, bricks, etc. reveal some features which are thought to be marks or symbols. These marks were affixed by the creator or maker of the article, or by public authorities as indicators for the payment of tax, for disclosing state monopoly, or devices for the settlement of accounts between an entrepreneur and his workmen. In the Middle Ages, the use of many kinds of marks on a variety of goods was commonplace. Fifteenth century England saw the compulsory use of identifying marks in certain trades. There were the baker's mark on bread, bottlemaker's marks, smith's marks, tanner's marks, watermarks on paper, etc. Every guild had its own mark and every master belonging to it had a special mark of his own. The marks were not trademarks but police marks compulsorily imposed by the sovereign to let the public know that the goods were not "foreign" goods smuggled into an area where the guild had a monopoly, as well as to aid in tracing defective work or poor craftsmanship to the artisan. For a similar reason, merchants also used merchants' marks. Merchants dealt
ETEPHA, A.G. V. DIRECTOR OF PATENTS AND WESTMONT PHARMACEUTICALS, INC. (1966) Refresher: Westmont Pharmaceuticals, Inc. – NY Corp. = sought to register “Atussin” in 1959 on its "medicinal preparation of expectorant antihistaminic, bronchodilator sedative, ascorbic acid (Vitamin C) used in the treatment of cough". Etepha, A. G. – Liechtenstein corp. = objected to the registration on the ground that “Atussin” is similar to its trademark “Pertussin” that they used on a preparation for the treatment of coughs. Doctrine: The objects of a trademark are "to point out distinctly the origin or ownership of the articles to which it is affixed, to secure to him who has been instrumental in bringing into market a superior article or merchandise the fruit of his industry and skill, and to prevent fraud and imposition." Confusion is likely between trademarks, however, only if their over-‐all presentations in any of the particulars of sound, appearance, or meaning are such as would lead the purchasing public into believing that the products to which the marks are applied emanated from the same source. In testing this issue, fixed legal rules exist — if not in harmony, certainly in abundance — but, in the final analysis, the application of these rules in any given situation necessarily reflects a matter of individual judgment largely predicated on opinion. There is, however, and can be no disagreement with the rule that the purchaser is confused, if at all, by the marks as a whole.
MIRPURI V. CA, DIRECTOR OF PATENTS AND BARBIZON CORP. (1999) Refresher: Trademark in issue = “Barbizon” Lolita Escobar (predecessor-‐in-‐interest of Mirpuiri) = sought to register “Barbizon” for use in brassieres and ladies undergarments. Escobar alleged that she had been manufacturing and selling these products under the firm name "L & BM Commercial" since March 3, 1970.
o 65
Katrina Michelle Mancao in goods acquired from many sources and the marks enabled them to identify and reclaim their goods upon recovery after shipwreck or piracy. With constant use, the mark acquired popularity and became voluntarily adopted. It was not intended to create or continue monopoly but to give the customer an index or guarantee of quality. It was in the late 18th century when the industrial revolution gave rise to mass production and distribution of consumer goods that the mark became an important instrumentality of trade and commerce. By this time, trademarks did not merely identify the goods; they also indicated the goods to be of satisfactory quality, and thereby stimulated further purchases by the consuming public. Eventually, they came to symbolize the goodwill and business reputation of the owner of the product and became a property right protected by law. The common law developed the doctrine of trademarks and tradenames "to prevent a person from palming off his goods as another's, from getting another's business or injuring his reputation by unfair means, and, from defrauding the public." Subsequently, England and the United States enacted national legislation on trademarks as part of the law regulating unfair trade. It became the right of the trademark owner to exclude others from the use of his mark, or of a confusingly similar mark where confusion resulted in diversion of trade or financial injury. At the same time, the trademark served as a warning against the imitation or faking of products to prevent the imposition of fraud upon the public. Today, the trademark is not merely a symbol of origin and goodwill; it is often the most effective agent for the actual creation and protection of goodwill. It imprints upon the public mind an anonymous and impersonal guaranty of satisfaction, creating a desire for further satisfaction. In other words, the mark actually sells the goods. The mark has become the "silent salesman," the conduit through which direct contact between the trademark owner and the consumer is assured. It has invaded popular culture in ways never anticipated that it has become a more convincing selling point than even the quality of the article to which it refers. In the last half century, the unparalleled growth of industry and the rapid development of communications technology have enabled trademarks, tradenames and other distinctive signs of a product to penetrate regions where the owner does not actually manufacture or sell the product itself. Goodwill is no longer confined to the territory of actual market penetration; it extends to zones where the marked article has been fixed in the public mind through advertising. Whether in the print, broadcast or electronic communications medium, particularly on the Internet, advertising has paved the way for growth and expansion of the product by creating and earning a reputation that crosses over borders, virtually turning the whole world into one vast marketplace.
D. How are trademarks acquired?
SECTION 122. HOW MARKS ARE ACQUIRED. – The rights in a mark shall be acquired through registration made validly in accordance with the provisions of this law. NOTE:
RA 166, SECTION 2-‐A. OWNERSHIP OF TRADE-‐MARKS, TRADE-‐NAMES AND SERVICE-‐MARKS, HOW ACQUIRED. – Anyone who lawfully produces or deals in merchandise of any kind or who engages in any lawful business, or who renders an lawful service in commerce, by actual use thereof in manufacture or trade, in business, and in the service rendered, may appropriate to his exclusive use a trade-‐mark, trade-‐name, or a service-‐mark not so appropriated by another, to distinguish his merchandise, business or service from the merchandise, business or service of others. The ownership or possession of a trade-‐mark, trade-‐name, service-‐mark, heretofore or hereafter appropriated, as in this section provided, shall be recognized and protected in the same manner and to the same extent as are other property rights known to the laws.
UNNO COMMERCIAL ENTERPRISES V. GENERAL MILLING CORPORATION (1983) Refresher: Trademark in issue: "All Montana" General Milling = used it on wheat flour in the Philippines since 1955 Unno Commercial = (prior) registrant of the mark since 1962, allegedly started using the mark since 1956 Doctrine: The right to register trademark is based on ownership. When the applicant is not the owner of the trademark being applied for, he has no right to apply for the registration of the same. Under the Trademark Law only the owner of the trademark, trade name or service mark used to distinguish his goods, business or service from the goods, business or service of others is entitled to register the same. The term owner does NOT include the importer of the goods bearing the trademark, trade name, service mark, or other mark of ownership, UNLESS such importer is actually the owner thereof in the country from which the goods are imported. A LOCAL IMPORTER, however, may make application for the registration of a foreign trademark, trade name or service mark if he is duly authorized by the actual owner of the name or other mark of ownership. A mere distributor of the merchandise covered by the TM cannot apply for the registration of the TM.
66 z
Intellectual Property Law Thus, this Court, has on several occasions ruled that where the applicant's alleged ownership is not shown in any notarial document and the applicant appears to be merely an importer or distributor of the merchandise covered by said trademark, its application cannot be granted. Ownership of a trademark is not acquired by the mere fact of registration alone. Registration merely creates a prima facie presumption of the validity of the registration, of the registrant's ownership of the trademark and of the exclusive right to the use thereof. Registration does not perfect a trademark right. As conceded itself by petitioner, evidence may be presented to overcome the presumption. Prior use by one will controvert a claim of legal appropriation, by subsequent users.
of the name in advertisements, circulars, price lists, and on signs and stationery. The Paris Convention for the Protection of Industrial Property does not automatically exclude all countries of the world which have signed it from using a tradename which happens to be used in one country. To illustrate -‐ If a taxicab or bus company in a town in the United Kingdom or India happens to use the tradename "Rapid Transportation", it does not necessarily follow that "Rapid" can no longer be registered in Uganda, Fiji, or the Philippines. Indeed, the Philippines is a signatory to this Treaty and, hence, we must honor our obligation thereunder on matters concerning internationally known or well known marks. However, this Treaty provision clearly indicated the conditions which must exist before any trademark owner can claim and be afforded rights such as the Petitioner herein seeks and those conditions are that: a) the mark must be internationally known or well known; b) the subject of the right must be a trademark, not a patent or copyright or anything else; c) the mark must be for use in the same or similar kinds of goods; and d) the person claiming must be the owner of the mark
KABUSHI KAISHA ISETAN V. IAC AND ISETANN DEPT. STORE (1991) Refresher: Trademark in issue: “Isetan” Kabushi Kaisha Isetan – Japan Corp., sought to cancel Isetann’s registration Isetann Dept. Store – domestic corp., registrant of “Isetann Dept. Store” Doctrine: A fundamental principle of Philippine Trademark Law is that actual use in commerce in the Philippines is a pre-‐ requisite to the acquisition of ownership over a trademark or a tradename. Adoption alone of a trademark would not give exclusive right thereto. Such right grows out of their actual use. Adoption is not use. One way make advertisements, issue circulars, give out price lists on certain goods; but these alone would not give exclusive right of use. For trademark is a creation of use. The underlying reason for all these is that purchasers have come to understand the mark as indicating the origin of the wares. Flowing from this is the trader's right to protection in the trade he has built up and the goodwill he has accumulated from use of the trademark. The records show that the petitioner has never conducted any business in the Philippines. It has never promoted its tradename or trademark in the Philippines. It has absolutely no business goodwill in the Philippines. It is unknown to Filipinos except the very few who may have noticed it while travelling abroad. It has never paid a single centavo of tax to the Philippine government. Under the law, it has no right to the remedy it seeks. The mere origination or adoption of a particular tradename without actual use thereof in the market is insufficient to give any exclusive right to its use, even though such adoption is publicly declared, such as by use
PHILIP MORRIS V. CA AND FORTUNE TOBACCO CORP. (1993) Refresher: Trademark in issue = “Mark” Philip Morris = Virginia corp. not doing business in the Philippines. Fortune Tobacco = domestic corp. Doctrine: A foreign corporation not doing business in the PH may have the right to sue before PH courts but existing adjective axioms require that qualifying circumstances necessary for the assertion of such right should first be affirmatively pleaded. Foreign corporation not engaged in business in the PH may maintain a cause of action for infringement primarily because of Sec. 21-‐A of TM Law when the legal standing to sue is alleged. Our municipal law on TM regarding the requirement of actual use in the PH must subordinate an international agreement inasmuch as the apparent clash is being decided by a municipal tribunal. Withal, the fact that international law has been made part of law of the land
o 67
Katrina Michelle Mancao does not by any means imply the primacy of international law over national law in the municipal sphere. Under the doctrine of incorporation as applied in most countries, rules of international law are given a standing equal, not superior to the national legislative enactments. A foreign corporation may have the personal to file a suit for infringement BUT it may not necessarily be entitled to protection due to absence of actual use of the emblem in the local market.
Developers Group of Companies, Inc. = claim ownership of the logo and the mark that they used for their restaurant business. They had the mark registered in 1983. Kuok family owns and operates a chain of hotels since 1962. Doctrine: Under the provisions of the former trademark law, RA 166, as amended, which was in effect up to December 31, 1997, hence, the law in force at the time of respondent's application for registration of trademark, the root of ownership of a trademark is actual use in commerce. Section 2 of said law requires that before a trademark can be registered, it must have been actually used in commerce and service for not less than two months in the Philippines prior to the filing of an application for its registration. Registration, without more, does not confer upon the registrant an absolute right to the registered mark. The certificate of registration is merely a prima facie proof that the registrant is the owner of the registered mark or trade name. Evidence of prior and continuous use of the mark or trade name by another can overcome the presumptive ownership of the registrant and may very well entitle the former to be declared owner in an appropriate case. Among the effects of registration of a mark, as catalogued by the Court in Lorenzana v. Macagba, are: 1. Registration in the Principal Register gives rise to a presumption of the validity of the registration, the registrant's ownership of the mark, and his right to the exclusive use thereof. x x x 2. Registration in the Principal Register is limited to the actual owner of the trademark and proceedings therein pass on the issue of ownership, which may be contested through opposition or interference proceedings, or, after registration, in a petition for cancellation. xxx Ownership of a mark or trade name may be acquired not necessarily by registration but by adoption and use in trade or commerce. As between actual use of a mark without registration, and registration of the mark without actual use thereof, the former prevails over the latter. For a rule widely accepted and firmly entrenched, because it has come down through the years, is that actual use in commerce or business is a pre-‐requisite to the acquisition of the right of ownership.
PHILIP MORRIS V. FORTUNE TOBACCO CORP. (2006) Doctrine: Foreign corporation may not successfully sue on the basis alone of their respective certificates of registration of TM, for as far as a condition to the availment of the rights and privileges vis-‐à-‐vis their TM in this country, they ought to show proof that, on top of the PH registration, their country grants substantially similar rights and privileges to Filipino citizens pursuant to Sec. 21-‐A of RA 166. Members of the Paris Union are not automatically entitled protection of TM in this country, absent actual use of the marks in the local commerce and trade. True, the PH adherence to Paris Convention effectively obligates the country to honor and enforce its provisions as regards the protection of industrial property of foreign nationals in this country. However, any protection accorded has to be made subject to the limitations of PH laws. The registration of TM cannot be deemed conclusive as to the actual use of such TM in local commerce. As it were, registration does not confer upon the registrant an absolute right to the registered mark. The certificate of registration merely constitutes prima facie evidence that the registrant is the owner of the registered mark. Evidence of non-‐usage of the mark rebuts the presumption of TM ownership, as what happened here when petitioners no less admitted not doing business in this country. Registration in the PH of TM does not ipso facto convey an absolute right or exclusive ownership thereof. The registration of TM unaccompanied by actual use thereof in the country accords the registrant only the standing to sue for infringement in the PH courts. Entitlement to protection of such TM in the country is entirely a different matter.
SHANGRI-‐LA INTERNATIONAL HOTEL MANAGEMENT, LTD. V. DEVELOPERS GROUP OF COMPANIES, INC. (2006) Refresher: Trademark in issue: “Shangri-‐la” and the “S” logo
68 z
Intellectual Property Law While the present law on trademarks has dispensed with the requirement of prior actual use at the time of registration, the law in force at the time of registration must be applied, and thereunder it was held that as a condition precedent to registration of trademark, trade name or service mark, the same must have been in actual use in the Philippines before the filing of the application for registration. Trademark is a creation of use and therefore actual use is a pre-‐requisite to exclusive ownership and its registration with the Philippine Patent Office is a mere administrative confirmation of the existence of such right. By itself, registration is not a mode of acquiring ownership. When the applicant is not the owner of the trademark being applied for, he has no right to apply for registration of the same. Registration merely creates a prima facie presumption of the validity of the registration, of the registrant's ownership of the trademark and of the exclusive right to the use thereof. Such presumption, just like the presumptive regularity in the performance of official functions, is rebuttable and must give way to evidence to the contrary.
or the name, signature, or portrait of a deceased President of the Philippines, during the life of his widow, if any, except by written consent of the widow; (d) Is identical with a registered mark belonging to a different proprietor or a mark with an earlier filing or priority date, in respect of: (i) The same goods or services, or (ii) Closely related goods or services, or (iii) If it nearly resembles such a mark as to be likely to deceive or cause confusion; (e) Is identical with, or confusingly similar to, or constitutes a translation of a mark which is considered by the competent authority of the Philippines to be well-‐ known internationally and in the Philippines, whether or not it is registered here, as being already the mark of a person other than the applicant for registration, and used for identical or similar goods or services: Provided, That in determining whether a mark is well-‐known, account shall be taken of the knowledge of the relevant sector of the public, rather than of the public at large, including knowledge in the Philippines which has been obtained as a result of the promotion of the mark; (f) Is identical with, or confusingly similar to, or constitutes a translation of a mark considered well-‐ known in accordance with the preceding paragraph, which is registered in the Philippines with respect to goods or services which are not similar to those with respect to which registration is applied for: Provided, That use of the mark in relation to those goods or services would indicate a connection between those goods or services, and the owner of the registered mark: Provided further, That the interests of the owner of the registered mark are likely to be damaged by such use; (g) Is likely to mislead the public, particularly as to the nature, quality, characteristics or geographical origin of the goods or services; (h) Consists exclusively of signs that are generic for the goods or services that they seek to identify; (i) Consists exclusively of signs or of indications that have become customary or usual to designate the goods or services in everyday language or in bona fide and established trade practice; (j) Consists exclusively of signs or of indications that may serve in trade to designate the kind, quality, quantity, intended purpose, value, geographical origin, time or production of the goods or rendering of the services, or other characteristics of the goods or services;
SHANGRI-‐LA INTERNATIONAL HOTEL MANAGEMENT, LTD. V. DEVELOPERS GROUP OF COMPANIES, INC. (2007) Court decision: The interpretation of Republic Act No. 166 in the Decision does not in any way diminish the protection afforded to valid trademark registrations made under said law. It was glaringly obvious, however, from the testimony of movant’s own witness that DGCI’s registration of the subject mark and logo was void due to the existence of bad faith and the absence of the requisite 2-‐month prior use.
E.
Standard for registrability
Distinctiveness.
F.
What marks may be registered?
SECTION 123. REGISTRABILITY. – 123.1. A mark cannot be registered if it: (a) Consists of immoral, deceptive or scandalous matter, or matter which may disparage or falsely suggest a connection with persons, living or dead, institutions, beliefs, or national symbols, or bring them into contempt or disrepute; (b) Consists of the flag or coat of arms or other insignia of the Philippines or any of its political subdivisions, or of any foreign nation, or any simulation thereof; (c) Consists of a name, portrait or signature identifying a particular living individual except by his written consent,
o 69
Katrina Michelle Mancao (k) Consists of shapes that may be necessitated by technical factors or by the nature of the goods themselves or factors that affect their intrinsic value; (l) Consists of color alone, unless defined by a given form; or (m) Is contrary to public order or morality. 123.2. As regards signs or devices mentioned in paragraphs (j), (k), and (l), nothing shall prevent the registration of any such sign or device which has become distinctive in relation to the goods for which registration is requested as a result of the use that have been made of it in commerce in the Philippines. The Office may accept as prima facie evidence that the mark has become distinctive, as used in connection with the applicant's goods or services in commerce, proof of substantially exclusive and continuous use thereof by the applicant in commerce in the Philippines for five (5) years before the date on which the claim of distinctiveness is made. 123.3. The nature of the goods to which the mark is applied will not constitute an obstacle to registration.
CIA GENERAL DE TOBACCO V. AHLAMBRA CIGAR (1916) Court descision: Where plaintiff’s registered TN is “La Flor de la Isabela” which it uses by way of advertisements, signs over the place of business, upon letter heads and in other ways to furnish the public a method of distinguishing the plaintiff’s business, and sends its manufactured products into the market with such TN upon them, the plaintiff acquire no trade-‐name rights in the single word “Isabela” where it is not claimed that the use of the word “Isabela” is an infringement of the trade-‐name “La Flor de la Isabela.” “Isabela” is a name, quality or description of the merchandise in connectin with which the word is used, or the geographical place of its production and origin, and the statute provides that such a name cannot be appropriated as a trademark or a trade-‐name.
ANG TIBAY V. TEODORO, SUPRA Court decision: "Ang Tibay" is not a descriptive term within the meaning of the Trade-‐Mark Law but rather a fanciful or coined phrase which may properly and legally be appropriated as a trademark or trade-‐name.
ETEPHA V. DIRECTOR OF PATENTS, SUPRA
ARCE SONS V. SELECTA BISCUITS (1961)
Court decision: Tussin is merely descriptive; it is generic; it furnishes to the buyer no indication of the origin of the goods; it is open for appropriation by anyone. It is accordingly barred from registration as a trademark. But while “Tussin” by itself cannot thus be used exclusively to identify one’s goods, it may properly become the subject of a trademark by “combination with another word or phrase."
Court decision: The word “Selecta” may be an ordinary or common word in the sense that it may be used or employed by anyone in promoting his business or enterprise BUT once adopted or coined in connection with one’s business as an emblem or sign to characterize its products, or as a badge of authenticity, it may acquire a secondary meaning as to be exclusively associated with its products and business. “DOCTRINE OF SECONDARY MEANING” a word or phrase originally incapable of exclusive appropriation with reference to an article on the market because geographically or otherwise descriptive, might nevertheless have been used so long and so exclusively by one producer with reference to his article that, in that trade and to that branch of the purchasing public, the word or phrase has come to mean that the article was his product. The word “Selecta” may be placed at par with the words “Ang Tibay” which this Court has considered not merely as descriptive term within the meaning of the TM Law but as “fanciful” or “coined phrase” or a trademark.
BAXTER V. ZUASUA (1905) Refresher: “Agua de Kananga” and “Kananga Superior” are toilet waters produced by two competing companies. Baxter acquired right to use TM of Agua de Kananga. Kananga is a flower. Doctrine: IF the goods in question had really nothing to do with the said flower, then it was not lawful for the plaintiffs to sell them to the public under the name of “Agua de Kananga” because the people might be deceived as to the nature of the goods, taking for “Kananga” an article which as a matter of fact, had nothing to do with the said flower. The names and denominations generally used in commerce to designate the class of goods or merchandise cannot be the subject of a trademark.
KABUSHI KAISHA ISETAN V. IAC, SUPRA Court decision: It might be pertinent at this point to stress that what is involved in this case is not so much a trademark as a tradename. Isetann Department Store, Inc. is the name
70 z
Intellectual Property Law of a store and not of product sold in various parts of the country. This case must be differentiated from cases involving products bearing such familiar names as "colgate", "Singer". "Toyota", or "Sony" where the products are marketed widely in the Philippines. There is not product with the name "Isetann" popularized with that brand name in the Philippines. Unless one goes to the store called Isetann in Manila, he would never know what the name means. Similarly, until a Filipino buyer steps inside a store called "Isetan" in Tokyo or Hongkong, that name would be completely alien to him. The records show that among Filipinos, the name cannot claim to be internationally well-‐known.
this ground alone, Fredco’s registration of the mark "Harvard" should have been disallowed. “Harvard” is a well-‐known name and mark not only in the United States but also internationally, including the Philippines. The mark "Harvard" is rated as one of the most famous marks in the world. It has been registered in at least 50 countries. It has been used and promoted extensively in numerous publications worldwide. It has established a considerable goodwill worldwide since the founding of Harvard University more than 350 years ago. It is easily recognizable as the trade name and mark of Harvard University of Cambridge, Massachusetts, U.S.A., internationally known as one of the leading educational institutions in the world. As such, even before Harvard University applied for registration of the mark "Harvard" in the Philippines, the mark was already protected under Article 6bis and Article 8 of the Paris Convention. Again, even without applying the Paris Convention, Harvard University can invoke Section 4(a) of R.A. No. 166 which prohibits the registration of a mark "which may disparage or falsely suggest a connection with persons, living or dead, institutions, beliefs x x x." 02 February 2012 DEFINITION OF “TRADEMARK” -‐ Scent and aural cannot be trademarks because Phil law on visible signs FUNCTIONS OF A TRADEMARK -‐ Traditional purpose – Point out the source of goods or services. o This began when artisans decided to put mark on their goods to indicate who made the goods, where the good were made. -‐ Why should trademarks be recognized and protected? o To encourage the enterprise to invest in the creation of durable goods through the guaranty that if they continue to bring goods in such market, their identity is protected, their market is secured because no other person can appropriate their marks. If the enterprise invest in their services and it acquires goodwill, the goodwill will be preserved because no other person allowed to make use of its mark that would undermine your integrity in the market. o In turn, the public will benefit from the creation of superior goods. -‐ Current purpose changed – Trademark has evolved with centuries of use. Because the marketplace has become global, trademarks has evolved from mere source indicators, to
ASIA BREWERY V. CA (1933) Court decision: The fact that the words “pale pilsen” are part of ABI’s trademark does not constitute an infringement of SMC’s trademark (San Miguel Pale Pilsen) for “pale pilsen” are generic words descriptive of the color (pale), of a strong type of beer (pilsen) which is a light bohemian beer with a strong hops flavor that originated in the City of Pilsen in Czechoslovakia and became famous in the Middle Ages. “Pilsen” is primarily geographically descriptive word, hence, non-‐registrabel and not appropriable by any beer manufacturer.
EMERALD GARMENT MANUFACTURING V. CA AND H.D. LEE CO., INC. (1995) Refresher: H.D. Lee Co. = a Delaware corp., owner of the “Lee” trademark Emerald Garment Manufacturing = domestic corp., registrant of “Stylistic Mr. Lee” trademark used on skirts, jeans, blouses, socks, briefs, jackets, jogging suits, dresses, shorts, shirts and lingerie under Class 25. Court decision: "LEE" is primarily a surname. Private respondent cannot, therefore, acquire exclusive ownership over and singular use of said term. It has been held that a personal name or surname may not be monopolized as a trademark or tradename as against others of the same name or surname. For in the absence of contract, fraud, or estoppel, any man may use his name or surname in all legitimate ways. Thus, "Wellington" is a surname, and its first user has no cause of action against the junior user of "Wellington" as it is incapable of exclusive appropriation.
FREDCO MANUFACTURING CORP. V. PRESIDENT AND FELLOWS OF HARVARD COLLEGE (2011) Court decision: Fredco’s use of the mark “Harvard,” coupled with its claimed origin in Cambridge, Massachusetts, obviously suggests a false connection with Harvard University. On
o 71
Katrina Michelle Mancao serving very critical functions especially in a crowded marketplace. o Associative quality for luxury brands – because of a particular trademark, consumers would choose to buy the product because they want to belong to a particular class. § Mark is associated with a particular class or lifestyle. People wearing a brand (if genuine) belong to a special class of people. § When a mark creates a certain kind of image, it can make consumers want it. § It creates a segment of the population. Without the consumers realizing it, they buy not just the product but also the lifestyle. o Bearer of information – it is the heart of a business. § It bears information that such thing was created by the manufacturer. o It is the hear of the business. § It has a direct effect on the company’s profits; whether the company that will flourish or become bankrupt. § Strength of the mark dictates the premium of the price. § Trademark has become an asset of the company. It is hard to distinguish between it and a business. If the company loses its right to a trademark, it also loses the business. § Note: As an asset, you can sell the trademark independently of the business. • Ex: Volkswagen and BMW bid for the trademark of Rolls-‐Royce. Volkswagen won the bidding. Why did they want to acquire the trademark? Because it is very difficult to build a brand. Unfortunately, the car companies did not do a due diligence investigation. They did not realize that the trademark did not belong to the car company. The trademark belonged to Aero Engine. What Volkswagen bought was the factory. Thus, it can build Rolls-‐
Royce but not call it as such. Later, BMW acquired the mark from Aero Engine. BMW gave Volkswagen a year to use its mark.
How are marks acquired? -‐ Now (under the IPC): Through registration. -‐ Previously: Through actual use. -‐ Because of this shift/change in the basis of ownership over trademarks, it presents a host of interesting transitional problems: o Under old law, the owner of the mark was prior user in Philippine commerce (Unno Commercial Enterprises vs. General Milling). First user/entity who used it in commerce owns the mark. o NOTE: Under the old law, local applicants were required to use the mark for at least 2 months prior to the filing of the application. If you want to file, you must have used the mark 2 months prior to filing of application. § It’s a very risky undertaking à To own the mark, one has to build the business and spend for investments, put up store, incorporate, hire accountant and signage maker, and do your marketing materials. Since one is required to have used mark for at least 2 month prior to the filing of application, for 2 months, you’re taking a risk that somebody else might use your mark. In the meantime, you already have spent your money and sunk in the cost. § That was the requirement because under the law, actual use was the basis of ownership. o Why change the law? § Because the Philippines became a party to the Paris Convention. It required us to adhere to the “national treatment” principle. Kabushi Kaisha Isetan v IAC -‐ Case between a local registrant and a foreign non-‐user. -‐ Local user prevailed because the old law based ownership of the trademark on actual use. Since there was no actual use for Kabushi Kaisha in the PH, there was no goodwill to be protected. -‐ BUT the Court recognized an exception à Under the Paris Convention, States are
72 z
Intellectual Property Law required to recognize well-‐known trademark. o Exception has following elements: 1. the mark must be internationally known or well known; 2. the subject of the mark must be a trademark; 3. the mark must be for use in the same or similar kinds of goods; 4. the person claiming must be the owner of the mark. Kabushi Kaisha was not able to prove the
ONLY EXCEPTION: internationally well-‐known mark The only way a prior registrant user will not prevail is if use was subsequent to the filing of the application. o
•
NOW à Registration is the basis of ownership. -‐ Under IPC, are you required to make use of mark prior to filing of application? No. Standard of registerability: Distinctiveness. WHAT MARKS MAY NOT BE REGISTERED? -‐ Basically, Section 123 IPC is an enumeration of marks that are not distinctive; i.e. Incapable of pointing our source of goods or services. Deceptive -‐ Cannot be distinctive because suggests a false connection. -‐ E.g. Jose Rizal Shoes Flag/ Coat of arms -‐ Paris Convention makes obligation not to allow registration of flags or national symbols. They are outside commerce of men. o Everyone can make use of the mark to indicate origin but it cannot be a trademark. It cannot be appropriated by manufacturer at the expense of the rest. -‐ BUT: There are certain countries that have acquired a certain reputation for producing goods of superior quality. Companies based here could use the country’s flag, not as trademark, but as an indicia of the country of origin of the goods. o E.g. Switzerland. Swiss have strong laws about who can use the mark. To be able to use the flag, the Swiss company must be able to meet a certain quality, and undergo quality control. The Swiss government wants to ensure that those who use Swiss flags would maintain the standard so that their reputation of producing superior goods will be maintained. Name, portrait or signature identifying a particular living individual -‐ If a famous doctor’s name is being used in relation to cosmetic products, will the trademark be allowed? o Yes, if done with the consent of the doctor. -‐ If the person has died, what will you do to make use of the name, etc? o Get heirs’ consent.
-‐ RECAP: GR: First user (Established in the law and Unno) EXCEPTION: First user who did not use in PH commerce EXCEPTION TO THE EXCEPTION: First user, unregistered and not used in PH Commerce AND internationally well-‐ known (e.g. Kabushi Kaisha) – Applying Paris Convention Philip Morris cases -‐ Between a registrant non-‐user vs. non-‐ registrant user of the mark in the Philippine commerce? o The user prevails because ownership of trademark is on the basis of use in the Philippine commerce. While Philip Morris was registrant, it did not make use of the same. Shangrila v. DGCI -‐ Case between a registrant user (the restaurant) and the hotel that wants to come in. -‐ The Hotel prevailed and restaurant’s registration was cancelled. Why? o Evidence showed that there was no actual use prior to the filing of application. There was a violation of the requirements the RA 166, thus the registration was not a valid registration. o You may be the registrant and prior user BUT if you failed to comply with requirement of actual use before filing of application, you will lose the registration because the registration will be invalid. The foregoing were cases decided under a different law (RA 166) – PRIOR to the passage of the IP Code. Thus: • Prior use was the basis of ownership of the trademark. • Prior user in Philippine commerce shall own the trademark. • Prior user in Philippine commerce will prevail as against a registrant non-‐user.
o 73
Katrina Michelle Mancao Identical with a registered mark, or a mark with an earlier filing/priority date -‐ Very important because this indicated the shift. -‐ “Likely to deceive/cause confusion” means you don’t even have to prove actual confusion for as long as there is likelihood of confusion. -‐ This shows to you that our TM regime has shifted to a first-‐to-‐file system because a mere application with an earlier filing date can prevent subsequent application for the same or confusingly similar mark subsequently filed. o It’s very critical now to file an application ASAP because owner of mark with earlier filing date would prevail as against the owner of a mark with a later filing date. Time is critical and essential. Well-‐known mark (123.1e and 123.1f) -‐ E and F both deal with WELL-‐KNOWN MARK locally and internationally. -‐ What’s the difference between the two? o Under E: speaks about well-‐known mark WON registered o Under F: speaks about registered well-‐known mark. -‐ What’s the difference in terms of scope of protection? o E: protected only against use for identical or similar goods or services. o F: protected also as against use for goods or services not similar to those with respect to which registration is applied for. -‐ The foregoing illustrated why registration in the Philippines of a well-‐known mark is still practical à scope of protection after registration would be broader: o They could oppose the filing of applications for the same or confusingly similar mark even for goods/services that are dissimilar if you had registered in the PH. -‐ In short, it appears that under the IPC, only registered internationally well-‐known mark in the Philippines can oppose on the ground of dilution. o On the other hand, an unregistered well-‐known mark cannot raise dilution. The scope of protection for an unregistered well-‐known mark is limited to similar or identical goods or services. Can mislead the public -‐ Because it’s misleading, therefore, it cannot be descriptive.
Generic -‐ E.g. shoes, bags, damit, blusa, pantalon, radyo. -‐ They refer to the goods/services themselves. Therefore, nobody can monopolize or appropriate them, it cannot be acquired by anyone. o If you allow somebody to exclusively use them, it would be to the detriment of everyone else. Imagine, you can no longer sell bags because bags have become a trademark! Become customary or usual to designate the goods or services in everyday language -‐ This refers to the phenomena called “genericide” à marks that used to be distinctive lose their distinctiveness because of popularity of the mark. -‐ The downside of being a strong mark that people stop referring to the generic term of good/services but actually use the trademark as generic term. -‐ E.g. “Kodak” in Ph; Xerox; Sunkist -‐ Xerox and Kodak are arbitrary trademarks; i.e. they were just coined by the manufacturers/ they never existed before. o In the realm of trademarks, arbitrary marks should be entitled to the strongest protection. o The Philippines seem to defy that. The more arbitrary the trademark is, the more we think we have license to use them. (ex. Colgate, Frigidaire) -‐ Other trademarks that have fallen in the public domain – escalator; elevator; nylon; thermos o Owners of these didn’t work hard enough to prevent the falling into public domain of their trademarks. -‐ What’s the best evidence to prove that the mark has become generic, a victim of genericide, has lost its distinctiveness and therefore, may be cancelled? o Existence of the word in the dictionary. § Existence in the dictionary means it forms part of the lexicon. § Companies spend a great deal for service watch to check US dictionaries to make sure that well-‐known marks have not been incorporated in the dictionary Characteristics of the goods or services -‐ Also known as “descriptive marks” -‐ Being merely descriptive, they do not help in pointing out the source or origin of the good.
74 z
Intellectual Property Law Contrary to public order or morality RECAP: GR: The foregoing cannot be registered because they do not meet the standards of distinctiveness. EXCEPTION: Certain marks may be registered provided they acquired secondary meaning. What are these marks that may be registered? 1. Descriptive marks 2. Shapes 3. Colors While not registrable at first instance because they are deemed non-‐distinctive, they may be registered provided they have acquired secondary meaning. What is “SECONDARY MEANING”? -‐ Originally not distinctive but through its use by relevant public has become distinctive. -‐ Under the old law (RA 166) à proprietor can use the supplemental register to support the claim that the term has acquired secondary meaning. o Now, there is no supplemental register. Propritor would have to convince IPO that while your mark may be descriptive, shapes, colors, he has exclusively and continuously used it for at 5 years à the mark has acquired a secondary meaning that would now entitle him to register. Choice of trademark is very critical to any business. If the proprietor makes a mistake and uses a non-‐registrable TM, there is really an economic impact on the business. -‐ Proprietor should be careful not to use generic terms as the same cannot be registered. If the proprietor has acquired goodwill for a generic term, he cannot prevent other people from using it à generic term is always in the public domain. o “Kamiseta” and “Bayo” (which is Ilonggo for clothes) are NOT registrable under the IPC. § They are, however, registrable under the old law.
G. Application for registration
1.
Application
SECTION 3. INTERNATIONAL RECIPROCITY. –
CONVENTIONS
AND
Any person who is a national or who is domiciled or has a real and effective industrial establishment in a country which is a party to any convention, treaty or agreement relating to intellectual property rights or the repression of unfair competition, to which the Philippines is also a party, or extends reciprocal rights to nationals of the Philippines by law, shall be entitled to benefits to the extent necessary to give effect to any provision of such convention, treaty or reciprocal law, IN ADDITION to the rights to which any owner of an intellectual property right is otherwise entitled by this Act.
SECTION 124. REQUIREMENTS OF APPLICATION. – 124.1. The application for the registration of the mark shall be in Filipino or in English and shall contain the following: (a) A request for registration; (b) The name and address of the applicant; (c) The name of a State of which the applicant is a national or where he has domicile; and the name of a State in which the applicant has a real and effective industrial or commercial establishment, if any; (d) Where the applicant is a juridical entity, the law under which it is organized and existing; (e) The appointment of an agent or representative, if the applicant is not domiciled in the Philippines; (f) Where the applicant claims the priority of an earlier application, an indication of: i) The name of the State with whose national office the earlier application was filed or if filed with an office other than a national office, the name of that office, ii) The date on which the earlier application was filed, and iii) Where available, the application number of the earlier application; (g) Where the applicant claims color as a distinctive feature of the mark, a statement to that effect as well as the name or names of the color or colors claimed and an indication, in respect of each color, of the principal parts of the mark which are in that color;
o 75
Katrina Michelle Mancao (h) Where the mark is a three-‐dimensional mark, a statement to that effect; (i) One or more reproductions of the mark, as prescribed in the Regulations; (j) A transliteration or translation of the mark or of some parts of the mark, as prescribed in the Regulations; (k) The names of the goods or services for which the registration is sought, grouped according to the classes of the Nice Classification, together with the number of the class of the said Classification to which each group of goods or services belongs; and (l) A signature by, or other self-‐identification of, the applicant or his representative. 124.2. The applicant or the registrant shall file a declaration of actual use of the mark with evidence to that effect, as prescribed by the Regulations within three (3) years from the filing date of the application. Otherwise, the application shall be refused or the mark shall be removed from the Register by the Director. 124.3. One (1) application may relate to several goods and/or services, whether they belong to one (1) class or to several classes of the Nice Classification. 124.4. If during the examination of the application, the Office finds factual basis to reasonably doubt the veracity of any indication or element in the application, it may require the applicant to submit sufficient evidence to remove the doubt.
became distinctive of the applicant's or owner's goods, business or services.
SECTION 128. SINGLE REGISTRATION FOR GOODS AND/OR SERVICES. – Where goods and/or services belonging to several classes of the Nice Classification have been included in one (1) application, such an application shall result in one registration.
SECTION 130. SIGNATURE AND OTHER MEANS OF SELF-‐ IDENTIFICATION. – 130.1. Where a signature is required, the Office shall accept: (a) A hand-‐written signature; or (b) The use of other forms of signature, such as a printed or stamped signature, or the use of a seal instead of a hand-‐written signature: Provided, That where a seal is used, it should be accompanied by an indication in letters of the name of the signatory. 130.2. The Office shall accept communications to it by telecopier, or by electronic means subject to the conditions or requirements that will be prescribed by the Regulations. When communications are made by telefacsimile, the reproduction of the signature, or the reproduction of the seal together with, where required, the indication in letters of the name of the natural person whose seal is used, appears. The original communications must be received by the Office within thirty (30) days from date of receipt of the telefacsimile. 130.3. No attestation, notarization, authentication, legalization or other certification of any signature or other means of self-‐identification referred to in the preceding paragraphs, will be required, except, where the signature concerns the surrender of a registration.
SECTION 125. REPRESENTATION; ADDRESS FOR SERVICE. – If the applicant is not domiciled or has no real and effective commercial establishment in the Philippines, he shall designate by a written document filed in the Office, the name and address of a Philippine resident who may be served notices or process in proceedings affecting the mark. Such notices or services may be served upon the person so designated by leaving a copy thereof at the address specified in the last designation filed. If the person so designated cannot be found at the address given in the last designation, such notice or process may be served upon the Director.
SECTION 131. PRIORITY RIGHT. – 131.1. An application for registration of a mark filed in the Philippines by a person referred to in Section 3, and who previously duly filed an application for registration of the same mark in one of those countries, shall be considered as filed as of the day the application was first filed in the foreign country. 131.2. No registration of a mark in the Philippines by a person described in this section shall be granted until such mark has been registered in the country of origin of the applicant. 131.3. Nothing in this section shall entitle the owner of a registration granted under this section to sue for acts committed prior to the date on which his mark was
SECTION 126. DISCLAIMERS. – The Office may allow or require the applicant to disclaim an unregistrable component of an otherwise registrable mark but such disclaimer shall not prejudice or affect the applicant's or owner's rights then existing or thereafter arising in the disclaimed matter, nor such shall disclaimer prejudice or affect the applicant's or owner's right on another application of later date if the disclaimed matter
76 z
Intellectual Property Law registered in this country: Provided, That, notwithstanding the foregoing, the owner of a well-‐ known mark as defined in Section 123.1(e) of this Act, that is not registered in the Philippines, may, against an identical or confusingly similar mark, oppose its registration, or petition the cancellation of its registration or sue for unfair competition, without prejudice to availing himself of other remedies provided for under the law. 131.4. In like manner and subject to the same conditions and requirements, the right provided in this section may be based upon a subsequent regularly filed application in the same foreign country: Provided, That any foreign application filed prior to such subsequent application has been withdrawn, abandoned, or otherwise disposed of, without having been laid open to public inspection and without leaving any rights outstanding, and has not served, nor thereafter shall serve, as a basis for claiming a right of priority. SECTION 239.2. Marks registered under Republic Act No. 166 shall remain in force but shall be deemed to have been granted under this Act and shall be due for renewal within the period provided for under this Act and, upon renewal shall be reclassified in accordance with the International Classification. Trade names and marks registered in the Supplemental Register under Republic Act No. 166 shall remain in force but shall no longer be subject to renewal.
2.
SECTION 132. APPLICATION NUMBER AND FILING DATE. – 132.1. The Office shall examine whether the application satisfies the requirements for the grant of a filing date as provided in Section 127 and Regulations relating thereto. If the application does not satisfy the filing requirements, the Office shall notify the applicant who shall within a period fixed by the Regulations complete or correct the application as required, otherwise, the application shall be considered withdrawn. 132.2. Once an application meets the filing requirements of Section 127, it shall be numbered in the sequential order, and the applicant shall be informed of the application number and the filing date of the application will be deemed to have been abandoned. What’s the difference between “filing date” and a “priority date”? FILING DATE -‐ date you actually filed the application, PROVIDED you met all the minimum requirements under Sec. 127. The IPO enforces this strictly, if inadvertently the duty officer of the day of IPO gave you a filing date but it turns out that you did not submit the list of goods and services, when the application is referred to trademark examiner, the trademark examiner will examine and if he discovers that you did not comply with the minimum requirement, the TE will revoke your filing date. Pag nirevoke niya, he will send communication that you forgot one requirement. Only when you submit list of goods and services that you will be given another filing date. Pati application number mo mawawala. In the meantime, if somebody else files and application for same or confusingly similar mark, lagot ka na. Because your filing date is only when you complied with the minimum requirement. It takes about 6 to 8 months before it is assigned to an examiner. So there is a high probability that within those 8 months somebody else filed an application and because that application meets the minimum requirement, it would have an earlier filing date. As between you with revoked filing date and somebody earlier filing date, the latter will prevail. Yours has been revoked, so critical. Filing date is date you actually filed provided you met the minimum requirements under 127. PRIORITY DATE -‐ date when you filed a foreign application for the same mark in another country, a Paris Convention country or TRIPS country. Then you filed the application in the Ph within 6 months, you can claim the earlier foreign filing date as your filing date in the Ph. So the law will deem your application filed as of that date of filing of the foreign application. BUT in order for you to avail of foreign priority date, you must have filed the Philippine application within 6 months from the time you filed the foreign application.
Assignment of Application Number and Filing Date
SECTION 127. FILING DATE. – 127.1. Requirements. – The filing date of an application shall be the date on which the Office received the following indications and elements in English or Filipino: (a) An express or implicit indication that the registration of a mark is sought; (b) The identity of the applicant; (c) Indications sufficient to contact the applicant or his representative, if any; (d) A reproduction of the mark whose registration is sought; and (e) The list of the goods or services for which the registration is sought. 127.2. No filing date shall be accorded until the required fee is paid.
o 77
Katrina Michelle Mancao the mark as defined in Section 121 is registrable under Section 123. 133.2. Where the Office finds that the conditions referred to in Subsection 133.1 are fulfilled, it shall upon payment of the prescribed fee, forthwith cause the application, as filed, to be published in the prescribed manner. 133.3. If after the examination, the applicant is not entitled to registration for any reason, the Office shall advise the applicant thereof and the reasons therefor. The applicant shall have a period of four (4) months in which to reply or amend his application, which shall then be re-‐examined. The Regulations shall determine the procedure for the re-‐examination or revival of an application as well as the appeal to the Director of Trademarks from any final action by the Examiner. 133.4. An abandoned application may be revived as a pending application within three (3) months from the date of abandonment, upon good cause shown and the payment of the required fee. 133.5. The final decision of refusal of the Director of Trademarks shall be appealable to the Director General in accordance with the procedure fixed by the Regulations.
NB Under IP Code Sec 131, there is no 6 months limitation. This is an error on the part of the drafters of IPC because priority date is actually a convention obligation (Paris Convention). Under which there is 6 mos limitation to avail of priority date. Since every other country in the world subjects 6-‐mo period, and because of reciprocity, why would we adapt a longer period to claim priority date? That seems to be the effect because of 131 which failed to reproduce the 6 months limitation under the Paris Convention. -‐ But what the IPO did was to issue an administrative circular that clarifies priority right can only be claimed if the Philippine application is filed within 6 months. -‐ Basis for that: a. Follow basic principles of reciprocity: Why should we give foreign countries who only limit to 6mos the right to claim priority date more rights? Why construe IPC to be limitless? That would be contrary to national interest. b. Convention obligation: The domestic law provision that implements the international treaty obligation should be read in conjunction with international treaty. So only within 6 months. NOTE: Priority dates are important because as between an earlier filing date and an earlier priority date, the priority date prevails. E.g. Jances files application in the PH on 01 Mar 2012, to comply with minimum requirements of Sec. 127. Earla filed application for the same mark or similar goods/services and she filed it on 12 Mar 2012 BUT she earlier filed in SG an application dated 15 Feb 2012. When Earla made a claim on the basis of that foreign application, a priority date of 15 Feb 2012, who would prevail? Jances or Earla? Earla! Although filing date is later, she has a claim of priority. The law will deem Earla’s application to have been filed as of 15 Feb 2012 (date she filed SG application) for as long as she complies with the requirements for a claim of priority, namely, • Within 6 months from application • Certified copy of the application and English translation • Submission of foreign registration
SECTION 126. DISCLAIMERS. – The Office may allow or require the applicant to disclaim an unregistrable component of an otherwise registrable mark but such disclaimer shall not prejudice or affect the applicant's or owner's rights then existing or thereafter arising in the disclaimed matter, nor such shall disclaimer prejudice or affect the applicant's or owner's right on another application of later date if the disclaimed matter became distinctive of the applicant's or owner's goods, business or services.
SECTION 129. DIVISION OF APPLICATION. – Any application referring to several goods or services, hereafter referred to as the "initial application," may be divided by the applicant into two (2) or more applications, hereafter referred to as the "divisional applications," by distributing among the latter the goods or services referred to in the initial application. The divisional applications shall preserve the filing date of the initial application or the benefit of the right of priority. Examination consists of a determination of compliance with the formal and substantive (WON the mark is registrable) requirements of registration.
3.
Examination
SECTION 133. EXAMINATION AND PUBLICATION. – 133.1. Once the application meets the filing requirements of Section 127, the Office shall examine whether the application meets the requirements of Section 124 and
78 z
Intellectual Property Law 4.
Evidence received by the Philippine Patent Office showed that Bata shoes made by Gerbec and Hrdina of Czechoslovakia were sold in the Philippines prior to World War II. Some shoes made by Bata of Canada were perhaps also sold in the Philippines until 1948. However, the trademark BATA was never registered in the Philippines by any foreign entity. Under the circumstances, it was concluded that opposer has, to all intents and purposes, technically abandoned its trademark BATA in the Philippines. Upon the other hand, the Philippine Patent Office found that New Olympian Rubber Products Co., Inc. has overwhelmingly and convincingly established its right to the trademark BATA and consequently, its use and registration in its favor. There is no gainsaying the truth that the respondent has spent a considerable amount of money and effort in popularizing the trademark BATA for shoes in the Philippines through the advertising media since it was lawfully used in commerce on July 1, 1970. It can not be denied, therefore, that it is the respondent-‐applicant's expense that created the enormous goodwill of the trademark BATA in the Philippines and not the opposer as claimed in its opposition to the registration of the BATA mark by the respondent. PPO dismissed the opposition and ordered the registration of Bata in favor of New Olympian Rubber Products Co. Doctrine: The Court was satisfied from the evidence that any slight goodwill generated by the Czechoslovakian product during the Commonwealth years was completely abandoned and lost in the more than 35 years that have passed since the liberation of Manila from the Japanese troops. New Olympian has reproduced excerpts from the testimonies of the opposer-‐appellant's witnesses to prove that the opposer-‐appellant was never a user of the trademark BATA either before or after the war, that the appellant is not the successor-‐in-‐interest of Gerbec and Hrdina who were not is representatives or agents, and could not have passed any rights to the appellant, that there was no privity of interest between the Czechoslovakian owner and the Canadian appellant and that the Czechoslovakian trademark has been abandoned in Czechoslovakia.
Publication
SECTION 133.2. Where the Office finds that the conditions referred to in Subsection 133.1 are fulfilled, it shall upon payment of the prescribed fee, forthwith cause the application, as filed, to be published in the prescribed manner. 5.
Opposition
SECTION 134. OPPOSITION. – Any person who believes that he would be damaged by the registration of a mark may, upon payment of the required fee and within thirty (30) days after the publication referred to in Subsection 133.2, file with the Office an opposition to the application. Such opposition shall be in writing and verified by the oppositor or by any person on his behalf who knows the facts, and shall specify the grounds on which it is based and include a statement of the facts relied upon. Copies of certificates of registration of marks registered in other countries or other supporting documents mentioned in the opposition shall be filed therewith, together with the translation in English, if not in the English language. For good cause shown and upon payment of the required surcharge, the time for filing an opposition may be extended by the Director of Legal Affairs, who shall notify the applicant of such extension. The Regulations shall fix the maximum period of time within which to file the opposition.
SECTION 135. NOTICE AND HEARING. – Upon the filing of an opposition, the Office shall serve notice of the filing on the applicant, and of the date of the hearing thereof upon the applicant and the oppositor and all other persons having any right, title or interest in the mark covered by the application, as appear of record in the Office.
BATA INDUSTRIES V. CA AND NEW OLYMPIAN RUBBER PRODUCTS CO. (1982) Refresher: New Olympian Rubber Products Co. wanted to register BATA for casual rubber shoes that it has allegedly used since 1 July 1970. Bata Industries, Ltd., a Canadian corporation, opposed the same on the ground that it owns, and has not abandoned, the trademark BATA. The parties stipulated to the following: 1. Bata Industries, Ltd. has no license to do business in the Philippines; 2. It is not presently selling footwear under the trademark BATA in the Philippines; and 3. It has no licensing agreement with any local entity or firm to sell its products in the Philippines.
MIRPURI V. CA (1999), SUPRA Refresher: Trademark involved in this case: "Barbizon" Relevant issue: Res judicata in 2 oppositions filed by the Private respondent. Opposition No. 1 à IPC No. 686:
o 79
Katrina Michelle Mancao (a) "confusing similarity" of its trademark with that of Escobar's; (b) that the registration of Escobar's similar trademark will cause damage to private respondent's business reputation and goodwill; and (c) that Escobar's use of the trademark amounts to an unlawful appropriation of a mark previously used in the Philippines which act is penalized under Section 4 (d) of the Trademark Law.
IPC No. 2049 raised the issue of ownership of the trademark, the first registration and use of the trademark in the United States and other countries, and the international recognition and reputation of the trademark established by extensive use and advertisement of private respondent's products for over forty years here and abroad. These are different from the issues of confusing similarity and damage in IPC No. 686. The issue of prior use may have been raised in IPC No. 686 but this claim was limited to prior use in the Philippines only. Prior use in IPC No. 2049 stems from private respondent's claim as originator of the word and symbol "Barbizon," as the first and registered user of the mark attached to its products which have been sold and advertised worldwide for a considerable number of years prior to petitioner's first application for registration of her trademark in the Philippines. Indeed, these are substantial allegations that raised new issues and necessarily gave private respondent a new cause of action. Res judicata does not apply to rights, claims or demands, although growing out of the same subject matter, which constitute separate or distinct causes of action and were not put in issue in the former action. Respondent corporation also introduced in the second case a fact that did not exist at the time the first case was filed and terminated. The cancellation of petitioner's certificate of registration for failure to file the affidavit of use arose only after IPC No. 686. It did not and could not have occurred in the first case, and this gave respondent another cause to oppose the second application. Res judicata extends only to facts and conditions as they existed at the time judgment was rendered and to the legal rights and relations of the parties fixed by the facts so determined. When new facts or conditions intervene before the second suit, furnishing a new basis for the claims and defenses of the parties, the issues are no longer the same, and the former judgment cannot be pleaded as a bar to the subsequent action. It is also noted that the oppositions in the first and second cases are based on different laws. The opposition in IPC No. 686 was based on specific provisions of the Trademark Law, i.e., Section 4 (d) on confusing similarity of trademarks and Section 8 on the requisite damage to file an opposition to a petition for registration. The opposition in IPC No. 2049 invoked the Paris Convention, particularly Article 6bis thereof, E.O. No. 913 and the two Memoranda of the Minister of Trade and Industry. This opposition also invoked Article 189 of the Revised Penal Code which is a statute totally different from the Trademark Law. Causes of action which are distinct and independent from each other, although arising out of the same contract, transaction, or state of facts, may be sued on separately, recovery on one being no bar to subsequent actions on others. The mere fact that the same relief is sought in the subsequent action will not render the judgment in the
Opposition No. 2 à IPC No. 2049: (a) as early as 1933, it adopted the word "BARBIZON" as trademark on its products such as robes, pajamas, lingerie, nightgowns and slips; (b) that the trademark "BARBIZON" was registered with the United States Patent Office in 1934 and 1949; and that variations of the same trademark, i.e., "BARBIZON" with Bee design and "BARBIZON" with the representation of a woman were also registered with the U.S. Patent Office in 1961 and 1976; (c) that these marks have been in use in the Philippines and in many countries all over the world for over forty years. "Barbizon" products have been advertised in international publications and the marks registered in 36 countries worldwide; (d) Escobar's registration of the similar trademark "BARBIZON" in 1974 was based on fraud; and this fraudulent registration was cancelled in 1979, stripping Escobar of whatsoever right she had to the said mark; (e) Private respondent's trademark is entitled to protection as a well-‐known mark under Article 6bis of the Paris Convention, Executive Order No. 913, and the two Memoranda dated November 20, 1980 and October 25, 1983 of the Minister of Trade and Industry to the Director of Patents; (f) Escobar's trademark is identical to private respondent's and its use on the same class of goods as the latter's amounts to a violation of the Trademark Law and Article 189 of the Revised Penal Code.
80 z
Intellectual Property Law prior action operative as res judicata, such as where the two actions are based on different statutes. Res judicata therefore does not apply to the instant case and respondent Court of Appeals did not err in so ruling. Intellectual and industrial property rights cases are not simple property cases. Trademarks deal with the psychological function of symbols and the effect of these symbols on the public at large. Trademarks play a significant role in communication, commerce and trade, and serve valuable and interrelated business functions, both nationally and internationally. For this reason, all agreements concerning industrial property, like those on trademarks and tradenames, are intimately connected with economic development. Industrial property encourages investments in new ideas and inventions and stimulates creative efforts for the satisfaction of human needs. They speed up transfer of technology and industrialization, and thereby bring about social and economic progress. These advantages have been acknowledged by the Philippine government itself. The Intellectual Property Code of the Philippines declares that "an effective intellectual and industrial property system is vital to the development of domestic and creative activity, facilitates transfer of technology, it attracts foreign investments, and ensures market access for our products." The Intellectual Property Code took effect on January 1, 1998 and by its express provision, repealed the Trademark Law, the Patent Law, Articles 188 and 189 of the Revised Penal Code, the Decree on Intellectual Property, and the Decree on Compulsory Reprinting of Foreign Textbooks. The Code was enacted to strengthen the intellectual and industrial property system in the Philippines as mandated by the country's accession to the Agreement Establishing the World Trade Organization (WTO). The law does not require the oppositor be a TM owner, so long as he will sustain damage by the registration, he may oppose.
SECTION 137. REGISTRATION OF MARK AND ISSUANCE OF A CERTIFICATE TO THE OWNER OR HIS ASSIGNEE. – 137.1. The Office shall maintain a Register in which shall be registered marks, numbered in the order of their registration, and all transactions in respect of each mark, required to be recorded by virtue of this law. 137.2. The registration of a mark shall include a reproduction of the mark and shall mention: its number; the name and address of the registered owner and, if the registered owner's address is outside the country, his address for service within the country; the dates of application and registration; if priority is claimed, an indication of this fact, and the number, date and country of the application, basis of the priority claims; the list of goods or services in respect of which registration has been granted, with the indication of the corresponding class or classes; and such other data as the Regulations may prescribe from time to time. 137.3. A certificate of registration of a mark may be issued to the assignee of the applicant: Provided, That the assignment is recorded in the Office. In case of a change of ownership, the Office shall at the written request signed by the owner, or his representative, or by the new owner, or his representative and upon a proper showing and the payment of the prescribed fee, issue to such assignee a new certificate of registration of the said mark in the name of such assignee, and for the unexpired part of the original period. 137.4. The Office shall record any change of address, or address for service, which shall be notified to it by the registered owner. 137.5. In the absence of any provision to the contrary in this Act, communications to be made to the registered owner by virtue of this Act shall be sent to him at his last recorded address and, at the same, at his last recorded address for service.
6.
Issuance and Publication of Certificate
SECTION 138. CERTIFICATES OF REGISTRATION. – A certificate of registration of a mark shall be prima facie evidence of the validity of the registration, the registrant's ownership of the mark, and of the registrant's exclusive right to use the same in connection with the goods or services and those that are related thereto specified in the certificate.
SECTION 136. ISSUANCE AND PUBLICATION OF CERTIFICATE. – When the period for filing the opposition has expired, or when the Director of Legal Affairs shall have denied the opposition, the Office upon payment of the required fee, shall issue the certificate of registration. Upon issuance of a certificate of registration, notice thereof making reference to the publication of the application shall be published in the IPO Gazette.
SECTION 139. PUBLICATION OF REGISTERED MARKS; INSPECTION OF REGISTER. – 139.1. The Office shall publish, in the form and within the period fixed by the Regulations, the marks registered, in the order of their registration, reproducing all the particulars referred to in Subsection 137.2.
o 81
Katrina Michelle Mancao 139.2. Marks registered at the Office may be inspected free of charge and any person may obtain copies thereof at his own expense. This provision shall also be applicable to transactions recorded in respect of any registered mark.
when IPO agrees to accept non-‐use are very rare and exceptional. It must be factors beyond your control. • Bankrupt? Financial circumstance not an acceptable ground • Independent of your will: o Constitutional prohibition for broadcasting network. IPO accepted it so the mark is still registered, not struck down for use. o Government restriction/regulation • Within 1st year from 5th anniversary of mark: file declaration of actual use to show that you are still using the mark • 10 years: file petition for renewal
SECTION 144. CLASSIFICATION OF GOODS AND SERVICES. – 144.1. Each registration, and any publication of the Office which concerns an application or registration effected by the Office shall indicate the goods or services by their names, grouped according to the classes of the Nice Classification, and each group shall be preceded by the number of the class of that Classification to which that group of goods or services belongs, presented in the order of the classes of the said Classification. 144.2. Goods or services may not be considered as being similar or dissimilar to each other on the ground that, in any registration or publication by the Office, they appear in different classes of the Nice Classification. SECTION 4.2. The term "TECHNOLOGY TRANSFER ARRANGEMENTS" refers to contracts or agreements involving the transfer of systematic knowledge for the manufacture of a product, the application of a process, or rendering of a service including management contracts; and the transfer, assignment or licensing of all forms of intellectual property rights, including licensing of computer software except computer software developed for mass market.
7.
8.
Voluntary Cancellation of Certificate
SECTION 140. CANCELLATION UPON APPLICATION BY REGISTRANT; AMENDMENT OR DISCLAIMER OF REGISTRATION. – Upon application of the registrant, the Office may permit any registration to be surrendered for cancellation, and upon cancellation the appropriate entry shall be made in the records of the Office. Upon application of the registrant and payment of the prescribed fee, the Office for good cause may permit any registration to be amended or to be disclaimed in part: Provided, That the amendment or disclaimer does not alter materially the character of the mark. Appropriate entry shall be made in the records of the Office upon the certificate of registration or, if said certificate is lost or destroyed, upon a certified copy thereto.
Duration of Certificate
SECTION 145. DURATION. – A certificate of registration shall remain in force for ten (10) years: Provided, That the registrant shall file a declaration of actual use and evidence to that effect, or shall show valid reasons based on the existence of obstacles to such use, as prescribed by the Regulations, within one (1) year from the fifth anniversary of the date of the registration of the mark. Otherwise, the mark shall be removed from the Register by the Office. Marks acquired under RA 166 have a 20-‐year duration until expiration. Once it expires, the mark can only be renewed for a period of 10 years. Why require use from a conceptual basis? Warehousing of trademarks is not allowed. What’s the point of protecting trademarks if not use for contributing the GDP, pay taxes, hire people, no economic activity at all. • There is single 6mon extension to make use of the mark. Can you file declaration of non-‐use? Yes, but the instances
9.
Correction of Mistakes
SECTION 142. CORRECTION OF MISTAKES MADE BY THE OFFICE. – Whenever a material mistake in a registration incurred through the fault of the Office is clearly disclosed by the records of the Office, a certificate stating the fact and nature of such mistake shall be issued without charge, recorded and a printed copy thereof shall be attached to each printed copy of the registration. Such corrected registration shall thereafter have the same effect as the original certificate; or in the discretion of the Director of the Administrative, Financial and Human Resource Development Service Bureau a new certificate of registration may be issued without charge. All certificates of correction heretofore issued in accordance with the Regulations and the registration to which they are attached shall have the same force and effect as if such certificates and their issuance had been authorized by this Act.
82 z
Intellectual Property Law
146.4. An applicant for renewal not domiciled in the Philippines shall be subject to and comply with the requirements of this Act. Can a trademark registration be in perpetuity forever and ever? Copyright can be lifetime. Yes. For as long as you keep renewing the mark. A trademark can be forever for as long as 6 mos prior to 10th year anniversary of registration, you file petition for renewal which is granted as a matter of course. The cycle goes on and on.
SECTION 143. CORRECTION OF MISTAKES MADE BY APPLICANT. – Whenever a mistake is made in a registration and such mistake occurred in good faith through the fault of the applicant, the Office may issue a certificate upon the payment of the prescribed fee: Provided, That the correction does not involve any change in the registration that requires republication of the mark.
10. Renewal
SECTION 146. RENEWAL. –
H. Rights conferred
146.1. A certificate of registration may be renewed for periods of ten (10) years at its expiration upon payment of the prescribed fee and upon filing of a request. The request shall contain the following indications: (a) An indication that renewal is sought; (b) The name and address of the registrant or his successor-‐in-‐interest, hereafter referred to as the "right holder"; (c) The registration number of the registration concerned; (d) The filing date of the application which resulted in the registration concerned to be renewed; (e) Where the right holder has a representative, the name and address of that representative; (f) The names of the recorded goods or services for which the renewal is requested or the names of the recorded goods or services for which the renewal is not requested, grouped according to the classes of the Nice Classification to which that group of goods or services belongs and presented in the order of the classes of the said Classification; and (g) A signature by the right holder or his representative. 146.2. Such request shall be in Filipino or English and may be made at any time within six (6) months before the expiration of the period for which the registration was issued or renewed, or it may be made within six (6) months after such expiration on payment of the additional fee herein prescribed. 146.3. If the Office refuses to renew the registration, it shall notify the registrant of his refusal and the reasons therefor.
SECTION 147. RIGHTS CONFERRED. – 147.1.* Except in cases of importation of drugs and medicines allowed under Section 72.1 of this Act and of off-‐patent drugs and medicines, the owner of a registered mark shall have the exclusive right to prevent all third parties not having the owner's consent from using in the course of trade identical or similar signs or containers for goods or services which are identical or similar to those in respect of which the trademark is registered where such use would result in a likelihood of confusion. In case of the use of an identical sign for identical goods or services, a likelihood of confusion shall be presumed. There shall be no infringement of trademarks or tradenames of imported or sold patented drugs and medicines allowed under Section 72.1 of this Act, as well as imported or sold off-‐patent drugs and medicines: Provided, That, said drugs and medicines bear the registered marks that have not been tampered, unlawfully modified, or infringed upon, under Section 155 of this Code. 147.2. The exclusive right of the owner of a well-‐known mark defined in Subsection 123.1(e)** which is registered in the Philippines, shall extend to goods and services which are not similar to those in respect of which the mark is registered: Provided, That use of that mark in
*
As amended by RA 9502 – Universally Accessible Cheaper and Quality Medicines Act of 2008 **
Section 123.1. A mark cannot be registered if: (e) Is identical with, or confusingly similar to, or constitutes a translation of a mark which is considered by the competent authority of the Philippines to be well-‐known internationally and in the Philippines, whether or not it is registered here, as being already the mark of a person other than the applicant for registration, and used for identical or similar goods or services: Provided, That in determining whether a mark is well-‐known, account shall be taken of the knowledge of the relevant sector of the public, rather than of the public at large, including knowledge in the Philippines which has been obtained as a result of the promotion of the mark
o 83
Katrina Michelle Mancao relation to those goods or services would indicate a connection between those goods or services and the owner of the registered mark: Provided further, That the interests of the owner of the registered mark are likely to be damaged by such use.
87.2. Those pursuant to which the licensor reserves the right to fix the sale or resale prices of the products manufactured on the basis of the license; 87.3. Those that contain restrictions regarding the volume and structure of production; 87.4. Those that prohibit the use of competitive technologies in a non-‐exclusive technology transfer agreement; 87.5. Those that establish a full or partial purchase option in favor of the licensor; 87.6. Those that obligate the licensee to transfer for free to the licensor the inventions or improvements that may be obtained through the use of the licensed technology; 87.7. Those that require payment of royalties to the owners of patents for patents which are not used; 87.8. Those that prohibit the licensee to export the licensed product unless justified for the protection of the legitimate interest of the licensor such as exports to countries where exclusive licenses to manufacture and/or distribute the licensed product(s) have already been granted; 87.9. Those which restrict the use of the technology supplied after the expiration of the technology transfer arrangement, except in cases of early termination of the technology transfer arrangement due to reason(s) attributable to the licensee; 87.10. Those which require payments for patents and other industrial property rights after their expiration, termination arrangement; 87.11. Those which require that the technology recipient shall not contest the validity of any of the patents of the technology supplier; 87.12. Those which restrict the research and development activities of the licensee designed to absorb and adapt the transferred technology to local conditions or to initiate research and development programs in connection with new products, processes or equipment; 87.13. Those which prevent the licensee from adapting the imported technology to local conditions, or introducing innovation to it, as long as it does not impair the quality standards prescribed by the licensor; 87.14. Those which exempt the licensor for liability for non-‐fulfillment of his responsibilities under the
SECTION 148. USE OF INDICATIONS BY THIRD PARTIES FOR PURPOSES OTHER THAN THOSE FOR WHICH THE MARK IS USED. – Registration of the mark shall not confer on the registered owner the right to preclude third parties from using bona fide their names, addresses, pseudonyms, a geographical name, or exact indications concerning the kind, quality, quantity, destination, value, place of origin, or time of production or of supply, of their goods or services: Provided, That such use is confined to the purposes of mere identification or information and cannot mislead the public as to the source of the goods or services.
SECTION 4.2.**
The term "TECHNOLOGY TRANSFER ARRANGEMENTS" refers to contracts or agreements involving the transfer of systematic knowledge for the manufacture of a product, the application of a process, or rendering of a service including management contracts; and the transfer, assignment or licensing of all forms of intellectual property rights, including licensing of computer software except computer software developed for mass market.
SECTION 87. PROHIBITED CLAUSES. – Except in cases under Section 91 7 , the following provisions shall be deemed prima facie to have an adverse effect on competition and trade: 87.1. Those which impose upon the licensee the obligation to acquire from a specific source capital goods, intermediate products, raw materials, and other technologies, or of permanently employing personnel indicated by the licensor;
**
Relate to Sections 87, 88, 92
7
Section 91. Exceptional Cases. -‐ In exceptional or meritorious cases where substantial benefits will accrue to the economy, such as high technology content, increase in foreign exchange earnings, employment generation, regional dispersal of industries and/or substitution with or use of local raw materials, or in the case of Board of Investments, registered companies with pioneer status, exemption from any of the above requirements may be allowed by the Documentation, Information and Technology Transfer Bureau after evaluation thereof on a case by case basis.
84 z
Intellectual Property Law technology transfer arrangement and/or liability arising from third party suits brought about by the use of the licensed product or the licensed technology; and 87.15. Other clauses with equivalent effects.
149.2. Such assignment or transfer shall, however, be null and void if it is liable to mislead the public, particularly as regards the nature, source, manufacturing process, characteristics, or suitability for their purpose, of the goods or services to which the mark is applied. 149.3. The assignment of the application for registration of a mark, or of its registration, shall be in writing and require the signatures of the contracting parties. Transfers by mergers or other forms of succession may be made by any document supporting such transfer. 149.4. Assignments and transfers of registrations of marks shall be recorded at the Office on payment of the prescribed fee; assignment and transfers of applications for registration shall, on payment of the same fee, be provisionally recorded, and the mark, when registered, shall be in the name of the assignee or transferee. 149.5. Assignments and transfers shall have no effect against third parties until they are recorded at the Office.
SECTION 88. MANDATORY PROVISIONS. – The following provisions shall be included in voluntary license contracts: 88.1. That the laws of the Philippines shall govern the interpretation of the same and in the event of litigation, the venue shall be the proper court in the place where the licensee has its principal office; 88.2. Continued access to improvements in techniques and processes related to the technology shall be made available during the period of the technology transfer arrangement; 88.3. In the event the technology transfer arrangement shall provide for arbitration, the Procedure of Arbitration of the Arbitration Law of the Philippines or the Arbitration Rules of the United Nations Commission on International Trade Law (UNCITRAL) or the Rules of Conciliation and Arbitration of the International Chamber of Commerce (ICC) shall apply and the venue of arbitration shall be the Philippines or any neutral country; and 88.4. The Philippine taxes on all payments relating to the technology transfer arrangement shall be borne by the licensor.
SECTION 150. LICENSE CONTRACTS. – 150.1. Any license contract concerning the registration of a mark, or an application therefor, shall provide for effective control by the licensor of the quality of the goods or services of the licensee in connection with which the mark is used. If the license contract does not provide for such quality control, or if such quality control is not effectively carried out, the license contract shall not be valid. 150.2. A license contract shall be submitted to the Office which shall keep its contents confidential but shall record it and publish a reference thereto. A license contract shall have no effect against third parties until such recording is effected. The Regulations shall fix the procedure for the recording of the license contract.
SECTION 92. NON-‐REGISTRATION WITH THE DOCUMENTATION, INFORMATION AND TECHNOLOGY TRANSFER BUREAU . – Technology transfer arrangements that conform with the provisions of Sections 86 and 87 need not be registered with the Documentation, Information and Technology Transfer Bureau. Non-‐conformance with any of the provisions of Sections 87 and 88, however, shall automatically render the technology transfer arrangement unenforceable, unless said technology transfer arrangement is approved and registered with the Documentation, Information and Technology Transfer Bureau under the provisions of Section 91 on exceptional cases.
SECTION 149. ASSIGNMENT AND APPLICATION AND REGISTRATION. –
TRANSFER
SECTION 231. REVERSE RECIPROCITY OF FOREIGN LAWS. – Any condition, restriction, limitation, diminution, requirement, penalty or any similar burden imposed by the law of a foreign country on a Philippine national seeking protection of intellectual property rights in that country, shall reciprocally be enforceable upon nationals of said country, within Philippine jurisdiction.
OF
149.1. An application for registration of a mark, or its registration, may be assigned or transferred with or without the transfer of the business using the mark.
o 85
Katrina Michelle Mancao and advertisements of the originator of said mark, to convey to the public a false impression of some supposed connection between the manufacturer of the article sold under the original mark and the new articles being tendered to the public under the same or similar mark. As trade has developed and commercial changes have come about, the law of unfair competition has expanded to keep pace with the times and the element of strict competition in itself has ceased to be the determining factor. The owner of a trade-‐mark or trade-‐ name has a property right in which he is entitled to protection, since there is damage to him from confusion of reputation or goodwill in the mind of the public as well as from confusion of goods. The modern trend is to give emphasis to the unfairness of the acts and to classify and treat the issue as a fraud.
Is there infringement even if the goods are non-‐ competing?
ANG V. TEODORO (1942), SUPRA Refresher: “Ang Tibay” used for slippers; tried to be registered for pants and shirts. Doctrine: This fundamental change in attitude first manifested itself in the year 1915-‐1917. Until about then, the courts had proceeded on the theory that the same trade-‐mark, used on un-‐like goods, could not cause confusion in trade and that, therefore, there could be no objection to the use and registration of a well-‐known mark by a third party for a different class of goods. Since 1916 however, a growing sentiment began to arise that in the selection of a famous mark by a third party, there was generally the hidden intention to "have a free ride" on the trade-‐ mark owner's reputation and good will. In the present state of development of the law on Trade-‐ Marks, Unfair Competition, and Unfair Trading, the TEST employed by the courts to determine whether noncompeting goods are or are not of the same class is confusion as to the origin of the goods of the second user. Although two noncompeting articles may be classified under two different classes by the Patent Office because they are deemed not to possess the same descriptive properties, they would, nevertheless, be held by the courts to belong to the same class if the simultaneous use on them of identical or closely similar trade-‐marks would be likely to cause confusion as to the origin, or personal source, of the second user's goods. They would be considered as not falling under the same class only if they are so dissimilar or so foreign to each other as to make it unlikely that the purchaser would think the first user made the second user's goods. Such construction of the law is induced by cogent reasons of equity and fair dealing. The courts have come to realize that there can be unfair competition or unfair trading even if the goods are non-‐competing, and that such unfair trading can cause INJURY OR DAMAGE TO THE FIRST USER of a given trade-‐mark, first, by prevention of the natural expansion of his business and, second, by having his business reputation confused with and put at the mercy of the second user. Then noncompetitive products are sold under the same mark, the gradual whittling away or dispersion of the identity and hold upon the public mind of the mark created by its first user, inevitably results. The original owner is entitled to the preservation of the valuable link between him and the public that has been created by his ingenuity and the merit of his wares or services. Experience has demonstrated that when a well-‐known trade-‐mark is adopted by another even for a totally different class of goods, it is done to get the benefit of the reputation
CHUA CHE V. PHILIPPINE PATENT OFFICE (1965) Refresher: “X-‐7” previously used as a brand of toilet articles was being registered for soap. Doctrine: Registration of a trademark should be refused in cases where there is a likelihood of confusion, mistake, or deception, even though the goods fall into different categories. The products of appellee are common household items nowadays, in the same manner as laundry soap. The likelihood of purchasers to associate those products to a common origin is not far-‐fetched. Both from the standpoint of priority of use and for the protection of the buying public and, of course, appellee's rights to the trademark "X-‐7", it becomes manifest that the registration of said trademark in favor of applicant-‐ appellant should be denied.
STA. ANA V. MALIWAT (1968) Refresher: Maliwat wanted to register the trademark FLORMANN, which is used on shirts, pants, jackets and shoes for ladies, men, and children. Sta. Ana filed an application for the registration of the tradename FLORMEN SHOE MANUFACTURERS, which he'll use for manufacturing ladies' and children's shoes. Doctrine: Modern law recognizes that the protection to which the owner of a trademark mark is entitled is not limited to guarding his goods or business from actual market competition with identical or similar products of the parties, but extends to all cases in which the use by a junior appropriator of a trademark or tradename is likely to lead to a confusion of source, as where prospective purchasers would be misled into thinking that the complaining party has extended his business into the field or is in any way connected with the activities of the infringer; or when it forestalls the normal potential expansion of his business.
86 z
Intellectual Property Law Mere dissimilarity of goods should not preclude relief where the junior user's goods are not too different or remote from any that the owner would be likely to make or sell; and in the present case, wearing apparel is not so far removed from shoes as to preclude relief.
be injured by any subsequent appropriation or imitation by others, and the public will not be deceived.
ESSO STANDARD EASTERN, INC. V. CA (1982) Refresher: “Esso” is used by Esso Standard for its petroleum products. Private respondent (United Cigarette Corp.) wanted to use it for its cigarettes. Doctrine: The law defines infringement as the use without consent of the trademark owner of any "reproduction, counterfeit, copy or colorable limitation of any registered mark or tradename in connection with the sale, offering for sale, or advertising of any goods, business or services on or in connection with which such use is likely to cause confusion or mistake or to deceive purchasers or others as to the source or origin of such goods or services, or Identity of such business; or reproduce, counterfeit, copy or colorably imitate any such mark or tradename and apply such reproduction, counterfeit, copy or colorable limitation to labels, signs, prints, packages, wrappers, receptacles or advertisements intended to be used upon or in connection with such goods, business or services." Implicit in this definition is the concept that the goods must be so related that there is a likelihood either of confusion of goods or business. But likelihood of confusion is a relative concept; to be determined only according to the particular, and sometimes peculiar, circumstances of each case. It is unquestionably true that, as stated in Coburn vs. Puritan Mills, Inc. "In trademark cases, even more than in other litigation, precedent must be studied in the light of the facts of the particular case. It is undisputed that the goods on which petitioner uses the trademark ESSO, petroleum products, and the product of respondent, cigarettes, are non-‐competing. But as to whether trademark infringement exists depends for the most part upon whether or not the goods are so related that the public may be, or is actually, deceived and misled that they came from the same maker or manufacturer. For non-‐competing goods may be those which, though they are not in actual competition, are so related to each other that it might reasonably be assumed that they originate from one manufacturer. Non-‐competing goods may also be those which, being entirely unrelated, could not reasonably be assumed to have a common source. In the former case of related goods, confusion of business could arise out of the use of similar marks; in the latter case of non-‐related goods, it could not. The vast majority of courts today follow the modern theory or concept of "related goods" which the Court has likewise adopted and uniformly recognized and applied. Goods are related when they belong to the same class or have the same descriptive properties; when they possess
PHILIPPINE REFINING COMPANY V. NG SAM (1982) Refresher: “Camia” was used by Philippine Refining Sugar for its lard, butter, cooking oil, and soap. Ng Sam wanted to register it for its ham. Doctrine: A rudimentary precept in trademark protection is that "the right to a trademark is a limited one, in the sense that others may use the same mark on unrelated goods." Thus, as pronounced by the United States Supreme Court in the case of American Foundries vs. Robertson, "the mere fact that one person has adopted and used a trademark on his goods does not prevent the adoption and use of the same trademark by others on articles of a different description." Such restricted right over a trademark is likewise reflected in our Trademark law. Under Section 4(d) of the law, registration of a trademark which so resembles another already registered or in use should be denied, where to allow such registration could likely result in confusion, mistake or deception to the consumers. Conversely, where no confusion is likely to arise, as in this case, registration of a similar or even Identical mark may be allowed. The term "CAMIA" is descriptive of a whole genus of garden plants with fragrant white flowers. Some people call the "CAMIA" the "white ginger plant" because of its tuberous roots, while children refer to it as the butterfly flower because of its shape. Being a generic and common term, its appropriation as a trademark, albeit in a fanciful manner in that it bears no relation to the product it Identifies, is valid. However, the degree of exclusiveness accorded to each user is closely restricted. A trademark is designed to Identify the user. But it should be so distinctive and sufficiently original as to enable those who come into contact with it to recognize instantly the Identity of the user. " It must be affirmative and definite, significant and distinctive, capable to indicate origin." It is evident that "CAMIA" as a trademark is far from being distinctive. By itself, it does not Identify petitioner as the manufacturer or producer of the goods upon which said mark is used, as contra-‐distinguished to trademarks derived from coined words such as "Rolex", "Kodak" or "Kotex". It has been held that if a mark is so commonplace that it cannot be readily distinguished from others, then it is apparent that it cannot Identify a particular business; and he who first adopted it cannot
o 87
Katrina Michelle Mancao deodorant, talcum powder, and toilet soap. Thereafter, respondent court, through Justice Gopengco with Justices Patajo and Racela, Jr. concurring, was initially persuaded by petitioner's plea for reversal directed against the permission granted by the Director of Patents, but the decision of the Second Special Cases Division handed down on April 29, 1983 was later reconsidered in favor of herein private respondent. Doctrine: Having thus reviewed the laws applicable to the case before Us, it is not difficult to discern from the foregoing statutory enactments that private respondent may be permitted to register the trademark "BRUTE" for briefs produced by it notwithstanding petitioner's vehement protestations of unfair dealings in marketing its own set of items which are limited to: after-‐shave lotion, shaving cream, deodorant, talcum powder and toilet soap. In as much as petitioner has not ventured in the production of briefs, an item which is not listed in its certificate of registration, petitioner can not and should not be allowed to feign that private respondent had invaded petitioner's exclusive domain. To be sure, it is significant that petitioner failed to annex in its Brief the so-‐called "eloquent proof that petitioner indeed intended to expand its mark "BRUT" to other goods." Even then, a mere application by petitioner in this aspect does not suffice and may not vest an exclusive right in its favor that can ordinarily be protected by the Trademark Law. In short, paraphrasing Section 20 of the Trademark Law as applied to the documentary evidence adduced by petitioner, the certificate of registration issued by the Director of Patents can confer upon petitioner the exclusive right to use its own symbol only to those goods specified in the certificate, subject to any conditions and limitations stated therein. How do We now reconcile the apparent conflict between Section 4(d) which was relied upon by Justice JBL Reyes in the Sta. Ana case and Section 20? It would seem that Section 4(d) does not require that the goods manufactured by the second user be related to the goods produced by the senior user while Section 20 limits the exclusive right of the senior user only to those goods specified in the certificate of registration. But the rule has been laid down that the clause which comes later shall be given paramount significance over an anterior proviso upon the presumption that it expresses the latest and dominant purpose. It ineluctably follows that Section 20 is controlling and, therefore, private respondent can appropriate its symbol for the briefs it manufactures because as aptly remarked by Justice Sanchez in Sterling Products International Inc. vs. Farbenfabriken Bayer: Really, if the certificate of registration were to be deemed as including goods not specified therein, then a
the same physical attributes or essential characteristics with reference to their form, composition, texture or quality. They may also be related because they serve the same purpose or are sold in grocery stores. In the situation before us, the goods are obviously different from each other with "absolutely no iota of similitude" as stressed in respondent court's judgment. They are so foreign to each other as to make it unlikely that purchasers would think that petitioner is the manufacturer of respondent's goods. The mere fact that one person has adopted and used a trademark on his goods does not prevent the adoption and use of the same trademark by others on unrelated articles of a different kind. Although petitioner's products are numerous, they are of the same class or line of merchandise which are non-‐ competing with respondent's product of cigarettes, which as pointed out in the appealed judgment is beyond petitioner's "zone of potential or natural and logical expansion." When a trademark is used by a party for a product in which the other party does not deal, the use of the same trademark on the latter's product cannot be validly objected to. Another factor that shows that the goods involved are non-‐competitive and non-‐related is the appellate court's finding that they flow through different channels of trade, thus: "The products of each party move along and are disposed through different channels of distribution.
HICKOK MANUFACTURING CO. V. CA (1982) Refresher: Petitioner: diverse articles of men's wear such as wallets, belts and men's briefs Respondent: shoes Doctrine: It is established doctrine, as held in the above-‐cited cases, that "emphasis should be on the similarity of the products involved and not on the arbitrary classification or general description of their properties or characteristics" and that "the mere fact that one person has adopted and used a trademark on his goods does not prevent the adoption and use of the same trademark by others on unrelated articles of a different kind."
FABERGE V. IAC (1992) Refresher: The Director of Patents authorized herein private respondent Co Beng Kay to register the trademark "BRUTE" for the briefs manufactured and sold by his Corporation in the domestic market vis-‐a-‐vis petitioner's opposition grounded on similarity of said trademark with petitioner's own symbol "BRUT" which it previously registered for after shave lotion, shaving cream,
88 z
Intellectual Property Law situation may arise whereby an applicant may be tempted to register a trademark on any and all goods which his mind may conceive even if he had never intended to use the trademark for the said goods. We believe that such omnibus registration is not contemplated by our Trademark Law.
attributes or essential characteristics with reference to their form, composition, texture or quality. They may also be related because they serve the same purpose or are sold in grocery stores.
PEARL & DEAN (PHIL.), INC. V. SHOEMART, SUPRA Refresher: Light boxes Doctrine: On trademark infringement: The certificate of registration issued by the Director of Patents can confer the exclusive right to use its own symbol only to those goods specified in the certificate, subject to any conditions and limitations specified in the certificate. One who has adopted and used a trademark on his goods does not prevent the adoption and use of the same trademark by others for products which are of a different description. Moreover, the failure of P & D to secure a trademark registration for specific use on the light boxes meant that there could not have been any trademark infringement since registration was an essential element thereof.
CANON KABUSHIKI V. CA (2000) Refresher: Private respondent NSR Rubber Corporation (private respondent) filed an application for registration of the mark CANON for sandals. Doctrine: We find the arguments of petitioner to be unmeritorious. Ordinarily, the ownership of a trademark or tradename is a property right that the owner is entitled to protect as mandated by the Trademark Law. However, when a trademark is used by a party for a product in which the other party does not deal, the use of the same trademark on the latter's product cannot be validly objected to. Herein petitioner has not made known that it intends to venture into the business of producing sandals. This is clearly shown in its Trademark Principal Register where the products of the said petitioner had been clearly and specifically described as "Chemical products, dyestuffs, pigments, toner developing preparation, shoe polisher, polishing agent". It would be taxing one's credibility to aver at this point that the production of sandals could be considered as a possible "natural or normal expansion" of its business operation. The likelihood of confusion of goods or business is a relative concept, to be determined only according to the particular, and sometimes peculiar, circumstances of each case.16 Indeed, in trademark law cases, even more than in other litigation, precedent must be studied in the light of the facts of the particular case. Contrary to petitioner's supposition, the facts of this case will show that the cases of Sta. Ana vs. Maliwat, Ang vs. Teodoro and Converse Rubber Corporation vs. Universal Rubber Products, Inc. are hardly in point. The just cited cases involved goods that were confusingly similar, if not identical, as in the case of Converse Rubber Corporation vs. Universal Rubber Products, Inc. Here, the products involved are so unrelated that the public will not be misled that there is the slightest nexus between petitioner and the goods of private respondent. In cases of confusion of business or origin, the question that usually arises is whether the respective goods or services of the senior user and the junior user are so related as to likely cause confusion of business or origin, and thereby render the trademark or tradenames confusingly similar. Goods are related when they belong to the same class or have the same descriptive properties; when they possess the same physical
246 CORPORATION V. DAWAY (2003) Refresher: 246 Corporation: Rolex Music Lounge Doctrine: Under the old Trademark Law15 where the goods for which the identical marks are used are unrelated, there can be no likelihood of confusion and there is therefore no infringement in the use by the junior user of the registered mark on the entirely different goods. This ruling, however, has been to some extent, modified by Section 123.1(f) of the Intellectual Property Code (Republic Act No. 8293), which took effect on January 1, 1998. The said section reads: Sec. 123. Registrability. – 123.1. A mark cannot be registered if it: x x x x x x x x x (f) Is identical with, or confusingly similar to, or constitutes a translation of a mark considered well-‐known in accordance with the preceding paragraph, which is registered in the Philippines with respect to goods or services which are not similar to those with respect to which registration is applied for: Provided, That use of the mark in relation to those goods or services would indicate a connection between those goods or services, and the owner of the registered mark: Provided, further, That the
o 89
Katrina Michelle Mancao interest of the owner of the registered mark are likely to be damaged by such use;
(k) the outcome of litigations dealing with the issue of whether the mark is a well-‐known mark; and (l) the presence of absence of identical or similar marks validly registered for or used on identical or similar goods or services and owned by persons other than the person claiming that his mark is a well-‐known mark.
A junior user of a well-‐known mark on goods or services which are not similar to the goods or services, and are therefore unrelated, to those specified in the certificate of registration of the well-‐known mark is precluded from using the same on the entirely unrelated goods or services, subject to the following REQUISITES, to wit: 1. The mark is well-‐known internationally and in the Philippines. Under Rule 102 of the Rules and Regulations on Trademarks, Service Marks, Trade Names and Marked or Stamped Containers, in determining whether a mark is well known, the following criteria or any combination thereof may be taken into account: (a) the duration, extent and geographical area of any use of the mark, in particular, the duration, extent and geographical area of any promotion of the mark, including advertising or publicity and presentation, at fairs or exhibitions, of the goods and/or services to which the mark applies; (b) the market share in the Philippines and in other countries, of the goods and/or services to which the mark applies; (c) the degree of the inherent or acquired distinction of the mark; (d) the quality-‐image or reputation acquired by the mark; (e) the extent to which the mark has been registered in the world; (f) the exclusivity of the registration attained by the mark in the world; (g) the extent to which the mark has been used in the world; (h) the exclusivity of use attained by the mark in the world; (i) the commercial value attributed to the mark in the world; (j) the record of successful protection of the rights in the mark;
2. The use of the well-‐known mark on the entirely unrelated goods or services would indicate a connection between such unrelated goods or services and those goods or services specified in the certificate of registration in the well known mark. This requirement refers to the likelihood of confusion of origin or business or some business connection or relationship between the registrant and the user of the mark. 3. The interests of the owner of the well-‐known mark are likely to be damaged. For instance, if the registrant will be precluded from expanding its business to those unrelated good or services, or if the interests of the registrant of the well-‐known mark will be damaged because of the inferior quality of the good or services of the user. Section 123.1(f) is clearly in point because the Music Lounge of petitioner is entirely unrelated to respondents’ business involving watches, clocks, bracelets, etc. However, the Court cannot yet resolve the merits of the present controversy considering that the requisites for the application of Section 123.1(f), which constitute the kernel issue at bar, clearly require determination facts of which need to be resolved at the trial court. The existence or absence of these requisites should be addressed in a full-‐blown hearing and not on a mere preliminary hearing. The respondent must be given ample opportunity to prove its claim, and the petitioner to debunk the same.
SOCIETE DES PRODUITS NESTLE V. CA (2001) Refresher: CFC Corporation (Respondent): “Flavor Master” for coffee Nestle: “Master Roast” and “Master Blend” for coffee Doctrine: The protection of trade-‐marks is the law’s recognition of the psychological function of symbols. If it is true that we live by symbols, it is no less true that we purchase goods by them. A trade-‐mark is a merchandising short-‐
90 z
Intellectual Property Law cut which induces a purchaser to select what he wants, or what he has been led to believe he wants. The owner of a mark exploits this human propensity by making every effort to impregnate the atmosphere of the market with the drawing power of a congenial symbol. Whatever the means employed, the aim is the same -‐-‐-‐ to convey through the mark, in the minds of potential customers, the desirability of the commodity upon which it appears. Once this is attained, the trade-‐mark owner has something of value. If another poaches upon the commercial magnetism of the symbol he has created, the owner can obtain legal redress. Colorable imitation denotes such a close or ingenious imitation as to be calculated to deceive ordinary persons, or such a resemblance to the original as to deceive an ordinary purchaser giving such attention as a purchaser usually gives, as to cause him to purchase the one supposing it to be the other. In determining if colorable imitation exists, jurisprudence has developed two kinds of tests -‐ the Dominancy Test and the Holistic Test. The test of dominancy focuses on the similarity of the prevalent features of the competing trademarks which might cause confusion or deception and thus constitute infringement. On the other side of the spectrum, the holistic test mandates that the entirety of the marks in question must be considered in determining confusing similarity. Nestle points out that the dominancy test should have been applied to determine whether there is a confusing similarity between CFC’s FLAVOR MASTER and Nestle’s MASTER ROAST and MASTER BLEND. We agree. As the Court of Appeals itself has stated, "[t]he determination of whether two trademarks are indeed confusingly similar must be taken from the viewpoint of the ordinary purchasers who are, in general, undiscerningly rash in buying the more common and less expensive household products like coffee, and are therefore less inclined to closely examine specific details of similarities and dissimilarities between competing products." The basis for the Court of Appeals’ application of the totality or holistic test is the "ordinary purchaser" buying the product under "normally prevalent conditions in trade" and the attention such products normally elicit from said ordinary purchaser. An ordinary purchaser or buyer does not usually make such scrutiny nor does he usually have the time to do so. The average shopper is usually in a hurry and does not inspect every product on the shelf as if he were browsing in a library. The Court of Appeals held that the test to be applied should be the totality or holistic test reasoning, since
what is of paramount consideration is the ordinary purchaser who is, in general, undiscerningly rash in buying the more common and less expensive household products like coffee, and is therefore less inclined to closely examine specific details of similarities and dissimilarities between competing products. This Court cannot agree with the above reasoning. If the ordinary purchaser is "undiscerningly rash" in buying such common and inexpensive household products as instant coffee, and would therefore be "less inclined to closely examine specific details of similarities and dissimilarities" between the two competing products, then it would be less likely for the ordinary purchaser to notice that CFC’s trademark FLAVOR MASTER carries the colors orange and mocha while that of Nestle’s uses red and brown. The application of the totality or holistic test is improper since the ordinary purchaser would not be inclined to notice the specific features, similarities or dissimilarities, considering that the product is an inexpensive and common household item. It must be emphasized that the products bearing the trademarks in question are "inexpensive and common" household items bought off the shelf by "undiscerningly rash" purchasers. As such, if the ordinary purchaser is "undiscerningly rash", then he would not have the time nor the inclination to make a keen and perceptive examination of the physical discrepancies in the trademarks of the products in order to exercise his choice. While this Court agrees with the Court of Appeals’ detailed enumeration of differences between the respective trademarks of the two coffee products, this Court cannot agree that totality test is the one applicable in this case. Rather, this Court believes that the dominancy test is more suitable to this case in light of its peculiar factual milieu. Moreover, the totality or holistic test is contrary to the elementary postulate of the law on trademarks and unfair competition that confusing similarity is to be determined on the basis of visual, aural, connotative comparisons and overall impressions engendered by the marks in controversy as they are encountered in the realities of the marketplace. The totality or holistic test only relies on visual comparison between two trademarks whereas the dominancy test relies not only on the visual but also on the aural and connotative comparisons and overall impressions between the two trademarks. For this reason, this Court agrees with the BPTTT when it applied the test of dominancy and held that: From the evidence at hand, it is sufficiently established that the word MASTER is the dominant feature of
o 91
Katrina Michelle Mancao opposer’s mark. The word MASTER is printed across the middle portion of the label in bold letters almost twice the size of the printed word ROAST. Further, the word MASTER has always been given emphasis in the TV and radio commercials and other advertisements made in promoting the product. This can be gleaned from the fact that Robert Jaworski and Atty. Ric Puno Jr.., the personalities engaged to promote the product, are given the titles Master of the Game and Master of the Talk Show, respectively. In due time, because of these advertising schemes the mind of the buying public had come to learn to associate the word MASTER with the opposer’s goods. x x x. It is the observation of this Office that much of the dominance which the word MASTER has acquired through Opposer’s advertising schemes is carried over when the same is incorporated into respondent-‐ applicant’s trademark FLAVOR MASTER. Thus, when one looks at the label bearing the trademark FLAVOR MASTER (Exh. 4) one’s attention is easily attracted to the word MASTER, rather than to the dissimilarities that exist. Therefore, the possibility of confusion as to the goods which bear the competing marks or as to the origins thereof is not farfetched. x x x. In addition, the word "MASTER" is neither a generic nor a descriptive term. As such, said term can not be invalidated as a trademark and, therefore, may be legally protected. Generic terms are those which constitute "the common descriptive name of an article or substance," or comprise the "genus of which the particular product is a species," or are "commonly used as the name or description of a kind of goods," or "imply reference to every member of a genus and the exclusion of individuating characters," or "refer to the basic nature of the wares or services provided rather than to the more idiosyncratic characteristics of a particular product," and are not legally protectable. On the other hand, a term is descriptive and therefore invalid as a trademark if, as understood in its normal and natural sense, it "forthwith conveys the characteristics, functions, qualities or ingredients of a product to one who has never seen it and does not know what it is," or "if it forthwith conveys an immediate idea of the ingredients, qualities or characteristics of the goods," or if it clearly denotes what goods or services are provided in such a way that the consumer does not have to exercise powers of perception or imagination. Rather, the term "MASTER" is a suggestive term brought about by the advertising scheme of Nestle. Suggestive terms are those which, in the phraseology of one court, require "imagination, thought and perception to reach a conclusion as to the nature of the goods." Such terms, "which subtly connote something about the product," are eligible for protection in the absence of secondary meaning. While suggestive marks are capable of
shedding "some light" upon certain characteristics of the goods or services in dispute, they nevertheless involve "an element of incongruity," "figurativeness," or " imaginative effort on the part of the observer."
LEVI STRAUSS V. CLINTON APPARELLE (2005) Refresher: The Complaint alleged that LS & Co., a foreign corporation duly organized and existing under the laws of the State of Delaware, U.S.A., and engaged in the apparel business, is the owner by prior adoption and use since 1986 of the internationally famous "Dockers and Design" trademark. This ownership is evidenced by its valid and existing registrations in various member countries of the Paris Convention. In the Philippines, it has a Certificate of Registration No. 46619 in the Principal Register for use of said trademark on pants, shirts, blouses, skirts, shorts, sweatshirts and jackets under Class 25. The "Dockers and Design" trademark was first used in the Philippines in or about May 1988, by LSPI, a domestic corporation engaged in the manufacture, sale and distribution of various products bearing trademarks owned by LS & Co. To date, LSPI continues to manufacture and sell Dockers Pants with the "Dockers and Design" trademark. LS & Co. and LSPI further alleged that they discovered the presence in the local market of jeans under the brand name "Paddocks" using a device which is substantially, if not exactly, similar to the "Dockers and Design" trademark owned by and registered in the name of LS & Co., without its consent. Based on their information and belief, they added, Clinton Apparelle manufactured and continues to manufacture such "Paddocks" jeans and other apparel. Doctrine: Petitioners anchor their legal right to "Dockers and Design" trademark on the Certificate of Registration issued in their favor by the Bureau of Patents, Trademarks and Technology Transfer.* According to Section 138 of Republic Act No. 8293,42 this Certificate of Registration is prima facie evidence of the validity of the registration, the registrant’s ownership of the mark and of the exclusive right to use the same in connection with the goods or services and those that are related thereto specified in the certificate. Section 147.1 of said law likewise grants the owner of the registered mark the exclusive right to prevent all third parties not having the owner’s consent from using in the course of trade identical or similar signs for goods or services which are identical or similar to those in respect of which the trademark is registered if such use results in a likelihood of confusion.
92 z
Intellectual Property Law However, attention should be given to the fact that petitioners’ registered trademark consists of two elements: (1) the word mark "Dockers" and (2) the wing-‐ shaped design or logo. Notably, there is only one registration for both features of the trademark giving the impression that the two should be considered as a single unit. Clinton Apparelle’s trademark, on the other hand, uses the "Paddocks" word mark on top of a logo which according to petitioners is a slavish imitation of the "Dockers" design. The two trademarks apparently differ in their word marks ("Dockers" and "Paddocks"), but again according to petitioners, they employ similar or identical logos. It could thus be said that respondent only "appropriates" petitioners’ logo and not the word mark "Dockers"; it uses only a portion of the registered trademark and not the whole. Given the single registration of the trademark "Dockers and Design" and considering that respondent only uses the assailed device but a different word mark, the right to prevent the latter from using the challenged "Paddocks" device is far from clear. Stated otherwise, it is not evident whether the single registration of the trademark "Dockers and Design" confers on the owner the right to prevent the use of a fraction thereof in the course of trade. It is also unclear whether the use without the owner’s consent of a portion of a trademark registered in its entirety constitutes material or substantial invasion of the owner’s right. It is likewise not settled whether the wing-‐shaped logo, as opposed to the word mark, is the dominant or central feature of petitioners’ trademark—the feature that prevails or is retained in the minds of the public—an imitation of which creates the likelihood of deceiving the public and constitutes trademark infringement. In sum, there are vital matters which have yet and may only be established through a full-‐blown trial. From the above discussion, we find that petitioners’ right to injunctive relief has not been clearly and unmistakably demonstrated. The right has yet to be determined. Petitioners also failed to show proof that there is material and substantial invasion of their right to warrant the issuance of an injunctive writ. Neither were petitioners able to show any urgent and permanent necessity for the writ to prevent serious damage. Trademark dilution is the lessening of the capacity of a famous mark to identify and distinguish goods or services, regardless of the presence or absence of: (1) competition between the owner of the famous mark and other parties; or (2) likelihood of confusion, mistake or deception. Subject to the principles of equity, the owner of a famous mark is entitled to an injunction "against another person’s commercial use in commerce of a mark or trade name, if such use begins after the mark has become famous and causes dilution of the distinctive
quality of the mark." This is intended to protect famous marks from subsequent uses that blur distinctiveness of the mark or tarnish or disparage it. Based on the foregoing, to be eligible for protection from dilution, there has to be a finding that: (1) the trademark sought to be protected is famous and distinctive; (2) the use by respondent of "Paddocks and Design" began after the petitioners’ mark became famous; and (3) such subsequent use defames petitioners’ mark. In the case at bar, petitioners have yet to establish whether "Dockers and Design" has acquired a strong degree of distinctiveness and whether the other two elements are present for their cause to fall within the ambit of the invoked protection. The Trends MBL Survey Report which petitioners presented in a bid to establish that there was confusing similarity between two marks is not sufficient proof of any dilution that the trial court must enjoin.
RA 166, SECTION 20. CERTIFICATE OF REGISTRATION PRIMA FACIE EVIDENCE O F VALIDITY. – A certificate of registration of a mark or trade-‐name shall be prima facie evidence of the validity of the registration, the registrant's ownership of the mark or trade-‐name, and of the registrant's exclusive right to use the same in connection with the goods, business or services specified in the certificate, subject to any conditions and limitations stated therein.
SECTION 138. CERTIFICATES OF REGISTRATION. – A certificate of registration of a mark shall be prima facie evidence of the validity of the registration, the registrant's ownership of the mark, and of the registrant's exclusive right to use the same in connection with the goods or services and THOSE THAT ARE RELATED THERETO specified in the certificate. Question: Was Faberge v. IAC (215 SCRA 316) the controlling precedent under RA 166? State reasons for your answer. Research the cases cited in your syllabus on "Is there infringement if goods are not competing?" section of your syllabus. HW Answer (Di Bonilla): No, Faberge v. IAC was not the controlling precedent under RA 166. Since Faberge pronounced that Sec. 20 is controlling over Sec 4(d), it effectively reversed earlier decisions which merely used the "likelihood of confusion, deception or mistake" standard and which did not require similarity between the goods or articles to which the mark was being applied. Faberge held that the statement of the Court in the case of Sta. Ana vs. Maliwat that Sec 4(d) of RA 166 does not require that the subject
o 93
Katrina Michelle Mancao articles possess the same descriptive property or fall in the same category and that dissimilarity will not preclude relief if the junior user's goods are not remote from any other product which the first user would likely make or sell cannot prevail over the positive requirement of Sec. 20 of the same law that only those specified in the certificate of registration are protected. Effectively then, the Supreme Court in the Faberge case reversed its earlier ruling in Sta. Ana and other cases which merely relied on the "likelihood of confusion, deception or mistake" standard and which did not require similarity between the goods or articles. This was done with the Supreme Court not sitting en banc, in violation of the Constitution which requires the Supreme Court to be sitting en banc in order to reverse an earlier doctrine enunciated by it (Art. VIII, Sec. 3(3)).
use the mark in connection with the goods, business or services specified in the certificate is merely prima facie and is thus rebuttable. Third, the present IP Code enumerates the rights of the owner of a registered mark under Sec. 147. Subsection 147.1 grants the trademark owner the “exclusive right to prevent all third parties not having the owner's consent from using in the course of trade identical or similar signs or containers for goods or services which are identical or similar to those in respect of which the trademark is registered where such use would result in a likelihood of confusion.” In other words, the protection granted by law to a trademark owner extends to goods or services which are only SIMILAR to those in respect of which the mark is registered. Furthermore, Subsection 147.2 also recognizes the protection as applicable even to goods and services NOT SIMILAR to those in respect of which the mark is registered in the case of an internationally well known mark which is registered in the Philippines provided that the use would indicate a connection between the goods or services and the owner of the registered mark and the interests of the owner or the registered mark are likely to be damaged by such use.
Question: Is the Faberge case still good law under the IP Code? State the reasons for your answer. HW Answer (Di Bonilla) No, Faberge vs. IAC is no longer good law under the IP Code. First, the present law (under Sec. 138) now includes related goods, to wit: A certificate of registration of a mark shall be prima facie evidence of the validity of the registration, the registrant's ownership of the mark, and of the registrant's exclusive right to use the same in connection with the goods or services and those that are related thereto specified in the certificate. (emphasis supplied) This provision adds related goods or services specified in the certificate, compared to Sec. 20 of the old IP law which states that: A certificate of registration of a mark or trade-‐name shall be prima facie evidence of the validity of the registration, the registrant's ownership of the mark or trade-‐name, and of the registrant's exclusive right to use the same in connection with the goods, business or services specified in the certificate, subject to any conditions and limitations stated therein. The addition of the phrase “and those that are related thereto” should be given its significance. Second, Faberge relied heavily upon Sec. 20 of RA 166, which did not even define the rights of a trademark owner but merely established the prima facie evidentiary value of a certificate of registration. The evidentiary value of the certificate as the registrant's exclusive right to
Issue of parallel importation
YU V. CA AND UNISIA MERCHANDISING CO. (1993) Doctrine: To Our mind, the right to perform an exclusive distributorship agreement and to reap the profits resulting from such performance are proprietary rights which a party may protect which may otherwise not be diminished, nay, rendered illusory by the expedient act of utilizing or interposing a person or firm to obtain goods from the supplier to defeat the very purpose for which the exclusive distributorship was conceptualized, at the expense of the sole authorized distributor. RIGHTS CONFERRED: -‐ Under the IP Code, we have shifted to the REGISTRATION as the basis of ownership o registration is the source of right – major change in the IP Code (Jan. 1, 1998) o prior to IP Code – use was the basis of ownership of tm (RA 166) § Prior user can trump the rights of a registrant o Who would prevail between the prior user before Jan. 1, 1998 (since 1980), but neglected to file registration, and another
94 z
Intellectual Property Law
-‐
entity (registrant) who has registered its trademark before the prior user? § Prior user would prevail. Section 236 – § Ma’am: this is a grey area because it was brought about by the transition • Right has been vested prior to the effectivity of the IP Code. Such vested right cannot be impaired by the IP Code. Rights of the Registrant: o Right to prevent others from using your mark, or a confusingly similar mark, in respect to goods and services § Actual confusion is not required. What is required is a likelihood of confusion. • Used for the very same goods or services = presumption that there is a likelihood of confusion. § Problem: • Senior registrant: Cosmetics and toiletries • Junior registrant: Pesticides • Question: Competing? • Answer: o Note the IPO ruling: if the goods the same to the same class, even if not identical = subsequent registration is NOT allowed § “International class” = based on a Treaty § major factor to determine if they are similar = CLASSIFICATION o Should not be allowed even if not in the same class. The business, goodwill, image and reputation of the senior registrant may be damaged. It will cause dilution. § Buyers might think that the origin of the junior registrant’s product is the senior registrant. Thus, they might
§
§
§
§
§
o 95
be using the same harmful chemicals. o Dilution introduces confusion to the market. (term coined in Ang Tibay v. Teodoro) § Junior registrant’s use of the mark may undermine the strength of the senior registrant’s mark. It might also confuse the consuming public. Question: Would an ordinary registrant be able to claim protection from dilution under Section 147? • Answer: Depends on how “similar” would be defined. Existing jurisprudence provide for factors. Question: Why aren’t ordinary registrants not accorded protection against use by dissimilar goods? Why are there 2 different rules? • Obligation to recognize well-‐known marks is a treaty obligation. o Ma’am: But the treaty does not require broader protection. • Note: “Well-‐known” – under the IP Code, internationally AND locally Ma’am: Our law does not sufficiently protect locally-‐ established brands. à From a policy perspective, we should protect local brands, because it contributes to our national economy. Ma’am: For local brands, you should focus your arguments in similarity. Argue that it is broad enough to cover the junior goods. Then argue the “trademark dilution” doctrine from jurisprudence. You can also argue on the reasonable scope of expansion. Ma’am: Being “well-‐known” internationally is difficult to prove. There are several factors
Katrina Michelle Mancao
-‐
-‐
-‐
that should be shown. Note enumeration in 246 Corporation v. Daway. Aside: Article 8, Sec. 3, Constitution – on setting aside decision. (Important in arguing that Faberge is not the prevailing ruling under RA 166) o Under RA 166, the prevailing doctrine is provided in Chua Che and Sta. Ana cases (en banc decisions). The goods need not be similar. o Subsequent cases (Philippine Refining, Esso, Hickok) decided by division – goods must be competing. o Faberge – an extraordinary case, which is also decided by division – goods must be specifically identified in the certificate of registration (relying on Section 20 of the old law). § Ma’am: But Section 20 does not deal with the scope of protection. It only stated that the certificate of registration shall be a prima facie evidence of tm ownership. • This ruling requires applicant to apply for a lot of goods, and applicant must also use the same mark for the products enumerated therein. § But this case found its way in Canon Kabushiki and Pearl & Dean cases, giving the impression that it is the prevailing ruling. • Canon and Pearl & Dean were decided after the IP Code, but involved facts occurring prior to the effectivity of the IP Code. § Faberge is no longer effective under the IP Code. There is now a change in the phraseology of the similar provision – Section 138 under the IP Code. Furthermore, Section 147 was clear in defining the scope of protection under the IP Code. o 246 Corporation – recognized the changed in phraseology. Illustration of dilution: Seiko watch and Seiko wallet à The two have different markets. Because of the use of Seiko wallet, Seiko watch lost its aspirational value. What about a well-‐known mark that is not
-‐
registered? Not protected against dilution. o Ma’am: But easy way out is invoking Levi Strauss by Justice Tinga. Issue of Parallel importation: Does a registrant have the right to prevent the importation of genuine goods by third parties? NO. o Note the strategy of pharmaceutical companies – entering into licensing agreements. This is a tax saving strategy. o The IP Code did not give a trademark registrant the exclusive right to import (they do not have the right given to patent holders). Importation of genuine goods manufactured elsewhere is not prohibited from entering the Philippines. o NOW: Cheaper Medicines Act o Yu v. CA – not about trademark law. This is about contract law. Injunction was issued because the other party fraudulently misled the trademark owner à the Court used third party contractual tort. Thus, this may not be used to argue that trademark owner has the right to prevent parallel importation. o Ma’am: Recourse of the exclusive distributor should be against the trademark owner. It’s hard to go against the parallel importer because it is not privy to the exclusive distributorship agreement. Thus, make sure that the agreement allows the exclusive distributor to go against the trademark owner, or at least to ask compensation for any parallel importation.
I.
Remedies
SECTION 3. INTERNATIONAL RECIPROCITY. –
CONVENTIONS
AND
Any person who is a national or who is domiciled or has a real and effective industrial establishment in a country which is a party to any convention, treaty or agreement relating to intellectual property rights or the repression of unfair competition, to which the Philippines is also a party, or extends reciprocal rights to nationals of the Philippines by law, shall be entitled to benefits to the extent necessary to give effect to any provision of such convention, treaty or reciprocal law, IN ADDITION to the rights to which any owner of an intellectual property right is otherwise entitled by this Act.
SECTION 160. RIGHT OF FOREIGN CORPORATION TO SUE IN TRADEMARK OR SERVICE MARK ENFORCEMENT ACTION. – Any foreign national or juridical person who meets the requirements of Section 3 of this Act and does not engage in business in the Philippines may bring a CIVIL
96 z
Intellectual Property Law "facts showing the capacity of a party to sue or be sued or the authority of a party to sue or be sued in a representative capacity or the legal existence of an organized association of persons that is made a party, must be averred " In the case at bar, private respondent has chosen to anchor its action under the Trademark Law of the Philippines, a law which, as pointed out, explicitly sets down the conditions precedent for the successful prosecution thereof. It is therefore incumbent upon private respondent to comply with these requirements or aver its exemption therefrom, if such be the case. It may be that private respondent has the right to sue before Philippine courts, but our rules on pleadings require that the necessary qualifying circumstances which clothe it with such right be affirmatively pleaded.
or ADMINISTRATIVE action hereunder for opposition, cancellation, infringement, unfair competition, or false designation of origin and false description, whether or not it is licensed to do business in the Philippines under existing laws.
SECTION 231. REVERSE RECIPROCITY OF FOREIGN LAWS. – Any condition, restriction, limitation, diminution, requirement, penalty or any similar burden imposed by the law of a foreign country on a Philippine national seeking protection of intellectual property rights in that country, shall reciprocally be enforceable upon nationals of said country, within Philippine jurisdiction.
LEVITON INDUSTRIES V. SALVADOR (1982) Doctrine: We agree with petitioners that respondent Leviton Marketing Co., Inc. had failed to allege the essential facts bearing upon its capacity to sue before Philippine courts. Private respondent's action is squarely founded on Section 21-‐A of Republic Act No. 166, as amended, which we quote: Sec. 21-‐A. Any foreign corporation or juristic person to which a mark or tradename has been registered or assigned under this Act may bring an action hereunder for infringement, for unfair competition, or false designation of origin and false description, whether or not it has been licensed to do business in the Philippines under Act numbered Fourteen Hundred and Fifty-‐Nine, as amended, otherwise known as the Corporation Law, at the time it brings the complaint; Provided, That the country of which the said foreign corporation or juristic person is a citizen, or in which it is domiciled, by treaty, convention or law, grants a similar privilege to corporate or juristic persons of the Philippines. (As amended by R.A. No. 638) Undoubtedly, the foregoing section grants to a foreign corporation, whether or not licensed to do business in the Philippines, the right to seek redress for unfair competition before Philippine courts. But the said law is not without qualifications. Its literal tenor indicates as a condition sine qua non the registration of the trade mark of the suing foreign corporation with the Philippine Patent Office or, in the least, that it be an asignee of such registered trademark. The said section further requires that the country, of which the plaintiff foreign corporation or juristic person is a citizen or domicilliary, grants to Filipino corporations or juristic entities the same reciprocal treatment, either thru treaty, convention or law, All that is alleged in private respondent's complaint is that it is a foreign corporation. Such bare averment not only fails to comply with the requirements imposed by the aforesaid Section 21-‐A but violates as well the directive of Section 4, Rule 8 of the Rules of Court that
PUMA V. IAC (1988) Refresher: The petitioner, a foreign corporation duly organized and existing under the laws of the Federal Republic of Germany and the manufacturer and producer of "PUMA PRODUCTS," filed a complaint for infringement of patent or trademark with a prayer for the issuance of a writ of preliminary injunction against the private respondent before the Regional Trial Court of Makati. Doctrine: The Petitioner may still bring the action. Court quoted the decision in La Chemise Lacoste v. Fernandez (infra). Petitioner maintains that it has substantially complied with the requirements of Section 21-‐A of Republic Act R.A. No. 166, as amended. According to the petitioner, its complaint specifically alleged that it is not doing business in the Philippines and is suing under the said Republic Act; that Section 21-‐A thereof provides that "the country of which the said corporation or juristic person is a citizen, or in which it is domiciled, by treaty, convention or law, grants a similar privilege to corporate or juristic persons of the Philippines" but does not mandatorily require that such reciprocity between the Federal Republic of Germany and the Philippines be pleaded; that such reciprocity arrangement is embodied in and supplied by the Union Convention for the Protection of Industrial Property Paris Convention) to which both the Philippines and Federal Republic of Germany are signatories and that since the Paris Convention is a treaty which, pursuant to our Constitution, forms part of the law of the land, our courts are bound to take judicial notice of such treaty, and, consequently, this fact need not be averred in the complaint. The Court agreed.
o 97
Katrina Michelle Mancao Corporation The latter is an independent entity which buys and then markets not only products of the petitioner but also many other products bearing equally well-‐known and established trademarks and tradenames. in other words, Rustan is not a mere agent or conduit of the petitioner. The rules and regulations promulgated by the Board of Investments pursuant to its rule-‐making power under Presidential Decree No. 1789, otherwise known as the Omnibus Investment Code, support a finding that the petitioner is not doing business in the Philippines. Rule I, Sec. 1 (g) of said rules and regulations defines "doing business" as one" which includes, inter alia: (1) A foreign firm which does business through middlemen acting on their own names, such as indentors, commercial brokers or commission merchants, shall not be deemed doing business in the Philippines. But such indentors, commercial brokers or commission merchants shall be the ones deemed to be doing business in the Philippines. (2) Appointing a representative or distributor who is domiciled in the Philippines, unless said representative or distributor has an independent status, i.e., it transacts business in its name and for its account, and not in the name or for the account of a principal Thus, where a foreign firm is represented by a person or local company which does not act in its name but in the name of the foreign firm the latter is doing business in the Philippines. Applying the above provisions to the facts of this case, we find and conclude that the petitioner is not doing business in the Philippines. Rustan is actually a middleman acting and transacting business in its own name and or its own account and not in the name or for the account of the petitioner. But even assuming the truth of the private respondent's allegation that the petitioner failed to allege material facts in its petition relative to capacity to sue, the petitioner may still maintain the present suit against respondent Hemandas. As early as 1927, this Court was, and it still is, of the view that a foreign corporation not doing business in the Philippines needs no license to sue before Philippine courts for infringement of trademark and unfair competition. Thus, in Western Equipment and Supply Co. v. Reyes (51 Phil. 115), this Court held that a foreign corporation which has never done any business in the Philippines and which is unlicensed and unregistered to do business here, but is widely and favorably known in the Philippines through the use therein of its products bearing its corporate and tradename, has a legal right to maintain an action in the
LA CHEMISE LACOSTE V. FERNANDEZ (1984) Doctrine: Hemandas argues in his comment on the petition for certiorari that the petitioner being a foreign corporation failed to allege essential facts bearing upon its capacity to sue before Philippine courts. He states that not only is the petitioner not doing business in the Philippines but it also is not licensed to do business in the Philippines. He also cites the case of Leviton Industries v. Salvador to support his contention The Leviton case, however, involved a complaint for unfair competition under Section 21-‐A of Republic Act No. 166 which provides: Sec. 21 — A. Any foreign corporation or juristic person to which a mark or tradename has been registered or assigned under this Act may bring an action hereunder for infringement, for unfair competition, or false designation of origin and false description, whether or not it has been licensed to do business in the Philippines under Act numbered Fourteen Hundred and Fifty-‐Nine, as amended, otherwise known as the Corporation Law, at the time it brings the complaint; Provided, That the country of which the said foreign corporation or juristic person is a citizen, or in which it is domiciled, by treaty, convention or law, grants a similar privilege to corporate or juristic persons of the Philippines. We held that it was not enough for Leviton, a foreign corporation organized and existing under the laws of the State of New York, United States of America, to merely allege that it is a foreign corporation. It averred in Paragraph 2 of its complaint that its action was being filed under the provisions of Section 21-‐A of Republic Act No. 166, as amended. Compliance with the requirements imposed by the abovecited provision was necessary because Section 21-‐A of Republic Act No. 166 having explicitly laid down certain conditions in a specific proviso, the same must be expressly averred before a successful prosecution may ensue. It is therefore, necessary for the foreign corporation to comply with these requirements or aver why it should be exempted from them, if such was the case. The foreign corporation may have the right to sue before Philippine courts, but our rules on pleadings require that the qualifying circumstances necessary for the assertion of such right should first be affirmatively pleaded. In contradistinction, the present case involves a complaint for violation of Article 189 of the Revised Penal Code. The Leviton case is not applicable. In the present case, however, the petitioner is a foreign corporation not doing business in the Philippines. The marketing of its products in the Philippines is done through an exclusive distributor, Rustan Commercial
98 z
Intellectual Property Law Philippines to restrain the residents and inhabitants thereof from organizing a corporation therein bearing the same name as the foreign corporation, when it appears that they have personal knowledge of the existence of such a foreign corporation, and it is apparent that the purpose of the proposed domestic corporation is to deal and trade in the same goods as those of the foreign corporation. We further held: That company is not here seeking to enforce any legal or control rights arising from, or growing out of, any business which it has transacted in the Philippine Islands. The sole purpose of the action is to protect its reputation, its corporate name, its goodwill, whenever that reputation, corporate name or goodwill have, through the natural development of its trade, established themselves.' And it contends that its rights to the use of its corporate and trade name is a property right, a right in rem, which it may assert and protect against all the world, in any of the courts of the world-‐even in jurisdictions where it does not transact business-‐just the same as it may protect its tangible property, real or personal, against trespass, or conversion. That point is sustained by the authorities, and is well stated in Hanover Star Mining Co. v. Allen and Wheeler Co. in which the syllabus says, “Since it is the trade and not the mark that is to be protected, a trade-‐mark acknowledges no territorial boundaries of municipalities or states or nations, but extends to every market where the trader's goods have become known and Identified by the use of the mark.” Our recognizing the capacity of the petitioner to sue is not by any means novel or precedent setting. Our jurisprudence is replete with cases illustrating instances when foreign corporations not doing business in the Philippines may nonetheless sue in our courts. In East Board Navigation Ltd, v. Ysmael and Co., Inc., we recognized a right of foreign corporation to sue on isolated transactions. In General Garments Corp. v. Director of Patents, we sustained the right of Puritan Sportswear Corp., a foreign corporation not licensed to do and not doing business in the Philippines, to file a petition for cancellation of a trademark before the Patent Office. In upholding the right of the petitioner to maintain the present suit before our courts for unfair competition or infringement of trademarks of a foreign corporation, we are moreover recognizing our duties and the rights of foreign states under the Paris Convention for the
Protection of Industrial Property to which the Philippines and France are parties. We are simply interpreting and enforcing a solemn international commitment of the Philippines embodied in a multilateral treaty to which we are a party and which we entered into because it is in our national interest to do so. REMEDIES 1) Administrative a. Opposition b. Cancellation c. Intellectual Property right Violation (IPV – new to IP Code) d. Customs: Border enforcement (for counterfeit goods) 2) Criminal a. Infringement b. Unfair competition c. False designation of origin d. Section 162 – no specific criminal remedy, but you can file for perjury 3) Civil a. Infringement b. Unfair competition c. False designation of origin d. False or fraudulent declaration (most useless) Who has the remedy? All persons who satisfies the requisite in Section 3. -‐ Plead capacity to sue. Otherwise, petition will be vulnerable to a Motion to Dismiss. -‐ Easiest: Allege that it is a domiciliary of a state that is a party to a Convention or Treaty with the Philippines. This is easier because then, the court can take judicial notice. -‐ If foreign corporation not registered BUT doing business in the Philippines, no remedy. General rule in Corporation Code applies. Foreign corporation not given a remedy. Note: Causes of action in criminal and civil remedies may also be filed as IPV. IPV is broad enough to cover infringement, unfair competition, false designation of origin, false or fraudulent declaration.
1.
Administrative
a.
Cancellation proceedings
SECTION 151. CANCELLATION. -‐ 151.1. A PETITION TO CANCEL a registration of a mark under this Act may be filed with the Bureau of Legal Affairs by any person who believes that he is or will be
o 99
Katrina Michelle Mancao damaged by the registration of a mark under this Act as follows: (a) Within five (5) years from the date of the registration of the mark under this Act. (b) AT ANY TIME, if the registered mark becomes the generic name for the goods or services, or a portion thereof, for which it is registered, or has been abandoned, or its registration was obtained fraudulently or contrary to the provisions of this Act, or if the registered mark is being used by, or with the permission of, the registrant so as to misrepresent the source of the goods or services on or in connection with which the mark is used. If the registered mark becomes the generic name for less than all of the goods or services for which it is registered, a petition to cancel the registration for only those goods or services may be filed. A registered mark shall not be deemed to be the generic name of goods or services solely because such mark is also used as a name of or to identify a unique product or service. The primary significance of the registered mark to the relevant public rather than purchaser motivation shall be the TEST FOR DETERMINING WHETHER THE REGISTERED MARK HAS BECOME THE GENERIC name of goods or services on or in connection with which it has been used. (c) At any time, if the registered owner of the mark without legitimate reason fails to use the mark within the Philippines, or to cause it to be used in the Philippines by virtue of a license during an uninterrupted period of three (3) years or longer. 151.2. Notwithstanding the foregoing provisions, the court or the administrative agency vested with jurisdiction to hear and adjudicate any action to enforce the rights to a registered mark shall likewise exercise jurisdiction to determine whether the registration of said mark may be cancelled in accordance with this Act. The filing of a suit to enforce the registered mark with the proper court or agency shall exclude any other court or agency from assuming jurisdiction over a subsequently filed petition to cancel the same mark. On the other hand, the earlier filing of petition to cancel the mark with the Bureau of Legal Affairs shall not constitute a prejudicial question that must be resolved before an action to enforce the rights to same registered mark may be decided.
152.2. The use of the mark in a form different from the form in which it is registered, which does not alter its distinctive character, shall not be ground for cancellation or removal of the mark and shall not diminish the protection granted to the mark. 152.3. The use of a mark in connection with one or more of the goods or services belonging to the class in respect of which the mark is registered shall prevent its cancellation or removal in respect of all other goods or services of the same class. 152.4. The use of a mark by a company related with the registrant or applicant shall inure to the latter's benefit, and such use shall not affect the validity of such mark or of its registration: PROVIDED, That such mark is not used in such manner as to deceive the public. If use of a mark by a person is controlled by the registrant or applicant with respect to the nature and quality of the goods or services, such use shall inure to the benefit of the registrant or applicant.
SECTION 153. REQUIREMENTS OF PETITION; NOTICE AND HEARING. – Insofar as applicable, the petition for cancellation shall be in the same form as that provided in Section 1348 hereof, and notice and hearing shall be as provided in Section 1359 hereof.
SECTION 154. CANCELLATION OF REGISTRATION. – If the Bureau of Legal Affairs finds that a case for cancellation has been made out, it shall order the cancellation of the registration. When the order or judgment BECOMES FINAL, any right conferred by such registration upon the registrant or any person in interest of record shall terminate. Notice of cancellation shall be published in the IPO Gazette.
8 Section 134. Opposition. – x x x Such opposition shall be in writing and verified by the oppositor or by any person on his behalf who knows the facts, and shall specify the grounds on which it is based and include a statement of the facts relied upon. Copies of certificates of registration of marks registered in other countries or other supporting documents mentioned in the opposition shall be filed therewith, together with the translation in English, if not in the English language. For good cause shown and upon payment of the required surcharge, the time for filing an opposition may be extended by the Director of Legal Affairs, who shall notify the applicant of such extension. The Regulations shall fix the maximum period of time within which to file the opposition. 9 Section 135. Notice and Hearing. -‐ Upon the filing of an opposition, the Office shall serve notice of the filing on the applicant, and of the date of the hearing thereof upon the applicant and the oppositor and all other persons having any right, title or interest in the mark covered by the application, as appear of record in the Office.
SECTION 152. NON-‐USE OF A MARK WHEN EXCUSED. – 152.1. Non-‐use of a mark may be excused if caused by circumstances arising independently of the will of the trademark owner. LACK OF FUNDS shall not excuse non-‐ use of a mark.
100 z
Intellectual Property Law brassiere, does not affect its registrability as a trademark.” There being no evidence of use of the mark by others before 1932, or that appellee abandoned use thereof, the registration of the mark was made in accordance with the Trademark Law. Granting that appellant used the mark when appellee stopped using it during the period of time that the Government imposed restrictions on importation of respondent's brassiere bearing the trademark, such temporary non-‐use did not affect the rights of appellee because it was occasioned by government restrictions and was not permanent, intentional, and voluntary. To work an abandonment, the disuse must be permanent and not ephemeral; it must be intentional and voluntary, and not involuntary or even compulsory. There must be a thorough-‐going discontinuance of any trade-‐ mark use of the mark in question. The use of the trademark by other manufacturers did not indicate an intention on the part of appellee to abandon it. Non-‐use because of legal restrictions is not evidence of an intent to abandon. Non-‐use of their ancient trade-‐ mark and the adoption of new marks by the Carthusian Monks after they had been compelled to leave France was consistent with an intention to retain their right to use their old mark. Abandonment will not be inferred from a disuse over a period of years occasioned by statutory restrictions on the name of liquor.
SECTION 230. EQUITABLE PRINCIPLES TO GOVERN PROCEEDINGS. – In all inter partes proceedings in the Office under this Act, the equitable principles of laches, estoppel, and acquiescence where applicable, may be considered and applied.
SECTION 232.2. Unless expressly provided in this Act or other statutes, appeals from decisions of administrative officials shall be provided in the Regulations.
ROMERO V. MAIDEN FORM BRASSIERE CO. (1964) Refresher: Maiden Form filed an application for registration of the trademark "Adagio" for the brassieres manufactured by it with the Director of Patents. In its application, respondent company alleged that said trademark was first used by it in the United States on October 26, 1937, and in the Philippines on August 31, 1946; that it had been continuously used by it in trade in, or with the Philippines for over 10 years; that said trademark "is on the date of this application, actually used by respondent company on the following goods, classified according to the official classification of goods (Rule 82) -‐ Brassieres, Class 40"; and that said trademark is applied or affixed by respondent to the goods by placing thereon a woven label on which the trademark is shown. Petitioner filed with the Director a petition for cancellation of the trademark on the grounds that it is a common descriptive name of an article or substance on which the patent has expired. Petitioner alleged that said trademark has not become distinctive of respondent company's goods or business; that it has been used by respondent company to classify the goods (the brassieres) manufactured by it, in the same manner as petitioner uses the same; that said trademark has been used by petitioner for almost 6 years; that it has become a common descriptive name; and that it is not registered in accordance with the requirements of Section 37(a), Chapter XI of Republic Act No. 166. Doctrine: The evidence shows that the trademark "Adagio" is a musical term, which means slowly or in an easy manner, and was used as a trademark by the owners thereof (the Rosenthals of Maiden Form Co., New York) because they are musically inclined. Being a musical term, it is used in an arbitrary (fanciful) sense as a trademark for brassieres manufactured by respondent company. It also appears that respondent company has, likewise, adopted other musical terms such as "Etude", "Chansonette", "Prelude", "Over-‐ture", and "Concerto", to identify, as a trademark, the different styles or types of its brassieres. As respondent Director pointed out, "the fact that said mark is used also to designate a particular style of
PHILIPPINE NUT INDUSTRY V. STANDARD BRANDS, INC. (1975) Refresher: Philippine Nut, a domestic corporation, obtained from the Patent office a certificate of registration covering the trademark "Philippine Planters Cordial Peanuts," the label used on its product of salted peanuts. Standard Brands, a foreign corporation, filed a case for the cancellation of Philippine Nut's certificate of registration on the ground that such trademark closely resembles, and is confusingly similar to its trademark "Planters Cocktail Peanuts." Issue 1: The first argument advanced by petitioner which We believe goes to the core of the matter in litigation is that the Director of Patents erred in holding that the dominant portion of the label of Standard Brands in its cans of salted peanuts consists of the word PLANTERS which has been used in the label of Philippine Nut for its own product. According to petitioner, PLANTERS cannot be considered as the dominant feature of the
o 101
Katrina Michelle Mancao trademarks in question because it is a mere descriptive term, an ordinary word which is defined in Webster International Dictionary as "one who or that which plants or sows, a farmer or an agriculturist." Held: We find the argument without merit. While it is true that PLANTERS is an ordinary word, nevertheless it is used in the labels not to describe the nature of the product, but to project the source or origin of the salted peanuts contained in the cans. The word PLANTERS printed across the upper portion of the label in bold letters easily attracts and catches the eye of the ordinary consumer and it is that word and none other that sticks in his mind when he thinks of salted peanuts. In cases of this nature there can be no better evidence as to what is the dominant feature of a label and as to whether there is a confusing similarity in the contesting trademarks than the labels themselves. A visual and graphic presentation of the labels will constitute the best argument for one or the other. It is true that there are other words used such as "Cordial" in petitioner's can and "Cocktail" in Standard Brands', which are also prominently displayed, but these words are mere adjectives describing the type of peanuts in the labeled containers and are not sufficient to warn the unwary customer that the two products come form distinct sources. As a whole it is the word PLANTERS which draws the attention of the buyer and leads him to conclude that the salted peanuts contained in the two cans originate from one and the same manufacturer. In fact, when a housewife sends her housemaid to the market to buy canned salted peanuts, she will describe the brand she wants by using the word PLANTERS and not "Cordial" nor "Cocktail". Issue 2:* What is next submitted by petitioner is that it was error for respondent Director to have enjoined it from using PLANTERS in the absence of evidence showing that the term has acquired secondary meaning. Petitioner, invoking American jurisprudence, asserts that the first user of a tradename composed of common words is given no special preference unless it is shown that such words have acquired secondary meaning, and this, respondent Standard Brands failed to do when no evidence was presented to establish that fact. Held: This Court held that the doctrine is to the effect that a word or phrase originally incapable of exclusive appropriation with reference to an article on the market, because geographically or otherwise descriptive, might nevertheless have been used so long and so exclusively by one producer with reference to his article that, in that trade and to that branch of the purchasing public, the
word or phrase has come to mean that the article was his product. The applicability of the doctrine of secondary meaning to the situation now before Us is appropriate because there is oral and documentary evidence showing that the word PLANTERS has been used by and closely associated with Standard Brands for its canned salted peanuts since 1938 in this country. Not only is that fact admitted by petitioner in the amended stipulation of facts, but the matter has been established by testimonial and documentary evidence consisting of invoices covering the sale of "PLANTERS cocktail peanuts". In other words, there is evidence to show that the term PLANTERS has become a distinctive mark or symbol insofar as salted peanuts are concerned, and by priority of use dating as far back as 1938, respondent Standard Brands has acquired a preferential right to its adoption as its trademark warranting protection against its usurpation by another. Ubi jus ibi remedium. Where there is a right there is a remedy. Standard Brands has shown the existence of a property right and respondent Director, has afforded the remedy. Still on this point, petitioner contends that Standard Brands' use of the trademark PLANTERS was interrupted during the Japanese occupation and in fact was discontinued when the importation of peanuts was prohibited by Central Bank regulations effective July 1, 1953, hence it cannot be presumed that it has acquired a secondary meaning. We hold otherwise. Respondent Director correctly applied the rule that non-‐use of a trademark on an article of merchandize due to legal restrictions or circumstances beyond one's control is not to be considered as an abandonment.
ANCHOR TRADING CO. V. DIRECTOR OF PATENTS (1956) Doctrine: The principal legal issue here is whether or not the failure of the respondent-‐appellee to register his opposition to the petition for registration in due time estops him from asking for the cancellation of the certificate of registration issued in favor of the petitioner-‐appellant. The issue should be resolved in the negative. The only consequence resulting from a late filing of an opposition to any application for registration of a trademark is the oppositor’s relinquishment of the privilege given to him by laws to object to such registration, but such cannot prevent him from asking later for its cancellation when there are good grounds justifying it.
THE C LOROX CO. V. DIRECTOR OF PATENTS (1967) Refresher: On April 7, 1959, respondent Go Siu Gian filed with the Patent Office an application for registration of the trademark "OLDROX," with an accompanying statement that he is a citizen of China, residing and doing business
102 z
Intellectual Property Law in the Philippines at 838 Folgueras St., Manila; that he has adopted the trademark "OLDROX" for his goods (whitening agent for bleaching) in trade and commerce in the country; and that said trademark, shown on printed labels affixed to the goods, or to the containers thereof, has been used by him since February 1, 1959. The application was allowed and published in the issue of the Official Gazette dated April 25, 1960, which was released for circulation on August 22, 1960. On September 21 of the same year, or within 30 days from the date of its publication in the Official Gazette, an unverified opposition to the application was filed by the law firm Lichauco, Picazo and Agcaoili in behalf of the Clorox Company, herein petitioner. Doctrine: The rule is well settled that courts may vacate judgments and grant new trials or enter new judgments on the grounds of error in fact or in law. They have no power, of course, to vacate judgments after they have become final, in the sense that the party in whose favor they are rendered is entitled as of right, to have execution thereon, but prior thereto, the courts have plenary control over the proceedings including the judgment, and in the exercise of a sound judicial discretion, may take such proper action in this regard as truth and justice may require. The order of herein respondent dismissing the opposition of petitioner to the registration of the trademark in question may amount to considerable injustice to the opposer Clorox Company, the order having been entered not upon the merits of the controversy; and the possibility of such serious consequences necessitates a careful examination of the grounds upon which it requests that the order be set aside. It must be remembered that the only discretion conferred upon officers is a legal discretion, and when anything is left to any officer to be done according to his discretion, the law intends it to be done with a sound discretion and according to law. And when, as in this case, the allegation of the pleading clearly show circumstances constituting mistake and excusable negligence which are grounds for a motion for reconsideration of the order in question, a dismissal of the motion and a denial of the relief sought upon the flimsy excuse that the same was filed as a petition for relief, will amount to an abuse of that discretion. Neither may we consider the argument of herein respondent that the petitioner is not totally deprived of its right to question the registration of the trademark in question because it may still pursue a cancellation proceeding under Sections 17 to 19 of Republic Act No. 166, and Rules 191 to 197 of the Rules of Practice in Trademark Cases. The opposition to a registration and the petition for cancellation are alternative proceedings which a party may avail of according to his purposes, needs, and predicaments, and herein petitioner has the right to
choose which remedy it deems best for the protection of its rights.
WOLVERINE WORLDWIDE, INC. V. CA AND LOLITO CRUZ (1989) Refresher: On February 8, 1984, the petitioner, a foreign corporation organized and existing under the laws of the United States, brought a petition before the Philippine Patent Office, docketed as Inter Partes Case No. 1807, for the cancellation of Certificate of Registration No. 24986-‐B of the trademark HUSH PUPPIES and DOG DEVICE issued to the private respondent, a Filipino citizen. In support of its petition for cancellation, the petitioner alleged, inter alia, that it is the registrant of the internationally known trademark HUSH PUPPIES and the DEVICE of a Dog in the United States and in other countries which are members of the Paris Convention for the Protection of Industrial Property; that the goods sold by the private respondent, on the one hand, and by the petitioner, on the other hand, belong to the same class such that the private respondent's use of the same trademark in the Philippines (which is a member of said Paris Convention) in connection with the goods he sells constitutes an act of unfair competition, as denied in the Paris Convention. Subsequently, the private respondent moved to dismiss the petition on the ground of res judicata, averring that in 1973, or more than ten years before this petition (Inter Partes Case No. 1807) was filed, the same petitioner filed two petitions for cancellation (Inter Partes Cases Nos. 700 and 701) and was a party to an interference proceeding (Inter Partes Case No. 709), all of which involved the trademark HUSH PUPPIES and DEVICE, before the Philippine Patent Office. The Director of Patents had ruled in all three inter parties cases in favor of Ramon Angeles, the private respondent's predecessor-‐in-‐interest, to wit: WHEREFORE, for all the foregoing considerations, 1. The petitions seeking cancellation of Registration Nos. SR-‐1099 and SR-‐1526, respectively, are both denied and accordingly DISMISSED; 2. Respondent-‐Registrant/Junior Party-‐ Applicant, Roman Angeles, is hereby adjudged as the prior user and adopter of the trademark HUSH PUPPIES & DEVICE, under Appl. Serial No. 17174, and therefore, the same given due course; and
o 103
Katrina Michelle Mancao 3. Registration No. 14969 of Dexter Sales Company, assignor to Wolverine Worldwide, Inc., covering the trademark HUSH PUPPIES & Representation of a Dogie Head, is hereby CANCELLED.
Between the earlier petitions and the present one there is substantial identity of parties, subject matter, and cause of action. The petitioner in all of these cases is Wolverine Worldwide, Inc. The respondent-‐registrant in this case is the assignee of Randelson Agro-‐Industrial Development, Inc. (formerly known as Randelson Shoes, Inc.) which in turn, acquired its right from Ramon Angeles, the original respondents-‐registrant. As regards the subject matter, all of these cases refer to the cancellation of registration of the trademark HUSH PUPPIES and DEVICE of a Dog. Finally, there is identity of cause of action, which is the alleged wrongful or erroneous registration of the trademark.
Doctrine: The Court has repeatedly held that for a judgment to be a bar to a subsequent case, the following requisites must concur: (1) it must be a final judgment; (2) the court which rendered it had jurisdiction over the subject matter and the parties; (3) it must be a judgment on the merits; and (4) there must be Identity between the two cases, as to parties, — subject matter, and cause of action. Contrary to the petitioner's assertion, the judgment in Inter Partes Cases Nos. 700, 701, and 709 had long since become final and executory. That Sec. 17 of Republic Act 166, also known as the Trademark Law, allows the cancellation of a registered trademark is not a valid premise for the petitioner's proposition that a decision granting registration of a trademark cannot be imbued with the character of absolute finality as is required in res judicata. A judgment or order is final, as to give it the authority of res judicata, if it can no longer be modified by the court issuing it or by any other court. In the case at bar, the decision of the Court of Appeals affirming that of the Director of Patents, in the cancellation cases filed in 1973, was never appealed to us. Consequently, when the period to appeal from the Court of Appeals to this Court lapsed, with no appeal having been perfected, the foregoing judgment denying cancellation of registration in the name of private respondent's predecessor-‐in-‐interest but ordering cancellation of registration in the name of the petitioner's predecessor-‐ in-‐interest, became the settled law in the case. It must be stressed anew that, generally, the fundamental principle of res judicata applies to all cases and proceedings in whatever form they may be. We now expressly affirm that this principle applies, in the appropriate cases, to proceedings for cancellation of trademarks before the Philippine Patent Office (now Bureau of Patents, Trademarks and Technology Transfer). Undoubtedly, final decisions, orders, and resolutions, of the Director of Patents are clothed with a judicial character as they are, in fact, reviewable by the Court of Appeals and by us. The subject judgment is undeniably on the merits of the case, rendered after both parties and actually submitted their evidence.
SHANGRILA V. CA (1999) Court decision: The earlier institution of an inter partes case for cancellation of a registered service mark and device/logo with BPTTT cannot effectively bar the subsequent filing of an infringement case by the registrant. REASON: The certificate of registration upon which the infringement case is based remains valid and subsisting as long as it has not been cancelled by the Bureau or by an infringement court. As such, the registration continues as prima facie evidence of the validity of registration, registrant’s ownership of the mark and tradename and of the registrant’s exclusive right to use the same in connection with the goods, business or services specified in the certificate. In the same light that the infringement case can and should proceed independently from the cancellation case with the Bureau so as to afford the owner of certificates of registration redress and injunctive reliefs, so must the cancellation with the BPTTT continue independently from the infringement case so as to determine whether a registered mark may ultimately be cancelled. To provide a judicious resolution of the issues, the Court finds it apropos to order the suspension of the proceedings before the Bureau pending final determination of the infringement case, where the issue of validity of the registration of the subject TM and logo in the name of Developers group was passed upon. With the decision of the RTC upholding the validity of registration of the service mark “Shangri-‐La” and S logo in the name of Developers Group, the cancellation case filed with the Bureau becomes moot. To allow the Bureau to proceed with the cancellation case would lead
104 z
Intellectual Property Law to a possible result contradictory to what the RTC has rendered, albeit the same is on appeal.
voluntary assurance may include one or more of the following: (1) An assurance to comply with the provisions of the intellectual property law violated; (2) An assurance to refrain from engaging in unlawful and unfair acts and practices subject of the formal investigation; (3) An assurance to recall, replace, repair, or refund the money value of defective goods distributed in commerce; and (4) An assurance to reimburse the complainant the expenses and costs incurred in prosecuting the case in the Bureau of Legal Affairs. The Director of Legal Affairs may also require the respondent to submit periodic compliance reports and file a bond to guarantee compliance of his undertaking; (iii) The condemnation or seizure of products which are subject of the offense. The goods seized hereunder shall be disposed of in such manner as may be deemed appropriate by the Director of Legal Affairs, such as by sale, donation to distressed local governments or to charitable or relief institutions, exportation, recycling into other goods, or any combination thereof, under such guidelines as he may provide; (iv) The forfeiture of paraphernalia and all real and personal properties which have been used in the commission of the offense; (v) The imposition of administrative fines in such amount as deemed reasonable by the Director of Legal Affairs, which shall in no case be less than Five thousand pesos (P5,000) nor more than One hundred fifty thousand pesos (P150,000). In addition, an additional fine of not more than One thousand pesos (P1,000) shall be imposed for each day of continuing violation; (vi) The cancellation of any permit, license, authority, or registration which may have been granted by the Office, or the suspension of the validity thereof for such period of time as the Director of Legal Affairs may deem reasonable which shall not exceed one (1) year; (vii) The withholding of any permit, license, authority, or registration which is being secured by the respondent from the Office;
SUPERIOR COMMERCIAL ENTERPRISES V. KUNNAN ENTERPRISES (2010) Refresher: Sec. 22 RA 166 provides that only a registrant can file a case for infringement. Corollary to this, Sec. 19 RA 166 provides that nay right conferred upon the registration under the provision of RA 166 terminates when the judgment or order of cancellation has become final. Hence, the cancellation of a registration of TM has the effect of depriving the registrant of protection from infringement the moment judgment or order of cancellation has become final. In trademark infringement, title to TM is indispensable to a valid cause of action and such title is shown by its certificate of registration. In the absence of any inequitable conduct on the part of the manufacturer, an exclusive distributor who employs the TM of the manufacturer does not acquire proprietary rights of the manufacturer and a registration of the TM by the distributor as such belongs to the manufacturer, provided the fiduciary relationship does not terminate before application for registration is filed.
b.
Intellectual property rights violations
SECTION 10.2. THE BUREAU OF LEGAL AFFAIRS (a) Exercise original jurisdiction in administrative complaints for violations of laws involving intellectual property rights: Provided, That its jurisdiction is limited to complaints where the total damages claimed are not less than Two hundred thousand pesos (P200,000): Provided further, That availment of the provisional remedies may be granted in accordance with the Rules of Court. The Director of Legal Affairs shall have the power to hold and punish for contempt all those who disregard orders or writs issued in the course of the proceedings. (b) After formal investigation, the Director for Legal Affairs may impose one (1) or more of the following administrative penalties: (i) The issuance of a cease and desist order which shall specify the acts that the respondent shall cease and desist from and shall require him to submit a compliance report within a reasonable time which shall be fixed in the order; (ii) The acceptance of a voluntary assurance of compliance or discontinuance as may be imposed. Such
o 105
Katrina Michelle Mancao (viii) (ix) (x)
The assessment of damages;
Court of Appeals held that BLA did not have jurisdiction over the complaint for unfair competition, as jurisdiction is vested in the regular courts. The Supreme Court disagreed with the Court of Appeals. Section 10 of the Intellectual Property Code specifically identifies the functions of the Bureau of Legal Affairs. Unquestionably, petitioner’s complaint, which seeks the cancellation of the disputed mark in the name of respondent Sehwani, Incorporated, and damages for violation of petitioner’s intellectual property rights, falls within the jurisdiction of the IPO Director of Legal Affairs. The Intellectual Property Code also expressly recognizes the appellate jurisdiction of the IPO Director General over the decisions of the IPO Director of Legal Affairs, to wit: Section 7. The Director General and Deputies Director General. 7.1 Fuctions.–The Director General shall exercise the following powers and functions: x x x x b) Exercise exclusive appellate jurisdiction over all decisions rendered by the Director of Legal Affairs, the Director of Patents, the Director of Trademarks, and the Director of Documentation, Information and Technology Transfer Bureau. The decisions of the Director General in the exercise of his appellate jurisdiction in respect of the decisions of the Director of Patents, and the Director of Trademarks shall be appealable to the Court of Appeals in accordance with the Rules of Court; and those in respect of the decisions of the Director of Documentation, Information and Technology Transfer Bureau shall be appealable to the Secretary of Trade and Industry; The Court of Appeals erroneously reasoned that Section 10(a) of the Intellectual Property Code, conferring upon the BLA-‐IPO jurisdiction over administrative complaints for violations of intellectual property rights, is a general provision, over which the specific provision of Section 163 of the same Code, found under Part III thereof particularly governing trademarks, service marks, and tradenames, must prevail. Proceeding therefrom, the Court of Appeals incorrectly concluded that all actions involving trademarks, including charges of unfair competition, are under the exclusive jurisdiction of civil courts. Such interpretation is not supported by the provisions of the Intellectual Property Code. While Section 163 thereof
Censure; and Other analogous penalties or sanctions.
SECTION 232. APPEALS. – 232.1. Appeals from decisions of regular courts shall be governed by the Rules of Court. Unless restrained by a higher court, the judgment of the trial court shall be executory even pending appeal under such terms and conditions as the court may prescribe. 232.2. Unless expressly provided in this Act or other statutes, appeals from decisions of administrative officials shall be provided in the Regulations.
IN-‐N-‐OUT BURGER, INC. V. SEHWANI (2008) Refresher: Petitioner IN-‐N-‐OUT BURGER, INC., a business entity incorporated under the laws of California, United States (US) of America, which is a signatory to the Convention of Paris on Protection of Industrial Property and the Agreement on Trade Related Aspects of Intellectual Property Rights (TRIPS). Petitioner is engaged mainly in the restaurant business, but it has never engaged in business in the Philippines. Respondents Sehwani, Incorporated and Benita Frites, Inc. are corporations organized in the Philippines. On 2 June 1997, petitioner filed trademark and service mark applications with the Bureau of Trademarks (BOT) of the IPO for "IN-‐N-‐OUT" and "IN-‐N-‐OUT Burger & Arrow Design." Petitioner later found out, through the Official Action Papers issued by the IPO on 31 May 2000, that respondent Sehwani, Incorporated had already obtained Trademark Registration for the mark "IN N OUT (the inside of the letter "O" formed like a star)." By virtue of a licensing agreement, Benita Frites, Inc. was able to use the registered mark of respondent Sehwani, Incorporated. Petitioner eventually filed on 4 June 2001 before the Bureau of Legal Affairs (BLA) of the IPO an administrative complaint against respondents for unfair competition and cancellation of trademark registration. Petitioner averred in its complaint that it is the owner of the trade name IN-‐N-‐OUT and the following trademarks: (1) "IN-‐N-‐OUT"; (2) "IN-‐N-‐OUT Burger & Arrow Design"; and (3) "IN-‐N-‐OUT Burger Logo." These trademarks are registered with the Trademark Office of the US and in various parts of the world, are internationally well-‐ known, and have become distinctive of its business and goods through its long and exclusive commercial use. Doctrine: (jurisdiction of the Bureau of Legal Affairs)
106 z
Intellectual Property Law vests in civil courts jurisdiction over cases of unfair competition, nothing in the said section states that the regular courts have sole jurisdiction over unfair competition cases, to the exclusion of administrative bodies. On the contrary, Sections 160 and 170, which are also found under Part III of the Intellectual Property Code, recognize the concurrent jurisdiction of civil courts and the IPO over unfair competition cases. These two provisions read: Section 160. Right of Foreign Corporation to Sue in Trademark or Service Mark Enforcement Action.–Any foreign national or juridical person who meets the requirements of Section 3 of this Act and does not engage in business in the Philippines may bring a civil or administrative action hereunder for opposition, cancellation, infringement, unfair competition, or false designation of origin and false description, whether or not it is licensed to do business in the Philippines under existing laws. x x x x Section 170. Penalties.–Independent of the civil and administrative sanctions imposed by law, a criminal penalty of imprisonment from two (2) years to five (5) years and a fine ranging from Fifty thousand pesos (P50,000) to Two hundred thousand pesos (P200,000), shall be imposed on any person who is found guilty of committing any of the acts mentioned in Section 155, Section168, and Subsection169.1. Based on the foregoing discussion, the IPO Director of Legal Affairs had jurisdiction to decide the petitioner’s administrative case against respondents and the IPO Director General had exclusive jurisdiction over the appeal of the judgment of the IPO Director of Legal Affairs.
c.
Prohibition importation
officers of the customs service in enforcing this prohibition, any person who is entitled to the benefits of this Act, may require that his name and residence, and the name of the locality in which his goods are manufactured, a copy of the certificate of registration of his mark or trade name, to be recorded in books which shall be kept for this purpose in the Bureau of Customs, under such regulations as the Collector of Customs with the approval of the Secretary of Finance shall prescribe, and may furnish to the said Bureau facsimiles of his name, the name of the locality in which his goods are manufactured, or his registered mark or trade name, and thereupon the Collector of Customs shall cause one (1) or more copies of the same to be transmitted to each collector or to other proper officer of the Bureau of Customs. ADMINISTRATIVE REMEDIES Opposition -‐ Who can file: Any person who falls under Section 3. -‐ When: -‐ Remedy: Denial of application. No other remedies. Cancellation (Section 151) -‐ Who can file: Any person who falls under Section 3. -‐ When: After the mark has been registered. o From the point of view of when to file a petition à easier to file cancellation than opposition. o From the point of view of burden of proof à easier to file opposition. No prima facie presumptions of (1) validity of registration, (2) ownership of mark, and (3) exclusive right to use mark as conferred upon the registrant by Section 138. In opposition, all the petitioner has to prove is damage. § Ma’am: Why does Section 138 only give prima facie presumption if registration confers the right upon the registrant (i.e. registration is the basis of ownership)? Because registration is not a perfect process. Under the law, what actually confers ownership is a registration validly obtained, not just any kind of registration. Thus, a person can attack the registration as invalid. There’s always a possibility that the IPO made a mistake.
of
SECTION 166. GOODS BEARING INFRINGING MARKS OR TRADE NAMES. – No article of imported merchandise which shall copy or simulate the name of any domestic product, or manufacturer, or dealer, or which shall copy or simulate a mark registered in accordance with the provisions of this Act, or shall bear a mark or trade name calculated to induce the public to believe that the article is manufactured in the Philippines, or that it is manufactured in any foreign country or locality other than the country or locality where it is in fact manufactured, shall be admitted to entry at any customhouse of the Philippines. In order to aid the
o 107
Katrina Michelle Mancao -‐ -‐
-‐
advertising of any goods or services including other preparatory steps necessary to carry out the sale of any goods or services on or in connection with which such use is likely to cause confusion, or to cause mistake, or to deceive; or 155.2. Reproduce, counterfeit, copy or colorably imitate a registered mark or a dominant feature thereof and apply such reproduction, counterfeit, copy or colorable imitation to labels, signs, prints, packages, wrappers, receptacles or advertisements intended to be used in commerce upon or in connection with the sale, offering for sale, distribution, or advertising of goods or services on or in connection with which such use is likely to cause confusion, or to cause mistake, or to deceive, shall be liable in a civil action for infringement by the registrant for the remedies hereinafter set forth: Provided, That the infringement takes place at the moment any of the acts stated in Subsection 155.1 or this subsection are committed REGARDLESS OF WHETHER THERE IS ACTUAL SALE OF GOODS OR SERVICES USING THE INFRINGING MATERIAL.
Remedy: Cancellation of the registration. No other remedy. Grounds (does not prescribe): o Generic name § Test: primary significance to the relevant public o Abandonment § Requisites (as enumerated in Romero v. Maiden Form): VIP • Voluntary • Intentional • Permanent o Registration was obtained fraudulently o Misrepresents the source of the goods and services in connection with which the marks was used o Non-‐use of the mark for an uninterrupted period of 3 years § Circumstances which excuse non-‐use of trademark: Inter partes proceeding – expedited, based on documents. When you file petition, attach all exhibits.
SECTION 156. ACTIONS, AND DAMAGES AND INJUNCTION FOR INFRINGEMENT. –
Intellectual property violation (Section 10.2) -‐ Who can file: Any person who falls within Section 3. -‐ Venue: Bureau of Legal Affairs (BLA) o As long as the amount claimed is not LESS than P200,000.00 o It has the power of the courts, PLUS more. Except that this “more” has not really been exercised. -‐ Advantage of filing in the BLA à you are more likely to obtained a resolution at an earlier time. Also, no venue restrictions here. You can file in the BLA regardless of where the parties are located. o But BLA only works if the respondent is a legitimate business.
156.1. The owner of a registered mark may recover damages from any person who infringes his rights, and the measure of the damages suffered shall be either the reasonable profit which the complaining party would have made, had the defendant not infringed his rights, OR the profit which the defendant actually made out of the infringement, or in the event such measure of damages cannot be readily ascertained with reasonable certainty, then the court may award as damages a reasonable percentage based upon the amount of gross sales of the defendant or the value of the services in connection with which the mark or trade name was used in the infringement of the rights of the complaining party. 156.2. On application of the complainant, the court may impound during the pendency of the action, sales invoices and other documents evidencing sales. 156.3. In cases where actual intent to mislead the public or to defraud the complainant is shown, in the discretion of the court, the damages may be DOUBLED. 156.4. The complainant, upon proper showing, may also be granted injunction.
2.
Civil
a.
Infringement
SECTION 155. REMEDIES; INFRINGEMENT. – Any person who shall, without the consent of the owner of the registered mark: 155.1. Use in commerce any reproduction, counterfeit, copy, or colorable imitation of a registered mark or the same container or a dominant feature thereof in connection with the sale, offering for sale, distribution,
SECTION 157. POWER OF COURT TO ORDER INFRINGING MATERIAL DESTROYED. – 157.1 In any action arising under this Act, in which a violation of any right of the owner of the registered mark is established, the court may order that goods
108 z
Intellectual Property Law found to be infringing be, without compensation of any sort, disposed of outside the channels of commerce in such a manner as to avoid any harm caused to the right holder, or destroyed; and all labels, signs, prints, packages, wrappers, receptacles and advertisements in the possession of the defendant, bearing the registered mark or trade name or any reproduction, counterfeit, copy or colorable imitation thereof, all plates, molds, matrices and other means of making the same, shall be delivered up and destroyed. 157.2. In regard to counterfeit goods, the simple removal of the trademark affixed shall not be sufficient other than in exceptional cases which shall be determined by the Regulations, to permit the release of the goods into the channels of commerce.
limited to an injunction against the presentation of such advertising matter in future issues of such newspapers, magazines, or other similar periodicals or in future transmissions of such electronic communications. The limitations of this subparagraph shall apply only to innocent infringers: Provided, That SUCH INJUNCTIVE RELIEF SHALL NOT BE AVAILABLE to the owner of the right infringed with respect to an issue of a newspaper, magazine, or other similar periodical or an electronic communication containing infringing matter where restraining the dissemination of such infringing matter in any particular issue of such periodical or in an electronic communication would delay the delivery of such issue or transmission of such electronic communication is customarily conducted in accordance with the sound business practice, and not due to any method or device adopted to evade this section or to prevent or delay the issuance of an injunction or restraining order with respect to such infringing matter; and 159.4. There shall be no infringement of trademarks or tradenames of imported or sold drugs and medicines allowed under Section 72.1 of this Act, as well as imported or sold off-‐patent drugs and medicines: Provided, That said drugs and medicines bear the registered marks that have not been tampered, unlawfully modified, or infringed upon as defined under Section 155 of this Code.
SECTION 158. DAMAGES; REQUIREMENT OF NOTICE. – In any suit for infringement, the owner of the registered mark shall not be entitled to recover profits or damages unless the acts have been committed with knowledge that such imitation is likely to cause confusion, or to cause mistake, or to deceive. Such knowledge is PRESUMED if the registrant gives notice that his mark is registered by displaying with the mark the words '"Registered Mark" or the letter R within a circle or if the defendant had otherwise actual notice of the registration.
SECTION 159. LIMITATIONS TO ACTIONS FOR INFRINGEMENT. –
SECTION 161. AUTHORITY TO DETERMINE RIGHT TO REGISTRATION. –
Notwithstanding any other provision of this Act, the remedies given to the owner of a right infringed under this Act shall be limited as follows: 159.1. Notwithstanding the provisions of Section 155 hereof, a registered mark shall have no effect against any person who, in good faith, before the filing date or the priority date, was using the mark for the purposes of his business or enterprise: Provided, That his right may only be transferred or assigned together with his enterprise or business or with that part of his enterprise or business in which the mark is used. 159.2. Where an infringer who is engaged solely in the business of printing the mark or other infringing materials for others is an innocent infringer, the owner of the right infringed shall be entitled as against such infringer only to an injunction against future printing. 159.3. Where the infringement complained of is contained in or is part of paid advertisement in a newspaper, magazine, or other similar periodical or in an electronic communication, the remedies of the owner of the right infringed as against the publisher or distributor of such newspaper, magazine, or other similar periodical or electronic communication shall be
In any action involving a registered mark, the COURT may determine the right to registration, order the cancellation of a registration, in whole or in part, and otherwise rectify the register with respect to the registration of any party to the action in the exercise of this. Judgment and orders shall be certified by the court to the Director, who shall make appropriate entry upon the records of the Bureau, and shall be controlled thereby.
SECTION 163. JURISDICTION OF COURT. – All actions under Sections 150, 155, 164, and 166 to 169 shall be BROUGHT BEFORE THE PROPER COURTS with appropriate jurisdiction under existing laws.
SECTION 164. NOTICE OF FILING SUIT GIVEN TO THE DIRECTOR. – It shall be the duty of the clerks of such courts within one (1) month after the filing of any action, suit, or proceeding involving a mark registered under the provisions of this Act, to notify the Director in writing setting forth: the names and addresses of the litigants and designating the number of the registration or registrations and within one (1) month after the judgment is entered or an appeal is taken, the clerk of
o 109
Katrina Michelle Mancao court shall give notice thereof to the Office, and the latter shall endorse the same upon the filewrapper of the said registration or registrations and incorporate the same as a part of the contents of said filewrapper.
same trademark by others on unrelated articles of a different kind. Although petitioner's products are numerous, they are of the same class or line of merchandise which are non-‐competing with respondent's product of cigarettes, which as pointed out in the appealed judgment is beyond petitioner's "zone of potential or natural and logical expansion" 21 When a trademark is used by a party for a product in which the other party does not deal, the use of the same trademark on the latter's product cannot be validly objected to. Another factor that shows that the goods involved are non-‐competitive and non-‐related is the appellate court's finding that they flow through different channels of trade, thus: "The products of each party move along and are disposed through different channels of distribution. The petitioner's products are distributed principally through gasoline service and lubrication stations, automotive shops and hardware stores. On the other hand, the respondent's cigarettes are sold in sari-‐sari stores, grocery stores, and other small distributor outlets. Respondent's cigarettes are even peddled in the streets while petitioner's 'gasul' burners are not. Finally, there is a marked distinction between oil and tobacco, as well as between petroleum and cigarettes. Evidently, in kind and nature the products of respondent and of petitioner are poles apart."
SECTION 232. APPEALS. -‐ 232.1. Appeals from decisions of regular courts shall be governed by the Rules of Court. Unless restrained by a higher court, the judgment of the trial court shall be executory even pending appeal under such terms and conditions as the court may prescribe. 232.2. Unless expressly provided in this Act or other statutes, appeals from decisions of administrative officials shall be provided in the Regulations.
ETEPHA V. DIRECTOR OF PATENTS (1966), SUPRA Refresher: Atussin and Pertussin Doctrine: In the solution of a trademark infringement problem, regard too should be given to the class of persons who buy the particular product and the circumstances ordinarily attendant to its acquisition. The medicinal preparation clothed with the trademarks in question, are unlike articles of everyday use such as candies, ice cream, milk, soft drinks and the like which may be freely obtained by anyone, anytime, anywhere. Petitioner's and respondent's products are to be dispensed upon medical prescription. The respective labels say so. An intending buyer must have to go first to a licensed doctor of medicine; he receives instructions as to what to purchase; he reads the doctor's prescription; he knows what he is to buy. He is not of the incautious, unwary, unobservant or unsuspecting type; he examines the product sold to him; he checks to find out whether it conforms to the medical prescription. The common trade channel is the pharmacy or the drugstore. Similarly, the pharmacist or druggist verifies the medicine sold. The margin of error in the acquisition of one for the other is quite remote.
FRUIT OF THE LOOM V. CA AND GENERAL GARMENTS CORP. (1984) Refresher: Respondent: “Fruit for Eve” – women’s panties and pajamas Petitioner: “Fruit of the Loom” – men’s, women’s and children’s underwear The main issue involved in this case is whether or not private respondent's trademark FRUIT FOR EVE and its hang tag are confusingly similar to petitioner's trademark FRUIT OF THE LOOM and its hang tag so as to constitute an infringement of the latter's trademark rights and justify the cancellation of the former. Doctrine: In determining whether the trademarks are confusingly similar, a comparison of the words is not the only determinant factor. The trademarks in their entirety as they appear in their respective labels or hang tags must also be considered in relation to the goods to which they are attached. The discerning eye of the observer must focus not only on the predominant words but also on the other features appearing in both labels in order that he may draw his conclusion whether one is confusingly similar to the other.
ESSO STANDARD EASTERN, INC. V. CA AND UNITED CIGARETTES (1982) Refresher: “Esso” on petroleum and cigarette products Doctrine: In the situation before us, the goods are obviously different from each other with "absolutely no iota of similitude" as stressed in respondent court's judgment. They are so foreign to each other as to make it unlikely that purchasers would think that petitioner is the manufacturer of respondent's goods. The mere fact that one person has adopted and used a trademark on his goods does not prevent the adoption and use of the
110 z
Intellectual Property Law WE hold that the trademarks FRUIT OF THE LOOM and FRUIT FOR EVE do not resemble each other as to confuse or deceive an ordinary purchaser. The ordinary purchaser must be thought of as having, and credited with, at least a modicum of intelligence to be able to see the obvious differences between the two trademarks in question. Furthermore, We believe that a person who buys petitioner's products and starts to have a liking for it, will not get confused and reach out for private respondent's products when she goes to a garment store.
may be dissipated as soon as the court assumes to analyze carefully the respective features of the mark. It has also been held that it is not the function of the court in cases of infringement and unfair competition to educate purchasers but rather to take their carelessness for granted, and to be ever conscious of the fact that marks need not be identical. A confusing similarity will justify the intervention of equity. The judge must also be aware of the fact that usually a defendant in cases of infringement does not normally copy but makes only colorable changes. Well has it been said that the most successful form of copying is to employ enough points of similarity to confuse the public with enough points of difference to confuse the courts.
DEL MONTE CORP. AND PHIL. PACKING CORP. V. CA AND SUNSHINE SAUCE MANUFACTURING INDUSTRIES (1990) Doctrine: It has been correctly held that side-‐by-‐side comparison is not the final test of similarity. Such comparison requires a careful scrutiny to determine in what points the labels of the products differ, as was done by the trial judge. The ordinary buyer does not usually make such scrutiny nor does he usually have the time to do so. The average shopper is usually in a hurry and does not inspect every product on the shelf as if he were browsing in a library. Where the housewife has to return home as soon as possible to her baby or the working woman has to make quick purchases during her off hours, she is apt to be confused by similar labels even if they do have minute differences. The male shopper is worse as he usually does not bother about such distinctions. The question is not whether the two articles are distinguishable by their label when set side by side but whether the general confusion made by the article upon the eye of the casual purchaser who is unsuspicious and off his guard, is such as to likely result in his confounding it with the original. As observed in several cases, the general impression of the ordinary purchaser, buying under the normally prevalent conditions in trade and giving the attention such purchasers usually give in buying that class of goods is the touchstone. It has been held that in making purchases, the consumer must depend upon his recollection of the appearance of the product which he intends to purchase. The buyer having in mind the mark/label of the respondent must rely upon his memory of the petitioner's mark. Unlike the judge who has ample time to minutely examine the labels in question in the comfort of his sala, the ordinary shopper does not enjoy the same opportunity. A number of courts have held that to determine whether a trademark has been infringed, we must consider the mark as a whole and not as dissected. If the buyer is deceived, it is attributable to the marks as a totality, not usually to any part of it. The court therefore should be guided by its first impression, for a buyer acts quickly and is governed by a casual glance, the value of which
ASIA BREWERY, INC. V. CA AND SAN MIGUEL CORP. (1993), SUPRA Court held: Only registered trade marks, trade names and service marks are protected against infringement or unauthorized use by another or others. The use of someone else's registered trademark, trade name or service mark is unauthorized, hence, actionable, if it is done "without the consent of the registrant." Infringement is determined by the "test of dominancy" rather than by differences or variations in the details of one trademark and of another. It has been consistently held that the question of infringement of a trademark is to be determined by the test of dominancy. Similarity in size, form and color, while relevant, is not conclusive. If the competing trademark contains the main or essential or dominant features of another, and confusion and deception is likely to result, infringement takes place. Duplication or imitation is not necessary; nor it is necessary that the infringing label should suggest an effort to imitate. The question at issue in cases of infringement of trademarks is whether the use of the marks involved would be likely to cause confusion or mistakes in the mind of the public or deceive purchasers. The universal test question is whether the public is likely to be deceived. Nothing less than conduct tending to pass off one man's goods or business as that of another will constitute unfair competition. Actual or probable deception and confusion on the part of the customers by reason of defendant's practices must always appear.
CONRAD AND CO. V. CA, FITRITE INC. AND VICTORIA BISCUITS CO. (1995) Refresher: Trademark in question = “Sunshine” Doctrine: Whether CONRAD's acts of importing, selling and distributing biscuits, cookies and other food items
o 111
Katrina Michelle Mancao bearing said registered "SUNSHINE" trademark in the Philippines without the consent of its registrant (FITRITE) constitute infringement thereof in contemplation of Sec. 22 of Republic Act No. 166, as amended. Under Sec. 22, the elements that constitute infringement are simply (1) the use by any person, without the consent of the registrant, (2) of any registered mark or trade-‐name in connection with the sale, business or services, among other things, bearing such registered mark or trade-‐name. Thus, having the exclusive right over said trademark, FITRITE should be protected in the use thereof; and considering that it is apparent from the record that the invasion of the right FITRITE sought to protect is material and substantial; that such right of FITRITE is clear and unmistakable; and that there is an urgent necessity to prevent serious damage to FITRITE's business interest, goodwill and profit, thus under the authority of Sec. 23 of said Republic Act No. 166, as amended, a preliminary injunction may be issued in favor of FITRITE to maintain the status quo pending trial of the action a quo on the merits without prejudice to the suspension of such action if the aforesaid cancellation proceeding before the BPTTT has not been concluded.
As its title implies, the test of dominancy focuses on the similarity of the prevalent features of the competing trademarks which might cause confusion or deception and thus constitutes infringement. On the other side of the spectrum, the holistic test mandates that the entirety of the marks in question must be considered in determining confusing similarity. Applying the foregoing tenets to the present controversy and taking into account the factual circumstances of this case, we considered the trademarks involved as a whole and rule that petitioner's "STYLISTIC MR. LEE" is not confusingly similar to private respondent's "LEE" trademark. Petitioner's trademark is the whole "STYLISTIC MR. LEE." Although on its label the word "LEE" is prominent, the trademark should be considered as a whole and not piecemeal. The dissimilarities between the two marks become conspicuous, noticeable and substantial enough to matter especially in the light of the following variables that must be factored in. First, the products involved in the case at bar are, in the main, various kinds of jeans. These are not your ordinary household items like catsup, soysauce or soap which are of minimal cost. Maong pants or jeans are not inexpensive. Accordingly, the casual buyer is predisposed to be more cautious and discriminating in and would prefer to mull over his purchase. Confusion and deception, then, is less likely. Second, like his beer, the average Filipino consumer generally buys his jeans by brand. He does not ask the sales clerk for generic jeans but for, say, a Levis, Guess, Wrangler or even an Armani. He is, therefore, more or less knowledgeable and familiar with his preference and will not easily be distracted. Finally, in line with the foregoing discussions, more credit should be given to the "ordinary purchaser." Cast in this particular controversy, the ordinary purchaser is not the "completely unwary consumer" but is the "ordinarily intelligent buyer" considering the type of product involved. The definition laid down in Dy Buncio v. Tan Tiao Bok is better suited to the present case. There, the "ordinary purchaser" was defined as one "accustomed to buy, and therefore to some extent familiar with, the goods in question. The test of fraudulent simulation is to be found in the likelihood of the deception of some persons in some measure acquainted with an established design and desirous of purchasing the commodity with which that design has been associated. The test is not found in the deception, or the possibility of deception, of the
EMERALD GARMENT MANUFACTURING V. CA (1995), SUPRA Refresher: “Stylistics Mr. Lee” and “Lee” Doctrine: The essential element of infringement is colorable imitation. This term has been defined as "such a close or ingenious imitation as to be calculated to deceive ordinary purchasers, or such resemblance of the infringing mark to the original as to deceive an ordinary purchaser giving such attention as a purchaser usually gives, and to cause him to purchase the one supposing it to be the other." Colorable imitation does not mean such similitude as amounts to identity. Nor does it require that all the details be literally copied. Colorable imitation refers to such similarity in form, content, words, sound, meaning, special arrangement, or general appearance of the trademark or tradename with that of the other mark or tradename in their over-‐all presentation or in their essential, substantive and distinctive parts as would likely mislead or confuse persons in the ordinary course of purchasing the genuine article. In determining whether colorable imitation exists, jurisprudence has developed TWO KINDS OF TESTS — the Dominancy Test applied in Asia Brewery, Inc. v. Court of Appeals and other cases and the Holistic Test developed in Del Monte Corporation v. Court of Appeals and its proponent cases.
112 z
Intellectual Property Law person who knows nothing about the design which has been counterfeited, and who must be indifferent between that and the other. The simulation, in order to be objectionable, must be such as appears likely to mislead the ordinary intelligent buyer who has a need to supply and is familiar with the article that he seeks to purchase."
infringement. On the other side of the spectrum, the holistic test mandates that the entirety of the marks in question must be considered in determining confusing similarity. As this Court has often declared, each case must be studied according to the peculiar circumstances of each case. That is the reason why in trademark cases, jurisprudential precedents should be applied only to a case if they are specifically in point. In the case at bar, other than the fact that both Nestle’s and CFC’s products are inexpensive and common household items, the similarity ends there. What is being questioned here is the use by CFC of the trademark MASTER. In view of the difficulty of applying jurisprudential precedents to trademark cases due to the peculiarity of each case, judicial fora should not readily apply a certain test or standard just because of seeming similarities. As this Court has pointed above, there could be more telling differences than similarities as to make a jurisprudential precedent inapplicable. Nestle points out that the dominancy test should have been applied to determine whether there is a confusing similarity between CFC’s FLAVOR MASTER and Nestle’s MASTER ROAST and MASTER BLEND. We agree. It must be emphasized that the products bearing the trademarks in question are “inexpensive and common” household items bought off the shelf by “undiscerningly rash” purchasers. As such, if the ordinary purchaser is “undiscerningly rash”, then he would not have the time nor the inclination to make a keen and perceptive examination of the physical discrepancies in the trademarks of the products in order to exercise his choice. While this Court agrees with the Court of Appeals’ detailed enumeration of differences between the respective trademarks of the two coffee products, this Court cannot agree that totality test is the one applicable in this case. Rather, this Court believes that the dominancy test is more suitable to this case in light of its peculiar factual milieu. Moreover, the totality or holistic test is contrary to the elementary postulate of the law on trademarks and unfair competition that confusing similarity is to be determined on the basis of visual, aural, connotative comparisons and overall impressions engendered by the marks in controversy as they are encountered in the realities of the marketplace. The totality or holistic test only relies on visual comparison between two trademarks whereas the dominancy test relies not only on the visual but also on the aural and connotative
AMIGO V. CLUETT PEABODY (2001) Refresher: “Gold Top” and “Gold Toe” mark on socks. Court decision: In the present case, a resort to either the Dominancy Test or the Holistic Test shows that colorable imitation exists between respondent’s “Gold Toe” and petitioner’s “Gold Top.” A glance at petitioner’s mark shows that it definitely has a lot of similarities and in fact looks like a combination of the trademark and devices that respondent has already registered; namely, “Gold Toe,” the representation of a sock with a magnifying glass, the “Gold Toe” representation and “linenized.” Admittedly, there are some minor differences between the two sets of marks. The similarities, however, are of such degree, number and quality that the overall impression given is that the two brands of socks are deceptively the same, or at least very similar to each another. An examination of the products in question shows that their dominant features are gold checkered lines against a predominantly black background and a representation of a sock with a magnifying glass. In addition, both products use the same type of lettering. Both also include a representation of a man’s foot wearing a sock and the word “linenized” with arrows printed on the label. Lastly, the names of the brands are similar -‐-‐ “Gold Top” and “Gold Toe.” Moreover, it must also be considered that petitioner and respondent are engaged in the same line of business.
SOCIETES DES PRODUITS NESTLE V. CA (2001), SUPRA Court decision: Under Section 36 of Republic Act 165 (Patent Law), the Director of Patents is authorized, in case the parties failed to submit a licensing agreement, to fix the terms and conditions of the license. Colorable imitation denotes such a close or ingenious imitation as to be calculated to deceive ordinary persons, or such a resemblance to the original as to deceive an ordinary purchaser giving such attention as a purchaser usually gives, as to cause him to purchase the one supposing it to be the other. In determining if colorable imitation exists, jurisprudence has developed two kinds of tests -‐ the Dominancy Test and the Holistic Test. The test of dominancy focuses on the similarity of the prevalent features of the competing trademarks which might cause confusion or deception and thus constitute
o 113
Katrina Michelle Mancao comparisons and overall impressions between the two trademarks.
in which case defendant’s goods are then bought as the plaintiff’s and its poor quality reflects badly on the plaintiff’s reputation. The other is “confusion of business” wherein the goods of the parties are different but the defendant’s product can reasonably (though mistakenly) be assumed to originate from the plaintiff, thus deceiving the public into believing that there is some connection between the plaintiff and defendant which, in fact, does not exist. In determining the likelihood of confusion, the Court must consider: [a] the resemblance between the trademarks; [b] the similarity of the goods to which the trademarks are attached; [c] the likely effect on the purchaser and [d] the registrant’s express or implied consent and other fair and equitable considerations. Whether a trademark causes confusion and is likely to deceive the public hinges on “colorable imitation” which has been defined as “such similarity in form, content, words, sound, meaning, special arrangement or general appearance of the trademark or tradename in their overall presentation or in their essential and substantive and distinctive parts as would likely mislead or confuse persons in the ordinary course of purchasing the genuine article.” Jurisprudence has developed TWO TESTS in determining similarity and likelihood of confusion in trademark resemblance -‐ the Dominancy Test and the Holistic or Totality Test. The Dominancy Test focuses on the similarity of the prevalent features of the competing trademarks which might cause confusion or deception, and thus infringement. If the competing trademark contains the main, essential or dominant features of another, and confusion or deception is likely to result, infringement takes place. Duplication or imitation is not necessary; nor is it necessary that the infringing label should suggest an effort to imitate. The question is whether the use of the marks involved is likely to cause confusion or mistake in the mind of the public or deceive purchasers. On the other hand, the Holistic Test requires that the entirety of the marks in question be considered in resolving confusing similarity. Comparison of words is not the only determining factor. The trademarks in their entirety as they appear in their respective labels or hang tags must also be considered in relation to the goods to which they are attached. The discerning eye of the observer must focus not only on the predominant words but also on the other features appearing in both labels in order that he may draw his conclusion whether one is confusingly similar to the other.
MIGHTY CORP. V. E. & J. GALLO WINERY (2004) Court decision: Although the laws on trademark infringement and unfair competition have a common conception at their root, that is, a person shall not be permitted to misrepresent his goods or his business as the goods or business of another, the law on unfair competition is broader and more inclusive than the law on trademark infringement. The latter is more limited but it recognizes a more exclusive right derived from the trademark adoption and registration by the person whose goods or business is first associated with it. The law on trademarks is thus a specialized subject distinct from the law on unfair competition, although the two subjects are entwined with each other and are dealt with together in the Trademark Law (now, both are covered by the IP Code). Hence, even if one fails to establish his exclusive property right to a trademark, he may still obtain relief on the ground of his competitor’s unfairness or fraud. Conduct constitutes unfair competition if the effect is to pass off on the public the goods of one man as the goods of another. It is not necessary that any particular means should be used to this end. Trademark infringement v. Unfair competition: (1) Infringement of trademark is the unauthorized use of a trademark, whereas unfair competition is the passing off of one's goods as those of another. (2) In infringement of trademark fraudulent intent is unnecessary, whereas in unfair competition fraudulent intent is essential. (3) In infringement of trademark the prior registration of the trademark is a prerequisite to the action, whereas in unfair competition registration is not necessary. A crucial issue in any trademark infringement case is the likelihood of confusion, mistake or deceit as to the identity, source or origin of the goods or identity of the business as a consequence of using a certain mark. Likelihood of confusion is admittedly a relative term, to be determined rigidly according to the particular (and sometimes peculiar) circumstances of each case. Thus, in trademark cases, more than in other kinds of litigation, precedents must be studied in the light of each particular case. There are TWO TYPES OF CONFUSION IN TRADEMARK INFRINGEMENT. The first is “confusion of goods” when an otherwise prudent purchaser is induced to purchase one product in the belief that he is purchasing another,
114 z
Intellectual Property Law In comparing the resemblance or colorable imitation of marks, various factors have been considered, such as the dominant color, style, size, form, meaning of letters, words, designs and emblems used, the likelihood of deception of the mark or name's tendency to confuse and the commercial impression likely to be conveyed by the trademarks if used in conjunction with the respective goods of the parties.
confusion of business remains, since the low-‐income group might be led to believe that the “Big Mak” hamburgers are the low-‐end hamburgers marketed by petitioners. After all, petitioners have the exclusive right to use the “Big Mac” mark. On the other hand, respondents would benefit by associating their low-‐end hamburgers, through the use of the “Big Mak” mark, with petitioners’ high-‐end “Big Mac” hamburgers, leading to likelihood of confusion in the identity of business. The registered trademark owner may use his mark on the same or similar products, in different segments of the market, and at different price levels depending on variations of the products for specific segments of the market. The Court has recognized that the registered trademark owner enjoys protection in product and market areas that are the normal potential expansion of his business. Thus, the Court has declared: Modern law recognizes that the protection to which the owner of a trademark is entitled is not limited to guarding his goods or business from actual market competition with identical or similar products of the parties, but extends to all cases in which the use by a junior appropriator of a trade-‐mark or trade-‐name is likely to lead to a confusion of source, as where prospective purchasers would be misled into thinking that the complaining party has extended his business into the field or is in any way connected with the activities of the infringer; or when it forestalls the normal potential expansion of his business. Test to determine likelihood of confusion: This Court, however, has relied on the dominancy test rather than the holistic test. The dominancy test considers the dominant features in the competing marks in determining whether they are confusingly similar. Under the dominancy test, courts give greater weight to the similarity of the appearance of the product arising from the adoption of the dominant features of the registered mark, disregarding minor differences. Courts will consider more the aural and visual impressions created by the marks in the public mind, giving little weight to factors like prices, quality, sales outlets and market segments. The test of dominancy is now explicitly incorporated into law in Section 155.1 of the Intellectual Property Code which defines infringement as the “colorable imitation of a registered mark xxx or a dominant feature thereof.”
MCDONALD’S CORP V. L.C. BIG MAK (2004) Court decision: To establish trademark infringement, the following elements must be shown: (1) the validity of plaintiff’s mark; (2) the plaintiff’s ownership of the mark; and (3) the use of the mark or its colorable imitation by the alleged infringer results in “likelihood of confusion.” Of these, it is the element of likelihood of confusion that is the gravamen of trademark infringement. Section 22 covers two types of confusion arising from the use of similar or colorable imitation marks, namely, confusion of goods (product confusion) and confusion of business (source or origin confusion). In confusion of goods, the ordinarily prudent purchaser would be induced to purchase one product in the belief that he was purchasing the other. In confusion of business, though the goods of the parties are different, the defendant’s product is such as might reasonably be assumed to originate with the plaintiff, and the public would then be deceived either into that belief or into the belief that there is some connection between the plaintiff and defendant which, in fact, does not exist. Thus, while there is confusion of goods when the products are competing, confusion of business exists when the products are non-‐competing but related enough to produce confusion of affiliation. Petitioners claim that respondents’ use of the “Big Mak” mark on respondents’ hamburgers results in confusion of goods, particularly with respect to petitioners’ hamburgers labeled “Big Mac.” Since respondents used the “Big Mak” mark on the same goods, i.e. hamburger sandwiches, that petitioners’ “Big Mac” mark is used, trademark infringement through confusion of goods is a proper issue in this case. Respondents also admit that their business includes selling hamburger sandwiches, the same food product that petitioners sell using the “Big Mac” mark. Thus, trademark infringement through confusion of business is also a proper issue in this case. Respondents assert that their “Big Mak” hamburgers cater mainly to the low-‐income group while petitioners’ “Big Mac” hamburgers cater to the middle and upper income groups. Even if this is true, the likelihood of
o 115
Katrina Michelle Mancao In contrast, the Holistic or Totality Test necessitates a consideration of the entirety of the marks as applied to the products, including the labels and packaging, in determining confusing similarity. The discerning eye of the observer must focus not only on the predominant words, but also on the other features appearing on both labels so that the observer may draw conclusion on whether one is confusingly similar to the other. Relative to the question on confusion of marks and trade names, jurisprudence has noted 2 types of confusion, viz.: (1) confusion of goods (product confusion), where the ordinarily prudent purchaser would be induced to purchase one product in the belief that he was purchasing the other; and (2) confusion of business (source or origin confusion), where, although the goods of the parties are different, the product, the mark of which registration is applied for by one party, is such as might reasonably be assumed to originate with the registrant of an earlier product, and the public would then be deceived either into that belief or into the belief that there is some connection between the two parties, though inexistent. Applying the Dominancy Test to the case at bar, this Court finds that the use of the stylized "S" by respondent in its Strong rubber shoes infringes on the mark already registered by petitioner with the IPO. While it is undisputed that petitioner's stylized "S" is within an oval design, to this Court's mind, the dominant feature of the trademark is the stylized "S," as it is precisely the stylized "S" which catches the eye of the purchaser. Thus, even if respondent did not use an oval design, the mere fact that it used the same stylized "S", the same being the dominant feature of petitioner's trademark, already constitutes infringement under the Dominancy Test. Withal, the protection of trademarks as intellectual property is intended not only to preserve the goodwill and reputation of the business established on the goods bearing the mark through actual use over a period of time, but also to safeguard the public as consumers against confusion on these goods. Cases on bottles and containers
MCDONALD’S CORP. V. MACJOY FASTFOOD CORP. (2007) Court decision: Essentially, the issue here is whether there is a confusing similarity between the MCDONALD’S marks of the petitioner and the respondent’s “MACJOY & DEVICE” trademark when applied to Classes 29 and 30 of the International Classification of Goods, i.e., food and ingredients of food. In determining similarity and likelihood of confusion, jurisprudence has developed two tests, the dominancy test and the holistic test. The dominancy test focuses on the similarity of the prevalent features of the competing trademarks that might cause confusion or deception. In contrast, the holistic test requires the court to consider the entirety of the marks as applied to the products, including the labels and packaging, in determining confusing similarity. Under the latter test, a comparison of the words is not the only determinant factor. In trademark cases, particularly in ascertaining whether one trademark is confusingly similar to another, no set rules can be deduced because each case must be decided on its merits. In such cases, even more than in any other litigation, precedent must be studied in the light of the facts of the particular case. That is the reason why in trademark cases, jurisprudential precedents should be applied only to a case if they are specifically in point. While we agree with the CA’s detailed enumeration of differences between the 2 competing trademarks herein involved, we believe that the holistic test is not the one applicable in this case, the dominancy test being the one more suitable. In recent cases with a similar factual milieu as here, the Court has consistently used and applied the dominancy test in determining confusing similarity or likelihood of confusion between competing trademarks.
SKETCHERS V. INTERPACIFIC INDUSTRIAL TRADING (2011) Court decision: The essential element of infringement under R.A. No. 8293 is that the infringing mark is likely to cause confusion. In determining similarity and likelihood of confusion, jurisprudence has developed tests – the Dominancy Test and the Holistic or Totality Test. The Dominancy Test focuses on the similarity of the prevalent or dominant features of the competing trademarks that might cause confusion, mistake, and deception in the mind of the purchasing public. Duplication or imitation is not necessary; neither is it required that the mark sought to be registered suggests an effort to imitate. Given more consideration are the aural and visual impressions created by the marks on the buyers of goods, giving little weight to factors like prices, quality, sales outlets, and market segments.
CAGAYAN VALLEY ENTERPRISES V CA (1989) Court decision: RA 623 extends protection to qualified manufacturer who successfully registered with the PPO its duly stamped or marked bottles, boxes, casks, and other similar containers. It merely requires that bottles, in order to be eligible for registration must be stamped or marked with the names of their manufacturers or the names of their principals or products or other marks of ownership. No drawings or labels are required but instead, two photos of the container, duly signed by the
116 z
Intellectual Property Law applicant, showing clearly and legibly the names and other marks of ownership sought to be registered and a bottle showing the name or other mark of ownership, irremovably stamped or marked shall be submitted. Hard liquor, although regulated, is not probity by law, hence it is within the purview and coverage of RA 623.
to the backyard, cottage and small-‐scale manufacturers of indigenous native products such as patis, sisi and toyo who do not have the capital to buy brand new bottles as containers nor afford to pass the added cost to the majority of the poor Filipinos who use the products as their daily condiments or viands.
DISTILLERIA WASHINGTON V CA (1996)
b.
Court decision: RA 623 extends TM protection in the use of containers duly registered with the PPO. The mere use of registered bottles or containers without the written consent of the manufacturer is prohibited. Only exceptions: usage as container for sisi, bagoong, patis and similar native products. RA 623 does not disallow the sale or transfer of ownership of the marked bottles or containers. While it may be unwarranted for the registrant to simply seize the empty containers from one engaged in the unauthorized use of said containers, it would be legally absurd to still allow the latter to recover possession thereof—the practical and feasible alternative is to merely require the payment of just compensation to the latter.
Unfair competition
SECTION 168. UNFAIR COMPETITION, RIGHTS, REGULATION AND REMEDIES. – 168.1. A person who has identified in the mind of the public the goods he manufactures or deals in, his business or services from those of others, whether or not a registered mark is employed, has a property right in the goodwill of the said goods, business or services so identified, which will be protected in the same manner as other property rights. 168.2. Any person who shall employ deception or any other means contrary to good faith by which he shall pass off the goods manufactured by him or in which he deals, or his business, or services for those of the one having established such goodwill, or who shall commit any acts calculated to produce said result, shall be guilty of unfair competition, and shall be subject to an action therefor. 168.3. In particular, and without in any way limiting the scope of protection against unfair competition, the following shall be deemed guilty of unfair competition: (a) Any person, who is selling his goods and gives them the general appearance of goods of another manufacturer or dealer, either as to the goods themselves or in the wrapping of the packages in which they are contained, or the devices or words thereon, or in any other feature of their appearance, which would be likely to influence purchasers to believe that the goods offered are those of a manufacturer or dealer, other than the actual manufacturer or dealer, or who otherwise clothes the goods with such appearance as shall deceive the public and defraud another of his legitimate trade, or any subsequent vendor of such goods or any agent of any vendor engaged in selling such goods with a like purpose; (b) Any person who by any artifice, or device, or who employs any other means calculated to induce the false belief that such person is offering the services of another who has identified such services in the mind of the public; or
DISTILLERIA WASHINGTON V. LA TONDEÑA (1997) Court decision: It is implied that Section 2 and 3 of RA 623 apply only when the registered manufacturer, bottler or seller retain ownership of the bottles and when the bottles have been transferred by way of sale, Section 5 applies, thereby precluding the institution of any action under Sections 2 and 3. The transferee of the marked bottles transferred by way of sale is allowed to enjoy all the rights of an owner in regard to such bottles. What is proscribed is the use of bottles in infringement of another’s trademark or incorporeal rights. Where the marked bottles are transferred by way of sale, the registered owner relinquishes all its proprietary rights over the bottles in favor of the person who obtains them in due course.
TWIN ACE HOLDINGS V. CA (1997) Court decision: Registered containers of hard liquor (rhum, gin, brandy, and the like) are protected by RA 623. RA 623 was meant to protect the intellectual property rights fo the registrants of the containers and prevent unfair trade practices and fraud on the public. HOWEVER, the exemption granted under the same law was deemed extremely necessary to provide assistance and incentive
o 117
Katrina Michelle Mancao circumstances of the case. The affidavit of a former employee of Universal attesting to the illegal sale and manufacture of “Spalding” balls and seized “Spalding” products and instruments from Universal’s factory was sufficient prima facie evidence to warrant prosecution of private respondents. That a corporation other than the certified owner of the TM is engaged in the unauthorized manufacture of products bearing the same TM engenders a reasonable belief that a criminal offense for unfair competition is being committed. To hold that the act of selling is an indispensable element of the crime of unfair competition is illogical because if the law punishes the seller of imitation goods, then with more reason should the law penalize the manufacturer.
(c) Any person who shall make any false statement in the course of trade or who shall commit any other act contrary to good faith of a nature calculated to discredit the goods, business or services of another. 168.4. The remedies provided by Sections 15610, 15711 and 16112 shall apply mutatis mutandis.
SECTION 232. APPEALS. – 232.1. Appeals from decisions of regular courts shall be governed by the Rules of Court. Unless restrained by a higher court, the judgment of the trial court shall be executory even pending appeal under such terms and conditions as the court may prescribe. 232.2. Unless expressly provided in this Act or other statutes, appeals from decisions of administrative officials shall be provided in the Regulations. Difference between infringement and unfair
UNIVERSAL RUBBER PRODUCTS V. CA, CONVERSE RUBBER CORPORATION, EDWARSON MANUFACTURING CO. (1984) Court decision: General rule: On obtaining an injunction for infringement of a TM, complainant is entitled to an accounting and recovery of defendant’s profits on the goods sold under the TM as part of his property right. This rule applies also in cases of unfair competition. In such case, the infringer or unfair trader is required in equity to account for and yield up his gains on a principle analogous to that which charges as trustee with the profits acquired by the wrongful use of the property of the cestui que trust and defendant’s profits are regarded as an equitable measure of the compensation plaintiff should receive for the past harm suffered by him. In recovering the loss suffered by the aggrieved party due to “unfair competition,” Sec. 23 of RA 166 grants the complainant three options within which to ascertain the amount of damages recoverable either [1] reasonable profit with the complaining party would have made, had the defendant not infringed his said rights [2] profit which the defendant actually made out the infringement [3] the court may award as damages a reasonable percentage based upon the amount of gross sales of the defendant of the value of the services in connection with which the mark or tradename was issued in the infringement of the rights of the complaining party. Nature of complaint of unfair competition: it’s basically a suit for “injunction and damages” because… -‐ Injunction: for the purpose of enjoining the unlawful competitor from proceeding further with the unlawful competition
competition
DEL MONTE CORPORATION V. CA AND SUNSHINE SAUCE (1990) Court decision: Distinctions between infringement of TM and unfair competition. INFRINGEMENT UNFAIR COMPETITION Unauthorized use of a TM Passing off of one’s goods as those of another Intent is unnecessary Fraudulent intent is necessary Prior registration of the Registration is not TM is a prerequisite to the necessary action Lower court and CA dismissed Del Monte’s complaint against Sunshine Suace for infringement of TM and unfair competition because of Del Monte’s failure to establish defendant’s malice or bad faith, which was an essential element of infringement of TM or unfair competition. SC reversed the lower courts’ ruling and found Sunshine guilty of both infringement (logo was confusingly similar) and unfair competition (refilling of Del Monte’s bottles with their own product).
PRO LINE SPORTS CENTER V. CA, UNIVERSAL ATHLETICS INDUSTRIAL PRODUCTS INC, AND MONICO SEHWANI (1997) Court decision: The existence of probable cause for unfair competition by Universal is derivable from the facts and 10
For infringement For destroying the material 12 For cancelation of registration 11
118 z
Intellectual Property Law -‐ Damages: to allow the aggrieved party to recover the damage he has suffered by virtue of the said unlawful competition. To determine the amount of damages allowable only after final determination of the unfair competition case would render nugatory the right of the complainant under Sec. 23 RA 166 but would be a repetitious process causing only unnecessary delay.
SOLID TRIANGLE SALES CORPORATION V. SHERIFF OF RTC QC (2001) Court decision: Sanly Corporation did not pass off the subject goods as that of another. Indeed, it admits that the goods are genuine Mitsubishi photographic paper, which it purchased from a supplier in HK. Petitioner alleges that private respondents “made it appear that they were duly authorized to sell or distribute Mitsubishi Photo Paper in the PH.” Assuming this act constitutes a crime, there is no proof to establish such allegation.
CONVERSE RUBBER CORPORATION V. JACINTO RUBBER AND PLASTIC CO (1980) Court decision: The shoes manufactured by defendants (Custombuilt) contained practically all the features of those of the plaintiff (Chuck Taylors), except for the respective brands, of course. The respective designs, shapes, colors of the ankle patches, the bands, the toe patch and the soles of the two products are exactly the same such that at a distance of few meters, it is impossible to distinguish Custombuilt from Chuck Taylor. These elements are more than sufficient to serve as basis for a charge of unfair competition. Even if not all the details just mentioned were identical, with the general appearances alone of the two products, any ordinary or even perhaps even a not too perceptive and discriminating customer could be deceived, and therefore, Custombuilt could easily be passed off for Chuck Taylor. The statute on unfair competition extends protection to the goodwill of the manufacturer/dealer. It is not required that: Goodwill sought to be protected in an action for unfair competition must have been established in an actual competitive situation. Deception or other means contrary to good faith or any acts calculated to pass off other goods for those of one who has established good will must have been committed in an actual competitive situation.
SONY COMPUTER ENTERTAINMENT V. SUPERGREEN, INC. (2007) Court decision: Sony filed complaint with the NBI against Supergreen for reproduction and distribution of counterfeit Playstation console and game software. Manufacture is in Cavite and distribution is in Mandaluyong. Search warrant was issued by Manila RTC. Supergreen moved for quashal of the search warrants. Court upheld the warrants for Mandaluyong because it is within the judicial region of Manila. Where a person’s imitation of the general appearance of another person’s goods was done allegedly in Cavite but is sold the goods in Metro Manila, the alleged acts would constitute a transitory or continuing offense. The instant cause involves a transitory or continuing offense of unfair competition under Sec. 168 of RA 8293.
SEHWANI, INC AND BENITA’S FRIES, INC V. IN-‐N-‐OUT BURGER, INC (2007) Court decision: The question of WON a trademark is well-‐known is factual in nature, involving as it does the appreciation of evidence adduced before the BLA-‐IPO. Section 151 (b) of RA 8293 specifically provides that a petition to cancel the registration of a mark which is registered contrary to the provisions thereof, or which is used to misrepresent the source of the goods or services may be filed anytime. Laches may not prevail against a specific provision of law, since equity, which has been defined as “justice outside legality” is applied in the absence of and not against statutory law or rules of procedure.
ASIA BREWERY V. CA (1993) Court decision: In resolving cases of infringement and unfair competition, courts should take into consideration several factors which would affect its conclusion. E.g. age, training and education of the usual purchaser, the nature and cost of the article, whether the article is bought for immediate consumption and also the conditions under which it is usually purchased. The ruling in Del Monte should not be applied here because of the consideration of the aforementioned factors. Beer is not usually picked up from a store shelf but ordered by brand by the beer drinker himself from the storekeeper or waiter in a pub or restaurant.
COCA-‐COLA V. GOMEZ (2008) Court decision: Unfair competition previously defined in PH jurisprudence in relation with RA 166 and Articles 188 and 189 of RPC is now covered by Section 168 IPC as this Code has expressly repealed RA 165, RA 166 and Articles 188 and 189 of the RPC. The law does not cover every
o 119
Katrina Michelle Mancao unfair act committed in the course of business, it covers only acts characterized by “deception or any other means contrary to good faith” in the passing off of goods and services as those of another or who has established goodwill in relation with these goods or services or any other act calculated to produce the same result. Under Sec. 168 of RA 8293, [1] deception, [2] passing off and [3] fraud upon the public are still the key elements that must be present for unfair competition to exist. “True test of unfair competition” – whether the acts of defendant are such as calculated to deceive the ordinary buyer making his purchases under the ordinary conditions which prevail in the particular trade to which the controversy relates. Re hoarding (collection of the petitioner’s empty bottles so that they can be withdrawn from circulation, and thus impede the circulation of the petitioner’s bottled products) à The critical question is not the intrinsic unfairness of the act of hoarding—what is critical for purposes of Sec. 168 (3) is to determine if the hoarding, as charged, is “of a nature calculated to discredit the goods, business or services” of another. Given the IPC’s specific focus, FIRST TEST [that should be made when question arises on whether a matter is covered by the Code] -‐ to ask if it refers to an IP right as defined in the Code SECOND TEST – if a disputed matter does not expressly refer to an IP right as defined above, is whether it falls under the general “unfair competition” concept and definition under Sections 168.1 and 168.2 of the IPC Hoarding of a competitor’s product does not fall within the coverage of the IPC and of Sec. 168 in particular. It does not relate to any patent, TM, trade name or service that the respondents have invaded, intruded into or used without proper authority from the petitioner nor are the respondents alleged to have fraudulently passed of their products or services as those of the petitioner. Hoarding for purposes of destruction is closer to RA 623’s provision (which has not been expressly repealed by RA 8293). • Hoarding is not within purview of unfair competition; there could be no probable cause. Does not cover unfair act, only the deception and contrary to good faith. • Must be calculated to discredit • Adverse reputational impact on the business. • Obiter: in order for unfair competition committed, it must be related to intellectual property. It’s not just passing off, it’s broader than that. Objective is really
•
to protect the property rights of the business. Bottle is very critical, that’ the most expensive component.
SUPERIOR COMMERCIAL ENTERPRISES V. KUNNAN ENTERPRISES LTD (2010) Court decision: There can be trademark infringement without unfair competition, such as when the infringer discloses on the labels containing the mark that he manufactures the goods, thus preventing the public from being deceived that the goods originate from the TM owner. INFRINGEMENT: Elements of trademark infringement: 1) Validity of the plaintiff’s mark 2) Plaintiff’s ownership of the mark 3) Use of the mark or its colorable imitation by the alleged infringer results in “likelihood or confusion” UNFAIR COMPETITION: passing off (or palming off) or attempting to pass off upon the public of the goods or business of one person as the goods or business of another with the end and probable effect of deceive the public. Elements of unfair competition: 1) confusing similarity in the general appearance of goods 2) intent to deceive the public and defraud a competitor
c.
Action for false or fraudulent declaration
SECTION 162. ACTION FOR FALSE OR FRAUDULENT DECLARATION. – Any person who shall procure registration in the Office of a mark by a false or fraudulent declaration or representation, whether oral or in writing, or by any false means, shall be liable in a civil action by any person injured thereby for any damages sustained in consequence thereof
SECTION 163. JURISDICTION OF COURT. – All actions under Sections 150, 155, 164, and 166 to 169 shall be brought before the proper courts with appropriate jurisdiction under existing laws.
120 z
Intellectual Property Law 169.2. Any goods marked or labelled in contravention of the provisions of this Section shall not be imported into the Philippines or admitted entry at any customhouse of the Philippines. The owner, importer, or consignee of goods refused entry at any customhouse under this section may have any recourse under the customs revenue laws or may have the remedy given by this Act in cases involving goods refused entry or seized.
SECTION 164. NOTICE OF FILING SUIT GIVEN TO THE DIRECTOR. – It shall be the duty of the clerks of such courts within one (1) month after the filing of any action, suit, or proceeding involving a mark registered under the provisions of this Act, to notify the Director in writing setting forth: the names and addresses of the litigants and designating the number of the registration or registrations and within one (1) month after the judgment is entered or an appeal is taken, the clerk of court shall give notice thereof to the Office, and the latter shall endorse the same upon the filewrapper of the said registration or registrations and incorporate the same as a part of the contents of said filewrapper.
SECTION 232. APPEALS. – 232.1. Appeals from decisions of regular courts shall be governed by the Rules of Court. Unless restrained by a higher court, the judgment of the trial court shall be executory even pending appeal under such terms and conditions as the court may prescribe. 232.2. Unless expressly provided in this Act or other statutes, appeals from decisions of administrative officials shall be provided in the Regulations. CIVIL REMEDIES -‐ Venue: Commercial courts CRIMINAL REMEDIES -‐ Venue: Commercial courts CAUSES OF ACTION: Infringement v. Unfair Competition Infringement Unfair Competition Definition Unauthorized use Passing off of of the trademark one’s goods as goods of another Registration Required NOT required – that’s why it's the last resort Fraudulent Not required Required – harder intent to prove Wrongful Use of mark Damaged good act without consent will of the business Ma’am: Thus, better remedy is infringement. Unfair competition is resorted to only after infringement failed (if applicable). Infringement may be proved upon showing certificate of registration. Question: Why do we still have the remedy of unfair competition if registration is the basis of the right? To give a remedy to persons with well-‐known mark but are not registered in the Philippines. Basis of unfair competition: Protection of the goodwill of the business, which is a property right. Section 168.3’s enumeration – merely illustrative. 168.3 states “without in any way limiting the scope of
SECTION 232. APPEALS. – 232.1. Appeals from decisions of regular courts shall be governed by the Rules of Court. Unless restrained by a higher court, the judgment of the trial court shall be executory even pending appeal under such terms and conditions as the court may prescribe. 232.2. Unless expressly provided in this Act or other statutes, appeals from decisions of administrative officials shall be provided in the Regulations.
d.
Action for false declaration of origin
SECTION 169. FALSE DESIGNATIONS OF ORIGIN; FALSE DESCRIPTION OR REPRESENTATION. – 169.1. Any person who, on or in connection with any goods or services, or any container for goods, uses in commerce any word, term, name, symbol, or device, or any combination thereof, or any false designation of origin, false or misleading description of fact, or false or misleading representation of fact, which: (a) Is likely to cause confusion, or to cause mistake, or to deceive as to the affiliation, connection, or association of such person with another person, or as to the origin, sponsorship, or approval of his or her goods, services, or commercial activities by another person; or (b) In commercial advertising or promotion, misrepresents the nature, characteristics, qualities, or geographic origin of his or her or another person's goods, services, or commercial activities, shall be liable to a civil action for damages and injunction provided in Sections 156 and 157 of this Act by any person who believes that he or she is or is likely to be damaged by such act.
o 121
Katrina Michelle Mancao protection against unfair competition.” Note: Includes a false statement in the course of trade calculated to discredit the goods, business or services of another. à Spreading rumors about the competitor. Coca Cola case à hoarding does not fall within the purview of unfair competition. Unfair competition does not cover every act unfair to the business. Ma’am: Note the obiter, which states that for there to be unfair competition, the act must be related to IP. Infringement and unfair competition are based on CONFUSING SIMILARITY between goods, business, or service. Tests: 1. Dominancy test – does not require that every element be similar. Requires only that the dominant element be similar. Most strict test. More often than not, plaintiff will win. 2. Holistic test – requires a side-‐by-‐side comparison of the goods. Also not limited to the visual similarities and differences, includes a consideration of how the products are presented. Most lenient test. More often than not, defendant will win. Del Monte case à did not specify how to determine which test to use, but according to Ma’am, it provides a rational explanation. According to the Court, for ordinary, everyday, inexpensive goods, the test to be applied is the dominancy test, because buyers do not usually scrutinize inexpensive goods. On the other hand, for expensive goods, the test to be applied is the holistic test, because buyers usually take the time to scrutinize the products before buying. Ma’am: The guidelines as to what test to apply are not clear-‐cut. The courts are actually given wide latitude of discretion to choose which test to apply. It would depend on the court’s appreciation of the facts and the circumstances of each case. Note: McDonald’s case, SC held that dominancy test should be applied because it has legal basis. UNFAIR COMPETITION: Elements: 1. Confusing similarity 2. Fraudulent intent Solid Triangle case* à no unfair competition because the goods were genuine. Other party was a parallel importer of genuine products.
FALSE OR FRAUDULENT DECLARATION: Ma’am: This is the most superfluous provision. Only a civil remedy, no criminal aspect because not included in Section 170. Thus, it should not be used as an independent cause of action. It may be used only as an alternate cause of action. FALSE DESIGNATION OF ORIGIN OR FALSE DESCRIPTION: Ma’am: Example – In-‐n-‐Out Burger claiming the same as the foreign business.
3.
Criminal
SECTION 170. PENALTIES. – Independent of the civil and administrative sanctions imposed by law, a criminal penalty of imprisonment from TWO (2) YEARS TO FIVE (5) YEARS and a fine ranging from Fifty thousand pesos (P50,000) to Two hundred thousand pesos(P200,000), shall be imposed on any person who is found guilty of committing any of the acts mentioned in Section 155 (Infringement), Section 168 (Unfair competition) and Subsection 169.1 (False designation of origin).
SY V. CA (1982) Court decision: Where the accused was charged with unfair competition, but the TC, being of the opinion that the crime is infringement of TM, neither acquitted nor found the accused guilty but ordered the filing of a new and proper information, there can be no double jeopardy. There can be no double jeopardy where there was neither acquittal nor conviction, but merely an order to file a new information for the correct offense. A new preliminary investigation is not called for where court orders filing of correct information involving a cognate offense (unfair competition to infringement of TM).
SAMSON V. JUDGE DAWAY (2004) Court decision: Which court has jurisdiction over criminal and civil cases for violation of intellectual property rights? Regional Trial Courts Sec. 163 of IPC states that actions (both civil and criminal) under Sec. 150, 155, 164, 166, 167, 168 and 169 shall be brought before proper courts with appropriate jurisdiction “under existing laws.” The existing law referred to in Sec. 163 is Sec. 27 of RA 166. RA 166 is not repealed by RA 8293 because of the wording of the repealing clause in 239.1 wherein it mentioned “parts of Acts” and “inconsistent therewith.”
122 z
Intellectual Property Law The use of such phrases only mean that the repeal pertains only to provisions which are repugnant or not susceptible of harmonization with RA 8293.
CONVERSE RUBBER CORPORATION V. UNIVERSAL RUBBER PRODUCTS (1987) Court decision: WON the respondent’s partial appropriation of petitioner’s corporate name is of such character that it is calculated to deceive or confuse the public to the injury of the petitioner to which the name belongs? A tradename is any individual name or surname, firm name, device or word used by manufacturers, industrialists, merchants and others to identify their businesses, vocations or occupations. As the tradename refers to the business and its goodwill, the trademark refers to the goods. There is a basic similarity between the tradenames “Universal Converse and Device” and “Converse Rubber Corporation” as in both names CONVERSE is the dominant word and both parties engaged in similar business. A tradename may not be similar to a corporate name. – A corporation is entitled to the cancellation of a mark that is confusingly similar to its corporate name. Appropriation by another of the dominant part of a corporate name is an infringement. The determinative factor in ascertaining whether or not marks are confusingly similar to each other is “not whether the challenged mark would actually cause confusion or deception of the purchasers but whether use of such mark would likely cause confusion or mistake on the party of the buying public.” The risk of damage is not limited to a possible confusion of goods but also includes confusion of reputation if the public could reasonably assume that the goods of the parties originated from the same source. Converse Rubber Corporation has already earned a business reputation in this country which should be protected.
YAO, SR. V. PEOPLE OF THE PHILIPPINES (2007) Court decision: Corporation owned by Yao family were allegedly engaged in unlawful refilling and selling of LPG tanks owned by Shell and Petron. A search warrant was issued against their store by virtue of NBI seized several cylindrical LPG tanks. Yao family demands return of the items seized by the raiding team considering that no action for infringement has been filed against them or their corporation, invoking Sec. 20 of AM 02-‐01-‐06-‐SC which provides that: Sec. 20. Failure to file complaint. – The writ shall also, upon motion of the expected adverse party, be set aside and the seized documents and articles returned to the expected adverse party if no case is filed with the appropriate court or authority within 31 calendar days from the date of the issuance of the writ. AM 02-‐1-‐06-‐SC is not applicable in the present case because it governs only searches and seizures in civil actions for infringement of intellectual property rights. The offense complained of herein is for a criminal violation of Sec. 155 in relation Sec. 170 of RA 8293. Samson v. Daway case à justified the jurisdiction of the RTC on the basis of RA 166, in connection with IP Code’s repealing clause. Ma’am: RA 166 was resurrected! But actually, since the IP Code was a codification effort, there was actually an intent to repeal the entire Code. Better resolution would have been that the SC has a delegated power under BP 129 . Ownership and search warrant validity. Ownership is not material. Truck was seized and even if it did not belong, as long as it is found in crime scene. Ownership would not matter.
J.
2.
SECTION 165.1.
Tradenames
A name or designation may not be used as a trade name if by its nature or the use to which such name or designation may be put, it is contrary to public order or morals and if, in particular, it is liable to deceive trade circles or the public as to the nature of the enterprise identified by that name.
1.
Definition
SECTION 121.3.
What may not be used as a tradename?
"TRADE NAME" means the name or designation identifying or distinguishing an enterprise
o 123
Katrina Michelle Mancao 3.
This principle proceeds upon the theory that it is fraud on the corporation which has acquired a right to that name and perhaps carried on its business thereunder that another should attempt to use the same name or the same name with a slight variation in such a way as to induce persons to deal with it in the belief that they are dealing with the corporation which has given a reputation to the name.
Rights of the tradename owner
SECTION 165.2. (a) Notwithstanding any laws or regulations providing for any obligation to register trade names, such names shall be protected, even prior to or without registration, against any unlawful act committed by third parties. (b) In particular, any subsequent use of the trade name by a third party, whether as a trade name or a mark or collective mark, or any such use of a similar trade name or mark, likely to mislead the public, shall be deemed unlawful.
ARMCO STEEL CORPORATION V. SEC (1987) Court decision: A trademark is entitled to protection in the use thereof in the Philippines. The owner has the right to the exclusive use and enjoyment of the term. Armco Steel-‐Philippines and Armco Steel-‐Ohio have not only identical name but also a similarity in line of business. People who are buying and using products bearing the TM Armco might be led to believe that such products are manufactured by the respondent, when in fact, they might actually be produced by the petitioners. Thus, the goodwill that should grow and inure to the benefit of the petitioners could be impaired and prejudiced by the continued use of the same term by the respondent.
SECTION 165.3. The remedies provided for in Sections 153 to 156 and Sections 166 and 167 shall apply mutatis mutandis.
SECTION 165.4. Any change in the ownership of a trade name shall be made with the transfer of the enterprise or part thereof identified by that name. The provisions of Subsections 149.2 to 149.4 shall apply mutatis mutandis.
WESTERN EQUIPMENT & SUPPLY CO V. REYES (1927)
PHILIPS EXPORT V. CA (1992)
Court decision: An unregistered foreign corporation which has not personally transacted in the PH but which has acquired valuable goodwill and high reputation therein through the sale by importers and the extensive use within the territory of its products bearing either its corporate name or TM has a legal right to restrain an officer of the government, who has full knowledge of those facts, from issuing a certificate of incorporation to residents of the PH who are attempting to organize a corporation for the purpose of pirating the corporate name of the foreign corporation and of engaging in the same business, for the purpose of making the public believe that the goods which it proposes to sell are the goods of the foreign corporation and of defrauding it and its local dealers of their legitimate trade. The purpose of such a suit is to protect its reputation, corporate name and goodwill which have been established through the natural development of its trade over a long period of years, in the doing of which it does not seek to enforce any legal or contract rights arising from or growing out of any business which it has transacted in the Ph. Under such state of facts, the right to the use of the corporate and trade name of the foreign corporation is a property right, a right in rem, which it may assert and protect in any of the courts of the world even in countries where it does not personally transact any business.
Court decision: A corporation’s right to use its corporate and trade name is a property right, a right in rem which it may assert and protect against the world in the same manner as it may protect its tangible property, real or personal against trespass or conversion. It is regarded to a certain extent, as a property right and one which cannot be impaired or defeated by subsequent appropriation by another corporation in the same field. GR as to corporation: Each corporation must have a name by which it is to sue and be sued and do all legal acts. A corporation can no more use a corporate name in violation of the right of others than an individual can use his name legally acquired so as to mislead the public and injure another. The right to the exclusive use of a corporate name with freedom from infringement by similarity is determined with priority of adoption. In determining the existence of confusing similarity in corporate name, the test is whether the similar is such as to mislead a person using ordinary care and discrimination. A corporation has an exclusive right to the use of its name which may be protected by injunction upon a principle similar to that upon which persons are protected in the use of trademarks and tradenames. à
124 z
Intellectual Property Law It is the trade and not the mark that is to be protected, and a TM does not acknowledge any territorial boundaries, but extends to every market where the trader’s goods have become known and identified by the use of the mark.
licensed by, or sourced from, Harvard University without the latter's consent. TRADE NAME: Old law: Trademark same as trade name. IP Code: Trade name is different. Trade name is a name or designation identifying an enterprise. Differences in treatment between trademark and trade name: -‐ REGISTRATION – Under the IP Code, registration is not required to acquire protection of trade name. The basis of protection is prior use in Philippine commerce. You can set this up against the trademark owner. (Note 168.3) -‐ REMEDIES AVAILABLE – Note the problem with non-‐registration of trade name: There is a limitation of remedy available to the trade name owner. Trade name owner does not have criminal remedy. Only civil and administrative. -‐ ASSIGNABILITY – Trade name is NOT separate to the business. Trade name may be transferred only with the transfer of the business. Trademark or service mark may be assigned independently of the business. SERVICE MARK: Trademark used in services.
FREDCO MANUFACTURING CORPORATION V PRESIDENT AND FELLOWS OF HARVARD COLLEGE (2011) Court decision: Article 8 of the Paris Convention has been incorporated in Section 37 of R.A. No. 166, as follows: Article 8, Paris Convention -‐-‐ A trade name shall be protected in all the countries of the Union without the obligation of filing or registration, whether or not it forms part of a trademark. Section 37, RA 166. Rights of foreign registrants. -‐-‐ Persons who are nationals of, domiciled in, or have a bona fide or effective business or commercial establishment in any foreign country, which is a party to any international convention or treaty relating to marks or trade-‐names, or the repression of unfair competition to which the Philippines may be a party, shall be entitled to the benefits and subject to the provisions of this Act to the extent and under the conditions essential to give effect to any such convention and treaties so long as the Philippines shall continue to be a party thereto, except as provided in the following paragraphs of this section. xxxx Trade-‐names of persons described in the first paragraph of this section shall be protected without the obligation of filing or registration whether or not they form parts of marks. xxxx
K. Collective Marks
1.
Definition
SECTION 121.2.
Thus, under Philippine law, a trade name of a national of a State that is a party to the Paris Convention, whether or not the trade name forms part of a trademark, is protected “without the obligation of filing or registration.” “Harvard” is the trade name of the world famous Harvard University, and it is also a trademark of Harvard University. Under Article 8 of the Paris Convention, as well as Section 37 of R.A. No. 166, Harvard University is entitled to protection in the Philippines of its trade name "Harvard" even without registration of such trade name in the Philippines. This means that no educational entity in the Philippines can use the trade name "Harvard" without the consent of Harvard University. Likewise, no entity in the Philippines can claim, expressly or impliedly through the use of the name and mark "Harvard," that its products or services are authorized, approved, or
"COLLECTIVE MARK" means any visible sign designated as such in the application for registration and capable of distinguishing the origin or any other common characteristic, including the quality of goods or services of different enterprises which use the sign under the control of the registered owner of the collective mark
SECTION 167. COLLECTIVE MARKS. – 167.1. Subject to Subsections 167.2 and 167.3, Sections 122 to 164 and 166 shall apply to collective marks, except that references therein to "mark" shall be read as "collective mark". 167.2. (a) An application for registration of a collective mark shall designate the mark as a collective mark and shall be accompanied by a copy of the agreement, if any, governing the use of the collective mark.
o 125
Katrina Michelle Mancao (b) The registered owner of a collective mark shall notify the Director of any changes made in respect of the agreement referred to in paragraph (a). 167.3. In addition to the grounds provided in Section 149, the Court shall cancel the registration of a collective mark if the person requesting the cancellation proves that only the registered owner uses the mark, or that he uses or permits its use in contravention of the agreements referred to in Subsection 166.2 or that he uses or permits its use in a manner liable to deceive trade circles or the public as to the origin or any other common characteristics of the goods or services concerned. 167.4. The registration of a collective mark, or an application therefor shall not be the subject of a license contract. COLLECTIVE MARK Mark designed to be used by parties other than the registrant. This is usually used by cooperatives. The owner of the mark is the one who registers, but the members of the cooperative are the ones who use it. The registrant only holds it. But anyone who qualifies to be a member (or meets the standard of the agreement) may use the collective mark. In Canada, this is used to protect the products of indigenous communities. In the Philippines, it has not been used.
VII.
Law on Patents
PATENTS This field gives rise to serious issues on access. Unlike copyright that has a very low standard, patent is very exclusionary. Under patents, a parallel invention is not allowed. Patent is a monopoly. Patent owner is given exclusive right to import. Patents are costly to the society. Issuance of patent means that the state allows the patent holder to maximize the economic value of the invention to the exclusion of everybody else. Thus, for patents to work, the state must ensure that it is given only to the one who deserves it. In the Philippines, the patent system is identified as the major cause of the high prices in medicines. Philippines are among the states that has the highest patent standards, even before it was required to do so. Philippines also did not adopt the flexibilities under the TRIPS Agreement. For example, under the TRIPS, the country is allowed to adopt any exhaustion principle it chooses. A country may use the international exhaustion principle. The Philippines, however, decided to adopt the domestic exhaustion principle. Purpose of the patent – to promote the advancement of technology. It was established as an incentive system to encourage innovation. But the ultimate objective is to reward the public. KINDS OF PATENTS: 1. Invention 2. Utility model 3. Industrial design 4. Layout circuits – it was designed to encourage semi-‐ conductor companies to invest in the Philippines (but as of date, no registration for this). STANDARDS OF PATENT PROTECTION: -‐ Key to effective patent protection. -‐ Note: Different rules apply to different countries. You might have to consider this in advising clients on registration. 1. Novelty -‐ Not part of prior art. Prior art is anything made available on any part of the world. -‐ Anything found in the public domain is not novel part. Anything also part of prior application is part of prior art. -‐ Philippine law exception – 1 year of non-‐ prejudicial disclosure. 2. Inventiveness -‐ Anything that is not obvious to a person skilled
126 z
Intellectual Property Law in the art. “Person skilled in the art” has no uniform definition. Here in the Philippines, definition is textbook. In some countries, term limited to those with doctorate degree. “Evergreening of a patent” – attempt to prolong the patent monopoly by changing one element.
-‐
-‐
3.
SECTION 236. PRESERVATION OF EXISTING RIGHTS. – Nothing herein shall adversely affect the rights on the enforcement of rights in patents, utility models, industrial designs, marks and works, acquired in good faith prior to the effective date of this Act.
RA 9502 -‐ UNIVERSALLY ACCESSIBLE CHEAPER AND QUALITY MEDICINES ACT OF 2008 -‐
Industrial applicability -‐ Has some use.
SECTION 47. SEPARABILITY CLAUSE. – Any portion or provision of this Act that may be declared unconstitutional or invalid shall not have the effect of nullifying other portions and provisions hereof as long as such remaining portion or provision can still subsist and be given effect in their entirety. SECTION 48. REPEALING CLAUSE. – All laws, decrees, executive orders, proclamations and administrative regulations or parts thereof inconsistent herewith are hereby repealed or modified accordingly. SECTION 49. EFFECTIVITY CLAUSE. – This Act shall take effect fifteen (15) days after its publication in at least two (2) national papers of general circulation.
Invention Utility model Industrial design Layout circuit
Novelty
Inventiveness
Industrial Applicability
ü
ü
ü
ü
û
ü
ü
û
û
ü
(originality)
A. Legislative history
REPUBLIC ACT 165 -‐ AN ACT CREATING A PATENT OFFICE, PRESCRIBING ITS POWERS AND DUTIES, REGULATING THE ISSUANCE OF PATENTS, AND APPROPRIATING FUNDS THEREFOR -‐
amended certain provision of the Intellectual Property Code
B. Definition
Took effect on 20 June 1947
“The right of monopoly secured by statute to those who invent or discover new and useful devices and processes. The exclusive right of manufacture, sale, or use secured by statute to one who invents or discovers a new and useful device or process.” (US Dictionary)
SECTION 239. REPEALS. – 239.1. All Acts and parts of Acts inconsistent herewith, more particularly Republic Act No. 165, as amended; Republic Act No. 166, as amended; and Articles 188 and 189 of the Revised Penal Code; Presidential Decree No. 49, including Presidential Decree No. 285, as amended, are hereby repealed.
C.
Purpose
MANZANO V. CA (1997)
SECTION 235. APPLICATIONS PENDING ON EFFECTIVE DATE OF ACT. –
Court decision: The primary purpose of the patent system is not the reward of the individual but the advancement of the arts and sciences. The function of a patent is to add to the sum of useful knowledge and one of the purposes of the patent system is to encourage dissemination of information concerning discoveries and inventions. This is a matter which is properly within the competence of the Patent Office the official action of which has the presumption of correctness and may not be interfered with in the absence of new evidence carrying thorough conviction that the Office has erred. Since the Patent Office is an expert body preeminently qualified to determine questions of patentability, its findings must be accepted if they are consistent with the evidence, with doubts as to patentability resolved in favor of the Patent Office.
235.1. All applications for patents pending in the Bureau of Patents, Trademarks and Technology Transfer shall be proceeded with and patents thereon granted in accordance with the Acts under which said applications were filed, and said Acts are hereby continued to be enforced, to this extent and for this purpose only, notwithstanding the foregoing general repeal thereof: Provided, That applications for utility models or industrial designs pending at the effective date of this Act, shall be proceeded with in accordance with the provisions of this Act, unless the applicants elect to prosecute said applications in accordance with the Acts under which they were filed.
o 127
Katrina Michelle Mancao
advances in technology and design, in return for the exclusive right to practice the invention for a number of years. The inventor may keep his invention secret and reap its fruits indefinitely. In consideration of its disclosure and the consequent benefit to the community, the patent is granted. An exclusive enjoyment is guaranteed him for 17 years, but upon the expiration of that period, the knowledge of the invention inures to the people, who are thus enabled to practice it and profit by its use.”
PEARL & DEAN (PHIL.), INC. V. SHOEMART, SUPRA Refresher: Light boxes Doctrine: On patent infringement: For some reason or another, petitioner never secured a patent for the light boxes. It therefore acquired no patent rights which could have protected its invention, if in fact it really was. And because it had no patent, petitioner could not legally prevent anyone from manufacturing or commercially using the contraption. In Creser Precision Systems, Inc. vs. Court of Appeals, we held that “there can be no infringement of a patent until a patent has been issued, since whatever right one has to the invention covered by the patent arises alone from the grant of patent. An inventor has no common law right to a monopoly of his invention. He has the right to make use of and vend his invention, but if he voluntarily discloses it, such as by offering it for sale, the world is free to copy and use it with impunity. A patent, however, gives the inventor the right to exclude all others. As a patentee, he has the exclusive right of making, selling or using the invention. On the assumption that petitioner’s advertising units were patentable inventions, petitioner revealed them fully to the public by submitting the engineering drawings thereof to the National Library. To be able to effectively and legally preclude others from copying and profiting from the invention, a patent is a primordial requirement. No patent, no protection. The ultimate goal of a patent system is to bring new designs and technologies into the public domain through disclosure. Ideas, once disclosed to the public without the protection of a valid patent, are subject to appropriation without significant restraint. On one side of the coin is the public which will benefit from new ideas; on the other are the inventors who must be protected. As held in Bauer & Cie vs. O’Donnel, “The act secured to the inventor the exclusive right to make use, and vend the thing patented, and consequently to prevent others from exercising like privileges without the consent of the patentee. It was passed for the purpose of encouraging useful invention and promoting new and useful inventions by the protection and stimulation given to inventive genius, and was intended to secure to the public, after the lapse of the exclusive privileges granted the benefit of such inventions and improvements.” The law attempts to strike an ideal balance between the two interests: “The patent system thus embodies a carefully crafted bargain for encouraging the creation and disclosure of new useful and non-‐obvious
The patent law has a three-‐fold purpose: “first, patent law seeks to foster and reward invention; second, it promotes disclosures of inventions to stimulate further innovation and to permit the public to practice the invention once the patent expires; third, the stringent requirements for patent protection seek to ensure that ideas in the public domain remain there for the free use of the public.” It is only after an exhaustive examination by the patent office that a patent is issued. Such an in-‐depth investigation is required because “in rewarding a useful invention, the rights and welfare of the community must be fairly dealt with and effectively guarded. To that end, the prerequisites to obtaining a patent are strictly observed and when a patent is issued, the limitations on its exercise are equally strictly enforced. To begin with, a genuine invention or discovery must be demonstrated lest in the constant demand for new appliances, the heavy hand of tribute be laid on each slight technological advance in art.” There is no such scrutiny in the case of copyrights nor any notice published before its grant to the effect that a person is claiming the creation of a work. The law confers the copyright from the moment of creation and the copyright certificate is issued upon registration with the National Library of a sworn ex-‐parte claim of creation.
D. What are patentable?
1.
Inventions
SECTION 21. PATENTABLE INVENTIONS. – Any technical solution of a problem in any field of human activity which is new, involves an inventive step and is industrially applicable shall be Patentable. It may be, or may relate to, a product, or process, or an improvement of any of the foregoing.
128 z
Intellectual Property Law Standards: a. Novelty
MAGUAN V. CA (1986) Refresher: Patent for powder puff (in cosmetics) Doctrine: It has been repeatedly held that an invention must possess the essential elements of novelty , originality and precedence and for the patentee to be entitled to protection, the invention must be new to the world. Accordingly, a single instance of public use of the invention by a patentee for more than two years (now for more than one year only under Sec. 9 of the Patent Law) before the date of his application for his patent, will be fatal to, the validity of the patent when issued. It will be noted that the validity of petitioner's patents is in question for want of novelty. Private respondent contends that powder puffs Identical in appearance with that covered by petitioner's patents existed and were publicly known and used as early as 1963 long before petitioner was issued the patents in question. As correctly observed by respondent Court of Appeals, "since sufficient proofs have been introduced in evidence showing a fair question of the invalidity of the patents issued for such models, it is but right that the evidence be looked into, evaluated and determined on the merits so that the matter of whether the patents issued were in fact valid or not may be resolved." All these notwithstanding, the trial court nonetheless issued the writ of preliminary injunction which under the circumstances should be denied. For failure to determine first the validity of the patents before aforesaid issuance of the writ, the trial court failed to satisfy the two requisites necessary if an injunction is to issue, namely: the existence of the right to be protected and the violation of said right.
SECTION 23. NOVELTY. – An invention shall not be considered new if it forms part of a prior art.
SECTION 24. PRIOR ART. – Prior art shall consist of: 24.1. Everything which has been made available to the public anywhere in the world, before the filing date or the priority date of the application claiming the invention; and 24.2. The whole contents of an application for a patent, utility model, or industrial design registration, published in accordance with this Act, filed or effective in the Philippines, with a filing or priority date that is earlier than the filing or priority date of the application: Provided, That the application which has validly claimed the filing date of an earlier application under Section 31 of this Act, shall be prior art with effect as of the filing date of such earlier application: Provided further, That the applicant or the inventor identified in both applications are not one and the same.
SECTION 25. NON-‐PREJUDICIAL DISCLOSURE. – 25.1. The disclosure of information contained in the application during the twelve (12) months preceding the filing date or the priority date of the application shall not prejudice the applicant on the ground of lack of novelty if such disclosure was made by: (a) The inventor; (b) A patent office and the information was contained (a) in another application filed by the inventor and should not have been disclosed by the office, or (b) in an application filed without the knowledge or consent of the inventor by a third party which obtained the information directly or indirectly from the inventor; or (c) A third party which obtained the information directly or indirectly from the inventor. 25.2. For the purposes of Subsection 25.1, "inventor" also means any person who, at the filing date of application, had the right to the patent.
VARGAS V. F.M. YAPTICO & CO. (1919) Refresher: Adjustable plow. Court decision: When a patent is sought to be enforced, "the question of invention, novelty, or prior use, and each of them, are open to judicial examination." The burden of proof to substantiate a charge of infringement is with the plaintiff. Where, however, the plaintiff introduces the patent in evidence, if it is in due form, it affords a prima facie presumption of its correctness and validity. The decision of the Commissioner of Patents in granting the patent is always presumed to be correct. The burden then shifts to the defendant to overcome by competent evidence this legal presumption. With all due respect, therefore, for the critical and expert examination of the invention by the United States Patent Office, the question of the validity of the patent is one for judicial determination,
o 129
Katrina Michelle Mancao and since a patent has been submitted, the exact question is whether the defendant has assumed the burden of proof as to anyone of his defenses. On the facts, we think the testimony shows such a public use of the Vargas plow as to render the patent invalid. For more than two years before the application for the original letters patent, or before July 22, 1908, there was, by the consent and allowance of Vargas, a public use of the invention covered by them.
came into existence. It may also be noted that Adrian de Icsiar applied for a patent on an invention which resulted in the rejection by the United States Patent Office of the plaintiffs' original application for a patent on the so called "spindle" or conical drum which was then in actual use in the Dringman and Icsiar hemp stripping machines.
b.
Inventiveness
SECTION 26. INVENTIVE STEP. – 26.1. An invention involves an inventive step if, having regard to prior art, it is not obvious to a person skilled in the art at the time of the filing date or priority date of the application claiming the invention. 26.2. In the case of drugs and medicines, there is no inventive step if the invention results from the mere discovery of a new form or new property of a known substance which does not result in the enhancement of the known efficacy of that substance, or the mere discovery of any new property or new use for a known substance, or the mere use of a known process unless such known process results in a new product that employs at least one new reactant.
VARGAS V. CHUA (1933) Court decision: We have carefully examined all the plows presented as exhibits as well as the designs of those covered by the patent, and we are convinced that no substantial difference exists between the plow, Exhibit F, and the plow, Exhibit 3-‐Chua which was originally patented by the appellee, Vargas. The only difference noted by us is the suppression of the bolt and the three holes on the metal strap attached to the handle bar. These holes and bolt with its nut were suppressed in Exhibit F in which the beam is movable as in the original plow. The members of this court, with the plows in view, arrived at the conclusion that not only is there no fundamental difference between the two plows but no improvement whatever has been made on the latest model, for the same working and movement of the beam existed in the original model with the advantage, perhaps, that its graduation could be carried through with more certainty by the use of the bolt which as has already been stated, was adjustable and movable. In view of the foregoing, we are firmly convinced that the appellee is not entitled to the protection he seeks for the simple reason that his plow, Exhibit F, does not constitute an invention in the legal sense, and because, according to the evidence, the same type of plows had been manufactured in this country and had been in use in many parts of the Philippine Archipelago, especially in the Province of Iloilo, long before he obtained his last patent.
AGUAS V. DE LEON (1982) Court decision: The records disclose that de Leon's process is an improvement of the old process of tile making. The tiles produced from de Leon's process are suitable for construction and ornamentation, which previously had not been achieved by tiles made out of the old process of tile making. De Leon's invention has therefore brought about a new and useful kind of tile. The old type of tiles were usually intended for floors although there is nothing to prevent one from using them for walling purposes. These tiles are neither artistic nor ornamental. They are heavy and massive. The respondent's improvement is indeed inventive and goes beyond the exercise of mechanical skill. He has introduced a new kind of tile for a new purpose. He has improved the old method of making tiles and pre-‐cast articles which were not satisfactory because of an intolerable number of breakages, especially if deep engravings are made on the tile. He has overcome the problem of producing decorative tiles with deep engraving, but with sufficient durability. Durability inspite of the thinness and lightness of the tile, is assured, provided that a certain critical depth is maintained in relation to the dimensions of the tile.
FRANK V. KOSUYAMA (1933) Court decision: We agree with the trial court that, strictly speaking, the hemp stripping machine of the plaintiffs does not constitute an invention on the ground that it lacks the elements of novelty, originality and precedence. In fact, before the plaintiffs herein obtained their patent, they themselves had already publicly used the same kind of machine for some months, at least, and, various other machines, having in general, the same characteristics and important parts as that of the said plaintiffs, were known in the Province of Davao. Machines known as Molo, Riesgo, Crumb, Icsiar, Browne and McFie were already known in that locality and used by the owners of hemp plantations before the machine of the plaintiffs
MANZANO V. CA (1997), SUPRA Court decision: The element of novelty is an essential requisite of the patentability of an invention or discovery. If a device or process has been known or used by others prior to its
130 z
Intellectual Property Law invention or discovery by the applicant, an application for a patent therefor should be denied; and if the application has been granted, the court, in a judicial proceeding in which the validity of the patent is drawn in question, will hold it void and ineffective. It has been repeatedly held that an invention must possess the essential elements of novelty, originality and precedence, and for the patentee to be entitled to the protection the invention must be new to the world. In issuing Letters Patent No. UM-‐4609 to Melecia Madolaria for an "LPG Burner" on 22 July 1981, the Philippine Patent Office found her invention novel and patentable. The issuance of such patent creates a presumption which yields only to clear and cogent evidence that the patentee was the original and first inventor. The burden of proving want of novelty is on him who avers it and the burden is a heavy one which is met only by clear and satisfactory proof which overcomes every reasonable doubt.
c.
113.2. Industrial designs dictated essentially by technical or functional considerations to obtain a technical result or those that are contrary to public order, health or morals shall not be protected.
4.
REPUBLIC ACT 9150 -‐ AN ACT PROVIDING FOR THE PROTECTION OF LAYOUT-‐DESIGNS (TOPOGRAPHIES) OF INTEGRATED CIRCUITS, AMENDING FOR THE PURPOSE OF CERTAIN SECTIONS OF RA 8293, OTHERWISE KNOWN AS THE IPC SECTION 1. Sections 112,113,114,116,117,118,119 and 120 under Chapter XIII of R. A. No. 8293 are hereby amended to read as follows: CHAPTER XIII INDUSTRIAL DESIGN AND LAYOUT-‐DESIGNS (TOPOGRAPHIES) OF INTEGRATED CIRCUITS SEC. 112. DEFINITION OF TERMS: 1. An Industrial Design is any composition of lines or colors or any three-‐ dimensional form, whether or not associated with lines or colors: Provided, That such composition or form gives a special appearance to and can serve as pattern for an industrial product or handicraft; 2. Integrated Circuit means a product, in its final form, or an intermediate form, in which the elements, at least one of which is an active element, and some or all of the interconnections are integrally formed in and/or on a piece of material, and which is intended to perform an electronic function; and 3. Layout-‐Design is synonymous with 'Topography' and means the three-‐dimensional disposition, however expressed, of the elements, at least one of which is an active element, and of some or all of the interconnections of an integrated circuit, or such a three-‐dimensional disposition prepared for an integrated circuit intended for manufacture." SEC. 113. SUBSTANTIVE CONDITIONS FOR PROTECTION. – 113.1. Only industrial designs that are new or ornamental shall benefit from protection under this Act.
Industrial application
SECTION 27. INDUSTRIAL APPLICABILITY. – An invention that can be produced and used in any industry shall be industrially applicable.
Utility model
SECTION 109. SPECIAL PROVISIONS RELATING TO UTILITY MODELS. – 109.1. (a) An invention qualifies for registration as a utility model if it is new and industrially applicable. (b) Section 21, "Patentable Inventions", shall apply except the reference to inventive step as a condition of protection.
3.
– of
2.
Lay-‐out designs Topographies Integrated Circuits
Industrial designs
SECTION 112. DEFINITION OF INDUSTRIAL DESIGN. – An industrial design is any composition of lines or colors or any three-‐dimensional form, whether or not associated-‐with lines or colors: Provided, That such composition or form gives a special appearance to and can serve as pattern for an industrial product or handicraft.
SECTION 113. SUBSTANTIVE CONDITIONS FOR PROTECTION. – 113.1. Only industrial designs that are new or original shall benefit from protection under this Act.
o 131
Katrina Michelle Mancao 113.2. Industrial designs dictated essentially by technical or functional considerations to obtain a technical result or those that are contrary to public order, health or morals shall not be protected. 113.3. Only layout-‐designs of integrated circuits that are original shall benefit from protection under this Act. A layout-‐design shall be considered original if it is the result of its creator's own intellectual effort and is not commonplace among creators of layout-‐designs and manufacturers of integrated circuits at the time of its creation. 113.4. A layout-‐design consisting of a combination of elements and interconnections that are commonplace shall be protected only if the combination, taken as a whole, is original. SEC. 114. CONTENTS OF THE APPLICATION. – 114.1. Every application for registration of an industrial design or layout-‐design shall contain: (a) A request for registration of the industrial design or layout-‐design; (b) Information identifying the applicant; (c) An indication of the kind of article of manufacture or handicraft to which the industrial design or layout-‐design shall be applied: (d) A representation of the article of manufacture or handicraft by way of drawings, photographs or adequate graphic representation of the industrial design or of the layout-‐design as applied to the article of manufacture or handicraft which clearly and fully discloses those features for which protection is claimed; and (e) The name and address of the creator, or where the applicant is not the creator, a statement indicating the origin of the right to the industrial design or layout-‐design registration. 114.2. The application may be accompanied by a specimen of the article embodying the industrial design or layout-‐design and shall be subject to the payment of the prescribed fee. SEC. 116. EXAMINATION. – 116.1. The Office shall accord as the filing date the date of receipt of the application containing indications allowing the identity of the applicant to be established and a representation of the article embodying the industrial
design or the layout-‐design or a pictorial representation thereof. 116.2. If the application does not meet these requirements, the filing date should be that date when all the elements specified in Sec. 114 are filed or the mistakes corrected. Otherwise, if the requirements are not complied within the prescribed period, the application shall be considered withdrawn. 116.3 After the application has been accorded a filing date and the required fees paid on time, the applicant shall comply with the requirements of Sec. 114 within the prescribed period, otherwise the application shall be considered withdrawn. 116.4. The Office shall examine whether the industrial design or layout-‐design complies with requirements of Sec. 112 (Definitions) and Sec. 113 (Substantive Conditions for Protection). SEC. 117. REGISTRATION. – 117.1. Where the Office finds that the conditions referred to in Sec. 113 are fulfilled, it shall order that registration be effected in the industrial design or layout-‐design register and cause the issuance of an industrial design or layout-‐design certificate of registration; otherwise, it shall refuse the application. 117.2. The form and contents of an industrial design or layout-‐design certificate shall be established by the Regulations: Provided, That the name and address of the creator shall be mentioned in every case. 117.3. Registration shall be published in the form and within the period fixed by the Regulations. 117.4. The Office shall record in the register any change in the identity of the proprietor of the industrial design or layout-‐design or his representative, if proof thereof is furnished to it. A fee shall be paid, with the request to record the change in the identity of the proprietor. If the fee is not paid, the request shall be deemed not to have been filed. In such case, the former proprietor and the former representative shall remain subject to the rights and obligations as provided in this Act. 117.5. Anyone may inspect the Register and the files of registered industrial designs or layout-‐designs including files of cancellation proceedings. SEC. 118. THE TERM OF INDUSTRIAL DESIGN OR LAYOUT-‐DESIGN REGISTRATION. – 118.1. The registration of an industrial design shall be for a period of five (5) years from the filing date of the application.
132 z
Intellectual Property Law 118.2. The registration of an industrial design may be renewed for not more than two (2) consecutive periods of five (5) years each, by paying the renewal fee. 118.3. The renewal fee shall be paid within twelve (12) months preceding the expiration of the period of registration. However, a grace period of six (6) months shall be granted for payment of the fees after such expiration, upon payment of a surcharge. 118.4. The Regulations shall fix the amount of renewal fee, the surcharge and other requirements regarding the recording of renewals of registration. 118.5. Registration of a layout-‐design shall be valid for a period often (10) years, without renewal, and such validity to be counted from the date of commencement of the protection accorded to the layout-‐design. The protection of a layout-‐design under this Act shall commence: a) on the date of the first commercial exploitation, anywhere in the world, of the layout-‐design by or with the consent of the right holder: Provided, That an application for registration is filed with the Intellectual Property Office within two (2) years from such date of first commercial exploitation; or b) on the filing date accorded to the application for the registration of the layout-‐ design if the layout-‐design has not been previously exploited commercially anywhere in the world. SEC. 119. APPLICATION OF OTHER SECTIONS AND CHAPTERS. – 119.1. The following provisions relating to patents shall apply mutatis mutandis to an industrial design registration: SECTION 21 – Novelty; SECTION 24 – Prior art: Provided, That the disclosure is contained in printed documents or in any tangible form; SECTION 25 – Non-‐prejudicial Disclosure; SECTION 28 – Right to a Patent; SECTION 29 – First to File Rule; SECTION 30 – Inventions Created Pursuant to a Commission; SECTION 31 -‐ Right of Priority: Provided, That the application for industrial design shall be filed within six (6) months from the earliest
filing date of the corresponding foreign application; SECTION 33 – Appointment of Agent or Representative; SECTION 51 – Refusal of the Application; SECTIONS 56 to 60 – Surrender, Correction of and Changes in Patent; CHAPTER VII – Remedies of a Person with a Right to Patent; Chapter VIII – Rights of Patentees and Infringement of Patents; and Chapter XI – Assignment and Transmission of Rights. 119.2. If the essential elements of an industrial design which is the subject of an application have been obtained from the creation of another person without his consent, protection under this Chapter cannot be invoked against the injured party. 119.3. The following provisions relating to patents shall apply mutatis mutandis to a layout-‐design of integrated circuits registration: SECTION 28 – Right to a Patent; SECTION 29 – First to File Rule; SECTION 30 – Inventions Created Pursuant to a Commission; SECTION 33 – Appointment of Agent or Representative; SECTION 56 – Surrender of Patent; SECTION 57 – Correction of Mistakes of the Office; SECTION 58 – Correction of Mistakes in the Application; SECTION 59 – Changes in Patents; SECTION 60 – Form and Publication of Amendment; Chapter VII – Remedies of a Person with a Right to Patent; Chapter VIII – Rights of Patentees and Infringement of Patents: Provided, That the
o 133
Katrina Michelle Mancao layout-‐design rights and limitation of layout-‐ design rights provided hereunder shall govern; Chapter X – Compulsory Licensing; Chapter XI – Assignment and Transmission of Rights.
may perform any of the said acts only with respect to the stock on hand or ordered before such time and shall be liable to pay to the right holder a sum equivalent to at least 5% of net sales or such other reasonable royalty as would be payable under a freely negotiated license in respect of such layout-‐design; or (5) Where the act is performed in respect of an identical layout-‐design which is original and has been created independently by a third party.
119.4. Rights Conferred to the Owner of a Layout-‐Design Registration. – The owner of a layout-‐design registration shall enjoy the following rights: (1) to reproduce, whether by incorporation in an integrated circuit or otherwise, the registered layout-‐design in its entirety or any part thereof, except the act of reproducing any part that does not comply with the requirement of originality; and (2) to sell or otherwise distribute for commercial purposes the registered layout-‐ design, an article or an integrated circuit in which the registered layout-‐design is incorporated. 119.5. Limitations of Layout Rights. -‐ The owner of a layout design has no right to prevent third parties from reproducing selling or otherwise distributing for commercial purposes the registered layout-‐design in the following circumstances: (1) Reproduction of the registered layout-‐ design for private purposes or for the sole purpose of evaluation, analysis, research or teaching; (2) Where the act is performed in respect of a layout-‐design created on the basis of such analysis or evaluation and which is itself original in the meaning as provided herein; (3) Where the act is performed in respect of a registered layout-‐design, or in respect of an integrated circuit in which such a layout-‐design is incorporated, that has been put on the market by or with the consent of the right holder; (4) In respect of an integrated circuit where the person performing or ordering such an act did not know and had no reasonable ground to know when acquiring the integrated circuit or the article incorporating such an integrated circuit, that it incorporated an unlawfully reproduced layout-‐design: Provided, however, That after the time that such person has received sufficient notice that the layout-‐ design was unlawfully reproduced, that person
SEC. 120. CANCELLATION OF DESIGN REGISTRATION. – 120.1. At any time during the term of the industrial design registration, any person upon payment of the required fee, may petition the Director of Legal Affairs to cancel the industrial design on any of the following grounds: (a) If the subject matter of the industrial design is not registerable within the terms of Sections 112 and 113; (b) If the subject matter is not new; or (c) If the subject matter of the industrial design extends beyond the content of the application as originally filed. 120.2. Where the grounds for cancellation relate to a part of the industrial design, cancellation may be effected to such extent only. The restriction may be effected in the form of an alteration of the effected features of the design. 120.3. Grounds for Cancellation of Layout-‐Design of Integrated Circuits. -‐ Any interested person may petition that the registration of a layout-‐design be cancelled on the ground that: (i) the layout-‐design is not protectable under this Act; (ii) the right holder is not entitled to protection under this Act; or (iii) where the application for registration of the layout-‐design, was not filed within two (2) years from its first commercial exploitation anywhere in the world. Where the grounds for cancellation are established with respect only to a part of the layout-‐design, only the corresponding part of the registration shall be cancelled. Any cancelled layout-‐design registration or part thereof, shall be regarded as null and void from the beginning and may be expunged from the records of the Intellectual Property Office. Reference to all cancelled
134 z
Intellectual Property Law layout-‐design registration shall be published in the IPO Gazette. SEC. 2. IMPLEMENTING RULES AND REGULATIONS. – The Intellectual Property Office may issue Regulations prescribing details for the implementation of this law. The Regulations may, in particular, provide for the payment of fees in connection with applications for the registration of layout-‐designs of integrated circuits and matters related thereto, including Administrative Instructions relating to the procedures and other functions of the responsible unit duly designated by the Director General. SEC. 3. APPLICABILITY. – The provisions of this Act shall apply to layout-‐designs of integrated circuits that were commercially exploited anywhere in the world from and after January 1998 provided they meet the conditions for protection under this Act. SEC. 4. REPEALING CLAUSE. – All acts and parts of acts inconsistent herewith are hereby repealed or amended accordingly. SEC. 5. SEPARABILITY CLAUSE. – If any provision of this Act or the application of such provision to any circumstance is held invalid, the remainder of this Act shall not be affected thereby. SEC. 6. EFFECTIVITY. – This Act shall take effect fifteen (15) days after its publication in two (2) newspapers of general circulation.
E.
22.2. Schemes, rules and methods of performing mental acts, playing games or doing business, and programs for computers; 22.3. Methods for treatment of the human or animal body by surgery or therapy and diagnostic methods practiced on the human or animal body. This provision shall not apply to products and composition for use in any of these methods; 22.4. Plant varieties or animal breeds or essentially biological process for the production of plants or animals. This provision shall not apply to micro-‐organisms and non-‐biological and microbiological processes. Provisions under this subsection shall not preclude Congress to consider the enactment of a law providing sui generis protection of plant varieties and animal breeds and a system of community intellectual rights protection: 22.5. Aesthetic creations; and 22.6. Anything which is contrary to public order or morality.
UNDER RA 165 (OLD PATENT LAW): SECTION 8. INVENTIONS NOT PATENTABLE. – An invention shall not be patentable if it is contrary to public order or morals, or to public health or welfare, or if it constitutes a mere idea, scientific principle or abstract theorem not embodied in an invention as specified in section seven hereof, or any process not directed to the making or improving of a commercial product. SECTION 9. INVENTION NOT CONSIDERED NEW OR PATENTABLE. – An invention shall not be considered new or capable of being patented if it was known or used by others in the Philippines before the invention thereof by the inventor named in an application for patent for the invention; or if it was patented or described in any printed publication in the Philippines or any foreign country more than one year before the application for a patent therefor; or if it had been in public use or on sale in the Philippines for more than one year before the application for a patent therefor; or if it is the subject matter of a validly issued patent in the Philippines granted on an application filed before the filing of the application for patent therefor. SECTION 74. PENALTY FOR FALSE MARKING. – Any person who falsely represents or indicates that any device, article or product made or sold by him is patented, or is the subject of a registered design, by making or having on the device, article or product, or on their containers or packages, or using in advertising or displays used in connection with them, or with any
What are not patentable?
SEC. 22. NON-‐PATENTABLE INVENTIONS. – The following shall be excluded from patent protection: 22.1. Discoveries, scientific theories and mathematical methods, and in the case of drugs and medicines, the mere discovery of a new form or new property of a known substance which does not result in the enhancement of the known efficacy of that substance, or the mere discovery of any new property or new use for a known substance, or the mere use of a known process unless such known process results in a new product that employs at least one new reactant. For the purpose of this clause, salts, esters, ethers, polymorphs, metabolites, pure form, particle size, isomers, mixtures of isomers, complexes, combinations, and other derivatives of a known substance shall be considered to be the same substance, unless they differ significantly in properties with regard to efficacy;
o 135
Katrina Michelle Mancao process, words expressing or implying that the device, article, product or process is patented or registered, shall be subject to a fine of not less than one hundred pesos nor more than one thousand pesos, or imprisonment for not less than one month nor more than one year, or both, in the discretion of the court. Actions hereunder shall prescribe in two years.
property which is possessed by no naturally occurring bacteria. A patent examiner's rejection of the patent application's claims for the new bacteria was affirmed by the Patent Office Board of Appeals on the ground that living things are not patentable subject matter under § 101. The Court of Customs and Patent Appeals reversed, concluding that the fact that micro-‐organisms are alive is without legal significance for purposes of the patent law. Court decision: A live, human-‐made micro-‐organism is patentable subject matter under § 101. Respondent's micro-‐ organism constitutes a "manufacture" or "composition of matter" within that statute. In choosing such expansive terms as "manufacture" and "composition of matter," modified by the comprehensive "any," Congress contemplated that the patent laws should be given wide scope, and the relevant legislative history also supports a broad construction. While laws of nature, physical phenomena, and abstract ideas are not patentable, respondent's claim is not to a hitherto unknown natural phenomenon, but to a non-‐naturally occurring manufacture or composition of matter -‐-‐ a product of human ingenuity "having a distinctive name, character and use." The passage of the 1930 Plant Patent Act, which afforded patent protection to certain asexually reproduced plants, and the 1970 Plant Variety Protection Act, which authorized protection for certain sexually reproduced plants but excluded bacteria from its protection, does not evidence congressional understanding that the terms "manufacture" or "composition of matter" in § 101 do not include living things. Nor does the fact that genetic technology was unforeseen when Congress enacted § 101 require the conclusion that micro-‐organisms cannot qualify as patentable subject matter until Congress expressly authorizes such protection. The unambiguous language of § 101 fairly embraces respondent's invention. Arguments against patentability under § 101, based on potential hazards that may be generated by genetic research, should be addressed to the Congress and the Executive, not to the Judiciary. Remarks: US Case
UNDER THE TRIPS AGREEMENT: ARTICLE 27, PATENTABLE SUBJECT MATTER. 1. Subject to the provisions of paragraphs 2 and 3, patents shall be available for any inventions, whether products or processes, in all fields of technology, provided that they are new, involve an inventive step and are capable of industrial application. (5) Subject to paragraph 4 of Article 65, paragraph 8 of Article 70 and paragraph 3 of this Article, patents shall be available and patent rights enjoyable without discrimination as to the place of invention, the field of technology and whether products are imported or locally produced. 2. Members may exclude from patentability inventions, the prevention within their territory of the commercial exploitation of which is necessary to protect ordre public or morality, including to protect human, animal or plant life or health or to avoid serious prejudice to the environment, provided that such exclusion is not made merely because the exploitation is prohibited by their law. 3. Members may also exclude from patentability: (a) diagnostic, therapeutic and surgical methods for the treatment of humans or animals; (b) plants and animals other than micro-‐ organisms, and essentially biological processes for the production of plants or animals other than non-‐biological and microbiological processes. However, Members shall provide for the protection of plant varieties either by patents or by an effective sui generis system or by any combination thereof. The provisions of this subparagraph shall be reviewed four years after the date of entry into force of the WTO Agreement.
RA 9168 – PHILIPPINE PLANT VARIETY PROTECTION ACT OF 2002
DIAMOND V. CHAKRABARTY, 447 US 303 (1980) Refresher: Title 35 U.S.C. § 101 provides for the issuance of a patent to a person who invents or discovers "any" new and useful "manufacture" or "composition of matter." Respondent filed a patent application relating to his invention of a human-‐made, genetically engineered bacterium capable of breaking down crude oil, a
SECTION 2. STATEMENT OF POLICIES. – a) The State recognizes that an effective intellectual property system in general and the development of new plant variety in particular is vital in attaining food security for the country.
136 z
Intellectual Property Law To this end, it shall protect and secure the exclusive rights of breeders with respect to their new plant variety particularly when beneficial to the people for such periods as provided for in this Act. b) The use of intellectual property bears a socioeconomic function. To this end, the State shall promote the diffusion of technology and information for the promotion of national development and progress for the common good. c) The State recognizes the indispensable role of the private sector, encourages the participation of private enterprises and provides incentives to needed investments in the development of new plant varieties. d) The State recognizes that science and technology are essential for national development and promotes the adaptation of technology and knowledge from all sources for the national benefit. The State also recognizes the need to protect and secure the exclusive rights of scientists and other gifted citizens to their intellectual property and creations. e) The State, while recognizing intellectual property rights in the field of agriculture, does so in a manner supportive of and not inconsistent with its obligation to maintain a healthful ecology in accord with the rhythm and harmony of nature.
(4) years or, in the case of vines or tress, more than six (6) years before the date of filing of an application for Plant Variety Protection.
However, the requirement of novelty provided for in this Act shall not apply to varieties sold, offered for sale or disposed of to others for a period of five (5) years before the approval of this Act. Provided, That application for PVP shall be filed within one (1) year from the approval of this act. SECTION 6. DISTINCTNESS. -‐ A variety shall be deemed distinct if it is clearly distinguishable from any commonly known variety. The filing of an application for the granting of a plant variety protection or for the entering of a new variety in an official register of variety in the Philippines or in any country, shall render the said variety a matter of public knowledge from the date of the said application: Provided, That the application leads to the granting of a Certificate of Plant Variety Protection or the entering of the said other variety in the official register of variety as the case may be. SECTION 7. UNIFORMITY. -‐ The variety shall be deemed uniform if, subject to the variation that may be expected from the particular features of its propagation, it is sufficiently uniform in its relevant characteristics. SECTION 8. STABILITY. -‐ The variety shall be deemed stable if its relevant characteristics remain unchanged after repeated propagation or, in the case of a particular cycle of propagation, at the end of each such cycle. SECTION 17. ENTITLEMENT. -‐ Any breeder, with respect to the variety developed, may apply for a plant variety developed, may apply for a plant variety protection and obtain a Certificate of Plant Variety Protection upon compliance with the requirements of this Act. SECTION 18. CO-‐OWNERSHIP OF THE RIGHT. -‐ If two (2) or more persons contribute to the development of a new plant variety, all of them shall be named in the Certificate of Plant Variety Protection and shall be entitled to such rights as agreed upon in writing or in the absence thereof, the rights in proportion to their contribution in the development of plant variety. SECTION 19. EMPLOYEE-‐EMPLOYER RELATIONSHIP. -‐ in case an employee develops a plant variety in the course of his employment as a result of the performance of his regular duty, the plant variety protection shall belong to the employer, unless there is a written stipulation to the contrary. SECTION 20. FIRST TO FILE RULE. -‐ If two (2) or more persons develop a new plant variety separately and independently of each other, the Certificate of Plant
SECTION 4. The Certificate of Plant Variety Protection shall be granted for varieties that are: a) New; b) Distinct; c) Uniform; and d) Stable. SECTION 5. NEWNESS. -‐ A variety shall be deemed new if the propagating or harvested material of the variety has not been sold, offered for sale or otherwise disposed of to others, by or with the consent of the breeder, for purposes of exploitation of the variety; a) In the Philippines for more than one (1) year before the date of filing of an application for plant variety protection; or b) In other countries or territories in which the application has been filed, for more than four
o 137
Katrina Michelle Mancao Variety Protection shall belong to the person who files the application first. In case two (2) or more persons file an application for the same plant variety, the right shall be granted to the person who has the earliest filing date or priority date. SECTION 21. PRIORITY DATE. -‐ Any application for a Certificate of Plant Variety Protection previously filed by a breeder in another country, which by treaty, convention or law affords similar privileges to Filipino citizens, shall be considered as filed locally as of the date of filing of the foreign application: Provided, That: a) The local application expressly claims priority; b) It is filed within twelve (12) months from the filing date of the earliest foreign application; and c) The applicant submits, within six (6) months from the filing of the local application, authenticated copies of documents which constitute the foreign application, samples or other evidence showing that the variety which is being applied for protection is the same variety which has been applied for protection in a foreign country. SECTION 22. FOREIGN NATIONALS. -‐ For purposes of this Act, a person shall be considered a national of a foreign country if he is a citizen of such country according to its laws, a natural person residing therein, or is a legal entity whose office is registered in such foreign country. SECTION 23. NATIONAL TREATMENT. -‐ Any application filed locally for a Certificate of Plant Variety Protection previously granted to a breeder in another country, which by treaty, convention or law affords similar privileges to Filipino citizens, shall be issued a Certificate of Plant Variety Protection upon payment of dues and compliance to all the provisions of this Act. This Act shall also apply to the nationals of foreign countries that are members of intergovernmental organizations or party to any multilateral agreement or convention concerning the granting of intellectual property protection to plant varieties. SECTION 33. TERM OF PROTECTION. -‐ For trees and vines, the period of protection shall be twenty-‐five (25) years from the date of the grant of the Certificate of Plant Variety Protection and twenty (20) years from the said date for all other types of plants, unless declared void ab initio or cancelled otherwise, as provided under Section 61 and 62, respectively of this Act.
SECTION 36. RIGHTS OF HOLDERS OF PLANT VARIETY PROTECTION. -‐ In respect of the propagating materials, holders of a Certificate of Plant Variety Protection shall have the right to authorize any of the following acts: a) Production or reproduction; b) Conditioning for the purpose of propagation; c) Offering for sale; d) Selling or other marketing; e) Exporting; f) Importing; and g) Stocking for any purpose mentioned above. SECTION 37. THE HOLDER MAY MAKE HIS AUTHORIZATION SUBJECT TO CONDITIONS AND L IMITATIONS. SECTION 47. WHAT CONSTITUTES INFRINGEMENT. -‐ Except as otherwise provided in this Act, any person who without being entitled to do so, performs the following acts: a) Sell the novel variety, or offer it or expose it for sale, deliver it, ship it, consign it, exchange it, or solicit an offer to buy it, or any other transfer of title or possession of it; or b) Import the novel variety into, or export it from, the Philippines; or c) Sexually multiply the novel variety as a step in marketing (for growing purposes) the variety; or d) Use the novel variety in producing (as distinguished from developing) a hybrid or different variety therefrom; or e) Use seed which had been marked "unauthorized propagation prohibited" or "unauthorized seed multiplication prohibited" or progeny thereof to propagate the novel variety; or f) Dispense the novel variety to another, in a form which can be propagated, without notice as to being a protected variety under which it was received; or g) Fails to use a variety denomination the use of which is obligatory under Section 15; or
138 z
Intellectual Property Law h) Perform any of the foregoing acts even in instances in which the novel variety is multiplied other than sexually, except in pursuance of a valid Philippine plant patent; or i) Instigate or actively induce performance of any foregoing acts, may be sued by the holder, who may also avail of all such relief as are available in any proceeding involving infringements of other proprietary rights.
taken without their free and prior informed consent or in violation of their laws, traditions and customs. SEC. 34. RIGHT TO INDIGENOUS KNOWLEDGE SYSTEMS AND PRACTICES AND TO D EVELOP OWN SCIENCES AND TECHNOLOGIES.-‐ ICCs/IPs are entitled to the recognition of the full ownership and control and protection of their cultural and intellectual rights. They shall have the right to special measures to control, develop and protect their sciences, technologies and cultural manifestations, including human and other genetic resources, seeds, including derivatives of these resources, traditional medicines and health practices, vital medicinal plants, animals and minerals, indigenous knowledge systems and practices, knowledge of the properties of fauna and flora, oral traditions, literature, designs, and visual and performing arts.
SECTION 48. WHERE TO COMMENCE ACTION. -‐ Any holder may petition the proper regional trial court for infringement of his plant variety protection as defined in this Act. SECTION 49. PRESUMPTION OF VALIDITY. -‐ Certificate of Plant Variety Protection shall be presumed valid and the burden of proof of their invalidity shall rest on the party assailing them. SECTION 50. DEFENSES AGAINST INFRINGEMENT CHARGES. -‐ The following shall be valid defenses against infringement charges: a) Non-‐infringement; b) The plant variety does not possess at the time of its application criterion of novelty or distinctness; c) The alleged infringement was performed under a right adverse to it, prior to the notice of infringement; and/or d) Other defenses that are made available under this Act. SECTION 55. PRESCRIPTION. -‐ No recovery of damages for any infringement case shall prosper when the cause of action has reached more than six (6) years from the time the alleged infringement case was committed. SECTION 56. CRIMINAL PENALTY. -‐ Any person who violates any of the rights of the holder provided for in this Act may also suffer the penalty of imprisonment of not less than three (3) years but not more than six (6) years and/or a fine of up to three (3) times the profit derived by virtue of the infringement but in no case should be less than One Hundred Thousand pesos (P100,000.00).
F.
Ownership of patent
SECTION 28. RIGHT TO A PATENT. – The right to a patent belongs to the inventor, his heirs, or assigns. When two (2) or more persons have jointly made an invention, the right to a patent shall belong to them jointly.
SECTION 29. FIRST TO FILE RULE. – If two (2) or more persons have made the invention separately and independently of each other, the right to the patent shall belong to the person who filed an application for such invention, or where two or more applications are filed for the same invention, to the applicant who has the earliest filing date or, the earliest priority date.
SECTION 30. INVENTIONS CREATED PURSUANT TO A COMMISSION. – 30.1. The person who commissions the work shall own the patent, unless otherwise provided in the contract. 30.2. In case the employee made the invention in the course of his employment contract, the patent shall belong to: (a) The employee, if the inventive activity is not a part of his regular duties even if the employee uses the time, facilities and materials of the employer. (b) The employer, if the invention is the result of the performance of his regularly-‐assigned duties, unless there is an agreement, express or implied, to the contrary.
RA 8371 – INDIGENOUS PEOPLES RIGHTS ACT OF 1997 SEC. 32. COMMUNITY INTELLECTUAL RIGHTS.-‐ ICCs/IPs have the right to practice and revitalize their own cultural traditions and customs. The State shall preserve, protect and develop the past, present and future manifestations of their cultures as well as the right to the restitution of cultural, intellectual, religious, and spiritual property
o 139
Katrina Michelle Mancao SECTION 67. PATENT APPLICATION BY PERSONS NOT HAVING THE RIGHT TO A PATENT. . –
G. Regular application for patent
1.
67.1. If a person referred to in Section 29 other than the applicant, is declared by final court order or decision as having the right to the patent, such person may, within three (3) months after the decision has become final: (a) Prosecute the application as his own application in place of the applicant; (b) File a new patent application in respect of the same invention; (c) Request that the application be refused; or (d) Seek cancellation of the patent, if one has already been issued. 67.2. The provisions of Subsection 38.2 shall apply mutatis mutandis to a new application filed under Subsection 67. 1(b).
Who may apply?
SECTION 28. RIGHT TO A PATENT. – The right to a patent belongs to the inventor, his heirs, or assigns. When two (2) or more persons have jointly made an invention, the right to a patent shall belong to them jointly.
SECTION 29. FIRST TO FILE RULE. – If two (2) or more persons have made the invention separately and independently of each other, the right to the patent shall belong to the person who filed an application for such invention, or where two or more applications are filed for the same invention, to the applicant who has the earliest filing date or, the earliest priority date.
SECTION 30. INVENTIONS CREATED PURSUANT TO A COMMISSION. – 30.1. The person who commissions the work shall own the patent, unless otherwise provided in the contract. 30.2. In case the employee made the invention in the course of his employment contract, the patent shall belong to: (a) The employee, if the inventive activity is not a part of his regular duties even if the employee uses the time, facilities and materials of the employer. (b) The employer, if the invention is the result of the performance of his regularly-‐assigned duties, unless there is an agreement, express or implied, to the contrary.
SECTION 68. REMEDIES OF THE TRUE AND ACTUAL INVENTOR. – If a person, who was deprived of the patent without his consent or through fraud is declared by final court order or decision to be the true and actual inventor, the court shall order for his substitution as patentee, or at the option of the true inventor, cancel the patent, and award actual and other damages in his favor if warranted by the circumstances.
SECTION 69. PUBLICATION OF THE COURT ORDER. – The court shall furnish the Office a copy of the order or decision referred to in Sections 67 and 68, which shall be published in the IPO Gazette within three (3) months from the date such order or decision became final and executory, and shall be recorded in the register of the Office.
SECTION 68. REMEDIES OF THE TRUE AND ACTUAL INVENTOR. –
SECTION 70. TIME TO FILE ACTION IN COURT. –
If a person, who was deprived of the patent without his consent or through fraud is declared by final court order or decision to be the true and actual inventor, the court shall order for his substitution as patentee, or at the option of the true inventor, cancel the patent, and award actual and other damages in his favor if warranted by the circumstances.
The actions indicated in Sections 67 and 68 shall be filed within one (1) year from the date of publication made in accordance with Sections 44 and 51, respectively.
SECTION 236. PRESERVATION OF EXISTING RIGHTS. – Nothing herein shall adversely affect the rights on the enforcement of rights in patents, utility models, industrial designs, marks and works, acquired in good faith prior to the effective date of this Act.
SECTION 3. INTERNATIONAL RECIPROCITY. –
CONVENTIONS
AND
Any person who is a national or who is domiciled or has a real and effective industrial establishment in a country which is a party to any convention, treaty or agreement relating to intellectual property rights or the repression of unfair competition, to which the Philippines is also a
140 z
Intellectual Property Law party, or extends reciprocal rights to nationals of the Philippines by law, shall be entitled to benefits to the extent necessary to give effect to any provision of such convention, treaty or reciprocal law, in addition to the rights to which any owner of an intellectual property right is otherwise entitled by this Act.
(c) Drawings necessary for the understanding of the invention; (d) One or more claims; and (e) An abstract. 32.2. No patent may be granted unless the application identifies the inventor. If the applicant is not the inventor, the Office may require him to submit said authority.
SECTION 231. REVERSE RECIPROCITY OF FOREIGN LAWS. – Any condition, restriction, limitation, diminution, requirement, penalty or any similar burden imposed by the law of a foreign country on a Philippine national seeking protection of intellectual property rights in that country, shall reciprocally be enforceable upon nationals of said country, within Philippine jurisdiction.
SECTION 33. APPOINTMENT OF AGENT OR REPRESENTATIVE. – An applicant who is not a resident of the Philippines must appoint and maintain a resident agent or representative in the Philippines upon whom notice or process for judicial or administrative procedure relating to the application for patent or the patent may be served.
SECTION 235. APPLICATIONS PENDING ON EFFECTIVE DATE OF ACT. – 235.1. All applications for patents pending in the Bureau of Patents, Trademarks and Technology Transfer shall be proceeded with and patents thereon granted in accordance with the Acts under which said applications were filed, and said Acts are hereby continued to be enforced, to this extent and for this purpose only, notwithstanding the foregoing general repeal thereof: Provided, That applications for utility models or industrial designs pending at the effective date of this Act, shall be proceeded with in accordance with the provisions of this Act, unless the applicants elect to prosecute said applications in accordance with the Acts under which they were filed. 235.2. All applications for registration of marks or trade names pending in the Bureau of Patents, Trademarks and Technology Transfer at the effective date of this Act may be amended, if practicable to bring them under the provisions of this Act. The prosecution of such applications so amended and the grant of registrations thereon shall be proceeded with in accordance with the provisions of this Act. If such amendments are not made, the prosecution of said applications shall be proceeded with and registrations thereon granted in accordance with the Acts under which said applications were filed, and said Acts are hereby continued in force to this extent for this purpose only, notwithstanding the foregoing general repeal thereof.
2.
SECTION 34. THE REQUEST. – The request shall contain a petition for the grant of the patent, the name and other data of the applicant, the inventor and the agent and the title of the invention.
SECTION 35. DISCLOSURE AND DESCRIPTION OF THE INVENTION. – 35.1. D ISCLOSURE. -‐ The application shall disclose the invention in a manner sufficiently clear and complete for it to be carried out by a person skilled in the art. Where the application concerns a microbiological process or the product thereof and involves the use of a micro-‐ organism which cannot be sufficiently disclosed in the application in such a way as to enable the invention to be carried out by a person skilled in the art, and such material is not available to the public, the application shall be supplemented by a deposit of such material with an international depository institution. 35.2. DESCRIPTION. -‐ The Regulations shall prescribe the contents of the description and the order of presentation.
SECTION 36. THE CLAIMS. –
Application
36.1. The application shall contain one (1) or more claims which shall define the matter for which protection is sought. Each claim shall be clear and concise, and shall be supported by the description. 36.2. The Regulations shall prescribe the manner of the presentation of claims.
SECTION 32. THE APPLICATION. – 32.1. The patent application shall be in Filipino or English and shall contain the following: (a) A request for the grant of a patent; (b) A description of the invention;
SECTION 37. THE ABSTRACT. – The abstract shall consist of a concise summary of the disclosure of the invention as contained in the
o 141
Katrina Michelle Mancao description, claims and drawings in preferably not more than one hundred fifty (150) words. It must be drafted in a way which allows the clear understanding of the technical problem, the gist of the solution of that problem through the invention, and the principal use or uses of the invention. The abstract shall merely serve for technical information.
SECTION 108. APPLICABILITY OF PROVISIONS RELATING TO PATENTS. – 108.1. Subject to Section 109, the provisions governing patents shall apply, mutatis mutandis, to the registration of utility models. 108.2. Where the right to a patent conflicts with the right to a utility model registration in the case referred to in Section 29, the said provision shall apply as if the word "patent" were replaced by the words "patent or utility model registration".
SECTION 38. UNITY OF INVENTION. – 38.1. The application shall relate to one invention only or to a group of inventions forming a single general inventive concept. 38.2. If several independent inventions which do not form a single general inventive concept are claimed in one application, the Director may require that the application be restricted to a single invention. A later application filed for an invention divided out shall be considered as having been filed on the same day as the first application: Provided, That the later application is filed within four (4) months after the requirement to divide becomes final or within such additional time, not exceeding four (4) months, as may be granted: Provided further, That each divisional application shall not go beyond the disclosure in the initial application. 38.3. The fact that a patent has been granted on an application that did not comply with the requirement of unity of invention shall not be a ground to cancel the patent.
SECTION 109. SPECIAL PROVISIONS RELATING TO UTILITY MODELS. -‐ 109.2. Sections 43 to 49 shall not apply in the case of applications for registration of a utility model.
Industrial Design SECTION 114. CONTENTS OF THE APPLICATION. – 114.1. Every application for registration of an industrial design shall contain: (a) A request for registration of the industrial design; (b) Information identifying the applicant; (c) An indication of the kind of article of manufacture or handicraft to which the design shall be applied; (d) A representation of the article of manufacture or handicraft by way of drawings, photographs or other adequate graphic representation of the design as applied to the article of manufacture or handicraft which clearly and fully discloses those features for which design protection is claimed; and (e) The name and address of the creator, or where the applicant is not the creator, a statement indicating the origin of the right to the industrial design registration. 114.2. The application may be accompanied by a specimen of the article embodying the industrial design and shall be subject to the payment of the prescribed fee.
SECTION 39. INFORMATION CONCERNING CORRESPONDING FOREIGN APPLICATION FOR PATENTS. – The applicant shall, at the request of the Director, furnish him with the date and number of any application for a patent filed by him abroad, hereafter referred to as the "foreign application," relating to the same or essentially the same invention as that claimed in the application filed with the Office and other documents relating to the foreign application.
SECTION 8. THE BUREAU OF PATENTS. – The Bureau of Patents shall have the following functions: 8.1. Search and examination of patent applications and the grant of patents; 8.2. Registration of utility models, industrial designs, and integrated circuits; and 8.3. Conduct studies and researches in the field of patents in order to assist the Director General in formulating policies on the administration and examination of patents.
SECTION 115. SEVERAL INDUSTRIAL DESIGNS IN ONE APPLICATION. – Two (2) or more industrial designs may be the subject of the same application: Provided, That they relate to the same sub-‐class of the International Classification or to the same set or composition of articles.
142 z
Intellectual Property Law 3.
are received. If the deficiencies are not remedied within the prescribed time limit, the application shall be considered withdrawn.
Priority date
SECTION 31. RIGHT OF PRIORITY. . – An application for patent filed by any person who has previously applied for the same invention in another country which by treaty, convention, or law affords similar privileges to Filipino citizens, shall be considered as filed as of the date of filing the foreign application: Provided, That: (a) the local application expressly claims priority; (b) it is filed within twelve (12) months from the date the earliest foreign application was filed; and (c) a certified copy of the foreign application together with an English translation is filed within six (6) months from the date of filing in the Philippines.
SECTION 116. EXAMINATION. – 116.1. The Office shall accord as the filing date the date of receipt of the application containing indications allowing the identity of the applicant to be established and a representation of the article embodying the industrial design or a pictorial representation thereof. 116.2. If the application does not meet these requirements the filing date should be that date when all the elements specified in Section 105 are filed or the mistakes corrected. Otherwise if the requirements are not complied within the prescribed period, the application shall be considered withdrawn. 116.3. After the application has been accorded a filing date and the required fees paid on time, the applicant shall comply with the requirements of Section 114 within the prescribed period, otherwise the application shall be considered withdrawn. 116.4. The Office shall examine whether the industrial design complies with requirements of Section 112 and Subsections 113.2 and 113.3.
BOOTHE V. DIRECTOR OF PATENTS (1980) Court decision: Under the Revised Rules of Practice in Patent Cases, it is imperative that the application be complete in order that it may be accepted. It is essential to the validity of Letters Patent that the specifications be full, definite, and specific. The purpose of requiring a definite and accurate description of the process is to apprise the public of what the patentee claims as his invention, to inform the Courts as to what they are called upon to construe, and to convey to competing manufacturers and dealers information of exactly what they are bound to avoid.
4.
Filing date
5.
Formality examination
SECTION 40. FILING DATE REQUIREMENTS. –
SECTION 42. FORMALITY EXAMINATION. –
40.1. The filing date of a patent application shall be the date of receipt by the Office of at least the following elements: (a) An express or implicit indication that a Philippine patent is sought; (b) Information identifying the applicant; and (c) Description of the invention and one (1) or more claims in Filipino or English. 40.2. If any of these elements is not submitted within the period set by the Regulations, the application shall be considered withdrawn.
42.1. After the patent application has been accorded a filing date and the required fees have been paid on time in accordance with the Regulations, the applicant shall comply with the formal requirements specified by Section 32 and the Regulations within the prescribed period, otherwise the application shall be considered withdrawn. 42.2. The Regulations shall determine the procedure for the re-‐examination and revival of an application as well as the appeal to the Director of Patents from any final action by the examiner.
6.
Classification and search
SECTION 43. CLASSIFICATION AND SEARCH. –
SECTION 41. ACCORDING A FILING DATE. –
An application that has complied with the formal requirements shall be classified and a search conducted to determine the prior art.
The Office shall examine whether the patent application satisfies the requirements for the grant of date of filing as provided in Section 40 hereof. If the date of filing cannot be accorded, the applicant shall be given an opportunity to correct the deficiencies in accordance with the implementing Regulations. If the application does not contain all the elements indicated in Section 40, the filing date should be that date when all the elements
o 143
Katrina Michelle Mancao 7.
Confidentiality publication
before
10. Observation parties
by
third
SECTION 45. CONFIDENTIALITY BEFORE PUBLICATION. –
SECTION 47. OBSERVATION BY THIRD PARTIES. –
A patent application, which has not yet been published, and all related documents, shall not be made available for inspection without the consent of the applicant.
Following the publication of the patent application, any person may present observations in writing concerning the patentability of the invention. Such observations shall be communicated to the applicant who may comment on them. The Office shall acknowledge and put such observations and comment in the file of the application to which it relates.
8.
Publication application
of
patent
SECTION 44. PUBLICATION OF PATENT APPLICATION. –
11. Request for substantive examination
44.1. The patent application shall be published in the IPO Gazette together with a search document established by or on behalf of the Office citing any documents that reflect prior art, after the expiration of eighteen (18) months from the filing date or priority date. 44.2. After publication of a patent application, any interested party may inspect the application documents filed with the Office. 44.3. The Director General subject to the approval of the Secretary of Trade and Industry, may prohibit or restrict the publication of an application, if in his opinion, to do so would be prejudicial to the national security and interests of the Republic of the Philippines.
9.
SECTION 48. REQUEST FOR SUBSTANTIVE EXAMINATION. – 48.1. The application shall be deemed withdrawn unless within six (6) months from the date of publication under Section 41, a written request to determine whether a patent application meets the requirements of Sections 21 to 27 and Sections 32 to 39 and the fees have been paid on time. 48.2. Withdrawal of the request for examination shall be irrevocable and shall not authorize the refund of any fee.
12. Amendment
Rights conferred by a patent application
SECTION 49. AMENDMENT OF APPLICATION. – An applicant may amend the patent application during examination: Provided, That such amendment shall not include new matter outside the scope of the disclosure contained in the application as filed.
SECTION 46. RIGHTS CONFERRED BY A PATENT APPLICATION AFTER PUBLICATION. – The applicant shall have all the rights of a patentee under Section 76 against any person who, without his authorization, exercised any of the rights conferred under Section 71 of this Act in relation to the invention claimed in the published patent application, as if a patent had been granted for that invention: Provided, That the said person had: 46.1. Actual knowledge that the invention that he was using was the subject matter of a published application; or 46.2. Received written notice that the invention that he was using was the subject matter of a published application being identified in the said notice by its serial number: Provided, That the action may not be filed until after the grant of a patent on the published application and within four (4) years from the commission of the acts complained of.
13. Conversion
SECTION 110. CONVERSION OF PATENT APPLICATIONS OR APPLICATIONS FOR UTILITY MODEL REGISTRATION. – 110.1. At any time before the grant or refusal of a patent, an applicant for a patent may, upon payment of the prescribed fee, convert his application into an application for registration of a utility model, which shall be accorded the filing date of the initial application. An application may be converted only once. 110.2. At any time before the grant or refusal of a utility model registration, an applicant for a utility model registration may, upon payment of the prescribed fee, convert his application into a patent application, which shall be accorded the filing date of the initial application.
144 z
Intellectual Property Law Property Code of the Philippines, the following rules on PCT applications are hereby promulgated: PART I. GENERAL PROVISIONS RULE 1. TITLE. -‐ These rules shall be known as the Philippine Rules on PCT Applications or “PRo-‐PCT”. RULE 2. D EFINITIONS. -‐ Unless otherwise specified, the following terms and acronyms shall be understood as follows: (a) “Administrative Instructions” means that body of instructions for operating under the Patent Cooperation Treaty referred to in PCT Rule 89; (b) “designated Office” means a national patent office or intergovernmental organization of or acting for the State designated by the applicant under Chapter I of the Treaty; (c) “designated State” refers to the State in which protection for the invention is desired on the basis of the international application and which is designated under Chapter I of the Treaty; (d) “Director General” means the Head of the Intellectual Property Office of the Philippines; (e) “elected Office” means a national patent office or intergovernmental organization of or acting for the State elected by the applicant under Chapter II of the Treaty; (f) “elected State” refers to the State in which protection for the invention is desired on the basis of the international application and which has been elected under Chapter II of the Treaty; (g) “IB” and “International Bureau” mean the International Bureau of the World Intellectual Property Organization as referred to in PCT Article 2(xix); (h) “International Application” and “PCT Application” mean an application filed under the Treaty; (i) “International Searching Authority” or “ISA” and “International Preliminary Examining Authority” or “IPEA” mean a national patent office or intergovernmental organization as appointed under the Treaty which processes
SECTION 111. PROHIBITION AGAINST FILING OF PARALLEL APPLICATIONS. – An applicant may not file two (2) applications for the same subject, one for utility model registration and the other for the grant of a patent whether simultaneously or consecutively.
H. Philippine Rules on PCT Applications WHEREAS, the State recognizes that an effective industrial property system is vital to the development of domestic creativity, facilitates transfer of technology, attracts foreign investments and ensures market access for our products; WHEREAS, it is the policy of the State to streamline administrative procedures in granting patents and enhance the enforcement of intellectual property rights in the Philippines; WHEREAS, Senate Resolution No. 74 dated 05 February 2001 was passed whereby the Philippines ratified the Patent Cooperation Treaty (PCT); WHEREAS, since 1978, the PCT system has offered investors and industry an advantageous route for obtaining patent protection internationally; aims to facilitate procedures for obtaining legal protection for inventions and disseminate technical information; seeks to simplify and to render more effective and more economical – in the interest of the applicants and the patent offices – previously established means of applying in several countries for patent protection for invention; would substantially benefit Philippine patent agents and industry from the filing in the Philippines of international applications under the PCT; WHEREAS, in accordance with Article 63 of the PCT, the Treaty shall enter into force with respect to the Philippines, three (3) months after the date on which it has deposited its Instrument of Accession with the Director General of the World Intellectual Property Organization (WIPO); WHEREAS, the Philippines deposited its Instrument of Accession with the Director General of the WIPO on 17 May 2001; and, therefore, the PCT entered into effect with respect to the Philippines on 17 August 2001; WHEREAS, there is a need to guide applicants, patent attorneys/agents, and other interested parties in the Philippines on the PCT and its rules and regulations; NOW, THEREFORE, pursuant to Senate Resolution No. 74 dated 05 February 2001, the provisions of the Patent Cooperation Treaty and the Regulations thereunder, Republic Act No. 8293 also known as the Intellectual
o 145
Katrina Michelle Mancao international applications as prescribed by the Treaty and PCT Regulations; (j) “IP Code” means Republic Act No. 8293 otherwise known as the Intellectual Property Code of the Philippines; (k) “IPO” or “Office” means the Intellectual Property Office of the Philippines; (l) “IPO Fee Structure” means the rules and regulations establishing the fees and charges of the Intellectual Property Office of the Philippines; (m) “PCT” and “Treaty” mean the Patent Cooperation Treaty; (n) “PCT-‐EASY” means the software made available by the International Bureau for facilitating the preparation of the request part of an international application, and printing thereof, for filing together with a computer diskette prepared using that software, containing a copy in electronic form of the data contained in the request and of the abstract; (o) “PCT Regulations” means the set of rules, referred to in PCT Article 58 and annexed to the Treaty referred to as the Regulations Under the PCT; (p) “Priority Date” for the purposes of computing time limits under the PCT is defined in PCT Article 2(xi) as follows: (i) where the international application contains a priority claim under PCT Article 8, the filing date of the application whose priority is so claimed; (ii) where the international application contains several priority claims under PCT Article 8, the filing date of the earliest application whose priority is so claimed; (iii) where the international application does not contain any priority claim under PCT Article 8, the international filing date of such application; (q) “Receiving Office” means a national patent office or intergovernmental organization with which the international application has been filed;
Other terms and expressions not defined in this rule are to be taken in the sense of the Treaty and PCT Regulations. RULE 3. APPLICATION OF THE PCT AND PCT REGULATIONS. – In all matters not specifically provided for under these Rules, the provisions of the Treaty, PCT Regulations and PCT Administrative Instructionsshall apply in the processing of an international application during the international phase of the application. In the event of conflict between these Rules and those of the PCT, the provisions of the latter shall apply. The international phase covers the period from the filing of the application in accordance with the PCT until the application enters the national phase. RULE 4. EFFECT OF AN INTERNATIONAL APPLICATION. – 4.1. An international application designating the Philippines for the purposes of obtaining a national patent or a utility model, which has been accorded an international filing date in accordance with the Treaty and PCT regulations, shall have the effect of a national application for a patent or utility model, respectively, regularly filed with the Intellectual Property Office as of the international filing date, which date shall be considered to be the actual filing date in the Philippines. 4.2. Subject to Review by the IPO under Rule 37, an international application designating the Philippines that is withdrawn or considered withdrawn under the Treaty and PCT Regulations, before the applicant has entered the national phase in the Philippines as provided in Part 3 of these Rules, that international application is likewise withdrawn or considered withdrawn in the Philippines. RULE 5. SEEKING TWO (2) KINDS OF PROTECTION; PROHIBITION AGAINST FILING OF PARALLEL APPLICATIONS. -‐ In respect of the Philippines as a designated or elected State, an applicant cannot seek two (2) kinds of protection for the same subject, one for utility model registration and the other for the grant of a patent whether simultaneously or consecutively. RULE 6. DELIVERY SERVICE. – Where the applicant sends a document or letter through mail and that document or letter is lost or reaches the IPO after the expiration of the applicable time limit under the Treaty and PCT Regulations and these Rules, PCT Rules 82.1(a) to (c) shall apply provided that the delivery service used is registered mail by a postal authority. RULE 7. APPOINTMENT OF RESIDENT AGENT OR REPRESENTATIVE. – (a) An applicant who is not a resident of the Philippines shall appoint and maintain an agent or representative residing in the Philippines upon whom notices or processes for judicial or administrative procedure may
146 z
Intellectual Property Law be served relating to the international application filed with the IPO as a receiving, designated or elected Office. An applicant who is a resident of the Philippines may likewise appoint and maintain an agent or representative residing in the Philippines for the same purpose. The list of registered resident agents or representatives is available in the IPO website. (b) Where the international application is filed with the IPO as a receiving Office, the appointment of an agent must be effected in the Request form, signed by all applicants, or in a separate power of attorney submitted to the IPO where the Request form is signed by the appointed agent in accordance with PCT Rule 90.4. The separate power of attorney does not require legalization or notarization. (c) For purposes of the procedure before the IPO as a designated or elected Office, the manner of appointing an agent or representative shall comply with the requirements prescribed in the implementing rules and regulations of the IP Code. PART II. INTERNATIONAL PHASE RULE 8. THE IPO AS A RECEIVING OFFICE. – 8.1. The IPO shall act as a Receiving Office only for applicants who are residents or nationals of the Philippines. 8.2. The IPO, when acting as a Receiving Office, will be identified by the full title “Philippine Receiving Office” or by the abbreviation “RO/PH.” 8.3. Without prejudice to the national security and interests of the Republic of the Philippines, the IPO shall perform all acts connected with the discharge of duties of a Receiving Office under the Treaty and PCT Regulations. The functions of the IPO as a Receiving Office include: (1) According of international filing dates to international applications fulfilling the requirements of PCT Article 11(1) and in particular, PCT Rule 20; (2) Checking that international applications meet the standards for format and content of PCT Article 14(1) and in particular, PCT Rules 9, 26, 29.1, 37, 38, 91, and portions of PCT Rules 3 through 11; (3) Assessing, collecting, and transmitting fees due for processing international applications; (4) Transmitting the record and search copies to the IB and ISA, respectively; and
(5) Transmitting the international application to the IB for processing, where the IPO is not the competent Receiving Office as defined under PCT Rule 19. RULE 9. INTERNATIONAL SEARCH AND INTERNATIONAL SEARCHING AUTHORITY. – 9.1. An international application shall be the subject of an international search, which shall be carried out by an ISA. The objective of the international search is to discover relevant prior art. 9.2. The Director General shall specify one or more competent ISAs for international applications filed with the IPO and, subject to any required acceptance or compliance with applicable requirements, the IPO shall publish in its website a list of those competent ISAs. 9.3. The applicant shall, in his Request Form, indicate his choice of the competent ISA. RULE 10. INTERNATIONAL PRELIMINARY EXAMINATION AND INTERNATIONAL PRELIMINARY EXAMINING AUTHORITY. – 10.1. An international application may be the subject of an international preliminary examination which shall be carried out by an IPEA. 10.2. The objective of the international preliminary examination is to formulate a preliminary and non-‐ binding opinion on the questions whether the claimed invention appears to be novel, to involve an inventive step (to be non-‐obvious) and to be industrially applicable. 10.3. The Director General shall specify one or more competent IPEAs for international applications filed with the IPO and, subject to any required acceptance or compliance with applicable requirements, the IPO shall publish in its website a list of those competent IPEAs. 10.4 Where there are two or more competent IPEAs for applications filed with the IPO, the applicant shall, in his Demand Form, indicate his choice of the competent IPEA. RULE 11. LANGUAGE PRESCRIBED BY THE RECEIVING O FFICE. – The language accepted by the Philippines receiving Office for the filing of international application is Filipino or English, provided that it is sufficient that the description and claims be in Filipino or English for the purposes of according an international filing date under PCT Article 11(1). RULE12. TRANSLATION FOR THE PURPOSES OF INTERNATIONAL SEARCH. – (a) Where the international application is filed in Filipino, the applicant shall, within one (1) month from the date of receipt of the international application by the IPO,
o 147
Katrina Michelle Mancao furnish to the Office a translation of the international application into English in accordance with PCT Rule 12.3 (a) and (b). (b) If the Engish translation is not furnished within the time limit under paragraph (a), the Office shall invite the applicant to furnish it within one (1) month from the date of invitation or two (2) months from the date of receipt of the international application by the IPO, whichever expires later. (c) If the applicant does not furnish the English translation within the applicable time limit under paragraph (b), the international application shall be considered withdrawn and the IPO as a receiving Office shall so declare in accordance with PCT Rule 12.3(d). (d) The IPO shall promptly furnish the IB and the ISA a copy of the English translation. RULE 13. D OCUMENTS AND CORRESPONDENCE. – (a) Any paper submitted by the applicant, other than the international application itself, must, if not itself in the form of a letter, be accompanied by a letter identifying the international application to which it relates and must be signed by the applicant. (b) Correspondence intended for the applicant shall be sent to the agent appointed under Rule 7. RULE 14. KEEPING OF RECORDS AND FILES. -‐ The IPO shall keep the records relating to the international application or purported international application for at least (ten) 10 years from the international filing date or, where no international filing date is accorded, from the date of receipt of the purported application. RULE 15. THE INTERNATIONAL APPLICATION. -‐ An international application shall contain a request, a description, one or more claims, one or more drawings (where required), and an abstract. RULE 16. THE REQUEST. -‐ The request shall be made in accordance with PCT Rules 3 and 4. Copies of the Request form are available at the IPO. The Request Form may be made using PCT-‐EASY software made available by the IB, in which case, the applicant is required to submit the Request as a computer print-‐out together with a computer diskette containing a copy of the electronic form of the data contained in the Request and of the abstract prepared using that software. RULE 17. THE DESCRIPTION. – The description shall disclose the invention in a manner sufficiently clear and complete
for the invention to be carried out by a person skilled in the art. The requirements as to the format and content of the description are set forth, in particular, in PCT Rules 5, 9, 10, and 11, and Section 204 of the PCT Administrative Instructions. RULE 18. THE CLAIMS. -‐ The claim or claims shall define the matter for which protection is sought. Claims shall be clear and concise. They shall be fully supported by the description. The requirements as to the format and content of claims are set forth, in particular, in PCT Rules 6, 9, 10, and 11. RULE 19. THE D RAWINGS. -‐ Drawings which are necessary for the understanding of the invention must be part of the international application as originally filed in order to maintain the international filing date during the national stage. If drawings are referred to in the international application but they are missing upon filing, the IPO shall give the applicant thirty (30) days from the date of receipt of the international application to submit the drawings. If the missing drawings are received within the thirty-‐day period, the international filing date shall be the date on which such drawings are received. If such drawings are not timely received, all references to drawings in the international application shall be considered non-‐existent. The physical requirements for drawings are set forth in PCT Rule 11.3. RULE 20. THE ABSTRACT. -‐ The abstract shall merely serve for technical information. Lack of an abstract upon filing of an international application will not affect the granting of a filing date. If the abstract is missing, the IPO shall invite the applicant to submit the same within one (1) month from the date of the invitation and the IPO shall notify the ISA accordingly. The requirements as to the format and content of the abstract are set forth in PCT Rule 8. RULE 21. PRIORITY CLAIM. – Any declaration referred to in Article 8(1) (“priority claim”) shall be indicated in the Request Form as set forth in PCT Rule 4.10. Any correction or addition on the priority claim shall be made in accordance with PCT Rule 26bis. RULE 22. PHYSICAL REQUIREMENTS. – 22.1. An international application and each of the documents that may be referred to in the checklist of the Request Form shall be filed in one (1) copy only.
148 z
Intellectual Property Law 22.2. All sheets of the international application must be on A4 size paper (29.7 cm x 21 cm) which shall be flexible, strong, white, smooth, non-‐shiny, and durable. 22.3. In order to facilitate processing of the international applications, attention of applicants is directed to the other physical requirements for international application which are set forth in PCT Rule 11 and Section 207 of the PCT Administrative Instructions. RULE 23. WHO MAY FILE AN INTERNATIONAL APPLICATION. – A resident or national of the Philippines may file an international application. If there are two or more applicants, the right to file an international application shall exist if at least one of them is a resident or national of the Philippines. RULE 24. WHERE TO FILE AN INTERNATIONAL APPLICATION. – 24.1. A resident or national of the Philippines may file an international application with the IPO or the IB. 24.2. An international application may be filed directly with the IPO or through registered mail by postal authority. RULE 25. ACCORDING OF INTERNATIONAL FILING D ATE AND ITS EFFECT. – 25.1. The IPO, as the competent receiving Office, shall accord as the international filing date the date of receipt of the international application, provided that, at the time of receipt: (1) at least one applicant is a resident or national of the Philippines;chan robles virtual law library (2) the international application is in the Filipino or English language; andchan robles virtual law library (3) the international application contains at least the following elements:chan robles virtual law library (i) an indication that it is intended as an international application; (ii) the designation of at least one Contracting State; (iii) the name of the applicant; (iv) a part which on the face of it appears to be a description; and (v) a part which on the face of it appears to be a claim or claims.
25.2. If the IPO finds that the applicant or not one of the applicants is a resident or national of the Philippines or that the language is neither Filipino nor English, the international application shall be treated and transmitted to the IB in accordance with PCT Rule 19.4, subject to payment of the transmittal fee under PCT Rule 14. 25.3. If the IPO finds that the international application did not, at the time of receipt, contain the elements listed in Rule 25.1 item (3), it shall invite the applicant to file the required correction within thirty (30) days from the date of the invitation. If the applicant complies with the invitation, the IPO shall accord as the international filing date the date of receipt of the required correction. If the applicant does not comply with the invitation, the IPO shall promptly in accordance with PCT Rule 20.7(i) notify the applicant that his application is not and will not be treated as an international application and shall indicate the reasons therefor. RULE 26. TRANSMITTAL OF THE INTERNATIONAL APPLICATION. – 26.1. The IPO shall transmit the “record copy” of the international application to the IB in accordance with PCT Rule 22. 26.2. The IPO shall transmit the “search copy” of the international application to the competent ISA after payment of the search fee by the applicant in accordance with PCT Rule 23. 26.3. The Director General may prohibit or restrict the transmittal of an application, if in his opinion, to do so would be prejudicial to the national security and interests of the Republic of the Philippines. The Director General may apply other measures, as provided under PCT Article 27(8), that will not prevent the transmittal of the record copy and search copy but will restrict the contents of the transmittals. RULE 27. THE INTERNATIONAL PUBLICATION AND ITS EFFECT. – 27.1. In accordance with PCT Article 21 and PCT Rule 48, the IB shall publish the international application promptly after the expiration of eighteen (18) months from the priority date of the application. However, the applicant may ask the IB to publish his international application any time before the expiration of eighteen (18) months from the priority date in accordance with PCT Rule 48.4. 27.2. In accordance with PCT Article 29(1), as far as the protection of any rights of the applicant under Section 46 of the IP Code in the Philippines is concerned, the international publication in English of an international application shall have the same effect as a publication in the IPO Gazette under Section 44 of the IP Code and its implementing rules and regulations provided that notice of the international publication and copy of the
o 149
Katrina Michelle Mancao international application has been transmitted by the applicant to the actual unauthorized user of the invention claimed in the international application pursuant to Sec. 46.2 of the IP Code. 27.3. If the language in which the international publication has been effected is a language other than English, the protection of any right of the applicant under Section 46 of the IP Code in the Philippines shall be applicable only from such time as a translation into English has been published in the IPO Gazette under Section 44 of the IP Code and its implementing rules and regulations, and, such translation into English has been transmitted by the applicant to the actual unauthorized user of the invention claimed in the international application pursuant to Section 46.2 of the IP Code. 27.4. Where the international publication has been effected, on the request of the applicant, before the expiration of eighteen (18) months from the priority date, the rights provided for under Sec. 46 of the IP Code shall be applicable only from the expiration of eighteen (18) months from the priority date subject to the conditions mentioned in the preceding paragraphs. RULE 28. DEMAND FOR INTERNATIONAL PRELIMINARY EXAMINATION. – 28.1. On filing of a demand under PCT Article 31, the international application shall be the subject of an international preliminary examination. 28.2. The demand for international preliminary examination shall be filed directly by the applicant with the competent IPEA prior to the expiration of the nineteenth (19th) month from the priority date. 28.3. The demand shall indicate the States in which the applicant intends to use the results of the international preliminary examination. 28.4. In accordance with PCT Article 31 and PCT Rules 57 and 58, the demand shall be subject to the payment by the applicant of the prescribed fees directly to the IPEA within one (1) month from the date on which the demand was submitted. Copies of the Demand Form are available at the IPO. RULE 29. MANNER OF MAKING CORRECTIONS. – Any correction offered to the IPO as a receiving Office may be stated in a letter addressed to the IPO if the correction is of such a nature that it can be transferred from the letter to the record copy without adversely affecting the clarity and the direct reproducibility of the sheet on to which the correction is to be transferred; otherwise, the applicant shall be required to submit the missing requirement or a replacement sheet containing the correction
accompanied by a letter indicating the differences between the replaced sheet and the replacement sheet. RULE 30. DELAYS IN MEETING TIME LIMITS. Delays in meeting time limits during international processing of international applications may only be excused as provided in PCT Rule 82. RULE 31. RECORDING OF CHANGES. -‐ Before the expiration of the time limit referred to in PCT Articles 22(1) and 39(1)(a), the IPO may receive from the applicant a request for the recording of a change in the person, name, residence, nationality or address of the applicant or a change in the person, name or address of the inventor, the agent or the common representative. The IPO shall notify the IB of the request for recording a change. RULE 32. WITHDRAWALS. -‐ The IPO may receive from the applicant notice of withdrawal of the international application, any designation, or priority claims in accordance with PCT Rule 90bis(1), 90bis(2), 90bis(3). Withdrawal of a demand or any election shall be made directly to the IB in accordance with PCT Rule 90bis.4. RULE 33. CERTIFIED COPIES OF THE INTERNATIONAL APPLICATION AND PRIORITY DOCUMENT. – (a) The applicant may request the IPO to prepare a certified copy of the international application as filed and of any corrections thereto upon payment of the appropriate fee. (b) Where the priority document is issued by the IPO, the applicant may, pursuant to PCT Rule 17.1(b), request the IPO to prepare and transmit the priority document to the IB. Such request shall be made not later than sixteen (16) months after the priority date and shall be subject to payment of a fee. PART III. NATIONAL PHASE RULE 34. THE IPO AS A DESIGNATED OR ELECTED OFFICE. – 34.1. The IPO will act as the Designated or Elected Office for international applications in which the Philippines has been designated or elected. 34.2. The IPO, when acting as a Designated or Elected Office during international processing will be identified by the full title “Philippine Designated Office” or by abbreviation “DO/PH” and “Philippine Elected Office” or by abbreviation “EO/PH.” 34.3. The major functions of the IPO in respect to international applications in which the Philippines has been designated or elected, shall include:
150 z
Intellectual Property Law
with the IP Code and its implementing rules and regulations. RULE 36. ENTRY INTO THE NATIONAL PHASE OF AN INTERNATIONAL APPLICATION CLAIMING THE PRIORITY OF AN EARLIER PHILIPPINE NATIONAL APPLICATION. Within one (1) month from the date of entry into the national phase of an international application claiming the priority of an earlier Philippine national application, the IPO shall invite the applicant to elect only one (1) application for further prosecution within two (2) months from the date of the invitation, or within a longer period which the IPO may allow but not to exceed six (6) months from the date of entry into the national phase. RULE 37. REVIEW BY IPO AS A D ESIGNATED OFFICE. – (a) Where the receiving Office other than the IPO has refused to accord an international filing date to an international application designating the Philippines or has declared that the international application is considered withdrawn either generally or as to the Philippines, or where the IB has made a finding under PCT Article 12(3) that the international application is considered withdrawn because it has not received the record copy within the prescribed time limit, the applicant may request review of the matter by the IPO provided the requirements of and the time limit specified by the Treaty and PCT regulations as well as the requirements for entry into national phase in these Rules are complied with. (b) Where the refusal, declaration or finding mentioned in paragraph (a) was the result of an error or omission on the part of the receiving Office or that the finding was the result of an error or omission on the part of the IB, the IPO as a designated Office may treat the international application as if such error or omission has not occurred. RULE 38. TIME LIMIT TO SUBMIT THE PRIORITY DOCUMENT. – 38.1. The priority claim of an international application designating or electing the Philippines shall not be disregarded by IPO, where the priority document is received by the IB not later than sixteen (16) months after the priority date in accordance with PCT Rule 17.1 (a) and (b). 38.2. If the priority document is not submitted to the IB within sixteen (16) months after the priority date in accordance with PCT Rule 17.1 (a) and (b), the applicant may submit to the IPO a certification from the national Office concerned stating the following data regarding the application the priority of which is claimed: name of the applicant, filing date, application number and title of the application. The certification, and its English translation if not in English, together with payment of the fee for extension of time to submit priority document and the surcharge for late payment provided
(1) Receiving various notifications throughout the international phase; and (2) Accepting for national phase examination international applications which comply with Part 3 of these Rules.
RULE 35. ENTRY INTO NATIONAL PHASE. – (a) An international application enters the national phase when the applicant furnishes IPO a copy of the international application in English (unless already transmitted by the IB), or, if the application was filed in another language, its English translation not later than thirty (30) months from the priority date based on PCT Article 22 (1) and PCT Article 39 (1) (a). Subject to the payment of an extension fee for late entry equal to fifty percent (50%) of the filing fee prescribed in the IPO Fee Structure, the entry into the national phase may be extended by one (1) month. (b) The filing fee prescribed in the IPO fee structure shall be paid within one (1) month from the date of entry into the national phase; provided that an international application which claims the priority of an earlier Philippine national application shall be exempt from payment of the filing fee.If the applicant fails to pay the filing fee as herein provided, the application shall be deemed withdrawn in the Philippines. (c) The translation of the international application, as filed, referred to in paragraph (a) shall include a translation into English of: (1) the description; (2) the claims as filed and, where applicable, as amended under PCT Article 19; (3) any text matter of the drawings; (4) the abstract; and (5) where applicable, amendments to the description, claims or drawings under PCT Article 34, as contained in the annexes to the international preliminary examination report. If the translation of the amendments under PCT Article 19 or PCT Article 34, as the case may be, is not included, those amendments will be considered cancelled. (d) During the national phase, and at anytime before a patent is granted or refused on the international application, the applicant may present amendments to the specification, claims and drawings in accordance
o 151
Katrina Michelle Mancao for in the IPO Fee Structure, shall be submitted within six (6) months from the date of entry of the international application into the national phase without need of notice. The certification and its English translation if not in English do not require legalization. Failure of the applicant to comply with this rule shall be ground to disregard the priority claim. 38.3 Where the validity of the priority claim is relevant to the determination of whether the invention concerned is patentable, the applicant shall be required to submit the English translation of the priority document. RULE 39. PROCEDURE FOR GRANT OF PATENT. – An international application that has entered the national phase shall be governed by the applicable provisions under Republic Act No. 8293 and its implementing rules and regulations and the applicable provisions of the Treaty, PCT Regulations, and PCT Administrative Instructions. RULE 40. APPLICATION OF THE IPO FEE STRUCTURE. – The IPO Fee Structure shall apply after entry into the national phase of the international application provided that: (a) an international application which validly claims the priority of an earlier Philippine national application shall be exempt from payment of fees that were already paid in connection with that earlier Philippine national application; (b) the due date for payment of the first annual fee shall be computed from the date of the international publication regardless of the language of publication; and (c) the international application shall be deemed withdrawn if the substantive examination fee is not paid within six (6) months from the date of entry into the national phase. The applicant may use a form for payment prescribed by IPO for this purpose. PART III. FEES DURING INTERNATIONAL PHASE RULE 41. FEES TO BE COLLECTED BY THE IPO. – The following fees shall be collected by the IPO: 41.1. Fees for the benefit of the IPO. The type and amount of fees for the benefit of the IPO are: Ref. Code 901
Type of Fee Transmittal Fee PCT Rule 14
902 903
904
905 906
907
Priority Document Fee PCT Rule 17.1(b) Fee for Transmitting Priority Document PCT Rule 17.1(b) Confirmation Fee PCT Rules 4.9(c) and 15.5(a)
Late Payment Fee PCT Rule 16bis.2(a)(ii) Certified Copy of the International Application Fee PCT Rule 20.9 Transmittal Fee PCT Rule 19.4
500.00 1,750.00
Fifty (50%) of the sum of designation fees under PCT Rule 15.5 in Philippine currency at the prevailing rate of exchange at the time the payment is due 3,500.00 500.00
3,500.00
41.2. Fees for the benefit of the IB (International Fee) The amount of the basic fee referred to in PCT Rule 15.1(i), the designation fee referred to in PCT Rule 15.1(ii) for any designation made under PCT Rule 4.9(a), and the designation fee referred to in PCT Rule 15.5 for any designation made under PCT Rule 4.9(b) and confirmed under PCT Rule 4.9(c), are published in the PCT Gazette and are available at the IPO. 41.3. Fee for the benefit of the ISA (Search Fee)chan robles virtual law library The amount of the search fee referred to in PCT Rule 16.1 of the competent ISA or ISAs for the Philippines is published in the PCT Gazette. RULE 42. D UE DATES AND EXTENSION OF TIME LIMITS FOR PAYMENT OF FEES COLLECTED BY IPO. – All fees must be paid on or before the due date prescribed in the PCT Regulations and these Rules. 42.1. The transmittal fee, basic fee and search fee shall be paid by the applicant within one (1) month from the date of receipt by the IPO of the international application; 42.2. The designation fee for any designation made under PCT Rule 4.9(a) shall be paid within a time limit of: (a) one (1) year from the priority date, or (b) one (1) month from the date of receipt of the international application if that one-‐month period expires later than one (1) year from the priority date; 42.3. Where the fees due under this rule have not been paid, or that the amount paid is insufficient to cover the total amount due, the IPO shall invite the applicant to pay the required fees together with the late payment fee
Amount(Philippine Peso) 3,500.00
152 z
Intellectual Property Law within one (1) month without extension of time from the date of invitation; 42.4. The designation fee under PCT Rule 15.5 for any designation made under PCT Rule 4.9(b) and confirmed under PCT Rule 4.9(c), together with the confirmation fee under PCT Rule 15.5, shall be paid within fifteen (15) months from the priority date. RULE 43. FEES FOR THE BENEFIT OF IPO; FORM, MANNER AND CURRENCY OF PAYMENT. – (a) All fees for the benefit of the IPO shall be paid in cash, money order, bank drafts and/or checks in Philippine currency. In case of cash payment, the date of receipt is the date on which the full amount due was paid in cash. In case of money order, check or bank draft payment, the date of receipt of the money order, check or bank draft will be the date of presentment provided that the same is honored upon first presentment and provided that the payment covers the full amount due. (b) Payment shall be made to the IPO Cashier during regular working days and business hours from Monday to Friday, 8:00 a.m. to 12:00 noon and from 1:00 p.m. to 5:00 p.m. RULE 44. FEES FOR THE BENEFIT OF IB AND ISA; FORM, MANNER AND CURRENCY OF PAYMENT. – 44.1 All fees for the benefit of the IB and the ISA collected by IPO as receiving Office shall be paid to IPO in the currency or any of the currencies, if more than one, specified by the IB or the ISA as the case may be, in the form of cash or bank draft. In case of cash payment, the date of receipt is the date on which the full amount due was paid in cash. In case of bank draft payment, the date of receipt of the bank draft will be the date of presentment provided that the same is honored upon first presentment and provided further that the payment covers the full amount due. Payment shall be made to the IPO Cashier during regular working days and business hours from Monday to Friday, 8:00 a.m. to 12:00 noon and from 1:00 p.m. to 5:00 p.m. 44.2. Where the international application incurs an additional fee other than the search fee under PCT Rule 16, the applicant shall pay directly to the ISA such other fees required by the ISA in the currency, form and manner prescribed by it. 44.3. The Schedules of Fees of the IB and ISAs are published in the PCT Gazette and are available at the IPO. Rule 45. Fees Payable to the IPEA. -‐ The international preliminary examination fee and the handling fee under PCT Rule 58 shall be paid directly to the IPEA in the currency, form and manner prescribed by it.
Other fees due to the IPEA in accordance with the Treaty and PCT Regulations shall be paid directly to the IPEA in the currency, form and manner prescribed by it. The Schedules of Fees of the IPEAs are published in the PCT Gazette and are available at the IPO. RULE 46. REFUNDS. – 46.1 The IPO shall refund the international fee to the applicant in accordance with PCT Rule 15.6: (a) if the IPO has found that the international filing date cannot be accorded in accordance with PCT Article 11(1); (b) if, before the transmittal of the record copy to the IB, the international application is withdrawn or considered withdrawn; or (c) if, due to prescriptions concerning the national security, the international application is not treated as such. 46.2. The IPO shall refund the search fee to the applicant in accordance with PCT Rule 16.2: (a) if the IPO has found that the international filing date cannot be accorded in accordance with PCT Article 11(1); (b) if, before the transmittal of the search copy to the ISA, the international application is withdrawn or considered withdrawn; or (c) if, due to prescriptions concerning the national security, the international application is not treated as such. RULE 47. REDUCTION IN FEES. – An applicant may be entitled to a reduction in fees provided that the requirements, as specified by the IB and Authorities under the Treaty granting such reduction in fees, are fully satisfied and/or duly complied with. RULE 48. SEPARABILITY CLAUSE. – If any provision in these Rules or application of such provision to any circumstance is held invalid, the remainder of these Rules shall not be affected. RULE 49. FILING OF CERTIFIED COPIES. – Mr. Eduardo Joson, Records Officer II, is hereby directed to immediately file three (3) certified copies of these Rules with the University of the Philippines Law Center, and, one (1) certified copy each with the Office of the President, the Senate of the Philippines, the House of Representatives, the Supreme Court of the Philippines, and the National Library.
o 153
Katrina Michelle Mancao patentee, he has the exclusive right of making, selling or using the invention. On the assumption that petitioner’s advertising units were patentable inventions, petitioner revealed them fully to the public by submitting the engineering drawings thereof to the National Library. To be able to effectively and legally preclude others from copying and profiting from the invention, a patent is a primordial requirement. No patent, no protection. The ultimate goal of a patent system is to bring new designs and technologies into the public domain through disclosure. Ideas, once disclosed to the public without the protection of a valid patent, are subject to appropriation without significant restraint. On one side of the coin is the public which will benefit from new ideas; on the other are the inventors who must be protected. The act secured to the inventor the exclusive right to make use, and vend the thing patented, and consequently to prevent others from exercising like privileges without the consent of the patentee. It was passed for the purpose of encouraging useful invention and promoting new and useful inventions by the protection and stimulation given to inventive genius, and was intended to secure to the public, after the lapse of the exclusive privileges granted the benefit of such inventions and improvements. The law attempts to strike an ideal balance between the two interests: “The patent system thus embodies a carefully crafted bargain for encouraging the creation and disclosure of new useful and non-‐obvious advances in technology and design, in return for the exclusive right to practice the invention for a number of years. The inventor may keep his invention secret and reap its fruits indefinitely. In consideration of its disclosure and the consequent benefit to the community, the patent is granted. An exclusive enjoyment is guaranteed him for 17 years, but upon the expiration of that period, the knowledge of the invention inures to the people, who are thus enabled to practice it and profit by its use.” The patent law has a three-‐fold purpose: “first, patent law seeks to foster and reward invention; second, it promotes disclosures of inventions to stimulate further innovation and to permit the public to practice the invention once the patent expires; third, the stringent requirements for patent protection seek to ensure that ideas in the public domain remain there for the free use of the public.” It is only after an exhaustive examination by the patent office that a patent is issued. Such an in-‐depth investigation is required because in rewarding a useful
RULE 50. EFFECTIVITY. – Without prejudice to the performance by the IPO of its functions as a receiving Office, designated Office or elected Office under the Treaty, from and as of the entering into effect of the Treaty with respect to the Philippines on 17 August 2001, these Rules shall take effect on October 22, 2001 after publication in a newspaper of general circulation in the Philippines on October 4, 2001.
I.
Issuance or Refusal of Patents
SECTION 50. GRANT OF PATENT. – 50.1. If the application meets the requirements of this Act, the Office shall grant the patent: Provided, That all the fees are paid on time. 50.2. If the required fees for grant and printing are not paid in due time, the application shall be deemed to be withdrawn. 50.3. A patent shall take effect on the date of the publication of the grant of the patent in the IPO Gazette.
SECTION 51. REFUSAL OF THE APPLICATION. – 51.1. The final order of refusal of the examiner to grant the patent shall be appealable to the Director in accordance with this Act. 51.2. The Regulations shall provide for the procedure by which an appeal from the order of refusal from the Director shall be undertaken.
SECTION 53. CONTENTS OF PATENT. – The patent shall be issued in the name of the Republic of the Philippines under the seal of the Office and shall be signed by the Director, and registered together with the description, claims, and drawings, if any, in books and records of the Office.
PEARL & DEAN V. SHOEMART (2003), SUPRA Court decision: For some reason or another, petitioner never secured a patent for the light boxes. It therefore acquired no patent rights which could have protected its invention, if in fact it really was. And because it had no patent, petitioner could not legally prevent anyone from manufacturing or commercially using the contraption. There can be no infringement of a patent until a patent has been issued, since whatever right one has to the invention covered by the patent arises alone from the grant of patent. An inventor has no common law right to a monopoly of his invention. He has the right to make use of and vend his invention, but if he voluntarily discloses it, such as by offering it for sale, the world is free to copy and use it with impunity. A patent, however, gives the inventor the right to exclude all others. As a
154 z
Intellectual Property Law invention, the rights and welfare of the community must be fairly dealt with and effectively guarded. To that end, the prerequisites to obtaining a patent are strictly observed and when a patent is issued, the limitations on its exercise are equally strictly enforced. To begin with, a genuine invention or discovery must be demonstrated lest in the constant demand for new appliances, the heavy hand of tribute be laid on each slight technological advance in art. There is no such scrutiny in the case of copyrights nor any notice published before its grant to the effect that a person is claiming the creation of a work. The law confers the copyright from the moment of creation and the copyright certificate is issued upon registration with the National Library of a sworn ex-‐parte claim of creation. Therefore, not having gone through the arduous examination for patents, the petitioner cannot exclude others from the manufacture, sale or commercial use of the light boxes on the sole basis of its copyright certificate over the technical drawings. Stated otherwise, what petitioner seeks is exclusivity without any opportunity for the patent office (IPO) to scrutinize the light box’s eligibility as a patentable invention. The irony here is that, had petitioner secured a patent instead, its exclusivity would have been for 17 years only. But through the simplified procedure of copyright-‐registration with the National Library — without undergoing the rigor of defending the patentability of its invention before the IPO and the public — the petitioner would be protected for 50 years. This situation could not have been the intention of the law. In the oft-‐cited case of Baker vs. Selden, the United States Supreme Court held that only the expression of an idea is protected by copyright, not the idea itself. In that case, the plaintiff held the copyright of a book which expounded on a new accounting system he had developed. The publication illustrated blank forms of ledgers utilized in such a system. The defendant reproduced forms similar to those illustrated in the plaintiff’s copyrighted book. The US Supreme Court ruled that: “There is no doubt that a work on the subject of book-‐keeping, though only explanatory of well known systems, may be the subject of a copyright; but, then, it is claimed only as a book. x x x. But there is a clear distinction between the books, as such, and the art, which it is, intended to illustrate. The mere statement of the proposition is so evident that it requires hardly any argument to support it. The same distinction may be predicated of every other art as well as that of bookkeeping. A treatise
on the composition and use of medicines, be they old or new; on the construction and use of ploughs or watches or churns; or on the mixture and application of colors for painting or dyeing; or on the mode of drawing lines to produce the effect of perspective, would be the subject of copyright; but no one would contend that the copyright of the treatise would give the exclusive right to the art or manufacture described therein. The copyright of the book, if not pirated from other works, would be valid without regard to the novelty or want of novelty of its subject matter. The novelty of the art or thing described or explained has nothing to do with the validity of the copyright. To give to the author of the book an exclusive property in the art described therein, when no examination of its novelty has ever been officially made, would be a surprise and a fraud upon the public. That is the province of letters patent, not of copyright. The claim to an invention of discovery of an art or manufacture must be subjected to the examination of the Patent Office before an exclusive right therein can be obtained; and a patent from the government can only secure it. The difference between the two things, letters patent and copyright, may be illustrated by reference to the subjects just enumerated. Take the case of medicines. Certain mixtures are found to be of great value in the healing art. If the discoverer writes and publishes a book on the subject (as regular physicians generally do), he gains no exclusive right to the manufacture and sale of the medicine; he gives that to the public. If he desires to acquire such exclusive right, he must obtain a patent for the mixture as a new art, manufacture or composition of matter. He may copyright his book, if he pleases; but that only secures to him the exclusive right of printing and publishing his book. So of all other inventions or di